Feat: regenerate
continuous-integration/drone/push Build is passing Details

This commit is contained in:
Bertrand Benjamin 2021-05-20 07:23:31 +02:00
parent 3c4a16a982
commit 7bac6d2517
36 changed files with 1846 additions and 1880 deletions

View File

@ -3,7 +3,7 @@
% Title Page
\title{DM1 \hfill BALLARD Antoine}
\tribe{Maths complémentaire}
\tribe{Maths complémentaires}
\date{\hfillÀ render pour le jeudi 27 mai}
\xsimsetup{
@ -17,7 +17,7 @@ Les valeurs des exercices sont générés automatiquement. Si une valeur a un no
\begin{exercise}[subtitle={Optimisation de matière}]
\begin{minipage}{0.6\textwidth}
On se propose de fabriquer avec le moins de tôle possible une citerne fermée en forme de parallélépipède rectangle dont le volume intérieur doit être de $8m^3$. La longueur est aussi fixée à $4m$ par le cahier des charges.
On se propose de fabriquer avec le moins de tôle possible une citerne fermée en forme de parallélépipède rectangle dont le volume intérieur doit être de $20m^3$. La longueur est aussi fixée à $2m$ par le cahier des charges.
On peut donc faire varier uniquement la largeur (notée $x$) et la hauteur (notée $h$) de la cuve.
\end{minipage}
@ -28,25 +28,25 @@ Les valeurs des exercices sont générés automatiquement. Si une valeur a un no
\pgfmathsetmacro{\cubey}{1}
\pgfmathsetmacro{\cubez}{2}
\draw[black,fill=gray] (0,0,0) -- ++(-\cubex,0,0) -- ++(0,-\cubey,0) node [midway, left] {$h$} -- ++(\cubex,0,0) node [midway, below] {$x$} -- cycle;
\draw[black,fill=gray] (0,0,0) -- ++(0,0,-\cubez) -- ++(0,-\cubey,0) -- ++(0,0,\cubez) node [midway, right] {$4m$} -- cycle;
\draw[black,fill=gray] (0,0,0) -- ++(0,0,-\cubez) -- ++(0,-\cubey,0) -- ++(0,0,\cubez) node [midway, right] {$2m$} -- cycle;
\draw[black,fill=gray] (0,0,0) -- ++(-\cubex,0,0) -- ++(0,0,-\cubez) -- ++(\cubex,0,0) -- cycle;
\end{tikzpicture}
\end{minipage}
\begin{enumerate}
\item Expliquer pourquoi quand la largeur $x$ change, la hauteur $h$ doit elle aussi changer pour respecter les contraintes.
\item Démontrer que l'on doit avoir $h = \dfrac{2}{x}$.
\item Démontrer que l'on doit avoir $h = \dfrac{10}{x}$.
\item On note $S(x)$ l'aire totale de la citerne (c'est à dire la somme des aires des six faces). Montrer que l'on peut écrire
\[
S(x) = 8x + 4 + \frac{16}{x}
S(x) = 4x + 20 + \frac{40}{x}
\]
\item Démontrer que
\[
S(x) = \frac{8x^2 + 4x + 16}{x}
S(x) = \frac{4x^2 + 20x + 40}{x}
\]
\item Démontrer que
\[
S'(x) = \frac{8x^2 - 16}{x^2}
S'(x) = \frac{4x^2 - 40}{x^2}
\]
\item En déduire le tableau de variation de $S(x)$ sur $\intOF{0}{10}$.
\item Déterminer les valeurs de $x$ et $h$ correspondant à une utilisation minimal de tôle.
@ -57,62 +57,62 @@ Les valeurs des exercices sont générés automatiquement. Si une valeur a un no
\begin{enumerate}
\item Le volume étant fixe si l'on fait varier $x$, $h$ doit aussi varier.
\begin{itemize}
\item Si $x = 2$ alors conserver un volume de $V=8$, $h$ doit être égale à $2 / 2$
\item Si $x = 3$ alors conserver un volume de $V=8$, $h$ doit être égale à $2 / 3$
\item Si $x = 2$ alors conserver un volume de $V=20$, $h$ doit être égale à $10 / 2$
\item Si $x = 3$ alors conserver un volume de $V=20$, $h$ doit être égale à $10 / 3$
\end{itemize}
\item Pour calculer le volume, on a
\begin{eqnarray*}
V &=& h\times x \times 4 \\
8 &=& h\times x \times 4 \\
x &=& \frac{8}{h\times 4} = \frac{2}{h}
V &=& h\times x \times 2 \\
20 &=& h\times x \times 2 \\
x &=& \frac{20}{h\times 2} = \frac{10}{h}
\end{eqnarray*}
\item Pour calculer la surface totale, on ajoute la surface de chaque face. On a donc le calcul suivant
\begin{eqnarray*}
S(x) &=& x\times h \times 2 + x\times4\times2 + h\times 4\times 2\\
S(x) &=& x\times \frac{2}{x} \times 2 + x\times4\times2 + \frac{2}{x}\times 4\times 2\\
S(x) &=& 8x + 4 + \frac{16}{x}
S(x) &=& x\times h \times 2 + x\times2\times2 + h\times 2\times 2\\
S(x) &=& x\times \frac{10}{x} \times 2 + x\times2\times2 + \frac{10}{x}\times 2\times 2\\
S(x) &=& 4x + 20 + \frac{40}{x}
\end{eqnarray*}
\item Pour trouver cette nouvelle forme, on met chaque élément sur le même dénominateur
\begin{eqnarray*}
S(x) &=& 8x + 4 + \frac{16}{x}\\
S(x) &=& \frac{8x\times x}{x} + \frac{4\times x}{x} + \frac{16}{x}\\
S(x) &=& \frac{8x^2 + 4x + 16}{x}
S(x) &=& 4x + 20 + \frac{40}{x}\\
S(x) &=& \frac{4x\times x}{x} + \frac{20\times x}{x} + \frac{40}{x}\\
S(x) &=& \frac{4x^2 + 20x + 40}{x}
\end{eqnarray*}
\item On retrouve la formule $\frac{u}{v}$ à dériver
\[
u(x) = 8x^2 + 4x + 16 \Rightarrow u'(x) = 16x + 4
u(x) = 4x^2 + 20x + 40 \Rightarrow u'(x) = 8x + 20
\]
\[
v(x) = x \Rightarrow v'(x) = 1
\]
Donc au numérateur on obtient
\begin{eqnarray*}
u'(x)\times v(x) - u(x)\times v'(x) &=& (16x + 4)\times x - (8x^2 + 4x + 16)\times 1\\
&=& 8x^2 - 16
u'(x)\times v(x) - u(x)\times v'(x) &=& (8x + 20)\times x - (4x^2 + 20x + 40)\times 1\\
&=& 4x^2 - 40
\end{eqnarray*}
Donc
\[
S'(x) = \frac{8x^2 - 16}{x^2}
S'(x) = \frac{4x^2 - 40}{x^2}
\]
\item Tableau de variations de $S$
\begin{itemize}
\item Valeur interdite: $x^2 = 0 \equiv x = 0$
\item Signe de $8x^2 - 16$: c'est un polynôme du 2e degré
\item Signe de $4x^2 - 40$: c'est un polynôme du 2e degré
\[
\Delta = 512 > 0
\Delta = 640 > 0
\]
Il y a donc 2 racines
\[
x_1 = - 1.4142135623730951 \qquad
x_2 = 1.4142135623730951
x_1 = - 3.1622776601683795 \qquad
x_2 = 3.1622776601683795
\]
Et on sait que $8x^2 - 16$ est du signe de $a$ donc positif en dehors des racines
Et on sait que $4x^2 - 40$ est du signe de $a$ donc positif en dehors des racines
\item Le dénominateur $x^2$ est toujours positif.
\item Tableau de variations
\begin{tikzpicture}[baseline=(a.north)]
\tkzTabInit[lgt=3,espcl=3]{$x$/1,$8x^2 - 16$/1, $x^2$/1, $S'$/1, $S$/2}{$0$, $- 1.4142135623730951$, $10$}
\tkzTabInit[lgt=3,espcl=3]{$x$/1,$4x^2 - 40$/1, $x^2$/1, $S'$/1, $S$/2}{$0$, $- 3.1622776601683795$, $10$}
\tkzTabLine{d,-, z, +, }
\tkzTabLine{d,+, , +, }
\tkzTabLine{d,-, z, +, }
@ -120,7 +120,7 @@ Les valeurs des exercices sont générés automatiquement. Si une valeur a un no
\end{tikzpicture}
\end{itemize}
\item On a donc une surface minimal pour $x=1.4142135623730951$ et $h = 2.8284271247461902$.
\item On a donc une surface minimal pour $x=3.1622776601683795$ et $h = 31.6227766016837950$.
\end{enumerate}
\end{solution}
@ -133,14 +133,14 @@ Les valeurs des exercices sont générés automatiquement. Si une valeur a un no
Le tour d'un bassin au niveau du sol présente deux axes de symétrie : laxe des abscisses et la droite déquation $x=4$. Il est obtenu par symétrie de la courbe $\mathcal{C}_f$ sur $\intFF{0}{4}$$f$ est la fonction définie par
\[
f(x) = \left(- x^{2} + 0.3 x - 7.3\right) e^{- x} + 7.3
f(x) = \left(- x^{2} + 4.7 x - 7.9\right) e^{- x} + 7.9
\]
On admet que sur $\intFF{0}{4}$ la fonction $f$ est positive.
\begin{enumerate}
\item Sur un repère, tracer l'allure de la courbe $\mathcal{C}_f$, les axes de symétries puis compléter pour dessiner la forme du bassin.
\item Montrer que la fonction $f$ admet comme primitive sur $\R$ la fonction $F$ définie par
\[
F(x) = 7.3 x + \left( x^{2} + 1.7 x + 9.0\right) e^{- x}
F(x) = 7.9 x + \left( x^{2} - 2.7 x + 5.2\right) e^{- x}
\]
\item Calculer la quantité $\ds \int_0^4 f(x) \; dx$, vous donnerez le résultat sous forme exacte. Interpréter le résultat et reportez cette quantité sur le graphique.
\item On considère que l'échelle de votre graphique est de 1unité pour 15m. Calculer l'aire du bassin. Vous donnerez un résultat arrondi au $m^2$ près.
@ -156,15 +156,14 @@ Les valeurs des exercices sont générés automatiquement. Si une valeur a un no
\tkzGrid
\tkzAxeXY
\tkzFct[domain=0:10,color=red,very thick]%
{ (-x**2 + 0.3*x - 7.3)*exp(-x) + 7.3 };
{ (-x**2 + 4.7*x - 7.9)*exp(-x) + 7.9 };
\end{tikzpicture}
\item Il faut dériver $F(x)$ et vérifier que $F'(x) = f(x)$.
\item $\ds \int_0^4 f(x) \; dx = F(4) - F(0) = \frac{31.8}{e^{4}} + 20.2$
\item $\ds \int_0^4 f(x) \; dx = F(4) - F(0) = \frac{10.4}{e^{4}} + 26.4$
\item La quantité calculée à la question précédente se retrouve 4fois pour former le bassin. Il faut ensuite prendre en compte l'échelle, comme 1unité de longueur correspond à 15m, une unité d'air correspond à $15\times15 = 225m^2$. Ainsi l'aire du bassin est égale à
\[
(\frac{31.8}{e^{4}} + 20.2)\times 4 \times 15^2 = 18704.00000
(\frac{10.4}{e^{4}} + 26.4)\times 4 \times 15^2 = 23931.00000
\]
\end{enumerate}
\end{solution}
@ -185,9 +184,9 @@ Les valeurs des exercices sont générés automatiquement. Si une valeur a un no
Deux ateliers A et B fabriquent des stylos pour une entreprise.
L'atelier A fabrique 86.0\,\% des stylos, et parmi ceux-là, 34.0\,\% possèdent un défaut de fabrication.
L'atelier A fabrique 40.0\,\% des stylos, et parmi ceux-là, 24.0\,\% possèdent un défaut de fabrication.
De plus, 8.0\,\% des stylos possèdent un défaut de fabrication et sortent de l'atelier B.
De plus, 50.0\,\% des stylos possèdent un défaut de fabrication et sortent de l'atelier B.
Un stylo est prélevé au hasard dans le stock de l'entreprise.
@ -241,7 +240,7 @@ Les valeurs des exercices sont générés automatiquement. Si une valeur a un no
\item
\begin{enumerate}
\item Calculer la probabilité qu'un stylo provienne de l'atelier A et possède un défaut de fabrication.
\item En déduire que la probabilité qu'un stylo possède un défaut de fabrication est de $0.37$.
\item En déduire que la probabilité qu'un stylo possède un défaut de fabrication est de $0.6$.
\end{enumerate}
\item On prélève un stylo au hasard avec un défaut. Quelle est la probabilité qu'il vienne de l'atelier A?
\end{enumerate}
@ -251,7 +250,7 @@ Les valeurs des exercices sont générés automatiquement. Si une valeur a un no
\textbf{Partie B}
\medskip
Dans cette partie, on suppose que 37.0\,\% des stylos possèdent un défaut de fabrication.
Dans cette partie, on suppose que 60.0\,\% des stylos possèdent un défaut de fabrication.
L'entreprise confectionne des paquets contenant chacun $4$~stylos.
@ -266,7 +265,7 @@ Les valeurs des exercices sont générés automatiquement. Si une valeur a un no
\begin{enumerate}
\setcounter{enumi}{4}
\item Avec quelle loi peut-on modéliser $X$. Préciser les paramètres.
\item Calculer et interpréter la probabilité $P(X = 10)$.
\item Calculer et interpréter la probabilité $P(X = 12)$.
\item Le directeur de l'entreprise affirme qu'il y a plus d'une chance sur deux qu'un paquet ne comporte aucun stylo défectueux. A-t-il raison ?
\item Combien de stylos peut-on espérer avoir en moyenne?
\end{enumerate}
@ -282,27 +281,27 @@ Les valeurs des exercices sont générés automatiquement. Si une valeur a un no
child {node {$A$}
child {node {$D$}
edge from parent
node[above] {0.34}
node[above] {0.24}
}
child {node {$\overline{D}$}
edge from parent
node[above] {0.66}
node[above] {0.76}
}
edge from parent
node[above] {0.86}
node[above] {0.4}
}
child[missing] {}
child { node {$B$}
child {node {$D$}
edge from parent
node[above] {0.58}
node[above] {0.83}
}
child {node {$\overline{D}$}
edge from parent
node[above] {0.42}
node[above] {0.17}
}
edge from parent
node[above] {0.14}
node[above] {0.6}
} ;
\end{tikzpicture}
\end{center}
@ -310,51 +309,51 @@ Les valeurs des exercices sont générés automatiquement. Si une valeur a un no
\begin{itemize}
\item Probabilité que le stylo vienne de l'atelier A
\[
P(A) = 0.86
P(A) = 0.4
\]
\item Probabilité que le stylo vienne de l'atelier B
\[
P(B) = 0.14
P(B) = 0.6
\]
\item Probabilité que le stylo ait un défaut sachant qu'il vient de l'atelier A.
\[
P_A(D) = 0.34
P_A(D) = 0.24
\]
\item Probabilité que le stylo vienne de l'atelier B et qu'il ait un défaut.
\[
P(D \cap D) = 0.08
P(D \cap D) = 0.5
\]
\end{itemize}
\item
\begin{enumerate}
\item Probabilité qu'un stylo vienne de l'atelier A et qu'il ait un defaut
\[
P(A\cap D) = P(A) \times P_A(D) = 0.86 \times 0.34 = 0.29
P(A\cap D) = P(A) \times P_A(D) = 0.4 \times 0.24 = 0.1
\]
\item Probabilité que le stylo ai un défaut de fabrication.
\[
P(D) = P(A\cap D) + P(B\cap D) = 0.29 + 0.08 = 0.37
P(D) = P(A\cap D) + P(B\cap D) = 0.1 + 0.5 = 0.6
\]
\end{enumerate}
\item Probabilité qu'il vienne de l'atelier A sachant qu'il a un defaut
\[
P_D(A) = \frac{P(A\cap D)}{P(D)} = \frac{0.29}{0.37} = 0.78
P_D(A) = \frac{P(A\cap D)}{P(D)} = \frac{0.1}{0.6} = 0.17
\]
\item $X$ peut être modélisée par une loi binomiale de paramètres $n=20$ et $p=0.37$.
\item $X$ peut être modélisée par une loi binomiale de paramètres $n=15$ et $p=0.6$.
\item (\textit{par de correction automatique disponible pour le résultat final}
\[
P(X = 10) = \coefBino{20}{10}\times 0.37^{10} \times 0.63^{10}
P(X = 12) = \coefBino{15}{12}\times 0.6^{12} \times 0.4^{3}
\]
\item (\textit{par de correction automatique disponible pour le résultat final}
Il faut calculer la probabilité qu'il y ait 0 stylo avec un defaut.
\[
P(X = 0) = \coefBino{20}{0}\times 0.37^{0} \times 0.63^{20}
P(X = 0) = \coefBino{15}{0}\times 0.6^{0} \times 0.4^{15}
\]
Puis comparer ce nombre à 0,5.
\item Il faut calculer l'espérance
\[
E[X] = n\times p = 20 \times 0.37 = 7.4
E[X] = n\times p = 15 \times 0.6 = 9.0
\]
\end{enumerate}
\end{solution}

View File

@ -3,7 +3,7 @@
% Title Page
\title{DM1 \hfill BALUKHATYY Alexandre}
\tribe{Maths complémentaire}
\tribe{Maths complémentaires}
\date{\hfillÀ render pour le jeudi 27 mai}
\xsimsetup{
@ -17,7 +17,7 @@ Les valeurs des exercices sont générés automatiquement. Si une valeur a un no
\begin{exercise}[subtitle={Optimisation de matière}]
\begin{minipage}{0.6\textwidth}
On se propose de fabriquer avec le moins de tôle possible une citerne fermée en forme de parallélépipède rectangle dont le volume intérieur doit être de $15m^3$. La longueur est aussi fixée à $3m$ par le cahier des charges.
On se propose de fabriquer avec le moins de tôle possible une citerne fermée en forme de parallélépipède rectangle dont le volume intérieur doit être de $14m^3$. La longueur est aussi fixée à $2m$ par le cahier des charges.
On peut donc faire varier uniquement la largeur (notée $x$) et la hauteur (notée $h$) de la cuve.
\end{minipage}
@ -28,25 +28,25 @@ Les valeurs des exercices sont générés automatiquement. Si une valeur a un no
\pgfmathsetmacro{\cubey}{1}
\pgfmathsetmacro{\cubez}{2}
\draw[black,fill=gray] (0,0,0) -- ++(-\cubex,0,0) -- ++(0,-\cubey,0) node [midway, left] {$h$} -- ++(\cubex,0,0) node [midway, below] {$x$} -- cycle;
\draw[black,fill=gray] (0,0,0) -- ++(0,0,-\cubez) -- ++(0,-\cubey,0) -- ++(0,0,\cubez) node [midway, right] {$3m$} -- cycle;
\draw[black,fill=gray] (0,0,0) -- ++(0,0,-\cubez) -- ++(0,-\cubey,0) -- ++(0,0,\cubez) node [midway, right] {$2m$} -- cycle;
\draw[black,fill=gray] (0,0,0) -- ++(-\cubex,0,0) -- ++(0,0,-\cubez) -- ++(\cubex,0,0) -- cycle;
\end{tikzpicture}
\end{minipage}
\begin{enumerate}
\item Expliquer pourquoi quand la largeur $x$ change, la hauteur $h$ doit elle aussi changer pour respecter les contraintes.
\item Démontrer que l'on doit avoir $h = \dfrac{5}{x}$.
\item Démontrer que l'on doit avoir $h = \dfrac{7}{x}$.
\item On note $S(x)$ l'aire totale de la citerne (c'est à dire la somme des aires des six faces). Montrer que l'on peut écrire
\[
S(x) = 6x + 10 + \frac{30}{x}
S(x) = 4x + 14 + \frac{28}{x}
\]
\item Démontrer que
\[
S(x) = \frac{6x^2 + 10x + 30}{x}
S(x) = \frac{4x^2 + 14x + 28}{x}
\]
\item Démontrer que
\[
S'(x) = \frac{6x^2 - 30}{x^2}
S'(x) = \frac{4x^2 - 28}{x^2}
\]
\item En déduire le tableau de variation de $S(x)$ sur $\intOF{0}{10}$.
\item Déterminer les valeurs de $x$ et $h$ correspondant à une utilisation minimal de tôle.
@ -57,62 +57,62 @@ Les valeurs des exercices sont générés automatiquement. Si une valeur a un no
\begin{enumerate}
\item Le volume étant fixe si l'on fait varier $x$, $h$ doit aussi varier.
\begin{itemize}
\item Si $x = 2$ alors conserver un volume de $V=15$, $h$ doit être égale à $5 / 2$
\item Si $x = 3$ alors conserver un volume de $V=15$, $h$ doit être égale à $5 / 3$
\item Si $x = 2$ alors conserver un volume de $V=14$, $h$ doit être égale à $7 / 2$
\item Si $x = 3$ alors conserver un volume de $V=14$, $h$ doit être égale à $7 / 3$
\end{itemize}
\item Pour calculer le volume, on a
\begin{eqnarray*}
V &=& h\times x \times 3 \\
15 &=& h\times x \times 3 \\
x &=& \frac{15}{h\times 3} = \frac{5}{h}
V &=& h\times x \times 2 \\
14 &=& h\times x \times 2 \\
x &=& \frac{14}{h\times 2} = \frac{7}{h}
\end{eqnarray*}
\item Pour calculer la surface totale, on ajoute la surface de chaque face. On a donc le calcul suivant
\begin{eqnarray*}
S(x) &=& x\times h \times 2 + x\times3\times2 + h\times 3\times 2\\
S(x) &=& x\times \frac{5}{x} \times 2 + x\times3\times2 + \frac{5}{x}\times 3\times 2\\
S(x) &=& 6x + 10 + \frac{30}{x}
S(x) &=& x\times h \times 2 + x\times2\times2 + h\times 2\times 2\\
S(x) &=& x\times \frac{7}{x} \times 2 + x\times2\times2 + \frac{7}{x}\times 2\times 2\\
S(x) &=& 4x + 14 + \frac{28}{x}
\end{eqnarray*}
\item Pour trouver cette nouvelle forme, on met chaque élément sur le même dénominateur
\begin{eqnarray*}
S(x) &=& 6x + 10 + \frac{30}{x}\\
S(x) &=& \frac{6x\times x}{x} + \frac{10\times x}{x} + \frac{30}{x}\\
S(x) &=& \frac{6x^2 + 10x + 30}{x}
S(x) &=& 4x + 14 + \frac{28}{x}\\
S(x) &=& \frac{4x\times x}{x} + \frac{14\times x}{x} + \frac{28}{x}\\
S(x) &=& \frac{4x^2 + 14x + 28}{x}
\end{eqnarray*}
\item On retrouve la formule $\frac{u}{v}$ à dériver
\[
u(x) = 6x^2 + 10x + 30 \Rightarrow u'(x) = 12x + 10
u(x) = 4x^2 + 14x + 28 \Rightarrow u'(x) = 8x + 14
\]
\[
v(x) = x \Rightarrow v'(x) = 1
\]
Donc au numérateur on obtient
\begin{eqnarray*}
u'(x)\times v(x) - u(x)\times v'(x) &=& (12x + 10)\times x - (6x^2 + 10x + 30)\times 1\\
&=& 6x^2 - 30
u'(x)\times v(x) - u(x)\times v'(x) &=& (8x + 14)\times x - (4x^2 + 14x + 28)\times 1\\
&=& 4x^2 - 28
\end{eqnarray*}
Donc
\[
S'(x) = \frac{6x^2 - 30}{x^2}
S'(x) = \frac{4x^2 - 28}{x^2}
\]
\item Tableau de variations de $S$
\begin{itemize}
\item Valeur interdite: $x^2 = 0 \equiv x = 0$
\item Signe de $6x^2 - 30$: c'est un polynôme du 2e degré
\item Signe de $4x^2 - 28$: c'est un polynôme du 2e degré
\[
\Delta = 720 > 0
\Delta = 448 > 0
\]
Il y a donc 2 racines
\[
x_1 = - 2.23606797749979 \qquad
x_2 = 2.23606797749979
x_1 = - 2.6457513110645907 \qquad
x_2 = 2.6457513110645907
\]
Et on sait que $6x^2 - 30$ est du signe de $a$ donc positif en dehors des racines
Et on sait que $4x^2 - 28$ est du signe de $a$ donc positif en dehors des racines
\item Le dénominateur $x^2$ est toujours positif.
\item Tableau de variations
\begin{tikzpicture}[baseline=(a.north)]
\tkzTabInit[lgt=3,espcl=3]{$x$/1,$6x^2 - 30$/1, $x^2$/1, $S'$/1, $S$/2}{$0$, $- 2.23606797749979$, $10$}
\tkzTabInit[lgt=3,espcl=3]{$x$/1,$4x^2 - 28$/1, $x^2$/1, $S'$/1, $S$/2}{$0$, $- 2.6457513110645907$, $10$}
\tkzTabLine{d,-, z, +, }
\tkzTabLine{d,+, , +, }
\tkzTabLine{d,-, z, +, }
@ -120,7 +120,7 @@ Les valeurs des exercices sont générés automatiquement. Si une valeur a un no
\end{tikzpicture}
\end{itemize}
\item On a donc une surface minimal pour $x=2.23606797749979$ et $h = 11.18033988749895$.
\item On a donc une surface minimal pour $x=2.6457513110645907$ et $h = 18.5202591774521349$.
\end{enumerate}
\end{solution}
@ -133,14 +133,14 @@ Les valeurs des exercices sont générés automatiquement. Si une valeur a un no
Le tour d'un bassin au niveau du sol présente deux axes de symétrie : laxe des abscisses et la droite déquation $x=4$. Il est obtenu par symétrie de la courbe $\mathcal{C}_f$ sur $\intFF{0}{4}$$f$ est la fonction définie par
\[
f(x) = \left(- x^{2} + 8.4 x - 0.6\right) e^{- x} + 0.6
f(x) = \left(- x^{2} + 7.7 x - 8.4\right) e^{- x} + 8.4
\]
On admet que sur $\intFF{0}{4}$ la fonction $f$ est positive.
\begin{enumerate}
\item Sur un repère, tracer l'allure de la courbe $\mathcal{C}_f$, les axes de symétries puis compléter pour dessiner la forme du bassin.
\item Montrer que la fonction $f$ admet comme primitive sur $\R$ la fonction $F$ définie par
\[
F(x) = 0.6 x + \left( x^{2} - 6.4 x - 5.8\right) e^{- x}
F(x) = 8.4 x + \left( x^{2} - 5.7 x + 2.7\right) e^{- x}
\]
\item Calculer la quantité $\ds \int_0^4 f(x) \; dx$, vous donnerez le résultat sous forme exacte. Interpréter le résultat et reportez cette quantité sur le graphique.
\item On considère que l'échelle de votre graphique est de 1unité pour 15m. Calculer l'aire du bassin. Vous donnerez un résultat arrondi au $m^2$ près.
@ -156,15 +156,14 @@ Les valeurs des exercices sont générés automatiquement. Si une valeur a un no
\tkzGrid
\tkzAxeXY
\tkzFct[domain=0:10,color=red,very thick]%
{ (-x**2 + 8.4*x - 0.6)*exp(-x) + 0.6 };
{ (-x**2 + 7.7*x - 8.4)*exp(-x) + 8.4 };
\end{tikzpicture}
\item Il faut dériver $F(x)$ et vérifier que $F'(x) = f(x)$.
\item $\ds \int_0^4 f(x) \; dx = F(4) - F(0) = 8.2 - \frac{15.4}{e^{4}}$
\item $\ds \int_0^4 f(x) \; dx = F(4) - F(0) = 30.9 - \frac{4.1}{e^{4}}$
\item La quantité calculée à la question précédente se retrouve 4fois pour former le bassin. Il faut ensuite prendre en compte l'échelle, comme 1unité de longueur correspond à 15m, une unité d'air correspond à $15\times15 = 225m^2$. Ainsi l'aire du bassin est égale à
\[
(8.2 - \frac{15.4}{e^{4}})\times 4 \times 15^2 = 7126.000000
(30.9 - \frac{4.1}{e^{4}})\times 4 \times 15^2 = 27742.00000
\]
\end{enumerate}
\end{solution}
@ -185,9 +184,9 @@ Les valeurs des exercices sont générés automatiquement. Si une valeur a un no
Deux ateliers A et B fabriquent des stylos pour une entreprise.
L'atelier A fabrique 25.0\,\% des stylos, et parmi ceux-là, 84.0\,\% possèdent un défaut de fabrication.
L'atelier A fabrique 14.000000000000002\,\% des stylos, et parmi ceux-là, 51.0\,\% possèdent un défaut de fabrication.
De plus, 57.99999999999999\,\% des stylos possèdent un défaut de fabrication et sortent de l'atelier B.
De plus, 61.0\,\% des stylos possèdent un défaut de fabrication et sortent de l'atelier B.
Un stylo est prélevé au hasard dans le stock de l'entreprise.
@ -241,7 +240,7 @@ Les valeurs des exercices sont générés automatiquement. Si une valeur a un no
\item
\begin{enumerate}
\item Calculer la probabilité qu'un stylo provienne de l'atelier A et possède un défaut de fabrication.
\item En déduire que la probabilité qu'un stylo possède un défaut de fabrication est de $0.79$.
\item En déduire que la probabilité qu'un stylo possède un défaut de fabrication est de $0.68$.
\end{enumerate}
\item On prélève un stylo au hasard avec un défaut. Quelle est la probabilité qu'il vienne de l'atelier A?
\end{enumerate}
@ -251,7 +250,7 @@ Les valeurs des exercices sont générés automatiquement. Si une valeur a un no
\textbf{Partie B}
\medskip
Dans cette partie, on suppose que 79.0\,\% des stylos possèdent un défaut de fabrication.
Dans cette partie, on suppose que 68.0\,\% des stylos possèdent un défaut de fabrication.
L'entreprise confectionne des paquets contenant chacun $4$~stylos.
@ -266,7 +265,7 @@ Les valeurs des exercices sont générés automatiquement. Si une valeur a un no
\begin{enumerate}
\setcounter{enumi}{4}
\item Avec quelle loi peut-on modéliser $X$. Préciser les paramètres.
\item Calculer et interpréter la probabilité $P(X = 11)$.
\item Calculer et interpréter la probabilité $P(X = 10)$.
\item Le directeur de l'entreprise affirme qu'il y a plus d'une chance sur deux qu'un paquet ne comporte aucun stylo défectueux. A-t-il raison ?
\item Combien de stylos peut-on espérer avoir en moyenne?
\end{enumerate}
@ -282,27 +281,27 @@ Les valeurs des exercices sont générés automatiquement. Si une valeur a un no
child {node {$A$}
child {node {$D$}
edge from parent
node[above] {0.84}
node[above] {0.51}
}
child {node {$\overline{D}$}
edge from parent
node[above] {0.16}
node[above] {0.49}
}
edge from parent
node[above] {0.25}
node[above] {0.14}
}
child[missing] {}
child { node {$B$}
child {node {$D$}
edge from parent
node[above] {0.78}
node[above] {0.71}
}
child {node {$\overline{D}$}
edge from parent
node[above] {0.22}
node[above] {0.29}
}
edge from parent
node[above] {0.75}
node[above] {0.86}
} ;
\end{tikzpicture}
\end{center}
@ -310,51 +309,51 @@ Les valeurs des exercices sont générés automatiquement. Si une valeur a un no
\begin{itemize}
\item Probabilité que le stylo vienne de l'atelier A
\[
P(A) = 0.25
P(A) = 0.14
\]
\item Probabilité que le stylo vienne de l'atelier B
\[
P(B) = 0.75
P(B) = 0.86
\]
\item Probabilité que le stylo ait un défaut sachant qu'il vient de l'atelier A.
\[
P_A(D) = 0.84
P_A(D) = 0.51
\]
\item Probabilité que le stylo vienne de l'atelier B et qu'il ait un défaut.
\[
P(D \cap D) = 0.58
P(D \cap D) = 0.61
\]
\end{itemize}
\item
\begin{enumerate}
\item Probabilité qu'un stylo vienne de l'atelier A et qu'il ait un defaut
\[
P(A\cap D) = P(A) \times P_A(D) = 0.25 \times 0.84 = 0.21
P(A\cap D) = P(A) \times P_A(D) = 0.14 \times 0.51 = 0.07
\]
\item Probabilité que le stylo ai un défaut de fabrication.
\[
P(D) = P(A\cap D) + P(B\cap D) = 0.21 + 0.58 = 0.79
P(D) = P(A\cap D) + P(B\cap D) = 0.07 + 0.61 = 0.68
\]
\end{enumerate}
\item Probabilité qu'il vienne de l'atelier A sachant qu'il a un defaut
\[
P_D(A) = \frac{P(A\cap D)}{P(D)} = \frac{0.21}{0.79} = 0.27
P_D(A) = \frac{P(A\cap D)}{P(D)} = \frac{0.07}{0.68} = 0.1
\]
\item $X$ peut être modélisée par une loi binomiale de paramètres $n=12$ et $p=0.79$.
\item $X$ peut être modélisée par une loi binomiale de paramètres $n=16$ et $p=0.68$.
\item (\textit{par de correction automatique disponible pour le résultat final}
\[
P(X = 11) = \coefBino{12}{11}\times 0.79^{11} \times 0.21^{1}
P(X = 10) = \coefBino{16}{10}\times 0.68^{10} \times 0.32^{6}
\]
\item (\textit{par de correction automatique disponible pour le résultat final}
Il faut calculer la probabilité qu'il y ait 0 stylo avec un defaut.
\[
P(X = 0) = \coefBino{12}{0}\times 0.79^{0} \times 0.21^{12}
P(X = 0) = \coefBino{16}{0}\times 0.68^{0} \times 0.32^{16}
\]
Puis comparer ce nombre à 0,5.
\item Il faut calculer l'espérance
\[
E[X] = n\times p = 12 \times 0.79 = 9.48
E[X] = n\times p = 16 \times 0.68 = 10.88
\]
\end{enumerate}
\end{solution}

View File

@ -3,7 +3,7 @@
% Title Page
\title{DM1 \hfill CALES Mathis}
\tribe{Maths complémentaire}
\tribe{Maths complémentaires}
\date{\hfillÀ render pour le jeudi 27 mai}
\xsimsetup{
@ -17,7 +17,7 @@ Les valeurs des exercices sont générés automatiquement. Si une valeur a un no
\begin{exercise}[subtitle={Optimisation de matière}]
\begin{minipage}{0.6\textwidth}
On se propose de fabriquer avec le moins de tôle possible une citerne fermée en forme de parallélépipède rectangle dont le volume intérieur doit être de $6m^3$. La longueur est aussi fixée à $2m$ par le cahier des charges.
On se propose de fabriquer avec le moins de tôle possible une citerne fermée en forme de parallélépipède rectangle dont le volume intérieur doit être de $8m^3$. La longueur est aussi fixée à $2m$ par le cahier des charges.
On peut donc faire varier uniquement la largeur (notée $x$) et la hauteur (notée $h$) de la cuve.
\end{minipage}
@ -35,18 +35,18 @@ Les valeurs des exercices sont générés automatiquement. Si une valeur a un no
\begin{enumerate}
\item Expliquer pourquoi quand la largeur $x$ change, la hauteur $h$ doit elle aussi changer pour respecter les contraintes.
\item Démontrer que l'on doit avoir $h = \dfrac{3}{x}$.
\item Démontrer que l'on doit avoir $h = \dfrac{4}{x}$.
\item On note $S(x)$ l'aire totale de la citerne (c'est à dire la somme des aires des six faces). Montrer que l'on peut écrire
\[
S(x) = 4x + 6 + \frac{12}{x}
S(x) = 4x + 8 + \frac{16}{x}
\]
\item Démontrer que
\[
S(x) = \frac{4x^2 + 6x + 12}{x}
S(x) = \frac{4x^2 + 8x + 16}{x}
\]
\item Démontrer que
\[
S'(x) = \frac{4x^2 - 12}{x^2}
S'(x) = \frac{4x^2 - 16}{x^2}
\]
\item En déduire le tableau de variation de $S(x)$ sur $\intOF{0}{10}$.
\item Déterminer les valeurs de $x$ et $h$ correspondant à une utilisation minimal de tôle.
@ -57,62 +57,62 @@ Les valeurs des exercices sont générés automatiquement. Si une valeur a un no
\begin{enumerate}
\item Le volume étant fixe si l'on fait varier $x$, $h$ doit aussi varier.
\begin{itemize}
\item Si $x = 2$ alors conserver un volume de $V=6$, $h$ doit être égale à $3 / 2$
\item Si $x = 3$ alors conserver un volume de $V=6$, $h$ doit être égale à $3 / 3$
\item Si $x = 2$ alors conserver un volume de $V=8$, $h$ doit être égale à $4 / 2$
\item Si $x = 3$ alors conserver un volume de $V=8$, $h$ doit être égale à $4 / 3$
\end{itemize}
\item Pour calculer le volume, on a
\begin{eqnarray*}
V &=& h\times x \times 2 \\
6 &=& h\times x \times 2 \\
x &=& \frac{6}{h\times 2} = \frac{3}{h}
8 &=& h\times x \times 2 \\
x &=& \frac{8}{h\times 2} = \frac{4}{h}
\end{eqnarray*}
\item Pour calculer la surface totale, on ajoute la surface de chaque face. On a donc le calcul suivant
\begin{eqnarray*}
S(x) &=& x\times h \times 2 + x\times2\times2 + h\times 2\times 2\\
S(x) &=& x\times \frac{3}{x} \times 2 + x\times2\times2 + \frac{3}{x}\times 2\times 2\\
S(x) &=& 4x + 6 + \frac{12}{x}
S(x) &=& x\times \frac{4}{x} \times 2 + x\times2\times2 + \frac{4}{x}\times 2\times 2\\
S(x) &=& 4x + 8 + \frac{16}{x}
\end{eqnarray*}
\item Pour trouver cette nouvelle forme, on met chaque élément sur le même dénominateur
\begin{eqnarray*}
S(x) &=& 4x + 6 + \frac{12}{x}\\
S(x) &=& \frac{4x\times x}{x} + \frac{6\times x}{x} + \frac{12}{x}\\
S(x) &=& \frac{4x^2 + 6x + 12}{x}
S(x) &=& 4x + 8 + \frac{16}{x}\\
S(x) &=& \frac{4x\times x}{x} + \frac{8\times x}{x} + \frac{16}{x}\\
S(x) &=& \frac{4x^2 + 8x + 16}{x}
\end{eqnarray*}
\item On retrouve la formule $\frac{u}{v}$ à dériver
\[
u(x) = 4x^2 + 6x + 12 \Rightarrow u'(x) = 8x + 6
u(x) = 4x^2 + 8x + 16 \Rightarrow u'(x) = 8x + 8
\]
\[
v(x) = x \Rightarrow v'(x) = 1
\]
Donc au numérateur on obtient
\begin{eqnarray*}
u'(x)\times v(x) - u(x)\times v'(x) &=& (8x + 6)\times x - (4x^2 + 6x + 12)\times 1\\
&=& 4x^2 - 12
u'(x)\times v(x) - u(x)\times v'(x) &=& (8x + 8)\times x - (4x^2 + 8x + 16)\times 1\\
&=& 4x^2 - 16
\end{eqnarray*}
Donc
\[
S'(x) = \frac{4x^2 - 12}{x^2}
S'(x) = \frac{4x^2 - 16}{x^2}
\]
\item Tableau de variations de $S$
\begin{itemize}
\item Valeur interdite: $x^2 = 0 \equiv x = 0$
\item Signe de $4x^2 - 12$: c'est un polynôme du 2e degré
\item Signe de $4x^2 - 16$: c'est un polynôme du 2e degré
\[
\Delta = 192 > 0
\Delta = 256 > 0
\]
Il y a donc 2 racines
\[
x_1 = - 1.7320508075688772 \qquad
x_2 = 1.7320508075688772
x_1 = - 2 \qquad
x_2 = 2
\]
Et on sait que $4x^2 - 12$ est du signe de $a$ donc positif en dehors des racines
Et on sait que $4x^2 - 16$ est du signe de $a$ donc positif en dehors des racines
\item Le dénominateur $x^2$ est toujours positif.
\item Tableau de variations
\begin{tikzpicture}[baseline=(a.north)]
\tkzTabInit[lgt=3,espcl=3]{$x$/1,$4x^2 - 12$/1, $x^2$/1, $S'$/1, $S$/2}{$0$, $- 1.7320508075688772$, $10$}
\tkzTabInit[lgt=3,espcl=3]{$x$/1,$4x^2 - 16$/1, $x^2$/1, $S'$/1, $S$/2}{$0$, $- 2$, $10$}
\tkzTabLine{d,-, z, +, }
\tkzTabLine{d,+, , +, }
\tkzTabLine{d,-, z, +, }
@ -120,7 +120,7 @@ Les valeurs des exercices sont générés automatiquement. Si une valeur a un no
\end{tikzpicture}
\end{itemize}
\item On a donc une surface minimal pour $x=1.7320508075688772$ et $h = 5.1961524227066316$.
\item On a donc une surface minimal pour $x=2$ et $h = 8$.
\end{enumerate}
\end{solution}
@ -133,14 +133,14 @@ Les valeurs des exercices sont générés automatiquement. Si une valeur a un no
Le tour d'un bassin au niveau du sol présente deux axes de symétrie : laxe des abscisses et la droite déquation $x=4$. Il est obtenu par symétrie de la courbe $\mathcal{C}_f$ sur $\intFF{0}{4}$$f$ est la fonction définie par
\[
f(x) = \left(- x^{2} + 2.8 x - 8.7\right) e^{- x} + 8.7
f(x) = \left(- x^{2} + 9.4 x - 4.9\right) e^{- x} + 4.9
\]
On admet que sur $\intFF{0}{4}$ la fonction $f$ est positive.
\begin{enumerate}
\item Sur un repère, tracer l'allure de la courbe $\mathcal{C}_f$, les axes de symétries puis compléter pour dessiner la forme du bassin.
\item Montrer que la fonction $f$ admet comme primitive sur $\R$ la fonction $F$ définie par
\[
F(x) = 8.7 x + \left( x^{2} - 0.8 x + 7.9\right) e^{- x}
F(x) = 4.9 x + \left( x^{2} - 7.4 x - 2.5\right) e^{- x}
\]
\item Calculer la quantité $\ds \int_0^4 f(x) \; dx$, vous donnerez le résultat sous forme exacte. Interpréter le résultat et reportez cette quantité sur le graphique.
\item On considère que l'échelle de votre graphique est de 1unité pour 15m. Calculer l'aire du bassin. Vous donnerez un résultat arrondi au $m^2$ près.
@ -156,15 +156,14 @@ Les valeurs des exercices sont générés automatiquement. Si une valeur a un no
\tkzGrid
\tkzAxeXY
\tkzFct[domain=0:10,color=red,very thick]%
{ (-x**2 + 2.8*x - 8.7)*exp(-x) + 8.7 };
{ (-x**2 + 9.4*x - 4.9)*exp(-x) + 4.9 };
\end{tikzpicture}
\item Il faut dériver $F(x)$ et vérifier que $F'(x) = f(x)$.
\item $\ds \int_0^4 f(x) \; dx = F(4) - F(0) = \frac{20.7}{e^{4}} + 26.9$
\item $\ds \int_0^4 f(x) \; dx = F(4) - F(0) = 22.1 - \frac{16.1}{e^{4}}$
\item La quantité calculée à la question précédente se retrouve 4fois pour former le bassin. Il faut ensuite prendre en compte l'échelle, comme 1unité de longueur correspond à 15m, une unité d'air correspond à $15\times15 = 225m^2$. Ainsi l'aire du bassin est égale à
\[
(\frac{20.7}{e^{4}} + 26.9)\times 4 \times 15^2 = 24551.00000
(22.1 - \frac{16.1}{e^{4}})\times 4 \times 15^2 = 19625.00000
\]
\end{enumerate}
\end{solution}
@ -185,9 +184,9 @@ Les valeurs des exercices sont générés automatiquement. Si une valeur a un no
Deux ateliers A et B fabriquent des stylos pour une entreprise.
L'atelier A fabrique 48.0\,\% des stylos, et parmi ceux-là, 99.0\,\% possèdent un défaut de fabrication.
L'atelier A fabrique 37.0\,\% des stylos, et parmi ceux-là, 24.0\,\% possèdent un défaut de fabrication.
De plus, 10.0\,\% des stylos possèdent un défaut de fabrication et sortent de l'atelier B.
De plus, 22.0\,\% des stylos possèdent un défaut de fabrication et sortent de l'atelier B.
Un stylo est prélevé au hasard dans le stock de l'entreprise.
@ -241,7 +240,7 @@ Les valeurs des exercices sont générés automatiquement. Si une valeur a un no
\item
\begin{enumerate}
\item Calculer la probabilité qu'un stylo provienne de l'atelier A et possède un défaut de fabrication.
\item En déduire que la probabilité qu'un stylo possède un défaut de fabrication est de $0.58$.
\item En déduire que la probabilité qu'un stylo possède un défaut de fabrication est de $0.31$.
\end{enumerate}
\item On prélève un stylo au hasard avec un défaut. Quelle est la probabilité qu'il vienne de l'atelier A?
\end{enumerate}
@ -251,7 +250,7 @@ Les valeurs des exercices sont générés automatiquement. Si une valeur a un no
\textbf{Partie B}
\medskip
Dans cette partie, on suppose que 57.99999999999999\,\% des stylos possèdent un défaut de fabrication.
Dans cette partie, on suppose que 31.0\,\% des stylos possèdent un défaut de fabrication.
L'entreprise confectionne des paquets contenant chacun $4$~stylos.
@ -266,7 +265,7 @@ Les valeurs des exercices sont générés automatiquement. Si une valeur a un no
\begin{enumerate}
\setcounter{enumi}{4}
\item Avec quelle loi peut-on modéliser $X$. Préciser les paramètres.
\item Calculer et interpréter la probabilité $P(X = 10)$.
\item Calculer et interpréter la probabilité $P(X = 12)$.
\item Le directeur de l'entreprise affirme qu'il y a plus d'une chance sur deux qu'un paquet ne comporte aucun stylo défectueux. A-t-il raison ?
\item Combien de stylos peut-on espérer avoir en moyenne?
\end{enumerate}
@ -282,27 +281,27 @@ Les valeurs des exercices sont générés automatiquement. Si une valeur a un no
child {node {$A$}
child {node {$D$}
edge from parent
node[above] {0.99}
node[above] {0.24}
}
child {node {$\overline{D}$}
edge from parent
node[above] {0.01}
node[above] {0.76}
}
edge from parent
node[above] {0.48}
node[above] {0.37}
}
child[missing] {}
child { node {$B$}
child {node {$D$}
edge from parent
node[above] {0.2}
node[above] {0.35}
}
child {node {$\overline{D}$}
edge from parent
node[above] {0.8}
node[above] {0.65}
}
edge from parent
node[above] {0.52}
node[above] {0.63}
} ;
\end{tikzpicture}
\end{center}
@ -310,51 +309,51 @@ Les valeurs des exercices sont générés automatiquement. Si une valeur a un no
\begin{itemize}
\item Probabilité que le stylo vienne de l'atelier A
\[
P(A) = 0.48
P(A) = 0.37
\]
\item Probabilité que le stylo vienne de l'atelier B
\[
P(B) = 0.52
P(B) = 0.63
\]
\item Probabilité que le stylo ait un défaut sachant qu'il vient de l'atelier A.
\[
P_A(D) = 0.99
P_A(D) = 0.24
\]
\item Probabilité que le stylo vienne de l'atelier B et qu'il ait un défaut.
\[
P(D \cap D) = 0.1
P(D \cap D) = 0.22
\]
\end{itemize}
\item
\begin{enumerate}
\item Probabilité qu'un stylo vienne de l'atelier A et qu'il ait un defaut
\[
P(A\cap D) = P(A) \times P_A(D) = 0.48 \times 0.99 = 0.48
P(A\cap D) = P(A) \times P_A(D) = 0.37 \times 0.24 = 0.09
\]
\item Probabilité que le stylo ai un défaut de fabrication.
\[
P(D) = P(A\cap D) + P(B\cap D) = 0.48 + 0.1 = 0.58
P(D) = P(A\cap D) + P(B\cap D) = 0.09 + 0.22 = 0.31
\]
\end{enumerate}
\item Probabilité qu'il vienne de l'atelier A sachant qu'il a un defaut
\[
P_D(A) = \frac{P(A\cap D)}{P(D)} = \frac{0.48}{0.58} = 0.83
P_D(A) = \frac{P(A\cap D)}{P(D)} = \frac{0.09}{0.31} = 0.29
\]
\item $X$ peut être modélisée par une loi binomiale de paramètres $n=17$ et $p=0.58$.
\item $X$ peut être modélisée par une loi binomiale de paramètres $n=18$ et $p=0.31$.
\item (\textit{par de correction automatique disponible pour le résultat final}
\[
P(X = 10) = \coefBino{17}{10}\times 0.58^{10} \times 0.42^{7}
P(X = 12) = \coefBino{18}{12}\times 0.31^{12} \times 0.69^{6}
\]
\item (\textit{par de correction automatique disponible pour le résultat final}
Il faut calculer la probabilité qu'il y ait 0 stylo avec un defaut.
\[
P(X = 0) = \coefBino{17}{0}\times 0.58^{0} \times 0.42^{17}
P(X = 0) = \coefBino{18}{0}\times 0.31^{0} \times 0.69^{18}
\]
Puis comparer ce nombre à 0,5.
\item Il faut calculer l'espérance
\[
E[X] = n\times p = 17 \times 0.58 = 9.86
E[X] = n\times p = 18 \times 0.31 = 5.58
\]
\end{enumerate}
\end{solution}

View File

@ -3,7 +3,7 @@
% Title Page
\title{DM1 \hfill CHAKIR Iman}
\tribe{Maths complémentaire}
\tribe{Maths complémentaires}
\date{\hfillÀ render pour le jeudi 27 mai}
\xsimsetup{
@ -17,7 +17,7 @@ Les valeurs des exercices sont générés automatiquement. Si une valeur a un no
\begin{exercise}[subtitle={Optimisation de matière}]
\begin{minipage}{0.6\textwidth}
On se propose de fabriquer avec le moins de tôle possible une citerne fermée en forme de parallélépipède rectangle dont le volume intérieur doit être de $20m^3$. La longueur est aussi fixée à $4m$ par le cahier des charges.
On se propose de fabriquer avec le moins de tôle possible une citerne fermée en forme de parallélépipède rectangle dont le volume intérieur doit être de $16m^3$. La longueur est aussi fixée à $2m$ par le cahier des charges.
On peut donc faire varier uniquement la largeur (notée $x$) et la hauteur (notée $h$) de la cuve.
\end{minipage}
@ -28,25 +28,25 @@ Les valeurs des exercices sont générés automatiquement. Si une valeur a un no
\pgfmathsetmacro{\cubey}{1}
\pgfmathsetmacro{\cubez}{2}
\draw[black,fill=gray] (0,0,0) -- ++(-\cubex,0,0) -- ++(0,-\cubey,0) node [midway, left] {$h$} -- ++(\cubex,0,0) node [midway, below] {$x$} -- cycle;
\draw[black,fill=gray] (0,0,0) -- ++(0,0,-\cubez) -- ++(0,-\cubey,0) -- ++(0,0,\cubez) node [midway, right] {$4m$} -- cycle;
\draw[black,fill=gray] (0,0,0) -- ++(0,0,-\cubez) -- ++(0,-\cubey,0) -- ++(0,0,\cubez) node [midway, right] {$2m$} -- cycle;
\draw[black,fill=gray] (0,0,0) -- ++(-\cubex,0,0) -- ++(0,0,-\cubez) -- ++(\cubex,0,0) -- cycle;
\end{tikzpicture}
\end{minipage}
\begin{enumerate}
\item Expliquer pourquoi quand la largeur $x$ change, la hauteur $h$ doit elle aussi changer pour respecter les contraintes.
\item Démontrer que l'on doit avoir $h = \dfrac{5}{x}$.
\item Démontrer que l'on doit avoir $h = \dfrac{8}{x}$.
\item On note $S(x)$ l'aire totale de la citerne (c'est à dire la somme des aires des six faces). Montrer que l'on peut écrire
\[
S(x) = 8x + 10 + \frac{40}{x}
S(x) = 4x + 16 + \frac{32}{x}
\]
\item Démontrer que
\[
S(x) = \frac{8x^2 + 10x + 40}{x}
S(x) = \frac{4x^2 + 16x + 32}{x}
\]
\item Démontrer que
\[
S'(x) = \frac{8x^2 - 40}{x^2}
S'(x) = \frac{4x^2 - 32}{x^2}
\]
\item En déduire le tableau de variation de $S(x)$ sur $\intOF{0}{10}$.
\item Déterminer les valeurs de $x$ et $h$ correspondant à une utilisation minimal de tôle.
@ -57,62 +57,62 @@ Les valeurs des exercices sont générés automatiquement. Si une valeur a un no
\begin{enumerate}
\item Le volume étant fixe si l'on fait varier $x$, $h$ doit aussi varier.
\begin{itemize}
\item Si $x = 2$ alors conserver un volume de $V=20$, $h$ doit être égale à $5 / 2$
\item Si $x = 3$ alors conserver un volume de $V=20$, $h$ doit être égale à $5 / 3$
\item Si $x = 2$ alors conserver un volume de $V=16$, $h$ doit être égale à $8 / 2$
\item Si $x = 3$ alors conserver un volume de $V=16$, $h$ doit être égale à $8 / 3$
\end{itemize}
\item Pour calculer le volume, on a
\begin{eqnarray*}
V &=& h\times x \times 4 \\
20 &=& h\times x \times 4 \\
x &=& \frac{20}{h\times 4} = \frac{5}{h}
V &=& h\times x \times 2 \\
16 &=& h\times x \times 2 \\
x &=& \frac{16}{h\times 2} = \frac{8}{h}
\end{eqnarray*}
\item Pour calculer la surface totale, on ajoute la surface de chaque face. On a donc le calcul suivant
\begin{eqnarray*}
S(x) &=& x\times h \times 2 + x\times4\times2 + h\times 4\times 2\\
S(x) &=& x\times \frac{5}{x} \times 2 + x\times4\times2 + \frac{5}{x}\times 4\times 2\\
S(x) &=& 8x + 10 + \frac{40}{x}
S(x) &=& x\times h \times 2 + x\times2\times2 + h\times 2\times 2\\
S(x) &=& x\times \frac{8}{x} \times 2 + x\times2\times2 + \frac{8}{x}\times 2\times 2\\
S(x) &=& 4x + 16 + \frac{32}{x}
\end{eqnarray*}
\item Pour trouver cette nouvelle forme, on met chaque élément sur le même dénominateur
\begin{eqnarray*}
S(x) &=& 8x + 10 + \frac{40}{x}\\
S(x) &=& \frac{8x\times x}{x} + \frac{10\times x}{x} + \frac{40}{x}\\
S(x) &=& \frac{8x^2 + 10x + 40}{x}
S(x) &=& 4x + 16 + \frac{32}{x}\\
S(x) &=& \frac{4x\times x}{x} + \frac{16\times x}{x} + \frac{32}{x}\\
S(x) &=& \frac{4x^2 + 16x + 32}{x}
\end{eqnarray*}
\item On retrouve la formule $\frac{u}{v}$ à dériver
\[
u(x) = 8x^2 + 10x + 40 \Rightarrow u'(x) = 16x + 10
u(x) = 4x^2 + 16x + 32 \Rightarrow u'(x) = 8x + 16
\]
\[
v(x) = x \Rightarrow v'(x) = 1
\]
Donc au numérateur on obtient
\begin{eqnarray*}
u'(x)\times v(x) - u(x)\times v'(x) &=& (16x + 10)\times x - (8x^2 + 10x + 40)\times 1\\
&=& 8x^2 - 40
u'(x)\times v(x) - u(x)\times v'(x) &=& (8x + 16)\times x - (4x^2 + 16x + 32)\times 1\\
&=& 4x^2 - 32
\end{eqnarray*}
Donc
\[
S'(x) = \frac{8x^2 - 40}{x^2}
S'(x) = \frac{4x^2 - 32}{x^2}
\]
\item Tableau de variations de $S$
\begin{itemize}
\item Valeur interdite: $x^2 = 0 \equiv x = 0$
\item Signe de $8x^2 - 40$: c'est un polynôme du 2e degré
\item Signe de $4x^2 - 32$: c'est un polynôme du 2e degré
\[
\Delta = 1280 > 0
\Delta = 512 > 0
\]
Il y a donc 2 racines
\[
x_1 = - 2.23606797749979 \qquad
x_2 = 2.23606797749979
x_1 = - 2.8284271247461903 \qquad
x_2 = 2.8284271247461903
\]
Et on sait que $8x^2 - 40$ est du signe de $a$ donc positif en dehors des racines
Et on sait que $4x^2 - 32$ est du signe de $a$ donc positif en dehors des racines
\item Le dénominateur $x^2$ est toujours positif.
\item Tableau de variations
\begin{tikzpicture}[baseline=(a.north)]
\tkzTabInit[lgt=3,espcl=3]{$x$/1,$8x^2 - 40$/1, $x^2$/1, $S'$/1, $S$/2}{$0$, $- 2.23606797749979$, $10$}
\tkzTabInit[lgt=3,espcl=3]{$x$/1,$4x^2 - 32$/1, $x^2$/1, $S'$/1, $S$/2}{$0$, $- 2.8284271247461903$, $10$}
\tkzTabLine{d,-, z, +, }
\tkzTabLine{d,+, , +, }
\tkzTabLine{d,-, z, +, }
@ -120,7 +120,7 @@ Les valeurs des exercices sont générés automatiquement. Si une valeur a un no
\end{tikzpicture}
\end{itemize}
\item On a donc une surface minimal pour $x=2.23606797749979$ et $h = 11.18033988749895$.
\item On a donc une surface minimal pour $x=2.8284271247461903$ et $h = 22.6274169979695224$.
\end{enumerate}
\end{solution}
@ -133,14 +133,14 @@ Les valeurs des exercices sont générés automatiquement. Si une valeur a un no
Le tour d'un bassin au niveau du sol présente deux axes de symétrie : laxe des abscisses et la droite déquation $x=4$. Il est obtenu par symétrie de la courbe $\mathcal{C}_f$ sur $\intFF{0}{4}$$f$ est la fonction définie par
\[
f(x) = \left(- x^{2} + 9.0 x - 9.0\right) e^{- x} + 9.0
f(x) = \left(- x^{2} + 6.1 x - 2.6\right) e^{- x} + 2.6
\]
On admet que sur $\intFF{0}{4}$ la fonction $f$ est positive.
\begin{enumerate}
\item Sur un repère, tracer l'allure de la courbe $\mathcal{C}_f$, les axes de symétries puis compléter pour dessiner la forme du bassin.
\item Montrer que la fonction $f$ admet comme primitive sur $\R$ la fonction $F$ définie par
\[
F(x) = 9.0 x + \left( x^{2} - 7.0 x + 2.0\right) e^{- x}
F(x) = 2.6 x + \left( x^{2} - 4.1 x - 1.5\right) e^{- x}
\]
\item Calculer la quantité $\ds \int_0^4 f(x) \; dx$, vous donnerez le résultat sous forme exacte. Interpréter le résultat et reportez cette quantité sur le graphique.
\item On considère que l'échelle de votre graphique est de 1unité pour 15m. Calculer l'aire du bassin. Vous donnerez un résultat arrondi au $m^2$ près.
@ -156,15 +156,14 @@ Les valeurs des exercices sont générés automatiquement. Si une valeur a un no
\tkzGrid
\tkzAxeXY
\tkzFct[domain=0:10,color=red,very thick]%
{ (-x**2 + 9.0*x - 9.0)*exp(-x) + 9.0 };
{ (-x**2 + 6.1*x - 2.6)*exp(-x) + 2.6 };
\end{tikzpicture}
\item Il faut dériver $F(x)$ et vérifier que $F'(x) = f(x)$.
\item $\ds \int_0^4 f(x) \; dx = F(4) - F(0) = 34.0 - \frac{10.0}{e^{4}}$
\item $\ds \int_0^4 f(x) \; dx = F(4) - F(0) = 11.9 - \frac{1.9}{e^{4}}$
\item La quantité calculée à la question précédente se retrouve 4fois pour former le bassin. Il faut ensuite prendre en compte l'échelle, comme 1unité de longueur correspond à 15m, une unité d'air correspond à $15\times15 = 225m^2$. Ainsi l'aire du bassin est égale à
\[
(34.0 - \frac{10.0}{e^{4}})\times 4 \times 15^2 = 30435.00000
(11.9 - \frac{1.9}{e^{4}})\times 4 \times 15^2 = 10679.00000
\]
\end{enumerate}
\end{solution}
@ -185,9 +184,9 @@ Les valeurs des exercices sont générés automatiquement. Si une valeur a un no
Deux ateliers A et B fabriquent des stylos pour une entreprise.
L'atelier A fabrique 27.0\,\% des stylos, et parmi ceux-là, 39.0\,\% possèdent un défaut de fabrication.
L'atelier A fabrique 57.99999999999999\,\% des stylos, et parmi ceux-là, 5.0\,\% possèdent un défaut de fabrication.
De plus, 4.0\,\% des stylos possèdent un défaut de fabrication et sortent de l'atelier B.
De plus, 14.000000000000002\,\% des stylos possèdent un défaut de fabrication et sortent de l'atelier B.
Un stylo est prélevé au hasard dans le stock de l'entreprise.
@ -241,7 +240,7 @@ Les valeurs des exercices sont générés automatiquement. Si une valeur a un no
\item
\begin{enumerate}
\item Calculer la probabilité qu'un stylo provienne de l'atelier A et possède un défaut de fabrication.
\item En déduire que la probabilité qu'un stylo possède un défaut de fabrication est de $0.15$.
\item En déduire que la probabilité qu'un stylo possède un défaut de fabrication est de $0.17$.
\end{enumerate}
\item On prélève un stylo au hasard avec un défaut. Quelle est la probabilité qu'il vienne de l'atelier A?
\end{enumerate}
@ -251,7 +250,7 @@ Les valeurs des exercices sont générés automatiquement. Si une valeur a un no
\textbf{Partie B}
\medskip
Dans cette partie, on suppose que 15.0\,\% des stylos possèdent un défaut de fabrication.
Dans cette partie, on suppose que 17.0\,\% des stylos possèdent un défaut de fabrication.
L'entreprise confectionne des paquets contenant chacun $4$~stylos.
@ -266,7 +265,7 @@ Les valeurs des exercices sont générés automatiquement. Si une valeur a un no
\begin{enumerate}
\setcounter{enumi}{4}
\item Avec quelle loi peut-on modéliser $X$. Préciser les paramètres.
\item Calculer et interpréter la probabilité $P(X = 9)$.
\item Calculer et interpréter la probabilité $P(X = 17)$.
\item Le directeur de l'entreprise affirme qu'il y a plus d'une chance sur deux qu'un paquet ne comporte aucun stylo défectueux. A-t-il raison ?
\item Combien de stylos peut-on espérer avoir en moyenne?
\end{enumerate}
@ -281,19 +280,6 @@ Les valeurs des exercices sont générés automatiquement. Si une valeur a un no
\node {.}
child {node {$A$}
child {node {$D$}
edge from parent
node[above] {0.39}
}
child {node {$\overline{D}$}
edge from parent
node[above] {0.61}
}
edge from parent
node[above] {0.27}
}
child[missing] {}
child { node {$B$}
child {node {$D$}
edge from parent
node[above] {0.05}
}
@ -302,7 +288,20 @@ Les valeurs des exercices sont générés automatiquement. Si une valeur a un no
node[above] {0.95}
}
edge from parent
node[above] {0.73}
node[above] {0.58}
}
child[missing] {}
child { node {$B$}
child {node {$D$}
edge from parent
node[above] {0.33}
}
child {node {$\overline{D}$}
edge from parent
node[above] {0.67}
}
edge from parent
node[above] {0.42}
} ;
\end{tikzpicture}
\end{center}
@ -310,51 +309,51 @@ Les valeurs des exercices sont générés automatiquement. Si une valeur a un no
\begin{itemize}
\item Probabilité que le stylo vienne de l'atelier A
\[
P(A) = 0.27
P(A) = 0.58
\]
\item Probabilité que le stylo vienne de l'atelier B
\[
P(B) = 0.73
P(B) = 0.42
\]
\item Probabilité que le stylo ait un défaut sachant qu'il vient de l'atelier A.
\[
P_A(D) = 0.39
P_A(D) = 0.05
\]
\item Probabilité que le stylo vienne de l'atelier B et qu'il ait un défaut.
\[
P(D \cap D) = 0.04
P(D \cap D) = 0.14
\]
\end{itemize}
\item
\begin{enumerate}
\item Probabilité qu'un stylo vienne de l'atelier A et qu'il ait un defaut
\[
P(A\cap D) = P(A) \times P_A(D) = 0.27 \times 0.39 = 0.11
P(A\cap D) = P(A) \times P_A(D) = 0.58 \times 0.05 = 0.03
\]
\item Probabilité que le stylo ai un défaut de fabrication.
\[
P(D) = P(A\cap D) + P(B\cap D) = 0.11 + 0.04 = 0.15
P(D) = P(A\cap D) + P(B\cap D) = 0.03 + 0.14 = 0.17
\]
\end{enumerate}
\item Probabilité qu'il vienne de l'atelier A sachant qu'il a un defaut
\[
P_D(A) = \frac{P(A\cap D)}{P(D)} = \frac{0.11}{0.15} = 0.73
P_D(A) = \frac{P(A\cap D)}{P(D)} = \frac{0.03}{0.17} = 0.18
\]
\item $X$ peut être modélisée par une loi binomiale de paramètres $n=17$ et $p=0.15$.
\item $X$ peut être modélisée par une loi binomiale de paramètres $n=19$ et $p=0.17$.
\item (\textit{par de correction automatique disponible pour le résultat final}
\[
P(X = 9) = \coefBino{17}{9}\times 0.15^{9} \times 0.85^{8}
P(X = 17) = \coefBino{19}{17}\times 0.17^{17} \times 0.83^{2}
\]
\item (\textit{par de correction automatique disponible pour le résultat final}
Il faut calculer la probabilité qu'il y ait 0 stylo avec un defaut.
\[
P(X = 0) = \coefBino{17}{0}\times 0.15^{0} \times 0.85^{17}
P(X = 0) = \coefBino{19}{0}\times 0.17^{0} \times 0.83^{19}
\]
Puis comparer ce nombre à 0,5.
\item Il faut calculer l'espérance
\[
E[X] = n\times p = 17 \times 0.15 = 2.55
E[X] = n\times p = 19 \times 0.17 = 3.23
\]
\end{enumerate}
\end{solution}

View File

@ -3,7 +3,7 @@
% Title Page
\title{DM1 \hfill GERMAIN Margot}
\tribe{Maths complémentaire}
\tribe{Maths complémentaires}
\date{\hfillÀ render pour le jeudi 27 mai}
\xsimsetup{
@ -17,7 +17,7 @@ Les valeurs des exercices sont générés automatiquement. Si une valeur a un no
\begin{exercise}[subtitle={Optimisation de matière}]
\begin{minipage}{0.6\textwidth}
On se propose de fabriquer avec le moins de tôle possible une citerne fermée en forme de parallélépipède rectangle dont le volume intérieur doit être de $35m^3$. La longueur est aussi fixée à $5m$ par le cahier des charges.
On se propose de fabriquer avec le moins de tôle possible une citerne fermée en forme de parallélépipède rectangle dont le volume intérieur doit être de $45m^3$. La longueur est aussi fixée à $5m$ par le cahier des charges.
On peut donc faire varier uniquement la largeur (notée $x$) et la hauteur (notée $h$) de la cuve.
\end{minipage}
@ -35,18 +35,18 @@ Les valeurs des exercices sont générés automatiquement. Si une valeur a un no
\begin{enumerate}
\item Expliquer pourquoi quand la largeur $x$ change, la hauteur $h$ doit elle aussi changer pour respecter les contraintes.
\item Démontrer que l'on doit avoir $h = \dfrac{7}{x}$.
\item Démontrer que l'on doit avoir $h = \dfrac{9}{x}$.
\item On note $S(x)$ l'aire totale de la citerne (c'est à dire la somme des aires des six faces). Montrer que l'on peut écrire
\[
S(x) = 10x + 14 + \frac{70}{x}
S(x) = 10x + 18 + \frac{90}{x}
\]
\item Démontrer que
\[
S(x) = \frac{10x^2 + 14x + 70}{x}
S(x) = \frac{10x^2 + 18x + 90}{x}
\]
\item Démontrer que
\[
S'(x) = \frac{10x^2 - 70}{x^2}
S'(x) = \frac{10x^2 - 90}{x^2}
\]
\item En déduire le tableau de variation de $S(x)$ sur $\intOF{0}{10}$.
\item Déterminer les valeurs de $x$ et $h$ correspondant à une utilisation minimal de tôle.
@ -57,62 +57,62 @@ Les valeurs des exercices sont générés automatiquement. Si une valeur a un no
\begin{enumerate}
\item Le volume étant fixe si l'on fait varier $x$, $h$ doit aussi varier.
\begin{itemize}
\item Si $x = 2$ alors conserver un volume de $V=35$, $h$ doit être égale à $7 / 2$
\item Si $x = 3$ alors conserver un volume de $V=35$, $h$ doit être égale à $7 / 3$
\item Si $x = 2$ alors conserver un volume de $V=45$, $h$ doit être égale à $9 / 2$
\item Si $x = 3$ alors conserver un volume de $V=45$, $h$ doit être égale à $9 / 3$
\end{itemize}
\item Pour calculer le volume, on a
\begin{eqnarray*}
V &=& h\times x \times 5 \\
35 &=& h\times x \times 5 \\
x &=& \frac{35}{h\times 5} = \frac{7}{h}
45 &=& h\times x \times 5 \\
x &=& \frac{45}{h\times 5} = \frac{9}{h}
\end{eqnarray*}
\item Pour calculer la surface totale, on ajoute la surface de chaque face. On a donc le calcul suivant
\begin{eqnarray*}
S(x) &=& x\times h \times 2 + x\times5\times2 + h\times 5\times 2\\
S(x) &=& x\times \frac{7}{x} \times 2 + x\times5\times2 + \frac{7}{x}\times 5\times 2\\
S(x) &=& 10x + 14 + \frac{70}{x}
S(x) &=& x\times \frac{9}{x} \times 2 + x\times5\times2 + \frac{9}{x}\times 5\times 2\\
S(x) &=& 10x + 18 + \frac{90}{x}
\end{eqnarray*}
\item Pour trouver cette nouvelle forme, on met chaque élément sur le même dénominateur
\begin{eqnarray*}
S(x) &=& 10x + 14 + \frac{70}{x}\\
S(x) &=& \frac{10x\times x}{x} + \frac{14\times x}{x} + \frac{70}{x}\\
S(x) &=& \frac{10x^2 + 14x + 70}{x}
S(x) &=& 10x + 18 + \frac{90}{x}\\
S(x) &=& \frac{10x\times x}{x} + \frac{18\times x}{x} + \frac{90}{x}\\
S(x) &=& \frac{10x^2 + 18x + 90}{x}
\end{eqnarray*}
\item On retrouve la formule $\frac{u}{v}$ à dériver
\[
u(x) = 10x^2 + 14x + 70 \Rightarrow u'(x) = 20x + 14
u(x) = 10x^2 + 18x + 90 \Rightarrow u'(x) = 20x + 18
\]
\[
v(x) = x \Rightarrow v'(x) = 1
\]
Donc au numérateur on obtient
\begin{eqnarray*}
u'(x)\times v(x) - u(x)\times v'(x) &=& (20x + 14)\times x - (10x^2 + 14x + 70)\times 1\\
&=& 10x^2 - 70
u'(x)\times v(x) - u(x)\times v'(x) &=& (20x + 18)\times x - (10x^2 + 18x + 90)\times 1\\
&=& 10x^2 - 90
\end{eqnarray*}
Donc
\[
S'(x) = \frac{10x^2 - 70}{x^2}
S'(x) = \frac{10x^2 - 90}{x^2}
\]
\item Tableau de variations de $S$
\begin{itemize}
\item Valeur interdite: $x^2 = 0 \equiv x = 0$
\item Signe de $10x^2 - 70$: c'est un polynôme du 2e degré
\item Signe de $10x^2 - 90$: c'est un polynôme du 2e degré
\[
\Delta = 2800 > 0
\Delta = 3600 > 0
\]
Il y a donc 2 racines
\[
x_1 = - 2.6457513110645907 \qquad
x_2 = 2.6457513110645907
x_1 = - 3 \qquad
x_2 = 3
\]
Et on sait que $10x^2 - 70$ est du signe de $a$ donc positif en dehors des racines
Et on sait que $10x^2 - 90$ est du signe de $a$ donc positif en dehors des racines
\item Le dénominateur $x^2$ est toujours positif.
\item Tableau de variations
\begin{tikzpicture}[baseline=(a.north)]
\tkzTabInit[lgt=3,espcl=3]{$x$/1,$10x^2 - 70$/1, $x^2$/1, $S'$/1, $S$/2}{$0$, $- 2.6457513110645907$, $10$}
\tkzTabInit[lgt=3,espcl=3]{$x$/1,$10x^2 - 90$/1, $x^2$/1, $S'$/1, $S$/2}{$0$, $- 3$, $10$}
\tkzTabLine{d,-, z, +, }
\tkzTabLine{d,+, , +, }
\tkzTabLine{d,-, z, +, }
@ -120,7 +120,7 @@ Les valeurs des exercices sont générés automatiquement. Si une valeur a un no
\end{tikzpicture}
\end{itemize}
\item On a donc une surface minimal pour $x=2.6457513110645907$ et $h = 18.5202591774521349$.
\item On a donc une surface minimal pour $x=3$ et $h = 27$.
\end{enumerate}
\end{solution}
@ -133,14 +133,14 @@ Les valeurs des exercices sont générés automatiquement. Si une valeur a un no
Le tour d'un bassin au niveau du sol présente deux axes de symétrie : laxe des abscisses et la droite déquation $x=4$. Il est obtenu par symétrie de la courbe $\mathcal{C}_f$ sur $\intFF{0}{4}$$f$ est la fonction définie par
\[
f(x) = \left(- x^{2} + 8.0 x - 1.1\right) e^{- x} + 1.1
f(x) = \left(- x^{2} + 1.2 x - 9.5\right) e^{- x} + 9.5
\]
On admet que sur $\intFF{0}{4}$ la fonction $f$ est positive.
\begin{enumerate}
\item Sur un repère, tracer l'allure de la courbe $\mathcal{C}_f$, les axes de symétries puis compléter pour dessiner la forme du bassin.
\item Montrer que la fonction $f$ admet comme primitive sur $\R$ la fonction $F$ définie par
\[
F(x) = 1.1 x + \left( x^{2} - 6.0 x - 4.9\right) e^{- x}
F(x) = 9.5 x + \left( x^{2} + 0.8 x + 10.3\right) e^{- x}
\]
\item Calculer la quantité $\ds \int_0^4 f(x) \; dx$, vous donnerez le résultat sous forme exacte. Interpréter le résultat et reportez cette quantité sur le graphique.
\item On considère que l'échelle de votre graphique est de 1unité pour 15m. Calculer l'aire du bassin. Vous donnerez un résultat arrondi au $m^2$ près.
@ -156,15 +156,14 @@ Les valeurs des exercices sont générés automatiquement. Si une valeur a un no
\tkzGrid
\tkzAxeXY
\tkzFct[domain=0:10,color=red,very thick]%
{ (-x**2 + 8.0*x - 1.1)*exp(-x) + 1.1 };
{ (-x**2 + 1.2*x - 9.5)*exp(-x) + 9.5 };
\end{tikzpicture}
\item Il faut dériver $F(x)$ et vérifier que $F'(x) = f(x)$.
\item $\ds \int_0^4 f(x) \; dx = F(4) - F(0) = 9.3 - \frac{12.9}{e^{4}}$
\item $\ds \int_0^4 f(x) \; dx = F(4) - F(0) = \frac{29.5}{e^{4}} + 27.7$
\item La quantité calculée à la question précédente se retrouve 4fois pour former le bassin. Il faut ensuite prendre en compte l'échelle, comme 1unité de longueur correspond à 15m, une unité d'air correspond à $15\times15 = 225m^2$. Ainsi l'aire du bassin est égale à
\[
(9.3 - \frac{12.9}{e^{4}})\times 4 \times 15^2 = 8157.000000
(\frac{29.5}{e^{4}} + 27.7)\times 4 \times 15^2 = 25416.00000
\]
\end{enumerate}
\end{solution}
@ -185,9 +184,9 @@ Les valeurs des exercices sont générés automatiquement. Si une valeur a un no
Deux ateliers A et B fabriquent des stylos pour une entreprise.
L'atelier A fabrique 81.0\,\% des stylos, et parmi ceux-là, 57.99999999999999\,\% possèdent un défaut de fabrication.
L'atelier A fabrique 31.0\,\% des stylos, et parmi ceux-là, 4.0\,\% possèdent un défaut de fabrication.
De plus, 2.0\,\% des stylos possèdent un défaut de fabrication et sortent de l'atelier B.
De plus, 15.0\,\% des stylos possèdent un défaut de fabrication et sortent de l'atelier B.
Un stylo est prélevé au hasard dans le stock de l'entreprise.
@ -241,7 +240,7 @@ Les valeurs des exercices sont générés automatiquement. Si une valeur a un no
\item
\begin{enumerate}
\item Calculer la probabilité qu'un stylo provienne de l'atelier A et possède un défaut de fabrication.
\item En déduire que la probabilité qu'un stylo possède un défaut de fabrication est de $0.49$.
\item En déduire que la probabilité qu'un stylo possède un défaut de fabrication est de $0.16$.
\end{enumerate}
\item On prélève un stylo au hasard avec un défaut. Quelle est la probabilité qu'il vienne de l'atelier A?
\end{enumerate}
@ -251,7 +250,7 @@ Les valeurs des exercices sont générés automatiquement. Si une valeur a un no
\textbf{Partie B}
\medskip
Dans cette partie, on suppose que 49.0\,\% des stylos possèdent un défaut de fabrication.
Dans cette partie, on suppose que 16.0\,\% des stylos possèdent un défaut de fabrication.
L'entreprise confectionne des paquets contenant chacun $4$~stylos.
@ -266,7 +265,7 @@ Les valeurs des exercices sont générés automatiquement. Si une valeur a un no
\begin{enumerate}
\setcounter{enumi}{4}
\item Avec quelle loi peut-on modéliser $X$. Préciser les paramètres.
\item Calculer et interpréter la probabilité $P(X = 14)$.
\item Calculer et interpréter la probabilité $P(X = 13)$.
\item Le directeur de l'entreprise affirme qu'il y a plus d'une chance sur deux qu'un paquet ne comporte aucun stylo défectueux. A-t-il raison ?
\item Combien de stylos peut-on espérer avoir en moyenne?
\end{enumerate}
@ -282,27 +281,27 @@ Les valeurs des exercices sont générés automatiquement. Si une valeur a un no
child {node {$A$}
child {node {$D$}
edge from parent
node[above] {0.58}
node[above] {0.04}
}
child {node {$\overline{D}$}
edge from parent
node[above] {0.42}
node[above] {0.96}
}
edge from parent
node[above] {0.81}
node[above] {0.31}
}
child[missing] {}
child { node {$B$}
child {node {$D$}
edge from parent
node[above] {0.13}
node[above] {0.22}
}
child {node {$\overline{D}$}
edge from parent
node[above] {0.87}
node[above] {0.78}
}
edge from parent
node[above] {0.19}
node[above] {0.69}
} ;
\end{tikzpicture}
\end{center}
@ -310,51 +309,51 @@ Les valeurs des exercices sont générés automatiquement. Si une valeur a un no
\begin{itemize}
\item Probabilité que le stylo vienne de l'atelier A
\[
P(A) = 0.81
P(A) = 0.31
\]
\item Probabilité que le stylo vienne de l'atelier B
\[
P(B) = 0.19
P(B) = 0.69
\]
\item Probabilité que le stylo ait un défaut sachant qu'il vient de l'atelier A.
\[
P_A(D) = 0.58
P_A(D) = 0.04
\]
\item Probabilité que le stylo vienne de l'atelier B et qu'il ait un défaut.
\[
P(D \cap D) = 0.02
P(D \cap D) = 0.15
\]
\end{itemize}
\item
\begin{enumerate}
\item Probabilité qu'un stylo vienne de l'atelier A et qu'il ait un defaut
\[
P(A\cap D) = P(A) \times P_A(D) = 0.81 \times 0.58 = 0.47
P(A\cap D) = P(A) \times P_A(D) = 0.31 \times 0.04 = 0.01
\]
\item Probabilité que le stylo ai un défaut de fabrication.
\[
P(D) = P(A\cap D) + P(B\cap D) = 0.47 + 0.02 = 0.49
P(D) = P(A\cap D) + P(B\cap D) = 0.01 + 0.15 = 0.16
\]
\end{enumerate}
\item Probabilité qu'il vienne de l'atelier A sachant qu'il a un defaut
\[
P_D(A) = \frac{P(A\cap D)}{P(D)} = \frac{0.47}{0.49} = 0.96
P_D(A) = \frac{P(A\cap D)}{P(D)} = \frac{0.01}{0.16} = 0.06
\]
\item $X$ peut être modélisée par une loi binomiale de paramètres $n=20$ et $p=0.49$.
\item $X$ peut être modélisée par une loi binomiale de paramètres $n=14$ et $p=0.16$.
\item (\textit{par de correction automatique disponible pour le résultat final}
\[
P(X = 14) = \coefBino{20}{14}\times 0.49^{14} \times 0.51^{6}
P(X = 13) = \coefBino{14}{13}\times 0.16^{13} \times 0.84^{1}
\]
\item (\textit{par de correction automatique disponible pour le résultat final}
Il faut calculer la probabilité qu'il y ait 0 stylo avec un defaut.
\[
P(X = 0) = \coefBino{20}{0}\times 0.49^{0} \times 0.51^{20}
P(X = 0) = \coefBino{14}{0}\times 0.16^{0} \times 0.84^{14}
\]
Puis comparer ce nombre à 0,5.
\item Il faut calculer l'espérance
\[
E[X] = n\times p = 20 \times 0.49 = 9.8
E[X] = n\times p = 14 \times 0.16 = 2.24
\]
\end{enumerate}
\end{solution}

View File

@ -3,7 +3,7 @@
% Title Page
\title{DM1 \hfill HOKELEKLI Damla}
\tribe{Maths complémentaire}
\tribe{Maths complémentaires}
\date{\hfillÀ render pour le jeudi 27 mai}
\xsimsetup{
@ -17,7 +17,7 @@ Les valeurs des exercices sont générés automatiquement. Si une valeur a un no
\begin{exercise}[subtitle={Optimisation de matière}]
\begin{minipage}{0.6\textwidth}
On se propose de fabriquer avec le moins de tôle possible une citerne fermée en forme de parallélépipède rectangle dont le volume intérieur doit être de $10m^3$. La longueur est aussi fixée à $2m$ par le cahier des charges.
On se propose de fabriquer avec le moins de tôle possible une citerne fermée en forme de parallélépipède rectangle dont le volume intérieur doit être de $21m^3$. La longueur est aussi fixée à $3m$ par le cahier des charges.
On peut donc faire varier uniquement la largeur (notée $x$) et la hauteur (notée $h$) de la cuve.
\end{minipage}
@ -28,25 +28,25 @@ Les valeurs des exercices sont générés automatiquement. Si une valeur a un no
\pgfmathsetmacro{\cubey}{1}
\pgfmathsetmacro{\cubez}{2}
\draw[black,fill=gray] (0,0,0) -- ++(-\cubex,0,0) -- ++(0,-\cubey,0) node [midway, left] {$h$} -- ++(\cubex,0,0) node [midway, below] {$x$} -- cycle;
\draw[black,fill=gray] (0,0,0) -- ++(0,0,-\cubez) -- ++(0,-\cubey,0) -- ++(0,0,\cubez) node [midway, right] {$2m$} -- cycle;
\draw[black,fill=gray] (0,0,0) -- ++(0,0,-\cubez) -- ++(0,-\cubey,0) -- ++(0,0,\cubez) node [midway, right] {$3m$} -- cycle;
\draw[black,fill=gray] (0,0,0) -- ++(-\cubex,0,0) -- ++(0,0,-\cubez) -- ++(\cubex,0,0) -- cycle;
\end{tikzpicture}
\end{minipage}
\begin{enumerate}
\item Expliquer pourquoi quand la largeur $x$ change, la hauteur $h$ doit elle aussi changer pour respecter les contraintes.
\item Démontrer que l'on doit avoir $h = \dfrac{5}{x}$.
\item Démontrer que l'on doit avoir $h = \dfrac{7}{x}$.
\item On note $S(x)$ l'aire totale de la citerne (c'est à dire la somme des aires des six faces). Montrer que l'on peut écrire
\[
S(x) = 4x + 10 + \frac{20}{x}
S(x) = 6x + 14 + \frac{42}{x}
\]
\item Démontrer que
\[
S(x) = \frac{4x^2 + 10x + 20}{x}
S(x) = \frac{6x^2 + 14x + 42}{x}
\]
\item Démontrer que
\[
S'(x) = \frac{4x^2 - 20}{x^2}
S'(x) = \frac{6x^2 - 42}{x^2}
\]
\item En déduire le tableau de variation de $S(x)$ sur $\intOF{0}{10}$.
\item Déterminer les valeurs de $x$ et $h$ correspondant à une utilisation minimal de tôle.
@ -57,62 +57,62 @@ Les valeurs des exercices sont générés automatiquement. Si une valeur a un no
\begin{enumerate}
\item Le volume étant fixe si l'on fait varier $x$, $h$ doit aussi varier.
\begin{itemize}
\item Si $x = 2$ alors conserver un volume de $V=10$, $h$ doit être égale à $5 / 2$
\item Si $x = 3$ alors conserver un volume de $V=10$, $h$ doit être égale à $5 / 3$
\item Si $x = 2$ alors conserver un volume de $V=21$, $h$ doit être égale à $7 / 2$
\item Si $x = 3$ alors conserver un volume de $V=21$, $h$ doit être égale à $7 / 3$
\end{itemize}
\item Pour calculer le volume, on a
\begin{eqnarray*}
V &=& h\times x \times 2 \\
10 &=& h\times x \times 2 \\
x &=& \frac{10}{h\times 2} = \frac{5}{h}
V &=& h\times x \times 3 \\
21 &=& h\times x \times 3 \\
x &=& \frac{21}{h\times 3} = \frac{7}{h}
\end{eqnarray*}
\item Pour calculer la surface totale, on ajoute la surface de chaque face. On a donc le calcul suivant
\begin{eqnarray*}
S(x) &=& x\times h \times 2 + x\times2\times2 + h\times 2\times 2\\
S(x) &=& x\times \frac{5}{x} \times 2 + x\times2\times2 + \frac{5}{x}\times 2\times 2\\
S(x) &=& 4x + 10 + \frac{20}{x}
S(x) &=& x\times h \times 2 + x\times3\times2 + h\times 3\times 2\\
S(x) &=& x\times \frac{7}{x} \times 2 + x\times3\times2 + \frac{7}{x}\times 3\times 2\\
S(x) &=& 6x + 14 + \frac{42}{x}
\end{eqnarray*}
\item Pour trouver cette nouvelle forme, on met chaque élément sur le même dénominateur
\begin{eqnarray*}
S(x) &=& 4x + 10 + \frac{20}{x}\\
S(x) &=& \frac{4x\times x}{x} + \frac{10\times x}{x} + \frac{20}{x}\\
S(x) &=& \frac{4x^2 + 10x + 20}{x}
S(x) &=& 6x + 14 + \frac{42}{x}\\
S(x) &=& \frac{6x\times x}{x} + \frac{14\times x}{x} + \frac{42}{x}\\
S(x) &=& \frac{6x^2 + 14x + 42}{x}
\end{eqnarray*}
\item On retrouve la formule $\frac{u}{v}$ à dériver
\[
u(x) = 4x^2 + 10x + 20 \Rightarrow u'(x) = 8x + 10
u(x) = 6x^2 + 14x + 42 \Rightarrow u'(x) = 12x + 14
\]
\[
v(x) = x \Rightarrow v'(x) = 1
\]
Donc au numérateur on obtient
\begin{eqnarray*}
u'(x)\times v(x) - u(x)\times v'(x) &=& (8x + 10)\times x - (4x^2 + 10x + 20)\times 1\\
&=& 4x^2 - 20
u'(x)\times v(x) - u(x)\times v'(x) &=& (12x + 14)\times x - (6x^2 + 14x + 42)\times 1\\
&=& 6x^2 - 42
\end{eqnarray*}
Donc
\[
S'(x) = \frac{4x^2 - 20}{x^2}
S'(x) = \frac{6x^2 - 42}{x^2}
\]
\item Tableau de variations de $S$
\begin{itemize}
\item Valeur interdite: $x^2 = 0 \equiv x = 0$
\item Signe de $4x^2 - 20$: c'est un polynôme du 2e degré
\item Signe de $6x^2 - 42$: c'est un polynôme du 2e degré
\[
\Delta = 320 > 0
\Delta = 1008 > 0
\]
Il y a donc 2 racines
\[
x_1 = - 2.23606797749979 \qquad
x_2 = 2.23606797749979
x_1 = - 2.6457513110645907 \qquad
x_2 = 2.6457513110645907
\]
Et on sait que $4x^2 - 20$ est du signe de $a$ donc positif en dehors des racines
Et on sait que $6x^2 - 42$ est du signe de $a$ donc positif en dehors des racines
\item Le dénominateur $x^2$ est toujours positif.
\item Tableau de variations
\begin{tikzpicture}[baseline=(a.north)]
\tkzTabInit[lgt=3,espcl=3]{$x$/1,$4x^2 - 20$/1, $x^2$/1, $S'$/1, $S$/2}{$0$, $- 2.23606797749979$, $10$}
\tkzTabInit[lgt=3,espcl=3]{$x$/1,$6x^2 - 42$/1, $x^2$/1, $S'$/1, $S$/2}{$0$, $- 2.6457513110645907$, $10$}
\tkzTabLine{d,-, z, +, }
\tkzTabLine{d,+, , +, }
\tkzTabLine{d,-, z, +, }
@ -120,7 +120,7 @@ Les valeurs des exercices sont générés automatiquement. Si une valeur a un no
\end{tikzpicture}
\end{itemize}
\item On a donc une surface minimal pour $x=2.23606797749979$ et $h = 11.18033988749895$.
\item On a donc une surface minimal pour $x=2.6457513110645907$ et $h = 18.5202591774521349$.
\end{enumerate}
\end{solution}
@ -133,14 +133,14 @@ Les valeurs des exercices sont générés automatiquement. Si une valeur a un no
Le tour d'un bassin au niveau du sol présente deux axes de symétrie : laxe des abscisses et la droite déquation $x=4$. Il est obtenu par symétrie de la courbe $\mathcal{C}_f$ sur $\intFF{0}{4}$$f$ est la fonction définie par
\[
f(x) = \left(- x^{2} + 5.6 x - 1.6\right) e^{- x} + 1.6
f(x) = \left(- x^{2} + 2.6 x - 4.1\right) e^{- x} + 4.1
\]
On admet que sur $\intFF{0}{4}$ la fonction $f$ est positive.
\begin{enumerate}
\item Sur un repère, tracer l'allure de la courbe $\mathcal{C}_f$, les axes de symétries puis compléter pour dessiner la forme du bassin.
\item Montrer que la fonction $f$ admet comme primitive sur $\R$ la fonction $F$ définie par
\[
F(x) = 1.6 x + \left( x^{2} - 3.6 x - 2.0\right) e^{- x}
F(x) = 4.1 x + \left( x^{2} - 0.6 x + 3.5\right) e^{- x}
\]
\item Calculer la quantité $\ds \int_0^4 f(x) \; dx$, vous donnerez le résultat sous forme exacte. Interpréter le résultat et reportez cette quantité sur le graphique.
\item On considère que l'échelle de votre graphique est de 1unité pour 15m. Calculer l'aire du bassin. Vous donnerez un résultat arrondi au $m^2$ près.
@ -156,15 +156,14 @@ Les valeurs des exercices sont générés automatiquement. Si une valeur a un no
\tkzGrid
\tkzAxeXY
\tkzFct[domain=0:10,color=red,very thick]%
{ (-x**2 + 5.6*x - 1.6)*exp(-x) + 1.6 };
{ (-x**2 + 2.6*x - 4.1)*exp(-x) + 4.1 };
\end{tikzpicture}
\item Il faut dériver $F(x)$ et vérifier que $F'(x) = f(x)$.
\item $\ds \int_0^4 f(x) \; dx = F(4) - F(0) = 8.4 - \frac{0.399999999999999}{e^{4}}$
\item $\ds \int_0^4 f(x) \; dx = F(4) - F(0) = \frac{17.1}{e^{4}} + 12.9$
\item La quantité calculée à la question précédente se retrouve 4fois pour former le bassin. Il faut ensuite prendre en compte l'échelle, comme 1unité de longueur correspond à 15m, une unité d'air correspond à $15\times15 = 225m^2$. Ainsi l'aire du bassin est égale à
\[
(8.4 - \frac{0.399999999999999}{e^{4}})\times 4 \times 15^2 = 7553.000000
(\frac{17.1}{e^{4}} + 12.9)\times 4 \times 15^2 = 11892.00000
\]
\end{enumerate}
\end{solution}
@ -185,9 +184,9 @@ Les valeurs des exercices sont générés automatiquement. Si une valeur a un no
Deux ateliers A et B fabriquent des stylos pour une entreprise.
L'atelier A fabrique 47.0\,\% des stylos, et parmi ceux-là, 25.0\,\% possèdent un défaut de fabrication.
L'atelier A fabrique 26.0\,\% des stylos, et parmi ceux-là, 60.0\,\% possèdent un défaut de fabrication.
De plus, 23.0\,\% des stylos possèdent un défaut de fabrication et sortent de l'atelier B.
De plus, 74.0\,\% des stylos possèdent un défaut de fabrication et sortent de l'atelier B.
Un stylo est prélevé au hasard dans le stock de l'entreprise.
@ -241,7 +240,7 @@ Les valeurs des exercices sont générés automatiquement. Si une valeur a un no
\item
\begin{enumerate}
\item Calculer la probabilité qu'un stylo provienne de l'atelier A et possède un défaut de fabrication.
\item En déduire que la probabilité qu'un stylo possède un défaut de fabrication est de $0.35$.
\item En déduire que la probabilité qu'un stylo possède un défaut de fabrication est de $0.9$.
\end{enumerate}
\item On prélève un stylo au hasard avec un défaut. Quelle est la probabilité qu'il vienne de l'atelier A?
\end{enumerate}
@ -251,7 +250,7 @@ Les valeurs des exercices sont générés automatiquement. Si une valeur a un no
\textbf{Partie B}
\medskip
Dans cette partie, on suppose que 35.0\,\% des stylos possèdent un défaut de fabrication.
Dans cette partie, on suppose que 90.0\,\% des stylos possèdent un défaut de fabrication.
L'entreprise confectionne des paquets contenant chacun $4$~stylos.
@ -266,7 +265,7 @@ Les valeurs des exercices sont générés automatiquement. Si une valeur a un no
\begin{enumerate}
\setcounter{enumi}{4}
\item Avec quelle loi peut-on modéliser $X$. Préciser les paramètres.
\item Calculer et interpréter la probabilité $P(X = 17)$.
\item Calculer et interpréter la probabilité $P(X = 13)$.
\item Le directeur de l'entreprise affirme qu'il y a plus d'une chance sur deux qu'un paquet ne comporte aucun stylo défectueux. A-t-il raison ?
\item Combien de stylos peut-on espérer avoir en moyenne?
\end{enumerate}
@ -282,27 +281,27 @@ Les valeurs des exercices sont générés automatiquement. Si une valeur a un no
child {node {$A$}
child {node {$D$}
edge from parent
node[above] {0.25}
node[above] {0.6}
}
child {node {$\overline{D}$}
edge from parent
node[above] {0.75}
node[above] {0.4}
}
edge from parent
node[above] {0.47}
node[above] {0.26}
}
child[missing] {}
child { node {$B$}
child {node {$D$}
edge from parent
node[above] {0.44}
node[above] {1.0}
}
child {node {$\overline{D}$}
edge from parent
node[above] {0.56}
node[above] {0.0}
}
edge from parent
node[above] {0.53}
node[above] {0.74}
} ;
\end{tikzpicture}
\end{center}
@ -310,51 +309,51 @@ Les valeurs des exercices sont générés automatiquement. Si une valeur a un no
\begin{itemize}
\item Probabilité que le stylo vienne de l'atelier A
\[
P(A) = 0.47
P(A) = 0.26
\]
\item Probabilité que le stylo vienne de l'atelier B
\[
P(B) = 0.53
P(B) = 0.74
\]
\item Probabilité que le stylo ait un défaut sachant qu'il vient de l'atelier A.
\[
P_A(D) = 0.25
P_A(D) = 0.6
\]
\item Probabilité que le stylo vienne de l'atelier B et qu'il ait un défaut.
\[
P(D \cap D) = 0.23
P(D \cap D) = 0.74
\]
\end{itemize}
\item
\begin{enumerate}
\item Probabilité qu'un stylo vienne de l'atelier A et qu'il ait un defaut
\[
P(A\cap D) = P(A) \times P_A(D) = 0.47 \times 0.25 = 0.12
P(A\cap D) = P(A) \times P_A(D) = 0.26 \times 0.6 = 0.16
\]
\item Probabilité que le stylo ai un défaut de fabrication.
\[
P(D) = P(A\cap D) + P(B\cap D) = 0.12 + 0.23 = 0.35
P(D) = P(A\cap D) + P(B\cap D) = 0.16 + 0.74 = 0.9
\]
\end{enumerate}
\item Probabilité qu'il vienne de l'atelier A sachant qu'il a un defaut
\[
P_D(A) = \frac{P(A\cap D)}{P(D)} = \frac{0.12}{0.35} = 0.34
P_D(A) = \frac{P(A\cap D)}{P(D)} = \frac{0.16}{0.9} = 0.18
\]
\item $X$ peut être modélisée par une loi binomiale de paramètres $n=19$ et $p=0.35$.
\item $X$ peut être modélisée par une loi binomiale de paramètres $n=15$ et $p=0.9$.
\item (\textit{par de correction automatique disponible pour le résultat final}
\[
P(X = 17) = \coefBino{19}{17}\times 0.35^{17} \times 0.65^{2}
P(X = 13) = \coefBino{15}{13}\times 0.9^{13} \times 0.1^{2}
\]
\item (\textit{par de correction automatique disponible pour le résultat final}
Il faut calculer la probabilité qu'il y ait 0 stylo avec un defaut.
\[
P(X = 0) = \coefBino{19}{0}\times 0.35^{0} \times 0.65^{19}
P(X = 0) = \coefBino{15}{0}\times 0.9^{0} \times 0.1^{15}
\]
Puis comparer ce nombre à 0,5.
\item Il faut calculer l'espérance
\[
E[X] = n\times p = 19 \times 0.35 = 6.65
E[X] = n\times p = 15 \times 0.9 = 13.5
\]
\end{enumerate}
\end{solution}

View File

@ -3,7 +3,7 @@
% Title Page
\title{DM1 \hfill KICHENASSAMY Kévin}
\tribe{Maths complémentaire}
\tribe{Maths complémentaires}
\date{\hfillÀ render pour le jeudi 27 mai}
\xsimsetup{
@ -133,14 +133,14 @@ Les valeurs des exercices sont générés automatiquement. Si une valeur a un no
Le tour d'un bassin au niveau du sol présente deux axes de symétrie : laxe des abscisses et la droite déquation $x=4$. Il est obtenu par symétrie de la courbe $\mathcal{C}_f$ sur $\intFF{0}{4}$$f$ est la fonction définie par
\[
f(x) = \left(- x^{2} + 0.4 x - 6.0\right) e^{- x} + 6.0
f(x) = \left(- x^{2} + 8.1 x - 0.8\right) e^{- x} + 0.8
\]
On admet que sur $\intFF{0}{4}$ la fonction $f$ est positive.
\begin{enumerate}
\item Sur un repère, tracer l'allure de la courbe $\mathcal{C}_f$, les axes de symétries puis compléter pour dessiner la forme du bassin.
\item Montrer que la fonction $f$ admet comme primitive sur $\R$ la fonction $F$ définie par
\[
F(x) = 6.0 x + \left( x^{2} + 1.6 x + 7.6\right) e^{- x}
F(x) = 0.8 x + \left( x^{2} - 6.1 x - 5.3\right) e^{- x}
\]
\item Calculer la quantité $\ds \int_0^4 f(x) \; dx$, vous donnerez le résultat sous forme exacte. Interpréter le résultat et reportez cette quantité sur le graphique.
\item On considère que l'échelle de votre graphique est de 1unité pour 15m. Calculer l'aire du bassin. Vous donnerez un résultat arrondi au $m^2$ près.
@ -156,15 +156,14 @@ Les valeurs des exercices sont générés automatiquement. Si une valeur a un no
\tkzGrid
\tkzAxeXY
\tkzFct[domain=0:10,color=red,very thick]%
{ (-x**2 + 0.4*x - 6.0)*exp(-x) + 6.0 };
{ (-x**2 + 8.1*x - 0.8)*exp(-x) + 0.8 };
\end{tikzpicture}
\item Il faut dériver $F(x)$ et vérifier que $F'(x) = f(x)$.
\item $\ds \int_0^4 f(x) \; dx = F(4) - F(0) = \frac{30.0}{e^{4}} + 16.4$
\item $\ds \int_0^4 f(x) \; dx = F(4) - F(0) = 8.5 - \frac{13.7}{e^{4}}$
\item La quantité calculée à la question précédente se retrouve 4fois pour former le bassin. Il faut ensuite prendre en compte l'échelle, comme 1unité de longueur correspond à 15m, une unité d'air correspond à $15\times15 = 225m^2$. Ainsi l'aire du bassin est égale à
\[
(\frac{30.0}{e^{4}} + 16.4)\times 4 \times 15^2 = 15255.00000
(8.5 - \frac{13.7}{e^{4}})\times 4 \times 15^2 = 7424.000000
\]
\end{enumerate}
\end{solution}
@ -185,9 +184,9 @@ Les valeurs des exercices sont générés automatiquement. Si une valeur a un no
Deux ateliers A et B fabriquent des stylos pour une entreprise.
L'atelier A fabrique 84.0\,\% des stylos, et parmi ceux-là, 85.0\,\% possèdent un défaut de fabrication.
L'atelier A fabrique 68.0\,\% des stylos, et parmi ceux-là, 79.0\,\% possèdent un défaut de fabrication.
De plus, 4.0\,\% des stylos possèdent un défaut de fabrication et sortent de l'atelier B.
De plus, 26.0\,\% des stylos possèdent un défaut de fabrication et sortent de l'atelier B.
Un stylo est prélevé au hasard dans le stock de l'entreprise.
@ -241,7 +240,7 @@ Les valeurs des exercices sont générés automatiquement. Si une valeur a un no
\item
\begin{enumerate}
\item Calculer la probabilité qu'un stylo provienne de l'atelier A et possède un défaut de fabrication.
\item En déduire que la probabilité qu'un stylo possède un défaut de fabrication est de $0.75$.
\item En déduire que la probabilité qu'un stylo possède un défaut de fabrication est de $0.8$.
\end{enumerate}
\item On prélève un stylo au hasard avec un défaut. Quelle est la probabilité qu'il vienne de l'atelier A?
\end{enumerate}
@ -251,7 +250,7 @@ Les valeurs des exercices sont générés automatiquement. Si une valeur a un no
\textbf{Partie B}
\medskip
Dans cette partie, on suppose que 75.0\,\% des stylos possèdent un défaut de fabrication.
Dans cette partie, on suppose que 80.0\,\% des stylos possèdent un défaut de fabrication.
L'entreprise confectionne des paquets contenant chacun $4$~stylos.
@ -266,7 +265,7 @@ Les valeurs des exercices sont générés automatiquement. Si une valeur a un no
\begin{enumerate}
\setcounter{enumi}{4}
\item Avec quelle loi peut-on modéliser $X$. Préciser les paramètres.
\item Calculer et interpréter la probabilité $P(X = 14)$.
\item Calculer et interpréter la probabilité $P(X = 12)$.
\item Le directeur de l'entreprise affirme qu'il y a plus d'une chance sur deux qu'un paquet ne comporte aucun stylo défectueux. A-t-il raison ?
\item Combien de stylos peut-on espérer avoir en moyenne?
\end{enumerate}
@ -282,27 +281,27 @@ Les valeurs des exercices sont générés automatiquement. Si une valeur a un no
child {node {$A$}
child {node {$D$}
edge from parent
node[above] {0.85}
node[above] {0.79}
}
child {node {$\overline{D}$}
edge from parent
node[above] {0.15}
node[above] {0.21}
}
edge from parent
node[above] {0.84}
node[above] {0.68}
}
child[missing] {}
child { node {$B$}
child {node {$D$}
edge from parent
node[above] {0.26}
node[above] {0.81}
}
child {node {$\overline{D}$}
edge from parent
node[above] {0.74}
node[above] {0.19}
}
edge from parent
node[above] {0.16}
node[above] {0.32}
} ;
\end{tikzpicture}
\end{center}
@ -310,51 +309,51 @@ Les valeurs des exercices sont générés automatiquement. Si une valeur a un no
\begin{itemize}
\item Probabilité que le stylo vienne de l'atelier A
\[
P(A) = 0.84
P(A) = 0.68
\]
\item Probabilité que le stylo vienne de l'atelier B
\[
P(B) = 0.16
P(B) = 0.32
\]
\item Probabilité que le stylo ait un défaut sachant qu'il vient de l'atelier A.
\[
P_A(D) = 0.85
P_A(D) = 0.79
\]
\item Probabilité que le stylo vienne de l'atelier B et qu'il ait un défaut.
\[
P(D \cap D) = 0.04
P(D \cap D) = 0.26
\]
\end{itemize}
\item
\begin{enumerate}
\item Probabilité qu'un stylo vienne de l'atelier A et qu'il ait un defaut
\[
P(A\cap D) = P(A) \times P_A(D) = 0.84 \times 0.85 = 0.71
P(A\cap D) = P(A) \times P_A(D) = 0.68 \times 0.79 = 0.54
\]
\item Probabilité que le stylo ai un défaut de fabrication.
\[
P(D) = P(A\cap D) + P(B\cap D) = 0.71 + 0.04 = 0.75
P(D) = P(A\cap D) + P(B\cap D) = 0.54 + 0.26 = 0.8
\]
\end{enumerate}
\item Probabilité qu'il vienne de l'atelier A sachant qu'il a un defaut
\[
P_D(A) = \frac{P(A\cap D)}{P(D)} = \frac{0.71}{0.75} = 0.95
P_D(A) = \frac{P(A\cap D)}{P(D)} = \frac{0.54}{0.8} = 0.68
\]
\item $X$ peut être modélisée par une loi binomiale de paramètres $n=17$ et $p=0.75$.
\item $X$ peut être modélisée par une loi binomiale de paramètres $n=16$ et $p=0.8$.
\item (\textit{par de correction automatique disponible pour le résultat final}
\[
P(X = 14) = \coefBino{17}{14}\times 0.75^{14} \times 0.25^{3}
P(X = 12) = \coefBino{16}{12}\times 0.8^{12} \times 0.2^{4}
\]
\item (\textit{par de correction automatique disponible pour le résultat final}
Il faut calculer la probabilité qu'il y ait 0 stylo avec un defaut.
\[
P(X = 0) = \coefBino{17}{0}\times 0.75^{0} \times 0.25^{17}
P(X = 0) = \coefBino{16}{0}\times 0.8^{0} \times 0.2^{16}
\]
Puis comparer ce nombre à 0,5.
\item Il faut calculer l'espérance
\[
E[X] = n\times p = 17 \times 0.75 = 12.75
E[X] = n\times p = 16 \times 0.8 = 12.8
\]
\end{enumerate}
\end{solution}

View File

@ -3,7 +3,7 @@
% Title Page
\title{DM1 \hfill MATHIEU Allan}
\tribe{Maths complémentaire}
\tribe{Maths complémentaires}
\date{\hfillÀ render pour le jeudi 27 mai}
\xsimsetup{
@ -17,7 +17,7 @@ Les valeurs des exercices sont générés automatiquement. Si une valeur a un no
\begin{exercise}[subtitle={Optimisation de matière}]
\begin{minipage}{0.6\textwidth}
On se propose de fabriquer avec le moins de tôle possible une citerne fermée en forme de parallélépipède rectangle dont le volume intérieur doit être de $40m^3$. La longueur est aussi fixée à $4m$ par le cahier des charges.
On se propose de fabriquer avec le moins de tôle possible une citerne fermée en forme de parallélépipède rectangle dont le volume intérieur doit être de $24m^3$. La longueur est aussi fixée à $3m$ par le cahier des charges.
On peut donc faire varier uniquement la largeur (notée $x$) et la hauteur (notée $h$) de la cuve.
\end{minipage}
@ -28,25 +28,25 @@ Les valeurs des exercices sont générés automatiquement. Si une valeur a un no
\pgfmathsetmacro{\cubey}{1}
\pgfmathsetmacro{\cubez}{2}
\draw[black,fill=gray] (0,0,0) -- ++(-\cubex,0,0) -- ++(0,-\cubey,0) node [midway, left] {$h$} -- ++(\cubex,0,0) node [midway, below] {$x$} -- cycle;
\draw[black,fill=gray] (0,0,0) -- ++(0,0,-\cubez) -- ++(0,-\cubey,0) -- ++(0,0,\cubez) node [midway, right] {$4m$} -- cycle;
\draw[black,fill=gray] (0,0,0) -- ++(0,0,-\cubez) -- ++(0,-\cubey,0) -- ++(0,0,\cubez) node [midway, right] {$3m$} -- cycle;
\draw[black,fill=gray] (0,0,0) -- ++(-\cubex,0,0) -- ++(0,0,-\cubez) -- ++(\cubex,0,0) -- cycle;
\end{tikzpicture}
\end{minipage}
\begin{enumerate}
\item Expliquer pourquoi quand la largeur $x$ change, la hauteur $h$ doit elle aussi changer pour respecter les contraintes.
\item Démontrer que l'on doit avoir $h = \dfrac{10}{x}$.
\item Démontrer que l'on doit avoir $h = \dfrac{8}{x}$.
\item On note $S(x)$ l'aire totale de la citerne (c'est à dire la somme des aires des six faces). Montrer que l'on peut écrire
\[
S(x) = 8x + 20 + \frac{80}{x}
S(x) = 6x + 16 + \frac{48}{x}
\]
\item Démontrer que
\[
S(x) = \frac{8x^2 + 20x + 80}{x}
S(x) = \frac{6x^2 + 16x + 48}{x}
\]
\item Démontrer que
\[
S'(x) = \frac{8x^2 - 80}{x^2}
S'(x) = \frac{6x^2 - 48}{x^2}
\]
\item En déduire le tableau de variation de $S(x)$ sur $\intOF{0}{10}$.
\item Déterminer les valeurs de $x$ et $h$ correspondant à une utilisation minimal de tôle.
@ -57,62 +57,62 @@ Les valeurs des exercices sont générés automatiquement. Si une valeur a un no
\begin{enumerate}
\item Le volume étant fixe si l'on fait varier $x$, $h$ doit aussi varier.
\begin{itemize}
\item Si $x = 2$ alors conserver un volume de $V=40$, $h$ doit être égale à $10 / 2$
\item Si $x = 3$ alors conserver un volume de $V=40$, $h$ doit être égale à $10 / 3$
\item Si $x = 2$ alors conserver un volume de $V=24$, $h$ doit être égale à $8 / 2$
\item Si $x = 3$ alors conserver un volume de $V=24$, $h$ doit être égale à $8 / 3$
\end{itemize}
\item Pour calculer le volume, on a
\begin{eqnarray*}
V &=& h\times x \times 4 \\
40 &=& h\times x \times 4 \\
x &=& \frac{40}{h\times 4} = \frac{10}{h}
V &=& h\times x \times 3 \\
24 &=& h\times x \times 3 \\
x &=& \frac{24}{h\times 3} = \frac{8}{h}
\end{eqnarray*}
\item Pour calculer la surface totale, on ajoute la surface de chaque face. On a donc le calcul suivant
\begin{eqnarray*}
S(x) &=& x\times h \times 2 + x\times4\times2 + h\times 4\times 2\\
S(x) &=& x\times \frac{10}{x} \times 2 + x\times4\times2 + \frac{10}{x}\times 4\times 2\\
S(x) &=& 8x + 20 + \frac{80}{x}
S(x) &=& x\times h \times 2 + x\times3\times2 + h\times 3\times 2\\
S(x) &=& x\times \frac{8}{x} \times 2 + x\times3\times2 + \frac{8}{x}\times 3\times 2\\
S(x) &=& 6x + 16 + \frac{48}{x}
\end{eqnarray*}
\item Pour trouver cette nouvelle forme, on met chaque élément sur le même dénominateur
\begin{eqnarray*}
S(x) &=& 8x + 20 + \frac{80}{x}\\
S(x) &=& \frac{8x\times x}{x} + \frac{20\times x}{x} + \frac{80}{x}\\
S(x) &=& \frac{8x^2 + 20x + 80}{x}
S(x) &=& 6x + 16 + \frac{48}{x}\\
S(x) &=& \frac{6x\times x}{x} + \frac{16\times x}{x} + \frac{48}{x}\\
S(x) &=& \frac{6x^2 + 16x + 48}{x}
\end{eqnarray*}
\item On retrouve la formule $\frac{u}{v}$ à dériver
\[
u(x) = 8x^2 + 20x + 80 \Rightarrow u'(x) = 16x + 20
u(x) = 6x^2 + 16x + 48 \Rightarrow u'(x) = 12x + 16
\]
\[
v(x) = x \Rightarrow v'(x) = 1
\]
Donc au numérateur on obtient
\begin{eqnarray*}
u'(x)\times v(x) - u(x)\times v'(x) &=& (16x + 20)\times x - (8x^2 + 20x + 80)\times 1\\
&=& 8x^2 - 80
u'(x)\times v(x) - u(x)\times v'(x) &=& (12x + 16)\times x - (6x^2 + 16x + 48)\times 1\\
&=& 6x^2 - 48
\end{eqnarray*}
Donc
\[
S'(x) = \frac{8x^2 - 80}{x^2}
S'(x) = \frac{6x^2 - 48}{x^2}
\]
\item Tableau de variations de $S$
\begin{itemize}
\item Valeur interdite: $x^2 = 0 \equiv x = 0$
\item Signe de $8x^2 - 80$: c'est un polynôme du 2e degré
\item Signe de $6x^2 - 48$: c'est un polynôme du 2e degré
\[
\Delta = 2560 > 0
\Delta = 1152 > 0
\]
Il y a donc 2 racines
\[
x_1 = - 3.1622776601683795 \qquad
x_2 = 3.1622776601683795
x_1 = - 2.82842712474619 \qquad
x_2 = 2.82842712474619
\]
Et on sait que $8x^2 - 80$ est du signe de $a$ donc positif en dehors des racines
Et on sait que $6x^2 - 48$ est du signe de $a$ donc positif en dehors des racines
\item Le dénominateur $x^2$ est toujours positif.
\item Tableau de variations
\begin{tikzpicture}[baseline=(a.north)]
\tkzTabInit[lgt=3,espcl=3]{$x$/1,$8x^2 - 80$/1, $x^2$/1, $S'$/1, $S$/2}{$0$, $- 3.1622776601683795$, $10$}
\tkzTabInit[lgt=3,espcl=3]{$x$/1,$6x^2 - 48$/1, $x^2$/1, $S'$/1, $S$/2}{$0$, $- 2.82842712474619$, $10$}
\tkzTabLine{d,-, z, +, }
\tkzTabLine{d,+, , +, }
\tkzTabLine{d,-, z, +, }
@ -120,7 +120,7 @@ Les valeurs des exercices sont générés automatiquement. Si une valeur a un no
\end{tikzpicture}
\end{itemize}
\item On a donc une surface minimal pour $x=3.1622776601683795$ et $h = 31.6227766016837950$.
\item On a donc une surface minimal pour $x=2.82842712474619$ et $h = 22.62741699796952$.
\end{enumerate}
\end{solution}
@ -133,14 +133,14 @@ Les valeurs des exercices sont générés automatiquement. Si une valeur a un no
Le tour d'un bassin au niveau du sol présente deux axes de symétrie : laxe des abscisses et la droite déquation $x=4$. Il est obtenu par symétrie de la courbe $\mathcal{C}_f$ sur $\intFF{0}{4}$$f$ est la fonction définie par
\[
f(x) = \left(- x^{2} + 6.1 x - 9.3\right) e^{- x} + 9.3
f(x) = \left(- x^{2} + 5.6 x - 1.3\right) e^{- x} + 1.3
\]
On admet que sur $\intFF{0}{4}$ la fonction $f$ est positive.
\begin{enumerate}
\item Sur un repère, tracer l'allure de la courbe $\mathcal{C}_f$, les axes de symétries puis compléter pour dessiner la forme du bassin.
\item Montrer que la fonction $f$ admet comme primitive sur $\R$ la fonction $F$ définie par
\[
F(x) = 9.3 x + \left( x^{2} - 4.1 x + 5.2\right) e^{- x}
F(x) = 1.3 x + \left( x^{2} - 3.6 x - 2.3\right) e^{- x}
\]
\item Calculer la quantité $\ds \int_0^4 f(x) \; dx$, vous donnerez le résultat sous forme exacte. Interpréter le résultat et reportez cette quantité sur le graphique.
\item On considère que l'échelle de votre graphique est de 1unité pour 15m. Calculer l'aire du bassin. Vous donnerez un résultat arrondi au $m^2$ près.
@ -156,15 +156,14 @@ Les valeurs des exercices sont générés automatiquement. Si une valeur a un no
\tkzGrid
\tkzAxeXY
\tkzFct[domain=0:10,color=red,very thick]%
{ (-x**2 + 6.1*x - 9.3)*exp(-x) + 9.3 };
{ (-x**2 + 5.6*x - 1.3)*exp(-x) + 1.3 };
\end{tikzpicture}
\item Il faut dériver $F(x)$ et vérifier que $F'(x) = f(x)$.
\item $\ds \int_0^4 f(x) \; dx = F(4) - F(0) = \frac{4.8}{e^{4}} + 32.0$
\item $\ds \int_0^4 f(x) \; dx = F(4) - F(0) = 7.5 - \frac{0.699999999999999}{e^{4}}$
\item La quantité calculée à la question précédente se retrouve 4fois pour former le bassin. Il faut ensuite prendre en compte l'échelle, comme 1unité de longueur correspond à 15m, une unité d'air correspond à $15\times15 = 225m^2$. Ainsi l'aire du bassin est égale à
\[
(\frac{4.8}{e^{4}} + 32.0)\times 4 \times 15^2 = 28879.00000
(7.5 - \frac{0.699999999999999}{e^{4}})\times 4 \times 15^2 = 6738.000000
\]
\end{enumerate}
\end{solution}
@ -185,9 +184,9 @@ Les valeurs des exercices sont générés automatiquement. Si une valeur a un no
Deux ateliers A et B fabriquent des stylos pour une entreprise.
L'atelier A fabrique 92.0\,\% des stylos, et parmi ceux-là, 47.0\,\% possèdent un défaut de fabrication.
L'atelier A fabrique 53.0\,\% des stylos, et parmi ceux-là, 59.0\,\% possèdent un défaut de fabrication.
De plus, 1.0\,\% des stylos possèdent un défaut de fabrication et sortent de l'atelier B.
De plus, 7.000000000000001\,\% des stylos possèdent un défaut de fabrication et sortent de l'atelier B.
Un stylo est prélevé au hasard dans le stock de l'entreprise.
@ -241,7 +240,7 @@ Les valeurs des exercices sont générés automatiquement. Si une valeur a un no
\item
\begin{enumerate}
\item Calculer la probabilité qu'un stylo provienne de l'atelier A et possède un défaut de fabrication.
\item En déduire que la probabilité qu'un stylo possède un défaut de fabrication est de $0.44$.
\item En déduire que la probabilité qu'un stylo possède un défaut de fabrication est de $0.38$.
\end{enumerate}
\item On prélève un stylo au hasard avec un défaut. Quelle est la probabilité qu'il vienne de l'atelier A?
\end{enumerate}
@ -251,7 +250,7 @@ Les valeurs des exercices sont générés automatiquement. Si une valeur a un no
\textbf{Partie B}
\medskip
Dans cette partie, on suppose que 44.0\,\% des stylos possèdent un défaut de fabrication.
Dans cette partie, on suppose que 38.0\,\% des stylos possèdent un défaut de fabrication.
L'entreprise confectionne des paquets contenant chacun $4$~stylos.
@ -266,7 +265,7 @@ Les valeurs des exercices sont générés automatiquement. Si une valeur a un no
\begin{enumerate}
\setcounter{enumi}{4}
\item Avec quelle loi peut-on modéliser $X$. Préciser les paramètres.
\item Calculer et interpréter la probabilité $P(X = 15)$.
\item Calculer et interpréter la probabilité $P(X = 7)$.
\item Le directeur de l'entreprise affirme qu'il y a plus d'une chance sur deux qu'un paquet ne comporte aucun stylo défectueux. A-t-il raison ?
\item Combien de stylos peut-on espérer avoir en moyenne?
\end{enumerate}
@ -282,27 +281,27 @@ Les valeurs des exercices sont générés automatiquement. Si une valeur a un no
child {node {$A$}
child {node {$D$}
edge from parent
node[above] {0.47}
node[above] {0.59}
}
child {node {$\overline{D}$}
edge from parent
node[above] {0.53}
node[above] {0.41}
}
edge from parent
node[above] {0.92}
node[above] {0.53}
}
child[missing] {}
child { node {$B$}
child {node {$D$}
edge from parent
node[above] {0.17}
node[above] {0.14}
}
child {node {$\overline{D}$}
edge from parent
node[above] {0.83}
node[above] {0.86}
}
edge from parent
node[above] {0.08}
node[above] {0.47}
} ;
\end{tikzpicture}
\end{center}
@ -310,51 +309,51 @@ Les valeurs des exercices sont générés automatiquement. Si une valeur a un no
\begin{itemize}
\item Probabilité que le stylo vienne de l'atelier A
\[
P(A) = 0.92
P(A) = 0.53
\]
\item Probabilité que le stylo vienne de l'atelier B
\[
P(B) = 0.08
P(B) = 0.47
\]
\item Probabilité que le stylo ait un défaut sachant qu'il vient de l'atelier A.
\[
P_A(D) = 0.47
P_A(D) = 0.59
\]
\item Probabilité que le stylo vienne de l'atelier B et qu'il ait un défaut.
\[
P(D \cap D) = 0.01
P(D \cap D) = 0.07
\]
\end{itemize}
\item
\begin{enumerate}
\item Probabilité qu'un stylo vienne de l'atelier A et qu'il ait un defaut
\[
P(A\cap D) = P(A) \times P_A(D) = 0.92 \times 0.47 = 0.43
P(A\cap D) = P(A) \times P_A(D) = 0.53 \times 0.59 = 0.31
\]
\item Probabilité que le stylo ai un défaut de fabrication.
\[
P(D) = P(A\cap D) + P(B\cap D) = 0.43 + 0.01 = 0.44
P(D) = P(A\cap D) + P(B\cap D) = 0.31 + 0.07 = 0.38
\]
\end{enumerate}
\item Probabilité qu'il vienne de l'atelier A sachant qu'il a un defaut
\[
P_D(A) = \frac{P(A\cap D)}{P(D)} = \frac{0.43}{0.44} = 0.98
P_D(A) = \frac{P(A\cap D)}{P(D)} = \frac{0.31}{0.38} = 0.82
\]
\item $X$ peut être modélisée par une loi binomiale de paramètres $n=17$ et $p=0.44$.
\item $X$ peut être modélisée par une loi binomiale de paramètres $n=12$ et $p=0.38$.
\item (\textit{par de correction automatique disponible pour le résultat final}
\[
P(X = 15) = \coefBino{17}{15}\times 0.44^{15} \times 0.56^{2}
P(X = 7) = \coefBino{12}{7}\times 0.38^{7} \times 0.62^{5}
\]
\item (\textit{par de correction automatique disponible pour le résultat final}
Il faut calculer la probabilité qu'il y ait 0 stylo avec un defaut.
\[
P(X = 0) = \coefBino{17}{0}\times 0.44^{0} \times 0.56^{17}
P(X = 0) = \coefBino{12}{0}\times 0.38^{0} \times 0.62^{12}
\]
Puis comparer ce nombre à 0,5.
\item Il faut calculer l'espérance
\[
E[X] = n\times p = 17 \times 0.44 = 7.48
E[X] = n\times p = 12 \times 0.38 = 4.56
\]
\end{enumerate}
\end{solution}

View File

@ -3,7 +3,7 @@
% Title Page
\title{DM1 \hfill MOLINIER Annelise}
\tribe{Maths complémentaire}
\tribe{Maths complémentaires}
\date{\hfillÀ render pour le jeudi 27 mai}
\xsimsetup{
@ -17,7 +17,7 @@ Les valeurs des exercices sont générés automatiquement. Si une valeur a un no
\begin{exercise}[subtitle={Optimisation de matière}]
\begin{minipage}{0.6\textwidth}
On se propose de fabriquer avec le moins de tôle possible une citerne fermée en forme de parallélépipède rectangle dont le volume intérieur doit être de $25m^3$. La longueur est aussi fixée à $5m$ par le cahier des charges.
On se propose de fabriquer avec le moins de tôle possible une citerne fermée en forme de parallélépipède rectangle dont le volume intérieur doit être de $20m^3$. La longueur est aussi fixée à $2m$ par le cahier des charges.
On peut donc faire varier uniquement la largeur (notée $x$) et la hauteur (notée $h$) de la cuve.
\end{minipage}
@ -28,25 +28,25 @@ Les valeurs des exercices sont générés automatiquement. Si une valeur a un no
\pgfmathsetmacro{\cubey}{1}
\pgfmathsetmacro{\cubez}{2}
\draw[black,fill=gray] (0,0,0) -- ++(-\cubex,0,0) -- ++(0,-\cubey,0) node [midway, left] {$h$} -- ++(\cubex,0,0) node [midway, below] {$x$} -- cycle;
\draw[black,fill=gray] (0,0,0) -- ++(0,0,-\cubez) -- ++(0,-\cubey,0) -- ++(0,0,\cubez) node [midway, right] {$5m$} -- cycle;
\draw[black,fill=gray] (0,0,0) -- ++(0,0,-\cubez) -- ++(0,-\cubey,0) -- ++(0,0,\cubez) node [midway, right] {$2m$} -- cycle;
\draw[black,fill=gray] (0,0,0) -- ++(-\cubex,0,0) -- ++(0,0,-\cubez) -- ++(\cubex,0,0) -- cycle;
\end{tikzpicture}
\end{minipage}
\begin{enumerate}
\item Expliquer pourquoi quand la largeur $x$ change, la hauteur $h$ doit elle aussi changer pour respecter les contraintes.
\item Démontrer que l'on doit avoir $h = \dfrac{5}{x}$.
\item Démontrer que l'on doit avoir $h = \dfrac{10}{x}$.
\item On note $S(x)$ l'aire totale de la citerne (c'est à dire la somme des aires des six faces). Montrer que l'on peut écrire
\[
S(x) = 10x + 10 + \frac{50}{x}
S(x) = 4x + 20 + \frac{40}{x}
\]
\item Démontrer que
\[
S(x) = \frac{10x^2 + 10x + 50}{x}
S(x) = \frac{4x^2 + 20x + 40}{x}
\]
\item Démontrer que
\[
S'(x) = \frac{10x^2 - 50}{x^2}
S'(x) = \frac{4x^2 - 40}{x^2}
\]
\item En déduire le tableau de variation de $S(x)$ sur $\intOF{0}{10}$.
\item Déterminer les valeurs de $x$ et $h$ correspondant à une utilisation minimal de tôle.
@ -57,62 +57,62 @@ Les valeurs des exercices sont générés automatiquement. Si une valeur a un no
\begin{enumerate}
\item Le volume étant fixe si l'on fait varier $x$, $h$ doit aussi varier.
\begin{itemize}
\item Si $x = 2$ alors conserver un volume de $V=25$, $h$ doit être égale à $5 / 2$
\item Si $x = 3$ alors conserver un volume de $V=25$, $h$ doit être égale à $5 / 3$
\item Si $x = 2$ alors conserver un volume de $V=20$, $h$ doit être égale à $10 / 2$
\item Si $x = 3$ alors conserver un volume de $V=20$, $h$ doit être égale à $10 / 3$
\end{itemize}
\item Pour calculer le volume, on a
\begin{eqnarray*}
V &=& h\times x \times 5 \\
25 &=& h\times x \times 5 \\
x &=& \frac{25}{h\times 5} = \frac{5}{h}
V &=& h\times x \times 2 \\
20 &=& h\times x \times 2 \\
x &=& \frac{20}{h\times 2} = \frac{10}{h}
\end{eqnarray*}
\item Pour calculer la surface totale, on ajoute la surface de chaque face. On a donc le calcul suivant
\begin{eqnarray*}
S(x) &=& x\times h \times 2 + x\times5\times2 + h\times 5\times 2\\
S(x) &=& x\times \frac{5}{x} \times 2 + x\times5\times2 + \frac{5}{x}\times 5\times 2\\
S(x) &=& 10x + 10 + \frac{50}{x}
S(x) &=& x\times h \times 2 + x\times2\times2 + h\times 2\times 2\\
S(x) &=& x\times \frac{10}{x} \times 2 + x\times2\times2 + \frac{10}{x}\times 2\times 2\\
S(x) &=& 4x + 20 + \frac{40}{x}
\end{eqnarray*}
\item Pour trouver cette nouvelle forme, on met chaque élément sur le même dénominateur
\begin{eqnarray*}
S(x) &=& 10x + 10 + \frac{50}{x}\\
S(x) &=& \frac{10x\times x}{x} + \frac{10\times x}{x} + \frac{50}{x}\\
S(x) &=& \frac{10x^2 + 10x + 50}{x}
S(x) &=& 4x + 20 + \frac{40}{x}\\
S(x) &=& \frac{4x\times x}{x} + \frac{20\times x}{x} + \frac{40}{x}\\
S(x) &=& \frac{4x^2 + 20x + 40}{x}
\end{eqnarray*}
\item On retrouve la formule $\frac{u}{v}$ à dériver
\[
u(x) = 10x^2 + 10x + 50 \Rightarrow u'(x) = 20x + 10
u(x) = 4x^2 + 20x + 40 \Rightarrow u'(x) = 8x + 20
\]
\[
v(x) = x \Rightarrow v'(x) = 1
\]
Donc au numérateur on obtient
\begin{eqnarray*}
u'(x)\times v(x) - u(x)\times v'(x) &=& (20x + 10)\times x - (10x^2 + 10x + 50)\times 1\\
&=& 10x^2 - 50
u'(x)\times v(x) - u(x)\times v'(x) &=& (8x + 20)\times x - (4x^2 + 20x + 40)\times 1\\
&=& 4x^2 - 40
\end{eqnarray*}
Donc
\[
S'(x) = \frac{10x^2 - 50}{x^2}
S'(x) = \frac{4x^2 - 40}{x^2}
\]
\item Tableau de variations de $S$
\begin{itemize}
\item Valeur interdite: $x^2 = 0 \equiv x = 0$
\item Signe de $10x^2 - 50$: c'est un polynôme du 2e degré
\item Signe de $4x^2 - 40$: c'est un polynôme du 2e degré
\[
\Delta = 2000 > 0
\Delta = 640 > 0
\]
Il y a donc 2 racines
\[
x_1 = - 2.23606797749979 \qquad
x_2 = 2.23606797749979
x_1 = - 3.1622776601683795 \qquad
x_2 = 3.1622776601683795
\]
Et on sait que $10x^2 - 50$ est du signe de $a$ donc positif en dehors des racines
Et on sait que $4x^2 - 40$ est du signe de $a$ donc positif en dehors des racines
\item Le dénominateur $x^2$ est toujours positif.
\item Tableau de variations
\begin{tikzpicture}[baseline=(a.north)]
\tkzTabInit[lgt=3,espcl=3]{$x$/1,$10x^2 - 50$/1, $x^2$/1, $S'$/1, $S$/2}{$0$, $- 2.23606797749979$, $10$}
\tkzTabInit[lgt=3,espcl=3]{$x$/1,$4x^2 - 40$/1, $x^2$/1, $S'$/1, $S$/2}{$0$, $- 3.1622776601683795$, $10$}
\tkzTabLine{d,-, z, +, }
\tkzTabLine{d,+, , +, }
\tkzTabLine{d,-, z, +, }
@ -120,7 +120,7 @@ Les valeurs des exercices sont générés automatiquement. Si une valeur a un no
\end{tikzpicture}
\end{itemize}
\item On a donc une surface minimal pour $x=2.23606797749979$ et $h = 11.18033988749895$.
\item On a donc une surface minimal pour $x=3.1622776601683795$ et $h = 31.6227766016837950$.
\end{enumerate}
\end{solution}
@ -133,14 +133,14 @@ Les valeurs des exercices sont générés automatiquement. Si une valeur a un no
Le tour d'un bassin au niveau du sol présente deux axes de symétrie : laxe des abscisses et la droite déquation $x=4$. Il est obtenu par symétrie de la courbe $\mathcal{C}_f$ sur $\intFF{0}{4}$$f$ est la fonction définie par
\[
f(x) = \left(- x^{2} + 6.9 x - 3.3\right) e^{- x} + 3.3
f(x) = \left(- x^{2} + 5.1 x - 3.5\right) e^{- x} + 3.5
\]
On admet que sur $\intFF{0}{4}$ la fonction $f$ est positive.
\begin{enumerate}
\item Sur un repère, tracer l'allure de la courbe $\mathcal{C}_f$, les axes de symétries puis compléter pour dessiner la forme du bassin.
\item Montrer que la fonction $f$ admet comme primitive sur $\R$ la fonction $F$ définie par
\[
F(x) = 3.3 x + \left( x^{2} - 4.9 x - 1.6\right) e^{- x}
F(x) = 3.5 x + \left( x^{2} - 3.1 x + 0.4\right) e^{- x}
\]
\item Calculer la quantité $\ds \int_0^4 f(x) \; dx$, vous donnerez le résultat sous forme exacte. Interpréter le résultat et reportez cette quantité sur le graphique.
\item On considère que l'échelle de votre graphique est de 1unité pour 15m. Calculer l'aire du bassin. Vous donnerez un résultat arrondi au $m^2$ près.
@ -156,15 +156,14 @@ Les valeurs des exercices sont générés automatiquement. Si une valeur a un no
\tkzGrid
\tkzAxeXY
\tkzFct[domain=0:10,color=red,very thick]%
{ (-x**2 + 6.9*x - 3.3)*exp(-x) + 3.3 };
{ (-x**2 + 5.1*x - 3.5)*exp(-x) + 3.5 };
\end{tikzpicture}
\item Il faut dériver $F(x)$ et vérifier que $F'(x) = f(x)$.
\item $\ds \int_0^4 f(x) \; dx = F(4) - F(0) = 14.8 - \frac{5.2}{e^{4}}$
\item $\ds \int_0^4 f(x) \; dx = F(4) - F(0) = \frac{4.0}{e^{4}} + 13.6$
\item La quantité calculée à la question précédente se retrouve 4fois pour former le bassin. Il faut ensuite prendre en compte l'échelle, comme 1unité de longueur correspond à 15m, une unité d'air correspond à $15\times15 = 225m^2$. Ainsi l'aire du bassin est égale à
\[
(14.8 - \frac{5.2}{e^{4}})\times 4 \times 15^2 = 13234.00000
(\frac{4.0}{e^{4}} + 13.6)\times 4 \times 15^2 = 12306.00000
\]
\end{enumerate}
\end{solution}
@ -185,9 +184,9 @@ Les valeurs des exercices sont générés automatiquement. Si une valeur a un no
Deux ateliers A et B fabriquent des stylos pour une entreprise.
L'atelier A fabrique 56.00000000000001\,\% des stylos, et parmi ceux-là, 22.0\,\% possèdent un défaut de fabrication.
L'atelier A fabrique 52.0\,\% des stylos, et parmi ceux-là, 81.0\,\% possèdent un défaut de fabrication.
De plus, 10.0\,\% des stylos possèdent un défaut de fabrication et sortent de l'atelier B.
De plus, 31.0\,\% des stylos possèdent un défaut de fabrication et sortent de l'atelier B.
Un stylo est prélevé au hasard dans le stock de l'entreprise.
@ -241,7 +240,7 @@ Les valeurs des exercices sont générés automatiquement. Si une valeur a un no
\item
\begin{enumerate}
\item Calculer la probabilité qu'un stylo provienne de l'atelier A et possède un défaut de fabrication.
\item En déduire que la probabilité qu'un stylo possède un défaut de fabrication est de $0.22$.
\item En déduire que la probabilité qu'un stylo possède un défaut de fabrication est de $0.73$.
\end{enumerate}
\item On prélève un stylo au hasard avec un défaut. Quelle est la probabilité qu'il vienne de l'atelier A?
\end{enumerate}
@ -251,7 +250,7 @@ Les valeurs des exercices sont générés automatiquement. Si une valeur a un no
\textbf{Partie B}
\medskip
Dans cette partie, on suppose que 22.0\,\% des stylos possèdent un défaut de fabrication.
Dans cette partie, on suppose que 73.0\,\% des stylos possèdent un défaut de fabrication.
L'entreprise confectionne des paquets contenant chacun $4$~stylos.
@ -266,7 +265,7 @@ Les valeurs des exercices sont générés automatiquement. Si une valeur a un no
\begin{enumerate}
\setcounter{enumi}{4}
\item Avec quelle loi peut-on modéliser $X$. Préciser les paramètres.
\item Calculer et interpréter la probabilité $P(X = 19)$.
\item Calculer et interpréter la probabilité $P(X = 6)$.
\item Le directeur de l'entreprise affirme qu'il y a plus d'une chance sur deux qu'un paquet ne comporte aucun stylo défectueux. A-t-il raison ?
\item Combien de stylos peut-on espérer avoir en moyenne?
\end{enumerate}
@ -282,27 +281,27 @@ Les valeurs des exercices sont générés automatiquement. Si une valeur a un no
child {node {$A$}
child {node {$D$}
edge from parent
node[above] {0.22}
node[above] {0.81}
}
child {node {$\overline{D}$}
edge from parent
node[above] {0.78}
node[above] {0.19}
}
edge from parent
node[above] {0.56}
node[above] {0.52}
}
child[missing] {}
child { node {$B$}
child {node {$D$}
edge from parent
node[above] {0.23}
node[above] {0.65}
}
child {node {$\overline{D}$}
edge from parent
node[above] {0.77}
node[above] {0.35}
}
edge from parent
node[above] {0.44}
node[above] {0.48}
} ;
\end{tikzpicture}
\end{center}
@ -310,51 +309,51 @@ Les valeurs des exercices sont générés automatiquement. Si une valeur a un no
\begin{itemize}
\item Probabilité que le stylo vienne de l'atelier A
\[
P(A) = 0.56
P(A) = 0.52
\]
\item Probabilité que le stylo vienne de l'atelier B
\[
P(B) = 0.44
P(B) = 0.48
\]
\item Probabilité que le stylo ait un défaut sachant qu'il vient de l'atelier A.
\[
P_A(D) = 0.22
P_A(D) = 0.81
\]
\item Probabilité que le stylo vienne de l'atelier B et qu'il ait un défaut.
\[
P(D \cap D) = 0.1
P(D \cap D) = 0.31
\]
\end{itemize}
\item
\begin{enumerate}
\item Probabilité qu'un stylo vienne de l'atelier A et qu'il ait un defaut
\[
P(A\cap D) = P(A) \times P_A(D) = 0.56 \times 0.22 = 0.12
P(A\cap D) = P(A) \times P_A(D) = 0.52 \times 0.81 = 0.42
\]
\item Probabilité que le stylo ai un défaut de fabrication.
\[
P(D) = P(A\cap D) + P(B\cap D) = 0.12 + 0.1 = 0.22
P(D) = P(A\cap D) + P(B\cap D) = 0.42 + 0.31 = 0.73
\]
\end{enumerate}
\item Probabilité qu'il vienne de l'atelier A sachant qu'il a un defaut
\[
P_D(A) = \frac{P(A\cap D)}{P(D)} = \frac{0.12}{0.22} = 0.55
P_D(A) = \frac{P(A\cap D)}{P(D)} = \frac{0.42}{0.73} = 0.58
\]
\item $X$ peut être modélisée par une loi binomiale de paramètres $n=19$ et $p=0.22$.
\item $X$ peut être modélisée par une loi binomiale de paramètres $n=10$ et $p=0.73$.
\item (\textit{par de correction automatique disponible pour le résultat final}
\[
P(X = 19) = \coefBino{19}{19}\times 0.22^{19} \times 0.78^{0}
P(X = 6) = \coefBino{10}{6}\times 0.73^{6} \times 0.27^{4}
\]
\item (\textit{par de correction automatique disponible pour le résultat final}
Il faut calculer la probabilité qu'il y ait 0 stylo avec un defaut.
\[
P(X = 0) = \coefBino{19}{0}\times 0.22^{0} \times 0.78^{19}
P(X = 0) = \coefBino{10}{0}\times 0.73^{0} \times 0.27^{10}
\]
Puis comparer ce nombre à 0,5.
\item Il faut calculer l'espérance
\[
E[X] = n\times p = 19 \times 0.22 = 4.18
E[X] = n\times p = 10 \times 0.73 = 7.3
\]
\end{enumerate}
\end{solution}

View File

@ -3,7 +3,7 @@
% Title Page
\title{DM1 \hfill MOUHOUBI Maïssa}
\tribe{Maths complémentaire}
\tribe{Maths complémentaires}
\date{\hfillÀ render pour le jeudi 27 mai}
\xsimsetup{
@ -17,7 +17,7 @@ Les valeurs des exercices sont générés automatiquement. Si une valeur a un no
\begin{exercise}[subtitle={Optimisation de matière}]
\begin{minipage}{0.6\textwidth}
On se propose de fabriquer avec le moins de tôle possible une citerne fermée en forme de parallélépipède rectangle dont le volume intérieur doit être de $35m^3$. La longueur est aussi fixée à $5m$ par le cahier des charges.
On se propose de fabriquer avec le moins de tôle possible une citerne fermée en forme de parallélépipède rectangle dont le volume intérieur doit être de $9m^3$. La longueur est aussi fixée à $3m$ par le cahier des charges.
On peut donc faire varier uniquement la largeur (notée $x$) et la hauteur (notée $h$) de la cuve.
\end{minipage}
@ -28,25 +28,25 @@ Les valeurs des exercices sont générés automatiquement. Si une valeur a un no
\pgfmathsetmacro{\cubey}{1}
\pgfmathsetmacro{\cubez}{2}
\draw[black,fill=gray] (0,0,0) -- ++(-\cubex,0,0) -- ++(0,-\cubey,0) node [midway, left] {$h$} -- ++(\cubex,0,0) node [midway, below] {$x$} -- cycle;
\draw[black,fill=gray] (0,0,0) -- ++(0,0,-\cubez) -- ++(0,-\cubey,0) -- ++(0,0,\cubez) node [midway, right] {$5m$} -- cycle;
\draw[black,fill=gray] (0,0,0) -- ++(0,0,-\cubez) -- ++(0,-\cubey,0) -- ++(0,0,\cubez) node [midway, right] {$3m$} -- cycle;
\draw[black,fill=gray] (0,0,0) -- ++(-\cubex,0,0) -- ++(0,0,-\cubez) -- ++(\cubex,0,0) -- cycle;
\end{tikzpicture}
\end{minipage}
\begin{enumerate}
\item Expliquer pourquoi quand la largeur $x$ change, la hauteur $h$ doit elle aussi changer pour respecter les contraintes.
\item Démontrer que l'on doit avoir $h = \dfrac{7}{x}$.
\item Démontrer que l'on doit avoir $h = \dfrac{3}{x}$.
\item On note $S(x)$ l'aire totale de la citerne (c'est à dire la somme des aires des six faces). Montrer que l'on peut écrire
\[
S(x) = 10x + 14 + \frac{70}{x}
S(x) = 6x + 6 + \frac{18}{x}
\]
\item Démontrer que
\[
S(x) = \frac{10x^2 + 14x + 70}{x}
S(x) = \frac{6x^2 + 6x + 18}{x}
\]
\item Démontrer que
\[
S'(x) = \frac{10x^2 - 70}{x^2}
S'(x) = \frac{6x^2 - 18}{x^2}
\]
\item En déduire le tableau de variation de $S(x)$ sur $\intOF{0}{10}$.
\item Déterminer les valeurs de $x$ et $h$ correspondant à une utilisation minimal de tôle.
@ -57,62 +57,62 @@ Les valeurs des exercices sont générés automatiquement. Si une valeur a un no
\begin{enumerate}
\item Le volume étant fixe si l'on fait varier $x$, $h$ doit aussi varier.
\begin{itemize}
\item Si $x = 2$ alors conserver un volume de $V=35$, $h$ doit être égale à $7 / 2$
\item Si $x = 3$ alors conserver un volume de $V=35$, $h$ doit être égale à $7 / 3$
\item Si $x = 2$ alors conserver un volume de $V=9$, $h$ doit être égale à $3 / 2$
\item Si $x = 3$ alors conserver un volume de $V=9$, $h$ doit être égale à $3 / 3$
\end{itemize}
\item Pour calculer le volume, on a
\begin{eqnarray*}
V &=& h\times x \times 5 \\
35 &=& h\times x \times 5 \\
x &=& \frac{35}{h\times 5} = \frac{7}{h}
V &=& h\times x \times 3 \\
9 &=& h\times x \times 3 \\
x &=& \frac{9}{h\times 3} = \frac{3}{h}
\end{eqnarray*}
\item Pour calculer la surface totale, on ajoute la surface de chaque face. On a donc le calcul suivant
\begin{eqnarray*}
S(x) &=& x\times h \times 2 + x\times5\times2 + h\times 5\times 2\\
S(x) &=& x\times \frac{7}{x} \times 2 + x\times5\times2 + \frac{7}{x}\times 5\times 2\\
S(x) &=& 10x + 14 + \frac{70}{x}
S(x) &=& x\times h \times 2 + x\times3\times2 + h\times 3\times 2\\
S(x) &=& x\times \frac{3}{x} \times 2 + x\times3\times2 + \frac{3}{x}\times 3\times 2\\
S(x) &=& 6x + 6 + \frac{18}{x}
\end{eqnarray*}
\item Pour trouver cette nouvelle forme, on met chaque élément sur le même dénominateur
\begin{eqnarray*}
S(x) &=& 10x + 14 + \frac{70}{x}\\
S(x) &=& \frac{10x\times x}{x} + \frac{14\times x}{x} + \frac{70}{x}\\
S(x) &=& \frac{10x^2 + 14x + 70}{x}
S(x) &=& 6x + 6 + \frac{18}{x}\\
S(x) &=& \frac{6x\times x}{x} + \frac{6\times x}{x} + \frac{18}{x}\\
S(x) &=& \frac{6x^2 + 6x + 18}{x}
\end{eqnarray*}
\item On retrouve la formule $\frac{u}{v}$ à dériver
\[
u(x) = 10x^2 + 14x + 70 \Rightarrow u'(x) = 20x + 14
u(x) = 6x^2 + 6x + 18 \Rightarrow u'(x) = 12x + 6
\]
\[
v(x) = x \Rightarrow v'(x) = 1
\]
Donc au numérateur on obtient
\begin{eqnarray*}
u'(x)\times v(x) - u(x)\times v'(x) &=& (20x + 14)\times x - (10x^2 + 14x + 70)\times 1\\
&=& 10x^2 - 70
u'(x)\times v(x) - u(x)\times v'(x) &=& (12x + 6)\times x - (6x^2 + 6x + 18)\times 1\\
&=& 6x^2 - 18
\end{eqnarray*}
Donc
\[
S'(x) = \frac{10x^2 - 70}{x^2}
S'(x) = \frac{6x^2 - 18}{x^2}
\]
\item Tableau de variations de $S$
\begin{itemize}
\item Valeur interdite: $x^2 = 0 \equiv x = 0$
\item Signe de $10x^2 - 70$: c'est un polynôme du 2e degré
\item Signe de $6x^2 - 18$: c'est un polynôme du 2e degré
\[
\Delta = 2800 > 0
\Delta = 432 > 0
\]
Il y a donc 2 racines
\[
x_1 = - 2.6457513110645907 \qquad
x_2 = 2.6457513110645907
x_1 = - 1.7320508075688774 \qquad
x_2 = 1.7320508075688774
\]
Et on sait que $10x^2 - 70$ est du signe de $a$ donc positif en dehors des racines
Et on sait que $6x^2 - 18$ est du signe de $a$ donc positif en dehors des racines
\item Le dénominateur $x^2$ est toujours positif.
\item Tableau de variations
\begin{tikzpicture}[baseline=(a.north)]
\tkzTabInit[lgt=3,espcl=3]{$x$/1,$10x^2 - 70$/1, $x^2$/1, $S'$/1, $S$/2}{$0$, $- 2.6457513110645907$, $10$}
\tkzTabInit[lgt=3,espcl=3]{$x$/1,$6x^2 - 18$/1, $x^2$/1, $S'$/1, $S$/2}{$0$, $- 1.7320508075688774$, $10$}
\tkzTabLine{d,-, z, +, }
\tkzTabLine{d,+, , +, }
\tkzTabLine{d,-, z, +, }
@ -120,7 +120,7 @@ Les valeurs des exercices sont générés automatiquement. Si une valeur a un no
\end{tikzpicture}
\end{itemize}
\item On a donc une surface minimal pour $x=2.6457513110645907$ et $h = 18.5202591774521349$.
\item On a donc une surface minimal pour $x=1.7320508075688774$ et $h = 5.1961524227066322$.
\end{enumerate}
\end{solution}
@ -133,14 +133,14 @@ Les valeurs des exercices sont générés automatiquement. Si une valeur a un no
Le tour d'un bassin au niveau du sol présente deux axes de symétrie : laxe des abscisses et la droite déquation $x=4$. Il est obtenu par symétrie de la courbe $\mathcal{C}_f$ sur $\intFF{0}{4}$$f$ est la fonction définie par
\[
f(x) = \left(- x^{2} + 1.1 x - 6.1\right) e^{- x} + 6.1
f(x) = \left(- x^{2} + 7.0 x - 7.2\right) e^{- x} + 7.2
\]
On admet que sur $\intFF{0}{4}$ la fonction $f$ est positive.
\begin{enumerate}
\item Sur un repère, tracer l'allure de la courbe $\mathcal{C}_f$, les axes de symétries puis compléter pour dessiner la forme du bassin.
\item Montrer que la fonction $f$ admet comme primitive sur $\R$ la fonction $F$ définie par
\[
F(x) = 6.1 x + \left( x^{2} + 0.9 x + 7.0\right) e^{- x}
F(x) = 7.2 x + \left( x^{2} - 5.0 x + 2.2\right) e^{- x}
\]
\item Calculer la quantité $\ds \int_0^4 f(x) \; dx$, vous donnerez le résultat sous forme exacte. Interpréter le résultat et reportez cette quantité sur le graphique.
\item On considère que l'échelle de votre graphique est de 1unité pour 15m. Calculer l'aire du bassin. Vous donnerez un résultat arrondi au $m^2$ près.
@ -156,15 +156,14 @@ Les valeurs des exercices sont générés automatiquement. Si une valeur a un no
\tkzGrid
\tkzAxeXY
\tkzFct[domain=0:10,color=red,very thick]%
{ (-x**2 + 1.1*x - 6.1)*exp(-x) + 6.1 };
{ (-x**2 + 7.0*x - 7.2)*exp(-x) + 7.2 };
\end{tikzpicture}
\item Il faut dériver $F(x)$ et vérifier que $F'(x) = f(x)$.
\item $\ds \int_0^4 f(x) \; dx = F(4) - F(0) = \frac{26.6}{e^{4}} + 17.4$
\item $\ds \int_0^4 f(x) \; dx = F(4) - F(0) = 26.6 - \frac{1.8}{e^{4}}$
\item La quantité calculée à la question précédente se retrouve 4fois pour former le bassin. Il faut ensuite prendre en compte l'échelle, comme 1unité de longueur correspond à 15m, une unité d'air correspond à $15\times15 = 225m^2$. Ainsi l'aire du bassin est égale à
\[
(\frac{26.6}{e^{4}} + 17.4)\times 4 \times 15^2 = 16098.00000
(26.6 - \frac{1.8}{e^{4}})\times 4 \times 15^2 = 23910.00000
\]
\end{enumerate}
\end{solution}
@ -185,9 +184,9 @@ Les valeurs des exercices sont générés automatiquement. Si une valeur a un no
Deux ateliers A et B fabriquent des stylos pour une entreprise.
L'atelier A fabrique 23.0\,\% des stylos, et parmi ceux-là, 66.0\,\% possèdent un défaut de fabrication.
L'atelier A fabrique 57.99999999999999\,\% des stylos, et parmi ceux-là, 48.0\,\% possèdent un défaut de fabrication.
De plus, 68.0\,\% des stylos possèdent un défaut de fabrication et sortent de l'atelier B.
De plus, 11.0\,\% des stylos possèdent un défaut de fabrication et sortent de l'atelier B.
Un stylo est prélevé au hasard dans le stock de l'entreprise.
@ -241,7 +240,7 @@ Les valeurs des exercices sont générés automatiquement. Si une valeur a un no
\item
\begin{enumerate}
\item Calculer la probabilité qu'un stylo provienne de l'atelier A et possède un défaut de fabrication.
\item En déduire que la probabilité qu'un stylo possède un défaut de fabrication est de $0.83$.
\item En déduire que la probabilité qu'un stylo possède un défaut de fabrication est de $0.39$.
\end{enumerate}
\item On prélève un stylo au hasard avec un défaut. Quelle est la probabilité qu'il vienne de l'atelier A?
\end{enumerate}
@ -251,7 +250,7 @@ Les valeurs des exercices sont générés automatiquement. Si une valeur a un no
\textbf{Partie B}
\medskip
Dans cette partie, on suppose que 83.0\,\% des stylos possèdent un défaut de fabrication.
Dans cette partie, on suppose que 39.0\,\% des stylos possèdent un défaut de fabrication.
L'entreprise confectionne des paquets contenant chacun $4$~stylos.
@ -266,7 +265,7 @@ Les valeurs des exercices sont générés automatiquement. Si une valeur a un no
\begin{enumerate}
\setcounter{enumi}{4}
\item Avec quelle loi peut-on modéliser $X$. Préciser les paramètres.
\item Calculer et interpréter la probabilité $P(X = 10)$.
\item Calculer et interpréter la probabilité $P(X = 11)$.
\item Le directeur de l'entreprise affirme qu'il y a plus d'une chance sur deux qu'un paquet ne comporte aucun stylo défectueux. A-t-il raison ?
\item Combien de stylos peut-on espérer avoir en moyenne?
\end{enumerate}
@ -282,27 +281,27 @@ Les valeurs des exercices sont générés automatiquement. Si une valeur a un no
child {node {$A$}
child {node {$D$}
edge from parent
node[above] {0.66}
node[above] {0.48}
}
child {node {$\overline{D}$}
edge from parent
node[above] {0.34}
node[above] {0.52}
}
edge from parent
node[above] {0.23}
node[above] {0.58}
}
child[missing] {}
child { node {$B$}
child {node {$D$}
edge from parent
node[above] {0.88}
node[above] {0.26}
}
child {node {$\overline{D}$}
edge from parent
node[above] {0.12}
node[above] {0.74}
}
edge from parent
node[above] {0.77}
node[above] {0.42}
} ;
\end{tikzpicture}
\end{center}
@ -310,51 +309,51 @@ Les valeurs des exercices sont générés automatiquement. Si une valeur a un no
\begin{itemize}
\item Probabilité que le stylo vienne de l'atelier A
\[
P(A) = 0.23
P(A) = 0.58
\]
\item Probabilité que le stylo vienne de l'atelier B
\[
P(B) = 0.77
P(B) = 0.42
\]
\item Probabilité que le stylo ait un défaut sachant qu'il vient de l'atelier A.
\[
P_A(D) = 0.66
P_A(D) = 0.48
\]
\item Probabilité que le stylo vienne de l'atelier B et qu'il ait un défaut.
\[
P(D \cap D) = 0.68
P(D \cap D) = 0.11
\]
\end{itemize}
\item
\begin{enumerate}
\item Probabilité qu'un stylo vienne de l'atelier A et qu'il ait un defaut
\[
P(A\cap D) = P(A) \times P_A(D) = 0.23 \times 0.66 = 0.15
P(A\cap D) = P(A) \times P_A(D) = 0.58 \times 0.48 = 0.28
\]
\item Probabilité que le stylo ai un défaut de fabrication.
\[
P(D) = P(A\cap D) + P(B\cap D) = 0.15 + 0.68 = 0.83
P(D) = P(A\cap D) + P(B\cap D) = 0.28 + 0.11 = 0.39
\]
\end{enumerate}
\item Probabilité qu'il vienne de l'atelier A sachant qu'il a un defaut
\[
P_D(A) = \frac{P(A\cap D)}{P(D)} = \frac{0.15}{0.83} = 0.18
P_D(A) = \frac{P(A\cap D)}{P(D)} = \frac{0.28}{0.39} = 0.72
\]
\item $X$ peut être modélisée par une loi binomiale de paramètres $n=17$ et $p=0.83$.
\item $X$ peut être modélisée par une loi binomiale de paramètres $n=20$ et $p=0.39$.
\item (\textit{par de correction automatique disponible pour le résultat final}
\[
P(X = 10) = \coefBino{17}{10}\times 0.83^{10} \times 0.17^{7}
P(X = 11) = \coefBino{20}{11}\times 0.39^{11} \times 0.61^{9}
\]
\item (\textit{par de correction automatique disponible pour le résultat final}
Il faut calculer la probabilité qu'il y ait 0 stylo avec un defaut.
\[
P(X = 0) = \coefBino{17}{0}\times 0.83^{0} \times 0.17^{17}
P(X = 0) = \coefBino{20}{0}\times 0.39^{0} \times 0.61^{20}
\]
Puis comparer ce nombre à 0,5.
\item Il faut calculer l'espérance
\[
E[X] = n\times p = 17 \times 0.83 = 14.11
E[X] = n\times p = 20 \times 0.39 = 7.8
\]
\end{enumerate}
\end{solution}

View File

@ -3,7 +3,7 @@
% Title Page
\title{DM1 \hfill PERDRIX Camille}
\tribe{Maths complémentaire}
\tribe{Maths complémentaires}
\date{\hfillÀ render pour le jeudi 27 mai}
\xsimsetup{
@ -17,7 +17,7 @@ Les valeurs des exercices sont générés automatiquement. Si une valeur a un no
\begin{exercise}[subtitle={Optimisation de matière}]
\begin{minipage}{0.6\textwidth}
On se propose de fabriquer avec le moins de tôle possible une citerne fermée en forme de parallélépipède rectangle dont le volume intérieur doit être de $8m^3$. La longueur est aussi fixée à $4m$ par le cahier des charges.
On se propose de fabriquer avec le moins de tôle possible une citerne fermée en forme de parallélépipède rectangle dont le volume intérieur doit être de $30m^3$. La longueur est aussi fixée à $5m$ par le cahier des charges.
On peut donc faire varier uniquement la largeur (notée $x$) et la hauteur (notée $h$) de la cuve.
\end{minipage}
@ -28,25 +28,25 @@ Les valeurs des exercices sont générés automatiquement. Si une valeur a un no
\pgfmathsetmacro{\cubey}{1}
\pgfmathsetmacro{\cubez}{2}
\draw[black,fill=gray] (0,0,0) -- ++(-\cubex,0,0) -- ++(0,-\cubey,0) node [midway, left] {$h$} -- ++(\cubex,0,0) node [midway, below] {$x$} -- cycle;
\draw[black,fill=gray] (0,0,0) -- ++(0,0,-\cubez) -- ++(0,-\cubey,0) -- ++(0,0,\cubez) node [midway, right] {$4m$} -- cycle;
\draw[black,fill=gray] (0,0,0) -- ++(0,0,-\cubez) -- ++(0,-\cubey,0) -- ++(0,0,\cubez) node [midway, right] {$5m$} -- cycle;
\draw[black,fill=gray] (0,0,0) -- ++(-\cubex,0,0) -- ++(0,0,-\cubez) -- ++(\cubex,0,0) -- cycle;
\end{tikzpicture}
\end{minipage}
\begin{enumerate}
\item Expliquer pourquoi quand la largeur $x$ change, la hauteur $h$ doit elle aussi changer pour respecter les contraintes.
\item Démontrer que l'on doit avoir $h = \dfrac{2}{x}$.
\item Démontrer que l'on doit avoir $h = \dfrac{6}{x}$.
\item On note $S(x)$ l'aire totale de la citerne (c'est à dire la somme des aires des six faces). Montrer que l'on peut écrire
\[
S(x) = 8x + 4 + \frac{16}{x}
S(x) = 10x + 12 + \frac{60}{x}
\]
\item Démontrer que
\[
S(x) = \frac{8x^2 + 4x + 16}{x}
S(x) = \frac{10x^2 + 12x + 60}{x}
\]
\item Démontrer que
\[
S'(x) = \frac{8x^2 - 16}{x^2}
S'(x) = \frac{10x^2 - 60}{x^2}
\]
\item En déduire le tableau de variation de $S(x)$ sur $\intOF{0}{10}$.
\item Déterminer les valeurs de $x$ et $h$ correspondant à une utilisation minimal de tôle.
@ -57,62 +57,62 @@ Les valeurs des exercices sont générés automatiquement. Si une valeur a un no
\begin{enumerate}
\item Le volume étant fixe si l'on fait varier $x$, $h$ doit aussi varier.
\begin{itemize}
\item Si $x = 2$ alors conserver un volume de $V=8$, $h$ doit être égale à $2 / 2$
\item Si $x = 3$ alors conserver un volume de $V=8$, $h$ doit être égale à $2 / 3$
\item Si $x = 2$ alors conserver un volume de $V=30$, $h$ doit être égale à $6 / 2$
\item Si $x = 3$ alors conserver un volume de $V=30$, $h$ doit être égale à $6 / 3$
\end{itemize}
\item Pour calculer le volume, on a
\begin{eqnarray*}
V &=& h\times x \times 4 \\
8 &=& h\times x \times 4 \\
x &=& \frac{8}{h\times 4} = \frac{2}{h}
V &=& h\times x \times 5 \\
30 &=& h\times x \times 5 \\
x &=& \frac{30}{h\times 5} = \frac{6}{h}
\end{eqnarray*}
\item Pour calculer la surface totale, on ajoute la surface de chaque face. On a donc le calcul suivant
\begin{eqnarray*}
S(x) &=& x\times h \times 2 + x\times4\times2 + h\times 4\times 2\\
S(x) &=& x\times \frac{2}{x} \times 2 + x\times4\times2 + \frac{2}{x}\times 4\times 2\\
S(x) &=& 8x + 4 + \frac{16}{x}
S(x) &=& x\times h \times 2 + x\times5\times2 + h\times 5\times 2\\
S(x) &=& x\times \frac{6}{x} \times 2 + x\times5\times2 + \frac{6}{x}\times 5\times 2\\
S(x) &=& 10x + 12 + \frac{60}{x}
\end{eqnarray*}
\item Pour trouver cette nouvelle forme, on met chaque élément sur le même dénominateur
\begin{eqnarray*}
S(x) &=& 8x + 4 + \frac{16}{x}\\
S(x) &=& \frac{8x\times x}{x} + \frac{4\times x}{x} + \frac{16}{x}\\
S(x) &=& \frac{8x^2 + 4x + 16}{x}
S(x) &=& 10x + 12 + \frac{60}{x}\\
S(x) &=& \frac{10x\times x}{x} + \frac{12\times x}{x} + \frac{60}{x}\\
S(x) &=& \frac{10x^2 + 12x + 60}{x}
\end{eqnarray*}
\item On retrouve la formule $\frac{u}{v}$ à dériver
\[
u(x) = 8x^2 + 4x + 16 \Rightarrow u'(x) = 16x + 4
u(x) = 10x^2 + 12x + 60 \Rightarrow u'(x) = 20x + 12
\]
\[
v(x) = x \Rightarrow v'(x) = 1
\]
Donc au numérateur on obtient
\begin{eqnarray*}
u'(x)\times v(x) - u(x)\times v'(x) &=& (16x + 4)\times x - (8x^2 + 4x + 16)\times 1\\
&=& 8x^2 - 16
u'(x)\times v(x) - u(x)\times v'(x) &=& (20x + 12)\times x - (10x^2 + 12x + 60)\times 1\\
&=& 10x^2 - 60
\end{eqnarray*}
Donc
\[
S'(x) = \frac{8x^2 - 16}{x^2}
S'(x) = \frac{10x^2 - 60}{x^2}
\]
\item Tableau de variations de $S$
\begin{itemize}
\item Valeur interdite: $x^2 = 0 \equiv x = 0$
\item Signe de $8x^2 - 16$: c'est un polynôme du 2e degré
\item Signe de $10x^2 - 60$: c'est un polynôme du 2e degré
\[
\Delta = 512 > 0
\Delta = 2400 > 0
\]
Il y a donc 2 racines
\[
x_1 = - 1.4142135623730951 \qquad
x_2 = 1.4142135623730951
x_1 = - 2.449489742783178 \qquad
x_2 = 2.449489742783178
\]
Et on sait que $8x^2 - 16$ est du signe de $a$ donc positif en dehors des racines
Et on sait que $10x^2 - 60$ est du signe de $a$ donc positif en dehors des racines
\item Le dénominateur $x^2$ est toujours positif.
\item Tableau de variations
\begin{tikzpicture}[baseline=(a.north)]
\tkzTabInit[lgt=3,espcl=3]{$x$/1,$8x^2 - 16$/1, $x^2$/1, $S'$/1, $S$/2}{$0$, $- 1.4142135623730951$, $10$}
\tkzTabInit[lgt=3,espcl=3]{$x$/1,$10x^2 - 60$/1, $x^2$/1, $S'$/1, $S$/2}{$0$, $- 2.449489742783178$, $10$}
\tkzTabLine{d,-, z, +, }
\tkzTabLine{d,+, , +, }
\tkzTabLine{d,-, z, +, }
@ -120,7 +120,7 @@ Les valeurs des exercices sont générés automatiquement. Si une valeur a un no
\end{tikzpicture}
\end{itemize}
\item On a donc une surface minimal pour $x=1.4142135623730951$ et $h = 2.8284271247461902$.
\item On a donc une surface minimal pour $x=2.449489742783178$ et $h = 14.696938456699068$.
\end{enumerate}
\end{solution}
@ -133,14 +133,14 @@ Les valeurs des exercices sont générés automatiquement. Si une valeur a un no
Le tour d'un bassin au niveau du sol présente deux axes de symétrie : laxe des abscisses et la droite déquation $x=4$. Il est obtenu par symétrie de la courbe $\mathcal{C}_f$ sur $\intFF{0}{4}$$f$ est la fonction définie par
\[
f(x) = \left(- x^{2} + 8.3 x - 5.0\right) e^{- x} + 5.0
f(x) = \left(- x^{2} + 6.1 x - 9.6\right) e^{- x} + 9.6
\]
On admet que sur $\intFF{0}{4}$ la fonction $f$ est positive.
\begin{enumerate}
\item Sur un repère, tracer l'allure de la courbe $\mathcal{C}_f$, les axes de symétries puis compléter pour dessiner la forme du bassin.
\item Montrer que la fonction $f$ admet comme primitive sur $\R$ la fonction $F$ définie par
\[
F(x) = 5.0 x + \left( x^{2} - 6.3 x - 1.3\right) e^{- x}
F(x) = 9.6 x + \left( x^{2} - 4.1 x + 5.5\right) e^{- x}
\]
\item Calculer la quantité $\ds \int_0^4 f(x) \; dx$, vous donnerez le résultat sous forme exacte. Interpréter le résultat et reportez cette quantité sur le graphique.
\item On considère que l'échelle de votre graphique est de 1unité pour 15m. Calculer l'aire du bassin. Vous donnerez un résultat arrondi au $m^2$ près.
@ -156,15 +156,14 @@ Les valeurs des exercices sont générés automatiquement. Si une valeur a un no
\tkzGrid
\tkzAxeXY
\tkzFct[domain=0:10,color=red,very thick]%
{ (-x**2 + 8.3*x - 5.0)*exp(-x) + 5.0 };
{ (-x**2 + 6.1*x - 9.6)*exp(-x) + 9.6 };
\end{tikzpicture}
\item Il faut dériver $F(x)$ et vérifier que $F'(x) = f(x)$.
\item $\ds \int_0^4 f(x) \; dx = F(4) - F(0) = 21.3 - \frac{10.5}{e^{4}}$
\item $\ds \int_0^4 f(x) \; dx = F(4) - F(0) = \frac{5.1}{e^{4}} + 32.9$
\item La quantité calculée à la question précédente se retrouve 4fois pour former le bassin. Il faut ensuite prendre en compte l'échelle, comme 1unité de longueur correspond à 15m, une unité d'air correspond à $15\times15 = 225m^2$. Ainsi l'aire du bassin est égale à
\[
(21.3 - \frac{10.5}{e^{4}})\times 4 \times 15^2 = 18997.00000
(\frac{5.1}{e^{4}} + 32.9)\times 4 \times 15^2 = 29694.00000
\]
\end{enumerate}
\end{solution}
@ -185,9 +184,9 @@ Les valeurs des exercices sont générés automatiquement. Si une valeur a un no
Deux ateliers A et B fabriquent des stylos pour une entreprise.
L'atelier A fabrique 66.0\,\% des stylos, et parmi ceux-là, 35.0\,\% possèdent un défaut de fabrication.
L'atelier A fabrique 12.0\,\% des stylos, et parmi ceux-là, 55.00000000000001\,\% possèdent un défaut de fabrication.
De plus, 28.000000000000004\,\% des stylos possèdent un défaut de fabrication et sortent de l'atelier B.
De plus, 81.0\,\% des stylos possèdent un défaut de fabrication et sortent de l'atelier B.
Un stylo est prélevé au hasard dans le stock de l'entreprise.
@ -241,7 +240,7 @@ Les valeurs des exercices sont générés automatiquement. Si une valeur a un no
\item
\begin{enumerate}
\item Calculer la probabilité qu'un stylo provienne de l'atelier A et possède un défaut de fabrication.
\item En déduire que la probabilité qu'un stylo possède un défaut de fabrication est de $0.51$.
\item En déduire que la probabilité qu'un stylo possède un défaut de fabrication est de $0.88$.
\end{enumerate}
\item On prélève un stylo au hasard avec un défaut. Quelle est la probabilité qu'il vienne de l'atelier A?
\end{enumerate}
@ -251,7 +250,7 @@ Les valeurs des exercices sont générés automatiquement. Si une valeur a un no
\textbf{Partie B}
\medskip
Dans cette partie, on suppose que 51.0\,\% des stylos possèdent un défaut de fabrication.
Dans cette partie, on suppose que 88.0\,\% des stylos possèdent un défaut de fabrication.
L'entreprise confectionne des paquets contenant chacun $4$~stylos.
@ -266,7 +265,7 @@ Les valeurs des exercices sont générés automatiquement. Si une valeur a un no
\begin{enumerate}
\setcounter{enumi}{4}
\item Avec quelle loi peut-on modéliser $X$. Préciser les paramètres.
\item Calculer et interpréter la probabilité $P(X = 11)$.
\item Calculer et interpréter la probabilité $P(X = 8)$.
\item Le directeur de l'entreprise affirme qu'il y a plus d'une chance sur deux qu'un paquet ne comporte aucun stylo défectueux. A-t-il raison ?
\item Combien de stylos peut-on espérer avoir en moyenne?
\end{enumerate}
@ -282,27 +281,27 @@ Les valeurs des exercices sont générés automatiquement. Si une valeur a un no
child {node {$A$}
child {node {$D$}
edge from parent
node[above] {0.35}
node[above] {0.55}
}
child {node {$\overline{D}$}
edge from parent
node[above] {0.65}
node[above] {0.45}
}
edge from parent
node[above] {0.66}
node[above] {0.12}
}
child[missing] {}
child { node {$B$}
child {node {$D$}
edge from parent
node[above] {0.81}
node[above] {0.92}
}
child {node {$\overline{D}$}
edge from parent
node[above] {0.19}
node[above] {0.08}
}
edge from parent
node[above] {0.34}
node[above] {0.88}
} ;
\end{tikzpicture}
\end{center}
@ -310,51 +309,51 @@ Les valeurs des exercices sont générés automatiquement. Si une valeur a un no
\begin{itemize}
\item Probabilité que le stylo vienne de l'atelier A
\[
P(A) = 0.66
P(A) = 0.12
\]
\item Probabilité que le stylo vienne de l'atelier B
\[
P(B) = 0.34
P(B) = 0.88
\]
\item Probabilité que le stylo ait un défaut sachant qu'il vient de l'atelier A.
\[
P_A(D) = 0.35
P_A(D) = 0.55
\]
\item Probabilité que le stylo vienne de l'atelier B et qu'il ait un défaut.
\[
P(D \cap D) = 0.28
P(D \cap D) = 0.81
\]
\end{itemize}
\item
\begin{enumerate}
\item Probabilité qu'un stylo vienne de l'atelier A et qu'il ait un defaut
\[
P(A\cap D) = P(A) \times P_A(D) = 0.66 \times 0.35 = 0.23
P(A\cap D) = P(A) \times P_A(D) = 0.12 \times 0.55 = 0.07
\]
\item Probabilité que le stylo ai un défaut de fabrication.
\[
P(D) = P(A\cap D) + P(B\cap D) = 0.23 + 0.28 = 0.51
P(D) = P(A\cap D) + P(B\cap D) = 0.07 + 0.81 = 0.88
\]
\end{enumerate}
\item Probabilité qu'il vienne de l'atelier A sachant qu'il a un defaut
\[
P_D(A) = \frac{P(A\cap D)}{P(D)} = \frac{0.23}{0.51} = 0.45
P_D(A) = \frac{P(A\cap D)}{P(D)} = \frac{0.07}{0.88} = 0.08
\]
\item $X$ peut être modélisée par une loi binomiale de paramètres $n=19$ et $p=0.51$.
\item $X$ peut être modélisée par une loi binomiale de paramètres $n=15$ et $p=0.88$.
\item (\textit{par de correction automatique disponible pour le résultat final}
\[
P(X = 11) = \coefBino{19}{11}\times 0.51^{11} \times 0.49^{8}
P(X = 8) = \coefBino{15}{8}\times 0.88^{8} \times 0.12^{7}
\]
\item (\textit{par de correction automatique disponible pour le résultat final}
Il faut calculer la probabilité qu'il y ait 0 stylo avec un defaut.
\[
P(X = 0) = \coefBino{19}{0}\times 0.51^{0} \times 0.49^{19}
P(X = 0) = \coefBino{15}{0}\times 0.88^{0} \times 0.12^{15}
\]
Puis comparer ce nombre à 0,5.
\item Il faut calculer l'espérance
\[
E[X] = n\times p = 19 \times 0.51 = 9.69
E[X] = n\times p = 15 \times 0.88 = 13.2
\]
\end{enumerate}
\end{solution}

View File

@ -3,7 +3,7 @@
% Title Page
\title{DM1 \hfill POISON Lorette}
\tribe{Maths complémentaire}
\tribe{Maths complémentaires}
\date{\hfillÀ render pour le jeudi 27 mai}
\xsimsetup{
@ -17,7 +17,7 @@ Les valeurs des exercices sont générés automatiquement. Si une valeur a un no
\begin{exercise}[subtitle={Optimisation de matière}]
\begin{minipage}{0.6\textwidth}
On se propose de fabriquer avec le moins de tôle possible une citerne fermée en forme de parallélépipède rectangle dont le volume intérieur doit être de $35m^3$. La longueur est aussi fixée à $5m$ par le cahier des charges.
On se propose de fabriquer avec le moins de tôle possible une citerne fermée en forme de parallélépipède rectangle dont le volume intérieur doit être de $16m^3$. La longueur est aussi fixée à $4m$ par le cahier des charges.
On peut donc faire varier uniquement la largeur (notée $x$) et la hauteur (notée $h$) de la cuve.
\end{minipage}
@ -28,25 +28,25 @@ Les valeurs des exercices sont générés automatiquement. Si une valeur a un no
\pgfmathsetmacro{\cubey}{1}
\pgfmathsetmacro{\cubez}{2}
\draw[black,fill=gray] (0,0,0) -- ++(-\cubex,0,0) -- ++(0,-\cubey,0) node [midway, left] {$h$} -- ++(\cubex,0,0) node [midway, below] {$x$} -- cycle;
\draw[black,fill=gray] (0,0,0) -- ++(0,0,-\cubez) -- ++(0,-\cubey,0) -- ++(0,0,\cubez) node [midway, right] {$5m$} -- cycle;
\draw[black,fill=gray] (0,0,0) -- ++(0,0,-\cubez) -- ++(0,-\cubey,0) -- ++(0,0,\cubez) node [midway, right] {$4m$} -- cycle;
\draw[black,fill=gray] (0,0,0) -- ++(-\cubex,0,0) -- ++(0,0,-\cubez) -- ++(\cubex,0,0) -- cycle;
\end{tikzpicture}
\end{minipage}
\begin{enumerate}
\item Expliquer pourquoi quand la largeur $x$ change, la hauteur $h$ doit elle aussi changer pour respecter les contraintes.
\item Démontrer que l'on doit avoir $h = \dfrac{7}{x}$.
\item Démontrer que l'on doit avoir $h = \dfrac{4}{x}$.
\item On note $S(x)$ l'aire totale de la citerne (c'est à dire la somme des aires des six faces). Montrer que l'on peut écrire
\[
S(x) = 10x + 14 + \frac{70}{x}
S(x) = 8x + 8 + \frac{32}{x}
\]
\item Démontrer que
\[
S(x) = \frac{10x^2 + 14x + 70}{x}
S(x) = \frac{8x^2 + 8x + 32}{x}
\]
\item Démontrer que
\[
S'(x) = \frac{10x^2 - 70}{x^2}
S'(x) = \frac{8x^2 - 32}{x^2}
\]
\item En déduire le tableau de variation de $S(x)$ sur $\intOF{0}{10}$.
\item Déterminer les valeurs de $x$ et $h$ correspondant à une utilisation minimal de tôle.
@ -57,62 +57,62 @@ Les valeurs des exercices sont générés automatiquement. Si une valeur a un no
\begin{enumerate}
\item Le volume étant fixe si l'on fait varier $x$, $h$ doit aussi varier.
\begin{itemize}
\item Si $x = 2$ alors conserver un volume de $V=35$, $h$ doit être égale à $7 / 2$
\item Si $x = 3$ alors conserver un volume de $V=35$, $h$ doit être égale à $7 / 3$
\item Si $x = 2$ alors conserver un volume de $V=16$, $h$ doit être égale à $4 / 2$
\item Si $x = 3$ alors conserver un volume de $V=16$, $h$ doit être égale à $4 / 3$
\end{itemize}
\item Pour calculer le volume, on a
\begin{eqnarray*}
V &=& h\times x \times 5 \\
35 &=& h\times x \times 5 \\
x &=& \frac{35}{h\times 5} = \frac{7}{h}
V &=& h\times x \times 4 \\
16 &=& h\times x \times 4 \\
x &=& \frac{16}{h\times 4} = \frac{4}{h}
\end{eqnarray*}
\item Pour calculer la surface totale, on ajoute la surface de chaque face. On a donc le calcul suivant
\begin{eqnarray*}
S(x) &=& x\times h \times 2 + x\times5\times2 + h\times 5\times 2\\
S(x) &=& x\times \frac{7}{x} \times 2 + x\times5\times2 + \frac{7}{x}\times 5\times 2\\
S(x) &=& 10x + 14 + \frac{70}{x}
S(x) &=& x\times h \times 2 + x\times4\times2 + h\times 4\times 2\\
S(x) &=& x\times \frac{4}{x} \times 2 + x\times4\times2 + \frac{4}{x}\times 4\times 2\\
S(x) &=& 8x + 8 + \frac{32}{x}
\end{eqnarray*}
\item Pour trouver cette nouvelle forme, on met chaque élément sur le même dénominateur
\begin{eqnarray*}
S(x) &=& 10x + 14 + \frac{70}{x}\\
S(x) &=& \frac{10x\times x}{x} + \frac{14\times x}{x} + \frac{70}{x}\\
S(x) &=& \frac{10x^2 + 14x + 70}{x}
S(x) &=& 8x + 8 + \frac{32}{x}\\
S(x) &=& \frac{8x\times x}{x} + \frac{8\times x}{x} + \frac{32}{x}\\
S(x) &=& \frac{8x^2 + 8x + 32}{x}
\end{eqnarray*}
\item On retrouve la formule $\frac{u}{v}$ à dériver
\[
u(x) = 10x^2 + 14x + 70 \Rightarrow u'(x) = 20x + 14
u(x) = 8x^2 + 8x + 32 \Rightarrow u'(x) = 16x + 8
\]
\[
v(x) = x \Rightarrow v'(x) = 1
\]
Donc au numérateur on obtient
\begin{eqnarray*}
u'(x)\times v(x) - u(x)\times v'(x) &=& (20x + 14)\times x - (10x^2 + 14x + 70)\times 1\\
&=& 10x^2 - 70
u'(x)\times v(x) - u(x)\times v'(x) &=& (16x + 8)\times x - (8x^2 + 8x + 32)\times 1\\
&=& 8x^2 - 32
\end{eqnarray*}
Donc
\[
S'(x) = \frac{10x^2 - 70}{x^2}
S'(x) = \frac{8x^2 - 32}{x^2}
\]
\item Tableau de variations de $S$
\begin{itemize}
\item Valeur interdite: $x^2 = 0 \equiv x = 0$
\item Signe de $10x^2 - 70$: c'est un polynôme du 2e degré
\item Signe de $8x^2 - 32$: c'est un polynôme du 2e degré
\[
\Delta = 2800 > 0
\Delta = 1024 > 0
\]
Il y a donc 2 racines
\[
x_1 = - 2.6457513110645907 \qquad
x_2 = 2.6457513110645907
x_1 = - 2 \qquad
x_2 = 2
\]
Et on sait que $10x^2 - 70$ est du signe de $a$ donc positif en dehors des racines
Et on sait que $8x^2 - 32$ est du signe de $a$ donc positif en dehors des racines
\item Le dénominateur $x^2$ est toujours positif.
\item Tableau de variations
\begin{tikzpicture}[baseline=(a.north)]
\tkzTabInit[lgt=3,espcl=3]{$x$/1,$10x^2 - 70$/1, $x^2$/1, $S'$/1, $S$/2}{$0$, $- 2.6457513110645907$, $10$}
\tkzTabInit[lgt=3,espcl=3]{$x$/1,$8x^2 - 32$/1, $x^2$/1, $S'$/1, $S$/2}{$0$, $- 2$, $10$}
\tkzTabLine{d,-, z, +, }
\tkzTabLine{d,+, , +, }
\tkzTabLine{d,-, z, +, }
@ -120,7 +120,7 @@ Les valeurs des exercices sont générés automatiquement. Si une valeur a un no
\end{tikzpicture}
\end{itemize}
\item On a donc une surface minimal pour $x=2.6457513110645907$ et $h = 18.5202591774521349$.
\item On a donc une surface minimal pour $x=2$ et $h = 8$.
\end{enumerate}
\end{solution}
@ -133,14 +133,14 @@ Les valeurs des exercices sont générés automatiquement. Si une valeur a un no
Le tour d'un bassin au niveau du sol présente deux axes de symétrie : laxe des abscisses et la droite déquation $x=4$. Il est obtenu par symétrie de la courbe $\mathcal{C}_f$ sur $\intFF{0}{4}$$f$ est la fonction définie par
\[
f(x) = \left(- x^{2} + 6.1 x - 3.4\right) e^{- x} + 3.4
f(x) = \left(- x^{2} + 6.8 x - 6.4\right) e^{- x} + 6.4
\]
On admet que sur $\intFF{0}{4}$ la fonction $f$ est positive.
\begin{enumerate}
\item Sur un repère, tracer l'allure de la courbe $\mathcal{C}_f$, les axes de symétries puis compléter pour dessiner la forme du bassin.
\item Montrer que la fonction $f$ admet comme primitive sur $\R$ la fonction $F$ définie par
\[
F(x) = 3.4 x + \left( x^{2} - 4.1 x - 0.7\right) e^{- x}
F(x) = 6.4 x + \left( x^{2} - 4.8 x + 1.6\right) e^{- x}
\]
\item Calculer la quantité $\ds \int_0^4 f(x) \; dx$, vous donnerez le résultat sous forme exacte. Interpréter le résultat et reportez cette quantité sur le graphique.
\item On considère que l'échelle de votre graphique est de 1unité pour 15m. Calculer l'aire du bassin. Vous donnerez un résultat arrondi au $m^2$ près.
@ -156,15 +156,14 @@ Les valeurs des exercices sont générés automatiquement. Si une valeur a un no
\tkzGrid
\tkzAxeXY
\tkzFct[domain=0:10,color=red,very thick]%
{ (-x**2 + 6.1*x - 3.4)*exp(-x) + 3.4 };
{ (-x**2 + 6.8*x - 6.4)*exp(-x) + 6.4 };
\end{tikzpicture}
\item Il faut dériver $F(x)$ et vérifier que $F'(x) = f(x)$.
\item $\ds \int_0^4 f(x) \; dx = F(4) - F(0) = 14.3 - \frac{1.1}{e^{4}}$
\item $\ds \int_0^4 f(x) \; dx = F(4) - F(0) = 24.0 - \frac{1.6}{e^{4}}$
\item La quantité calculée à la question précédente se retrouve 4fois pour former le bassin. Il faut ensuite prendre en compte l'échelle, comme 1unité de longueur correspond à 15m, une unité d'air correspond à $15\times15 = 225m^2$. Ainsi l'aire du bassin est égale à
\[
(14.3 - \frac{1.1}{e^{4}})\times 4 \times 15^2 = 12852.00000
(24.0 - \frac{1.6}{e^{4}})\times 4 \times 15^2 = 21574.00000
\]
\end{enumerate}
\end{solution}
@ -185,9 +184,9 @@ Les valeurs des exercices sont générés automatiquement. Si une valeur a un no
Deux ateliers A et B fabriquent des stylos pour une entreprise.
L'atelier A fabrique 47.0\,\% des stylos, et parmi ceux-là, 95.0\,\% possèdent un défaut de fabrication.
L'atelier A fabrique 61.0\,\% des stylos, et parmi ceux-là, 21.0\,\% possèdent un défaut de fabrication.
De plus, 50.0\,\% des stylos possèdent un défaut de fabrication et sortent de l'atelier B.
De plus, 37.0\,\% des stylos possèdent un défaut de fabrication et sortent de l'atelier B.
Un stylo est prélevé au hasard dans le stock de l'entreprise.
@ -241,7 +240,7 @@ Les valeurs des exercices sont générés automatiquement. Si une valeur a un no
\item
\begin{enumerate}
\item Calculer la probabilité qu'un stylo provienne de l'atelier A et possède un défaut de fabrication.
\item En déduire que la probabilité qu'un stylo possède un défaut de fabrication est de $0.95$.
\item En déduire que la probabilité qu'un stylo possède un défaut de fabrication est de $0.5$.
\end{enumerate}
\item On prélève un stylo au hasard avec un défaut. Quelle est la probabilité qu'il vienne de l'atelier A?
\end{enumerate}
@ -251,7 +250,7 @@ Les valeurs des exercices sont générés automatiquement. Si une valeur a un no
\textbf{Partie B}
\medskip
Dans cette partie, on suppose que 95.0\,\% des stylos possèdent un défaut de fabrication.
Dans cette partie, on suppose que 50.0\,\% des stylos possèdent un défaut de fabrication.
L'entreprise confectionne des paquets contenant chacun $4$~stylos.
@ -266,7 +265,7 @@ Les valeurs des exercices sont générés automatiquement. Si une valeur a un no
\begin{enumerate}
\setcounter{enumi}{4}
\item Avec quelle loi peut-on modéliser $X$. Préciser les paramètres.
\item Calculer et interpréter la probabilité $P(X = 12)$.
\item Calculer et interpréter la probabilité $P(X = 10)$.
\item Le directeur de l'entreprise affirme qu'il y a plus d'une chance sur deux qu'un paquet ne comporte aucun stylo défectueux. A-t-il raison ?
\item Combien de stylos peut-on espérer avoir en moyenne?
\end{enumerate}
@ -282,27 +281,27 @@ Les valeurs des exercices sont générés automatiquement. Si une valeur a un no
child {node {$A$}
child {node {$D$}
edge from parent
node[above] {0.95}
node[above] {0.21}
}
child {node {$\overline{D}$}
edge from parent
node[above] {0.05}
node[above] {0.79}
}
edge from parent
node[above] {0.47}
node[above] {0.61}
}
child[missing] {}
child { node {$B$}
child {node {$D$}
edge from parent
node[above] {0.95}
node[above] {0.96}
}
child {node {$\overline{D}$}
edge from parent
node[above] {0.05}
node[above] {0.04}
}
edge from parent
node[above] {0.53}
node[above] {0.39}
} ;
\end{tikzpicture}
\end{center}
@ -310,51 +309,51 @@ Les valeurs des exercices sont générés automatiquement. Si une valeur a un no
\begin{itemize}
\item Probabilité que le stylo vienne de l'atelier A
\[
P(A) = 0.47
P(A) = 0.61
\]
\item Probabilité que le stylo vienne de l'atelier B
\[
P(B) = 0.53
P(B) = 0.39
\]
\item Probabilité que le stylo ait un défaut sachant qu'il vient de l'atelier A.
\[
P_A(D) = 0.95
P_A(D) = 0.21
\]
\item Probabilité que le stylo vienne de l'atelier B et qu'il ait un défaut.
\[
P(D \cap D) = 0.5
P(D \cap D) = 0.37
\]
\end{itemize}
\item
\begin{enumerate}
\item Probabilité qu'un stylo vienne de l'atelier A et qu'il ait un defaut
\[
P(A\cap D) = P(A) \times P_A(D) = 0.47 \times 0.95 = 0.45
P(A\cap D) = P(A) \times P_A(D) = 0.61 \times 0.21 = 0.13
\]
\item Probabilité que le stylo ai un défaut de fabrication.
\[
P(D) = P(A\cap D) + P(B\cap D) = 0.45 + 0.5 = 0.95
P(D) = P(A\cap D) + P(B\cap D) = 0.13 + 0.37 = 0.5
\]
\end{enumerate}
\item Probabilité qu'il vienne de l'atelier A sachant qu'il a un defaut
\[
P_D(A) = \frac{P(A\cap D)}{P(D)} = \frac{0.45}{0.95} = 0.47
P_D(A) = \frac{P(A\cap D)}{P(D)} = \frac{0.13}{0.5} = 0.26
\]
\item $X$ peut être modélisée par une loi binomiale de paramètres $n=15$ et $p=0.95$.
\item $X$ peut être modélisée par une loi binomiale de paramètres $n=10$ et $p=0.5$.
\item (\textit{par de correction automatique disponible pour le résultat final}
\[
P(X = 12) = \coefBino{15}{12}\times 0.95^{12} \times 0.05^{3}
P(X = 10) = \coefBino{10}{10}\times 0.5^{10} \times 0.5^{0}
\]
\item (\textit{par de correction automatique disponible pour le résultat final}
Il faut calculer la probabilité qu'il y ait 0 stylo avec un defaut.
\[
P(X = 0) = \coefBino{15}{0}\times 0.95^{0} \times 0.05^{15}
P(X = 0) = \coefBino{10}{0}\times 0.5^{0} \times 0.5^{10}
\]
Puis comparer ce nombre à 0,5.
\item Il faut calculer l'espérance
\[
E[X] = n\times p = 15 \times 0.95 = 14.25
E[X] = n\times p = 10 \times 0.5 = 5.0
\]
\end{enumerate}
\end{solution}

View File

@ -3,7 +3,7 @@
% Title Page
\title{DM1 \hfill RODRIGUEZ Teddy}
\tribe{Maths complémentaire}
\tribe{Maths complémentaires}
\date{\hfillÀ render pour le jeudi 27 mai}
\xsimsetup{
@ -17,7 +17,7 @@ Les valeurs des exercices sont générés automatiquement. Si une valeur a un no
\begin{exercise}[subtitle={Optimisation de matière}]
\begin{minipage}{0.6\textwidth}
On se propose de fabriquer avec le moins de tôle possible une citerne fermée en forme de parallélépipède rectangle dont le volume intérieur doit être de $50m^3$. La longueur est aussi fixée à $5m$ par le cahier des charges.
On se propose de fabriquer avec le moins de tôle possible une citerne fermée en forme de parallélépipède rectangle dont le volume intérieur doit être de $24m^3$. La longueur est aussi fixée à $3m$ par le cahier des charges.
On peut donc faire varier uniquement la largeur (notée $x$) et la hauteur (notée $h$) de la cuve.
\end{minipage}
@ -28,25 +28,25 @@ Les valeurs des exercices sont générés automatiquement. Si une valeur a un no
\pgfmathsetmacro{\cubey}{1}
\pgfmathsetmacro{\cubez}{2}
\draw[black,fill=gray] (0,0,0) -- ++(-\cubex,0,0) -- ++(0,-\cubey,0) node [midway, left] {$h$} -- ++(\cubex,0,0) node [midway, below] {$x$} -- cycle;
\draw[black,fill=gray] (0,0,0) -- ++(0,0,-\cubez) -- ++(0,-\cubey,0) -- ++(0,0,\cubez) node [midway, right] {$5m$} -- cycle;
\draw[black,fill=gray] (0,0,0) -- ++(0,0,-\cubez) -- ++(0,-\cubey,0) -- ++(0,0,\cubez) node [midway, right] {$3m$} -- cycle;
\draw[black,fill=gray] (0,0,0) -- ++(-\cubex,0,0) -- ++(0,0,-\cubez) -- ++(\cubex,0,0) -- cycle;
\end{tikzpicture}
\end{minipage}
\begin{enumerate}
\item Expliquer pourquoi quand la largeur $x$ change, la hauteur $h$ doit elle aussi changer pour respecter les contraintes.
\item Démontrer que l'on doit avoir $h = \dfrac{10}{x}$.
\item Démontrer que l'on doit avoir $h = \dfrac{8}{x}$.
\item On note $S(x)$ l'aire totale de la citerne (c'est à dire la somme des aires des six faces). Montrer que l'on peut écrire
\[
S(x) = 10x + 20 + \frac{100}{x}
S(x) = 6x + 16 + \frac{48}{x}
\]
\item Démontrer que
\[
S(x) = \frac{10x^2 + 20x + 100}{x}
S(x) = \frac{6x^2 + 16x + 48}{x}
\]
\item Démontrer que
\[
S'(x) = \frac{10x^2 - 100}{x^2}
S'(x) = \frac{6x^2 - 48}{x^2}
\]
\item En déduire le tableau de variation de $S(x)$ sur $\intOF{0}{10}$.
\item Déterminer les valeurs de $x$ et $h$ correspondant à une utilisation minimal de tôle.
@ -57,62 +57,62 @@ Les valeurs des exercices sont générés automatiquement. Si une valeur a un no
\begin{enumerate}
\item Le volume étant fixe si l'on fait varier $x$, $h$ doit aussi varier.
\begin{itemize}
\item Si $x = 2$ alors conserver un volume de $V=50$, $h$ doit être égale à $10 / 2$
\item Si $x = 3$ alors conserver un volume de $V=50$, $h$ doit être égale à $10 / 3$
\item Si $x = 2$ alors conserver un volume de $V=24$, $h$ doit être égale à $8 / 2$
\item Si $x = 3$ alors conserver un volume de $V=24$, $h$ doit être égale à $8 / 3$
\end{itemize}
\item Pour calculer le volume, on a
\begin{eqnarray*}
V &=& h\times x \times 5 \\
50 &=& h\times x \times 5 \\
x &=& \frac{50}{h\times 5} = \frac{10}{h}
V &=& h\times x \times 3 \\
24 &=& h\times x \times 3 \\
x &=& \frac{24}{h\times 3} = \frac{8}{h}
\end{eqnarray*}
\item Pour calculer la surface totale, on ajoute la surface de chaque face. On a donc le calcul suivant
\begin{eqnarray*}
S(x) &=& x\times h \times 2 + x\times5\times2 + h\times 5\times 2\\
S(x) &=& x\times \frac{10}{x} \times 2 + x\times5\times2 + \frac{10}{x}\times 5\times 2\\
S(x) &=& 10x + 20 + \frac{100}{x}
S(x) &=& x\times h \times 2 + x\times3\times2 + h\times 3\times 2\\
S(x) &=& x\times \frac{8}{x} \times 2 + x\times3\times2 + \frac{8}{x}\times 3\times 2\\
S(x) &=& 6x + 16 + \frac{48}{x}
\end{eqnarray*}
\item Pour trouver cette nouvelle forme, on met chaque élément sur le même dénominateur
\begin{eqnarray*}
S(x) &=& 10x + 20 + \frac{100}{x}\\
S(x) &=& \frac{10x\times x}{x} + \frac{20\times x}{x} + \frac{100}{x}\\
S(x) &=& \frac{10x^2 + 20x + 100}{x}
S(x) &=& 6x + 16 + \frac{48}{x}\\
S(x) &=& \frac{6x\times x}{x} + \frac{16\times x}{x} + \frac{48}{x}\\
S(x) &=& \frac{6x^2 + 16x + 48}{x}
\end{eqnarray*}
\item On retrouve la formule $\frac{u}{v}$ à dériver
\[
u(x) = 10x^2 + 20x + 100 \Rightarrow u'(x) = 20x + 20
u(x) = 6x^2 + 16x + 48 \Rightarrow u'(x) = 12x + 16
\]
\[
v(x) = x \Rightarrow v'(x) = 1
\]
Donc au numérateur on obtient
\begin{eqnarray*}
u'(x)\times v(x) - u(x)\times v'(x) &=& (20x + 20)\times x - (10x^2 + 20x + 100)\times 1\\
&=& 10x^2 - 100
u'(x)\times v(x) - u(x)\times v'(x) &=& (12x + 16)\times x - (6x^2 + 16x + 48)\times 1\\
&=& 6x^2 - 48
\end{eqnarray*}
Donc
\[
S'(x) = \frac{10x^2 - 100}{x^2}
S'(x) = \frac{6x^2 - 48}{x^2}
\]
\item Tableau de variations de $S$
\begin{itemize}
\item Valeur interdite: $x^2 = 0 \equiv x = 0$
\item Signe de $10x^2 - 100$: c'est un polynôme du 2e degré
\item Signe de $6x^2 - 48$: c'est un polynôme du 2e degré
\[
\Delta = 4000 > 0
\Delta = 1152 > 0
\]
Il y a donc 2 racines
\[
x_1 = - 3.162277660168379 \qquad
x_2 = 3.162277660168379
x_1 = - 2.82842712474619 \qquad
x_2 = 2.82842712474619
\]
Et on sait que $10x^2 - 100$ est du signe de $a$ donc positif en dehors des racines
Et on sait que $6x^2 - 48$ est du signe de $a$ donc positif en dehors des racines
\item Le dénominateur $x^2$ est toujours positif.
\item Tableau de variations
\begin{tikzpicture}[baseline=(a.north)]
\tkzTabInit[lgt=3,espcl=3]{$x$/1,$10x^2 - 100$/1, $x^2$/1, $S'$/1, $S$/2}{$0$, $- 3.162277660168379$, $10$}
\tkzTabInit[lgt=3,espcl=3]{$x$/1,$6x^2 - 48$/1, $x^2$/1, $S'$/1, $S$/2}{$0$, $- 2.82842712474619$, $10$}
\tkzTabLine{d,-, z, +, }
\tkzTabLine{d,+, , +, }
\tkzTabLine{d,-, z, +, }
@ -120,7 +120,7 @@ Les valeurs des exercices sont générés automatiquement. Si une valeur a un no
\end{tikzpicture}
\end{itemize}
\item On a donc une surface minimal pour $x=3.162277660168379$ et $h = 31.622776601683790$.
\item On a donc une surface minimal pour $x=2.82842712474619$ et $h = 22.62741699796952$.
\end{enumerate}
\end{solution}
@ -133,14 +133,14 @@ Les valeurs des exercices sont générés automatiquement. Si une valeur a un no
Le tour d'un bassin au niveau du sol présente deux axes de symétrie : laxe des abscisses et la droite déquation $x=4$. Il est obtenu par symétrie de la courbe $\mathcal{C}_f$ sur $\intFF{0}{4}$$f$ est la fonction définie par
\[
f(x) = \left(- x^{2} + 1.0 x - 3.6\right) e^{- x} + 3.6
f(x) = \left(- x^{2} + 3.8 x - 4.0\right) e^{- x} + 4.0
\]
On admet que sur $\intFF{0}{4}$ la fonction $f$ est positive.
\begin{enumerate}
\item Sur un repère, tracer l'allure de la courbe $\mathcal{C}_f$, les axes de symétries puis compléter pour dessiner la forme du bassin.
\item Montrer que la fonction $f$ admet comme primitive sur $\R$ la fonction $F$ définie par
\[
F(x) = 3.6 x + \left( x^{2} + x + 4.6\right) e^{- x}
F(x) = 4.0 x + \left( x^{2} - 1.8 x + 2.2\right) e^{- x}
\]
\item Calculer la quantité $\ds \int_0^4 f(x) \; dx$, vous donnerez le résultat sous forme exacte. Interpréter le résultat et reportez cette quantité sur le graphique.
\item On considère que l'échelle de votre graphique est de 1unité pour 15m. Calculer l'aire du bassin. Vous donnerez un résultat arrondi au $m^2$ près.
@ -156,15 +156,14 @@ Les valeurs des exercices sont générés automatiquement. Si une valeur a un no
\tkzGrid
\tkzAxeXY
\tkzFct[domain=0:10,color=red,very thick]%
{ (-x**2 + 1.0*x - 3.6)*exp(-x) + 3.6 };
{ (-x**2 + 3.8*x - 4.0)*exp(-x) + 4.0 };
\end{tikzpicture}
\item Il faut dériver $F(x)$ et vérifier que $F'(x) = f(x)$.
\item $\ds \int_0^4 f(x) \; dx = F(4) - F(0) = \frac{24.6}{e^{4}} + 9.8$
\item $\ds \int_0^4 f(x) \; dx = F(4) - F(0) = \frac{11.0}{e^{4}} + 13.8$
\item La quantité calculée à la question précédente se retrouve 4fois pour former le bassin. Il faut ensuite prendre en compte l'échelle, comme 1unité de longueur correspond à 15m, une unité d'air correspond à $15\times15 = 225m^2$. Ainsi l'aire du bassin est égale à
\[
(\frac{24.6}{e^{4}} + 9.8)\times 4 \times 15^2 = 9226.000000
(\frac{11.0}{e^{4}} + 13.8)\times 4 \times 15^2 = 12601.00000
\]
\end{enumerate}
\end{solution}
@ -185,9 +184,9 @@ Les valeurs des exercices sont générés automatiquement. Si une valeur a un no
Deux ateliers A et B fabriquent des stylos pour une entreprise.
L'atelier A fabrique 64.0\,\% des stylos, et parmi ceux-là, 4.0\,\% possèdent un défaut de fabrication.
L'atelier A fabrique 56.99999999999999\,\% des stylos, et parmi ceux-là, 6.0\,\% possèdent un défaut de fabrication.
De plus, 6.0\,\% des stylos possèdent un défaut de fabrication et sortent de l'atelier B.
De plus, 42.0\,\% des stylos possèdent un défaut de fabrication et sortent de l'atelier B.
Un stylo est prélevé au hasard dans le stock de l'entreprise.
@ -241,7 +240,7 @@ Les valeurs des exercices sont générés automatiquement. Si une valeur a un no
\item
\begin{enumerate}
\item Calculer la probabilité qu'un stylo provienne de l'atelier A et possède un défaut de fabrication.
\item En déduire que la probabilité qu'un stylo possède un défaut de fabrication est de $0.09$.
\item En déduire que la probabilité qu'un stylo possède un défaut de fabrication est de $0.45$.
\end{enumerate}
\item On prélève un stylo au hasard avec un défaut. Quelle est la probabilité qu'il vienne de l'atelier A?
\end{enumerate}
@ -251,7 +250,7 @@ Les valeurs des exercices sont générés automatiquement. Si une valeur a un no
\textbf{Partie B}
\medskip
Dans cette partie, on suppose que 9.0\,\% des stylos possèdent un défaut de fabrication.
Dans cette partie, on suppose que 45.0\,\% des stylos possèdent un défaut de fabrication.
L'entreprise confectionne des paquets contenant chacun $4$~stylos.
@ -266,7 +265,7 @@ Les valeurs des exercices sont générés automatiquement. Si une valeur a un no
\begin{enumerate}
\setcounter{enumi}{4}
\item Avec quelle loi peut-on modéliser $X$. Préciser les paramètres.
\item Calculer et interpréter la probabilité $P(X = 9)$.
\item Calculer et interpréter la probabilité $P(X = 8)$.
\item Le directeur de l'entreprise affirme qu'il y a plus d'une chance sur deux qu'un paquet ne comporte aucun stylo défectueux. A-t-il raison ?
\item Combien de stylos peut-on espérer avoir en moyenne?
\end{enumerate}
@ -282,27 +281,27 @@ Les valeurs des exercices sont générés automatiquement. Si une valeur a un no
child {node {$A$}
child {node {$D$}
edge from parent
node[above] {0.04}
node[above] {0.06}
}
child {node {$\overline{D}$}
edge from parent
node[above] {0.96}
node[above] {0.94}
}
edge from parent
node[above] {0.64}
node[above] {0.57}
}
child[missing] {}
child { node {$B$}
child {node {$D$}
edge from parent
node[above] {0.18}
node[above] {0.97}
}
child {node {$\overline{D}$}
edge from parent
node[above] {0.82}
node[above] {0.03}
}
edge from parent
node[above] {0.36}
node[above] {0.43}
} ;
\end{tikzpicture}
\end{center}
@ -310,51 +309,51 @@ Les valeurs des exercices sont générés automatiquement. Si une valeur a un no
\begin{itemize}
\item Probabilité que le stylo vienne de l'atelier A
\[
P(A) = 0.64
P(A) = 0.57
\]
\item Probabilité que le stylo vienne de l'atelier B
\[
P(B) = 0.36
P(B) = 0.43
\]
\item Probabilité que le stylo ait un défaut sachant qu'il vient de l'atelier A.
\[
P_A(D) = 0.04
P_A(D) = 0.06
\]
\item Probabilité que le stylo vienne de l'atelier B et qu'il ait un défaut.
\[
P(D \cap D) = 0.06
P(D \cap D) = 0.42
\]
\end{itemize}
\item
\begin{enumerate}
\item Probabilité qu'un stylo vienne de l'atelier A et qu'il ait un defaut
\[
P(A\cap D) = P(A) \times P_A(D) = 0.64 \times 0.04 = 0.03
P(A\cap D) = P(A) \times P_A(D) = 0.57 \times 0.06 = 0.03
\]
\item Probabilité que le stylo ai un défaut de fabrication.
\[
P(D) = P(A\cap D) + P(B\cap D) = 0.03 + 0.06 = 0.09
P(D) = P(A\cap D) + P(B\cap D) = 0.03 + 0.42 = 0.45
\]
\end{enumerate}
\item Probabilité qu'il vienne de l'atelier A sachant qu'il a un defaut
\[
P_D(A) = \frac{P(A\cap D)}{P(D)} = \frac{0.03}{0.09} = 0.33
P_D(A) = \frac{P(A\cap D)}{P(D)} = \frac{0.03}{0.45} = 0.07
\]
\item $X$ peut être modélisée par une loi binomiale de paramètres $n=16$ et $p=0.09$.
\item $X$ peut être modélisée par une loi binomiale de paramètres $n=11$ et $p=0.45$.
\item (\textit{par de correction automatique disponible pour le résultat final}
\[
P(X = 9) = \coefBino{16}{9}\times 0.09^{9} \times 0.91^{7}
P(X = 8) = \coefBino{11}{8}\times 0.45^{8} \times 0.55^{3}
\]
\item (\textit{par de correction automatique disponible pour le résultat final}
Il faut calculer la probabilité qu'il y ait 0 stylo avec un defaut.
\[
P(X = 0) = \coefBino{16}{0}\times 0.09^{0} \times 0.91^{16}
P(X = 0) = \coefBino{11}{0}\times 0.45^{0} \times 0.55^{11}
\]
Puis comparer ce nombre à 0,5.
\item Il faut calculer l'espérance
\[
E[X] = n\times p = 16 \times 0.09 = 1.44
E[X] = n\times p = 11 \times 0.45 = 4.95
\]
\end{enumerate}
\end{solution}

View File

@ -3,7 +3,7 @@
% Title Page
\title{DM1 \hfill SAINT CYR Louis}
\tribe{Maths complémentaire}
\tribe{Maths complémentaires}
\date{\hfillÀ render pour le jeudi 27 mai}
\xsimsetup{
@ -17,7 +17,7 @@ Les valeurs des exercices sont générés automatiquement. Si une valeur a un no
\begin{exercise}[subtitle={Optimisation de matière}]
\begin{minipage}{0.6\textwidth}
On se propose de fabriquer avec le moins de tôle possible une citerne fermée en forme de parallélépipède rectangle dont le volume intérieur doit être de $45m^3$. La longueur est aussi fixée à $5m$ par le cahier des charges.
On se propose de fabriquer avec le moins de tôle possible une citerne fermée en forme de parallélépipède rectangle dont le volume intérieur doit être de $8m^3$. La longueur est aussi fixée à $2m$ par le cahier des charges.
On peut donc faire varier uniquement la largeur (notée $x$) et la hauteur (notée $h$) de la cuve.
\end{minipage}
@ -28,25 +28,25 @@ Les valeurs des exercices sont générés automatiquement. Si une valeur a un no
\pgfmathsetmacro{\cubey}{1}
\pgfmathsetmacro{\cubez}{2}
\draw[black,fill=gray] (0,0,0) -- ++(-\cubex,0,0) -- ++(0,-\cubey,0) node [midway, left] {$h$} -- ++(\cubex,0,0) node [midway, below] {$x$} -- cycle;
\draw[black,fill=gray] (0,0,0) -- ++(0,0,-\cubez) -- ++(0,-\cubey,0) -- ++(0,0,\cubez) node [midway, right] {$5m$} -- cycle;
\draw[black,fill=gray] (0,0,0) -- ++(0,0,-\cubez) -- ++(0,-\cubey,0) -- ++(0,0,\cubez) node [midway, right] {$2m$} -- cycle;
\draw[black,fill=gray] (0,0,0) -- ++(-\cubex,0,0) -- ++(0,0,-\cubez) -- ++(\cubex,0,0) -- cycle;
\end{tikzpicture}
\end{minipage}
\begin{enumerate}
\item Expliquer pourquoi quand la largeur $x$ change, la hauteur $h$ doit elle aussi changer pour respecter les contraintes.
\item Démontrer que l'on doit avoir $h = \dfrac{9}{x}$.
\item Démontrer que l'on doit avoir $h = \dfrac{4}{x}$.
\item On note $S(x)$ l'aire totale de la citerne (c'est à dire la somme des aires des six faces). Montrer que l'on peut écrire
\[
S(x) = 10x + 18 + \frac{90}{x}
S(x) = 4x + 8 + \frac{16}{x}
\]
\item Démontrer que
\[
S(x) = \frac{10x^2 + 18x + 90}{x}
S(x) = \frac{4x^2 + 8x + 16}{x}
\]
\item Démontrer que
\[
S'(x) = \frac{10x^2 - 90}{x^2}
S'(x) = \frac{4x^2 - 16}{x^2}
\]
\item En déduire le tableau de variation de $S(x)$ sur $\intOF{0}{10}$.
\item Déterminer les valeurs de $x$ et $h$ correspondant à une utilisation minimal de tôle.
@ -57,62 +57,62 @@ Les valeurs des exercices sont générés automatiquement. Si une valeur a un no
\begin{enumerate}
\item Le volume étant fixe si l'on fait varier $x$, $h$ doit aussi varier.
\begin{itemize}
\item Si $x = 2$ alors conserver un volume de $V=45$, $h$ doit être égale à $9 / 2$
\item Si $x = 3$ alors conserver un volume de $V=45$, $h$ doit être égale à $9 / 3$
\item Si $x = 2$ alors conserver un volume de $V=8$, $h$ doit être égale à $4 / 2$
\item Si $x = 3$ alors conserver un volume de $V=8$, $h$ doit être égale à $4 / 3$
\end{itemize}
\item Pour calculer le volume, on a
\begin{eqnarray*}
V &=& h\times x \times 5 \\
45 &=& h\times x \times 5 \\
x &=& \frac{45}{h\times 5} = \frac{9}{h}
V &=& h\times x \times 2 \\
8 &=& h\times x \times 2 \\
x &=& \frac{8}{h\times 2} = \frac{4}{h}
\end{eqnarray*}
\item Pour calculer la surface totale, on ajoute la surface de chaque face. On a donc le calcul suivant
\begin{eqnarray*}
S(x) &=& x\times h \times 2 + x\times5\times2 + h\times 5\times 2\\
S(x) &=& x\times \frac{9}{x} \times 2 + x\times5\times2 + \frac{9}{x}\times 5\times 2\\
S(x) &=& 10x + 18 + \frac{90}{x}
S(x) &=& x\times h \times 2 + x\times2\times2 + h\times 2\times 2\\
S(x) &=& x\times \frac{4}{x} \times 2 + x\times2\times2 + \frac{4}{x}\times 2\times 2\\
S(x) &=& 4x + 8 + \frac{16}{x}
\end{eqnarray*}
\item Pour trouver cette nouvelle forme, on met chaque élément sur le même dénominateur
\begin{eqnarray*}
S(x) &=& 10x + 18 + \frac{90}{x}\\
S(x) &=& \frac{10x\times x}{x} + \frac{18\times x}{x} + \frac{90}{x}\\
S(x) &=& \frac{10x^2 + 18x + 90}{x}
S(x) &=& 4x + 8 + \frac{16}{x}\\
S(x) &=& \frac{4x\times x}{x} + \frac{8\times x}{x} + \frac{16}{x}\\
S(x) &=& \frac{4x^2 + 8x + 16}{x}
\end{eqnarray*}
\item On retrouve la formule $\frac{u}{v}$ à dériver
\[
u(x) = 10x^2 + 18x + 90 \Rightarrow u'(x) = 20x + 18
u(x) = 4x^2 + 8x + 16 \Rightarrow u'(x) = 8x + 8
\]
\[
v(x) = x \Rightarrow v'(x) = 1
\]
Donc au numérateur on obtient
\begin{eqnarray*}
u'(x)\times v(x) - u(x)\times v'(x) &=& (20x + 18)\times x - (10x^2 + 18x + 90)\times 1\\
&=& 10x^2 - 90
u'(x)\times v(x) - u(x)\times v'(x) &=& (8x + 8)\times x - (4x^2 + 8x + 16)\times 1\\
&=& 4x^2 - 16
\end{eqnarray*}
Donc
\[
S'(x) = \frac{10x^2 - 90}{x^2}
S'(x) = \frac{4x^2 - 16}{x^2}
\]
\item Tableau de variations de $S$
\begin{itemize}
\item Valeur interdite: $x^2 = 0 \equiv x = 0$
\item Signe de $10x^2 - 90$: c'est un polynôme du 2e degré
\item Signe de $4x^2 - 16$: c'est un polynôme du 2e degré
\[
\Delta = 3600 > 0
\Delta = 256 > 0
\]
Il y a donc 2 racines
\[
x_1 = - 3 \qquad
x_2 = 3
x_1 = - 2 \qquad
x_2 = 2
\]
Et on sait que $10x^2 - 90$ est du signe de $a$ donc positif en dehors des racines
Et on sait que $4x^2 - 16$ est du signe de $a$ donc positif en dehors des racines
\item Le dénominateur $x^2$ est toujours positif.
\item Tableau de variations
\begin{tikzpicture}[baseline=(a.north)]
\tkzTabInit[lgt=3,espcl=3]{$x$/1,$10x^2 - 90$/1, $x^2$/1, $S'$/1, $S$/2}{$0$, $- 3$, $10$}
\tkzTabInit[lgt=3,espcl=3]{$x$/1,$4x^2 - 16$/1, $x^2$/1, $S'$/1, $S$/2}{$0$, $- 2$, $10$}
\tkzTabLine{d,-, z, +, }
\tkzTabLine{d,+, , +, }
\tkzTabLine{d,-, z, +, }
@ -120,7 +120,7 @@ Les valeurs des exercices sont générés automatiquement. Si une valeur a un no
\end{tikzpicture}
\end{itemize}
\item On a donc une surface minimal pour $x=3$ et $h = 27$.
\item On a donc une surface minimal pour $x=2$ et $h = 8$.
\end{enumerate}
\end{solution}
@ -133,14 +133,14 @@ Les valeurs des exercices sont générés automatiquement. Si une valeur a un no
Le tour d'un bassin au niveau du sol présente deux axes de symétrie : laxe des abscisses et la droite déquation $x=4$. Il est obtenu par symétrie de la courbe $\mathcal{C}_f$ sur $\intFF{0}{4}$$f$ est la fonction définie par
\[
f(x) = \left(- x^{2} + 8.0 x - 0.9\right) e^{- x} + 0.9
f(x) = \left(- x^{2} + 2.5 x - 6.7\right) e^{- x} + 6.7
\]
On admet que sur $\intFF{0}{4}$ la fonction $f$ est positive.
\begin{enumerate}
\item Sur un repère, tracer l'allure de la courbe $\mathcal{C}_f$, les axes de symétries puis compléter pour dessiner la forme du bassin.
\item Montrer que la fonction $f$ admet comme primitive sur $\R$ la fonction $F$ définie par
\[
F(x) = 0.9 x + \left( x^{2} - 6.0 x - 5.1\right) e^{- x}
F(x) = 6.7 x + \left( x^{2} - 0.5 x + 6.2\right) e^{- x}
\]
\item Calculer la quantité $\ds \int_0^4 f(x) \; dx$, vous donnerez le résultat sous forme exacte. Interpréter le résultat et reportez cette quantité sur le graphique.
\item On considère que l'échelle de votre graphique est de 1unité pour 15m. Calculer l'aire du bassin. Vous donnerez un résultat arrondi au $m^2$ près.
@ -156,15 +156,14 @@ Les valeurs des exercices sont générés automatiquement. Si une valeur a un no
\tkzGrid
\tkzAxeXY
\tkzFct[domain=0:10,color=red,very thick]%
{ (-x**2 + 8.0*x - 0.9)*exp(-x) + 0.9 };
{ (-x**2 + 2.5*x - 6.7)*exp(-x) + 6.7 };
\end{tikzpicture}
\item Il faut dériver $F(x)$ et vérifier que $F'(x) = f(x)$.
\item $\ds \int_0^4 f(x) \; dx = F(4) - F(0) = 8.7 - \frac{13.1}{e^{4}}$
\item $\ds \int_0^4 f(x) \; dx = F(4) - F(0) = \frac{20.2}{e^{4}} + 20.6$
\item La quantité calculée à la question précédente se retrouve 4fois pour former le bassin. Il faut ensuite prendre en compte l'échelle, comme 1unité de longueur correspond à 15m, une unité d'air correspond à $15\times15 = 225m^2$. Ainsi l'aire du bassin est égale à
\[
(8.7 - \frac{13.1}{e^{4}})\times 4 \times 15^2 = 7614.000000
(\frac{20.2}{e^{4}} + 20.6)\times 4 \times 15^2 = 18873.00000
\]
\end{enumerate}
\end{solution}
@ -185,9 +184,9 @@ Les valeurs des exercices sont générés automatiquement. Si une valeur a un no
Deux ateliers A et B fabriquent des stylos pour une entreprise.
L'atelier A fabrique 88.0\,\% des stylos, et parmi ceux-là, 75.0\,\% possèdent un défaut de fabrication.
L'atelier A fabrique 99.0\,\% des stylos, et parmi ceux-là, 28.000000000000004\,\% possèdent un défaut de fabrication.
De plus, 4.0\,\% des stylos possèdent un défaut de fabrication et sortent de l'atelier B.
De plus, 1.0\,\% des stylos possèdent un défaut de fabrication et sortent de l'atelier B.
Un stylo est prélevé au hasard dans le stock de l'entreprise.
@ -241,7 +240,7 @@ Les valeurs des exercices sont générés automatiquement. Si une valeur a un no
\item
\begin{enumerate}
\item Calculer la probabilité qu'un stylo provienne de l'atelier A et possède un défaut de fabrication.
\item En déduire que la probabilité qu'un stylo possède un défaut de fabrication est de $0.7$.
\item En déduire que la probabilité qu'un stylo possède un défaut de fabrication est de $0.29$.
\end{enumerate}
\item On prélève un stylo au hasard avec un défaut. Quelle est la probabilité qu'il vienne de l'atelier A?
\end{enumerate}
@ -251,7 +250,7 @@ Les valeurs des exercices sont générés automatiquement. Si une valeur a un no
\textbf{Partie B}
\medskip
Dans cette partie, on suppose que 70.0\,\% des stylos possèdent un défaut de fabrication.
Dans cette partie, on suppose que 28.999999999999996\,\% des stylos possèdent un défaut de fabrication.
L'entreprise confectionne des paquets contenant chacun $4$~stylos.
@ -266,7 +265,7 @@ Les valeurs des exercices sont générés automatiquement. Si une valeur a un no
\begin{enumerate}
\setcounter{enumi}{4}
\item Avec quelle loi peut-on modéliser $X$. Préciser les paramètres.
\item Calculer et interpréter la probabilité $P(X = 14)$.
\item Calculer et interpréter la probabilité $P(X = 9)$.
\item Le directeur de l'entreprise affirme qu'il y a plus d'une chance sur deux qu'un paquet ne comporte aucun stylo défectueux. A-t-il raison ?
\item Combien de stylos peut-on espérer avoir en moyenne?
\end{enumerate}
@ -282,27 +281,27 @@ Les valeurs des exercices sont générés automatiquement. Si une valeur a un no
child {node {$A$}
child {node {$D$}
edge from parent
node[above] {0.75}
node[above] {0.28}
}
child {node {$\overline{D}$}
edge from parent
node[above] {0.25}
node[above] {0.72}
}
edge from parent
node[above] {0.88}
node[above] {0.99}
}
child[missing] {}
child { node {$B$}
child {node {$D$}
edge from parent
node[above] {0.34}
node[above] {0.82}
}
child {node {$\overline{D}$}
edge from parent
node[above] {0.66}
node[above] {0.18}
}
edge from parent
node[above] {0.12}
node[above] {0.01}
} ;
\end{tikzpicture}
\end{center}
@ -310,51 +309,51 @@ Les valeurs des exercices sont générés automatiquement. Si une valeur a un no
\begin{itemize}
\item Probabilité que le stylo vienne de l'atelier A
\[
P(A) = 0.88
P(A) = 0.99
\]
\item Probabilité que le stylo vienne de l'atelier B
\[
P(B) = 0.12
P(B) = 0.01
\]
\item Probabilité que le stylo ait un défaut sachant qu'il vient de l'atelier A.
\[
P_A(D) = 0.75
P_A(D) = 0.28
\]
\item Probabilité que le stylo vienne de l'atelier B et qu'il ait un défaut.
\[
P(D \cap D) = 0.04
P(D \cap D) = 0.01
\]
\end{itemize}
\item
\begin{enumerate}
\item Probabilité qu'un stylo vienne de l'atelier A et qu'il ait un defaut
\[
P(A\cap D) = P(A) \times P_A(D) = 0.88 \times 0.75 = 0.66
P(A\cap D) = P(A) \times P_A(D) = 0.99 \times 0.28 = 0.28
\]
\item Probabilité que le stylo ai un défaut de fabrication.
\[
P(D) = P(A\cap D) + P(B\cap D) = 0.66 + 0.04 = 0.7
P(D) = P(A\cap D) + P(B\cap D) = 0.28 + 0.01 = 0.29
\]
\end{enumerate}
\item Probabilité qu'il vienne de l'atelier A sachant qu'il a un defaut
\[
P_D(A) = \frac{P(A\cap D)}{P(D)} = \frac{0.66}{0.7} = 0.94
P_D(A) = \frac{P(A\cap D)}{P(D)} = \frac{0.28}{0.29} = 0.97
\]
\item $X$ peut être modélisée par une loi binomiale de paramètres $n=15$ et $p=0.7$.
\item $X$ peut être modélisée par une loi binomiale de paramètres $n=10$ et $p=0.29$.
\item (\textit{par de correction automatique disponible pour le résultat final}
\[
P(X = 14) = \coefBino{15}{14}\times 0.7^{14} \times 0.3^{1}
P(X = 9) = \coefBino{10}{9}\times 0.29^{9} \times 0.71^{1}
\]
\item (\textit{par de correction automatique disponible pour le résultat final}
Il faut calculer la probabilité qu'il y ait 0 stylo avec un defaut.
\[
P(X = 0) = \coefBino{15}{0}\times 0.7^{0} \times 0.3^{15}
P(X = 0) = \coefBino{10}{0}\times 0.29^{0} \times 0.71^{10}
\]
Puis comparer ce nombre à 0,5.
\item Il faut calculer l'espérance
\[
E[X] = n\times p = 15 \times 0.7 = 10.5
E[X] = n\times p = 10 \times 0.29 = 2.9
\]
\end{enumerate}
\end{solution}

View File

@ -3,7 +3,7 @@
% Title Page
\title{DM1 \hfill SAVIN Lou-Ann}
\tribe{Maths complémentaire}
\tribe{Maths complémentaires}
\date{\hfillÀ render pour le jeudi 27 mai}
\xsimsetup{
@ -17,7 +17,7 @@ Les valeurs des exercices sont générés automatiquement. Si une valeur a un no
\begin{exercise}[subtitle={Optimisation de matière}]
\begin{minipage}{0.6\textwidth}
On se propose de fabriquer avec le moins de tôle possible une citerne fermée en forme de parallélépipède rectangle dont le volume intérieur doit être de $18m^3$. La longueur est aussi fixée à $3m$ par le cahier des charges.
On se propose de fabriquer avec le moins de tôle possible une citerne fermée en forme de parallélépipède rectangle dont le volume intérieur doit être de $4m^3$. La longueur est aussi fixée à $2m$ par le cahier des charges.
On peut donc faire varier uniquement la largeur (notée $x$) et la hauteur (notée $h$) de la cuve.
\end{minipage}
@ -28,25 +28,25 @@ Les valeurs des exercices sont générés automatiquement. Si une valeur a un no
\pgfmathsetmacro{\cubey}{1}
\pgfmathsetmacro{\cubez}{2}
\draw[black,fill=gray] (0,0,0) -- ++(-\cubex,0,0) -- ++(0,-\cubey,0) node [midway, left] {$h$} -- ++(\cubex,0,0) node [midway, below] {$x$} -- cycle;
\draw[black,fill=gray] (0,0,0) -- ++(0,0,-\cubez) -- ++(0,-\cubey,0) -- ++(0,0,\cubez) node [midway, right] {$3m$} -- cycle;
\draw[black,fill=gray] (0,0,0) -- ++(0,0,-\cubez) -- ++(0,-\cubey,0) -- ++(0,0,\cubez) node [midway, right] {$2m$} -- cycle;
\draw[black,fill=gray] (0,0,0) -- ++(-\cubex,0,0) -- ++(0,0,-\cubez) -- ++(\cubex,0,0) -- cycle;
\end{tikzpicture}
\end{minipage}
\begin{enumerate}
\item Expliquer pourquoi quand la largeur $x$ change, la hauteur $h$ doit elle aussi changer pour respecter les contraintes.
\item Démontrer que l'on doit avoir $h = \dfrac{6}{x}$.
\item Démontrer que l'on doit avoir $h = \dfrac{2}{x}$.
\item On note $S(x)$ l'aire totale de la citerne (c'est à dire la somme des aires des six faces). Montrer que l'on peut écrire
\[
S(x) = 6x + 12 + \frac{36}{x}
S(x) = 4x + 4 + \frac{8}{x}
\]
\item Démontrer que
\[
S(x) = \frac{6x^2 + 12x + 36}{x}
S(x) = \frac{4x^2 + 4x + 8}{x}
\]
\item Démontrer que
\[
S'(x) = \frac{6x^2 - 36}{x^2}
S'(x) = \frac{4x^2 - 8}{x^2}
\]
\item En déduire le tableau de variation de $S(x)$ sur $\intOF{0}{10}$.
\item Déterminer les valeurs de $x$ et $h$ correspondant à une utilisation minimal de tôle.
@ -57,62 +57,62 @@ Les valeurs des exercices sont générés automatiquement. Si une valeur a un no
\begin{enumerate}
\item Le volume étant fixe si l'on fait varier $x$, $h$ doit aussi varier.
\begin{itemize}
\item Si $x = 2$ alors conserver un volume de $V=18$, $h$ doit être égale à $6 / 2$
\item Si $x = 3$ alors conserver un volume de $V=18$, $h$ doit être égale à $6 / 3$
\item Si $x = 2$ alors conserver un volume de $V=4$, $h$ doit être égale à $2 / 2$
\item Si $x = 3$ alors conserver un volume de $V=4$, $h$ doit être égale à $2 / 3$
\end{itemize}
\item Pour calculer le volume, on a
\begin{eqnarray*}
V &=& h\times x \times 3 \\
18 &=& h\times x \times 3 \\
x &=& \frac{18}{h\times 3} = \frac{6}{h}
V &=& h\times x \times 2 \\
4 &=& h\times x \times 2 \\
x &=& \frac{4}{h\times 2} = \frac{2}{h}
\end{eqnarray*}
\item Pour calculer la surface totale, on ajoute la surface de chaque face. On a donc le calcul suivant
\begin{eqnarray*}
S(x) &=& x\times h \times 2 + x\times3\times2 + h\times 3\times 2\\
S(x) &=& x\times \frac{6}{x} \times 2 + x\times3\times2 + \frac{6}{x}\times 3\times 2\\
S(x) &=& 6x + 12 + \frac{36}{x}
S(x) &=& x\times h \times 2 + x\times2\times2 + h\times 2\times 2\\
S(x) &=& x\times \frac{2}{x} \times 2 + x\times2\times2 + \frac{2}{x}\times 2\times 2\\
S(x) &=& 4x + 4 + \frac{8}{x}
\end{eqnarray*}
\item Pour trouver cette nouvelle forme, on met chaque élément sur le même dénominateur
\begin{eqnarray*}
S(x) &=& 6x + 12 + \frac{36}{x}\\
S(x) &=& \frac{6x\times x}{x} + \frac{12\times x}{x} + \frac{36}{x}\\
S(x) &=& \frac{6x^2 + 12x + 36}{x}
S(x) &=& 4x + 4 + \frac{8}{x}\\
S(x) &=& \frac{4x\times x}{x} + \frac{4\times x}{x} + \frac{8}{x}\\
S(x) &=& \frac{4x^2 + 4x + 8}{x}
\end{eqnarray*}
\item On retrouve la formule $\frac{u}{v}$ à dériver
\[
u(x) = 6x^2 + 12x + 36 \Rightarrow u'(x) = 12x + 12
u(x) = 4x^2 + 4x + 8 \Rightarrow u'(x) = 8x + 4
\]
\[
v(x) = x \Rightarrow v'(x) = 1
\]
Donc au numérateur on obtient
\begin{eqnarray*}
u'(x)\times v(x) - u(x)\times v'(x) &=& (12x + 12)\times x - (6x^2 + 12x + 36)\times 1\\
&=& 6x^2 - 36
u'(x)\times v(x) - u(x)\times v'(x) &=& (8x + 4)\times x - (4x^2 + 4x + 8)\times 1\\
&=& 4x^2 - 8
\end{eqnarray*}
Donc
\[
S'(x) = \frac{6x^2 - 36}{x^2}
S'(x) = \frac{4x^2 - 8}{x^2}
\]
\item Tableau de variations de $S$
\begin{itemize}
\item Valeur interdite: $x^2 = 0 \equiv x = 0$
\item Signe de $6x^2 - 36$: c'est un polynôme du 2e degré
\item Signe de $4x^2 - 8$: c'est un polynôme du 2e degré
\[
\Delta = 864 > 0
\Delta = 128 > 0
\]
Il y a donc 2 racines
\[
x_1 = - 2.4494897427831783 \qquad
x_2 = 2.4494897427831783
x_1 = - 1.4142135623730951 \qquad
x_2 = 1.4142135623730951
\]
Et on sait que $6x^2 - 36$ est du signe de $a$ donc positif en dehors des racines
Et on sait que $4x^2 - 8$ est du signe de $a$ donc positif en dehors des racines
\item Le dénominateur $x^2$ est toujours positif.
\item Tableau de variations
\begin{tikzpicture}[baseline=(a.north)]
\tkzTabInit[lgt=3,espcl=3]{$x$/1,$6x^2 - 36$/1, $x^2$/1, $S'$/1, $S$/2}{$0$, $- 2.4494897427831783$, $10$}
\tkzTabInit[lgt=3,espcl=3]{$x$/1,$4x^2 - 8$/1, $x^2$/1, $S'$/1, $S$/2}{$0$, $- 1.4142135623730951$, $10$}
\tkzTabLine{d,-, z, +, }
\tkzTabLine{d,+, , +, }
\tkzTabLine{d,-, z, +, }
@ -120,7 +120,7 @@ Les valeurs des exercices sont générés automatiquement. Si une valeur a un no
\end{tikzpicture}
\end{itemize}
\item On a donc une surface minimal pour $x=2.4494897427831783$ et $h = 14.6969384566990698$.
\item On a donc une surface minimal pour $x=1.4142135623730951$ et $h = 2.8284271247461902$.
\end{enumerate}
\end{solution}
@ -133,14 +133,14 @@ Les valeurs des exercices sont générés automatiquement. Si une valeur a un no
Le tour d'un bassin au niveau du sol présente deux axes de symétrie : laxe des abscisses et la droite déquation $x=4$. Il est obtenu par symétrie de la courbe $\mathcal{C}_f$ sur $\intFF{0}{4}$$f$ est la fonction définie par
\[
f(x) = \left(- x^{2} + 2.7 x - 6.1\right) e^{- x} + 6.1
f(x) = \left(- x^{2} + 2.2 x - 5.8\right) e^{- x} + 5.8
\]
On admet que sur $\intFF{0}{4}$ la fonction $f$ est positive.
\begin{enumerate}
\item Sur un repère, tracer l'allure de la courbe $\mathcal{C}_f$, les axes de symétries puis compléter pour dessiner la forme du bassin.
\item Montrer que la fonction $f$ admet comme primitive sur $\R$ la fonction $F$ définie par
\[
F(x) = 6.1 x + \left( x^{2} - 0.7 x + 5.4\right) e^{- x}
F(x) = 5.8 x + \left( x^{2} - 0.2 x + 5.6\right) e^{- x}
\]
\item Calculer la quantité $\ds \int_0^4 f(x) \; dx$, vous donnerez le résultat sous forme exacte. Interpréter le résultat et reportez cette quantité sur le graphique.
\item On considère que l'échelle de votre graphique est de 1unité pour 15m. Calculer l'aire du bassin. Vous donnerez un résultat arrondi au $m^2$ près.
@ -156,15 +156,14 @@ Les valeurs des exercices sont générés automatiquement. Si une valeur a un no
\tkzGrid
\tkzAxeXY
\tkzFct[domain=0:10,color=red,very thick]%
{ (-x**2 + 2.7*x - 6.1)*exp(-x) + 6.1 };
{ (-x**2 + 2.2*x - 5.8)*exp(-x) + 5.8 };
\end{tikzpicture}
\item Il faut dériver $F(x)$ et vérifier que $F'(x) = f(x)$.
\item $\ds \int_0^4 f(x) \; dx = F(4) - F(0) = \frac{18.6}{e^{4}} + 19.0$
\item $\ds \int_0^4 f(x) \; dx = F(4) - F(0) = \frac{20.8}{e^{4}} + 17.6$
\item La quantité calculée à la question précédente se retrouve 4fois pour former le bassin. Il faut ensuite prendre en compte l'échelle, comme 1unité de longueur correspond à 15m, une unité d'air correspond à $15\times15 = 225m^2$. Ainsi l'aire du bassin est égale à
\[
(\frac{18.6}{e^{4}} + 19.0)\times 4 \times 15^2 = 17407.00000
(\frac{20.8}{e^{4}} + 17.6)\times 4 \times 15^2 = 16183.00000
\]
\end{enumerate}
\end{solution}
@ -185,9 +184,9 @@ Les valeurs des exercices sont générés automatiquement. Si une valeur a un no
Deux ateliers A et B fabriquent des stylos pour une entreprise.
L'atelier A fabrique 47.0\,\% des stylos, et parmi ceux-là, 45.0\,\% possèdent un défaut de fabrication.
L'atelier A fabrique 93.0\,\% des stylos, et parmi ceux-là, 62.0\,\% possèdent un défaut de fabrication.
De plus, 3.0\,\% des stylos possèdent un défaut de fabrication et sortent de l'atelier B.
De plus, 1.0\,\% des stylos possèdent un défaut de fabrication et sortent de l'atelier B.
Un stylo est prélevé au hasard dans le stock de l'entreprise.
@ -241,7 +240,7 @@ Les valeurs des exercices sont générés automatiquement. Si une valeur a un no
\item
\begin{enumerate}
\item Calculer la probabilité qu'un stylo provienne de l'atelier A et possède un défaut de fabrication.
\item En déduire que la probabilité qu'un stylo possède un défaut de fabrication est de $0.24$.
\item En déduire que la probabilité qu'un stylo possède un défaut de fabrication est de $0.59$.
\end{enumerate}
\item On prélève un stylo au hasard avec un défaut. Quelle est la probabilité qu'il vienne de l'atelier A?
\end{enumerate}
@ -251,7 +250,7 @@ Les valeurs des exercices sont générés automatiquement. Si une valeur a un no
\textbf{Partie B}
\medskip
Dans cette partie, on suppose que 24.0\,\% des stylos possèdent un défaut de fabrication.
Dans cette partie, on suppose que 59.0\,\% des stylos possèdent un défaut de fabrication.
L'entreprise confectionne des paquets contenant chacun $4$~stylos.
@ -266,7 +265,7 @@ Les valeurs des exercices sont générés automatiquement. Si une valeur a un no
\begin{enumerate}
\setcounter{enumi}{4}
\item Avec quelle loi peut-on modéliser $X$. Préciser les paramètres.
\item Calculer et interpréter la probabilité $P(X = 9)$.
\item Calculer et interpréter la probabilité $P(X = 11)$.
\item Le directeur de l'entreprise affirme qu'il y a plus d'une chance sur deux qu'un paquet ne comporte aucun stylo défectueux. A-t-il raison ?
\item Combien de stylos peut-on espérer avoir en moyenne?
\end{enumerate}
@ -282,27 +281,27 @@ Les valeurs des exercices sont générés automatiquement. Si une valeur a un no
child {node {$A$}
child {node {$D$}
edge from parent
node[above] {0.45}
node[above] {0.62}
}
child {node {$\overline{D}$}
edge from parent
node[above] {0.55}
node[above] {0.38}
}
edge from parent
node[above] {0.47}
node[above] {0.93}
}
child[missing] {}
child { node {$B$}
child {node {$D$}
edge from parent
node[above] {0.05}
node[above] {0.16}
}
child {node {$\overline{D}$}
edge from parent
node[above] {0.95}
node[above] {0.84}
}
edge from parent
node[above] {0.53}
node[above] {0.07}
} ;
\end{tikzpicture}
\end{center}
@ -310,51 +309,51 @@ Les valeurs des exercices sont générés automatiquement. Si une valeur a un no
\begin{itemize}
\item Probabilité que le stylo vienne de l'atelier A
\[
P(A) = 0.47
P(A) = 0.93
\]
\item Probabilité que le stylo vienne de l'atelier B
\[
P(B) = 0.53
P(B) = 0.07
\]
\item Probabilité que le stylo ait un défaut sachant qu'il vient de l'atelier A.
\[
P_A(D) = 0.45
P_A(D) = 0.62
\]
\item Probabilité que le stylo vienne de l'atelier B et qu'il ait un défaut.
\[
P(D \cap D) = 0.03
P(D \cap D) = 0.01
\]
\end{itemize}
\item
\begin{enumerate}
\item Probabilité qu'un stylo vienne de l'atelier A et qu'il ait un defaut
\[
P(A\cap D) = P(A) \times P_A(D) = 0.47 \times 0.45 = 0.21
P(A\cap D) = P(A) \times P_A(D) = 0.93 \times 0.62 = 0.58
\]
\item Probabilité que le stylo ai un défaut de fabrication.
\[
P(D) = P(A\cap D) + P(B\cap D) = 0.21 + 0.03 = 0.24
P(D) = P(A\cap D) + P(B\cap D) = 0.58 + 0.01 = 0.59
\]
\end{enumerate}
\item Probabilité qu'il vienne de l'atelier A sachant qu'il a un defaut
\[
P_D(A) = \frac{P(A\cap D)}{P(D)} = \frac{0.21}{0.24} = 0.88
P_D(A) = \frac{P(A\cap D)}{P(D)} = \frac{0.58}{0.59} = 0.98
\]
\item $X$ peut être modélisée par une loi binomiale de paramètres $n=19$ et $p=0.24$.
\item $X$ peut être modélisée par une loi binomiale de paramètres $n=13$ et $p=0.59$.
\item (\textit{par de correction automatique disponible pour le résultat final}
\[
P(X = 9) = \coefBino{19}{9}\times 0.24^{9} \times 0.76^{10}
P(X = 11) = \coefBino{13}{11}\times 0.59^{11} \times 0.41^{2}
\]
\item (\textit{par de correction automatique disponible pour le résultat final}
Il faut calculer la probabilité qu'il y ait 0 stylo avec un defaut.
\[
P(X = 0) = \coefBino{19}{0}\times 0.24^{0} \times 0.76^{19}
P(X = 0) = \coefBino{13}{0}\times 0.59^{0} \times 0.41^{13}
\]
Puis comparer ce nombre à 0,5.
\item Il faut calculer l'espérance
\[
E[X] = n\times p = 19 \times 0.24 = 4.56
E[X] = n\times p = 13 \times 0.59 = 7.67
\]
\end{enumerate}
\end{solution}

View File

@ -3,7 +3,7 @@
% Title Page
\title{DM1 \hfill SILVA LOPES Katleen}
\tribe{Maths complémentaire}
\tribe{Maths complémentaires}
\date{\hfillÀ render pour le jeudi 27 mai}
\xsimsetup{
@ -17,7 +17,7 @@ Les valeurs des exercices sont générés automatiquement. Si une valeur a un no
\begin{exercise}[subtitle={Optimisation de matière}]
\begin{minipage}{0.6\textwidth}
On se propose de fabriquer avec le moins de tôle possible une citerne fermée en forme de parallélépipède rectangle dont le volume intérieur doit être de $27m^3$. La longueur est aussi fixée à $3m$ par le cahier des charges.
On se propose de fabriquer avec le moins de tôle possible une citerne fermée en forme de parallélépipède rectangle dont le volume intérieur doit être de $10m^3$. La longueur est aussi fixée à $5m$ par le cahier des charges.
On peut donc faire varier uniquement la largeur (notée $x$) et la hauteur (notée $h$) de la cuve.
\end{minipage}
@ -28,25 +28,25 @@ Les valeurs des exercices sont générés automatiquement. Si une valeur a un no
\pgfmathsetmacro{\cubey}{1}
\pgfmathsetmacro{\cubez}{2}
\draw[black,fill=gray] (0,0,0) -- ++(-\cubex,0,0) -- ++(0,-\cubey,0) node [midway, left] {$h$} -- ++(\cubex,0,0) node [midway, below] {$x$} -- cycle;
\draw[black,fill=gray] (0,0,0) -- ++(0,0,-\cubez) -- ++(0,-\cubey,0) -- ++(0,0,\cubez) node [midway, right] {$3m$} -- cycle;
\draw[black,fill=gray] (0,0,0) -- ++(0,0,-\cubez) -- ++(0,-\cubey,0) -- ++(0,0,\cubez) node [midway, right] {$5m$} -- cycle;
\draw[black,fill=gray] (0,0,0) -- ++(-\cubex,0,0) -- ++(0,0,-\cubez) -- ++(\cubex,0,0) -- cycle;
\end{tikzpicture}
\end{minipage}
\begin{enumerate}
\item Expliquer pourquoi quand la largeur $x$ change, la hauteur $h$ doit elle aussi changer pour respecter les contraintes.
\item Démontrer que l'on doit avoir $h = \dfrac{9}{x}$.
\item Démontrer que l'on doit avoir $h = \dfrac{2}{x}$.
\item On note $S(x)$ l'aire totale de la citerne (c'est à dire la somme des aires des six faces). Montrer que l'on peut écrire
\[
S(x) = 6x + 18 + \frac{54}{x}
S(x) = 10x + 4 + \frac{20}{x}
\]
\item Démontrer que
\[
S(x) = \frac{6x^2 + 18x + 54}{x}
S(x) = \frac{10x^2 + 4x + 20}{x}
\]
\item Démontrer que
\[
S'(x) = \frac{6x^2 - 54}{x^2}
S'(x) = \frac{10x^2 - 20}{x^2}
\]
\item En déduire le tableau de variation de $S(x)$ sur $\intOF{0}{10}$.
\item Déterminer les valeurs de $x$ et $h$ correspondant à une utilisation minimal de tôle.
@ -57,62 +57,62 @@ Les valeurs des exercices sont générés automatiquement. Si une valeur a un no
\begin{enumerate}
\item Le volume étant fixe si l'on fait varier $x$, $h$ doit aussi varier.
\begin{itemize}
\item Si $x = 2$ alors conserver un volume de $V=27$, $h$ doit être égale à $9 / 2$
\item Si $x = 3$ alors conserver un volume de $V=27$, $h$ doit être égale à $9 / 3$
\item Si $x = 2$ alors conserver un volume de $V=10$, $h$ doit être égale à $2 / 2$
\item Si $x = 3$ alors conserver un volume de $V=10$, $h$ doit être égale à $2 / 3$
\end{itemize}
\item Pour calculer le volume, on a
\begin{eqnarray*}
V &=& h\times x \times 3 \\
27 &=& h\times x \times 3 \\
x &=& \frac{27}{h\times 3} = \frac{9}{h}
V &=& h\times x \times 5 \\
10 &=& h\times x \times 5 \\
x &=& \frac{10}{h\times 5} = \frac{2}{h}
\end{eqnarray*}
\item Pour calculer la surface totale, on ajoute la surface de chaque face. On a donc le calcul suivant
\begin{eqnarray*}
S(x) &=& x\times h \times 2 + x\times3\times2 + h\times 3\times 2\\
S(x) &=& x\times \frac{9}{x} \times 2 + x\times3\times2 + \frac{9}{x}\times 3\times 2\\
S(x) &=& 6x + 18 + \frac{54}{x}
S(x) &=& x\times h \times 2 + x\times5\times2 + h\times 5\times 2\\
S(x) &=& x\times \frac{2}{x} \times 2 + x\times5\times2 + \frac{2}{x}\times 5\times 2\\
S(x) &=& 10x + 4 + \frac{20}{x}
\end{eqnarray*}
\item Pour trouver cette nouvelle forme, on met chaque élément sur le même dénominateur
\begin{eqnarray*}
S(x) &=& 6x + 18 + \frac{54}{x}\\
S(x) &=& \frac{6x\times x}{x} + \frac{18\times x}{x} + \frac{54}{x}\\
S(x) &=& \frac{6x^2 + 18x + 54}{x}
S(x) &=& 10x + 4 + \frac{20}{x}\\
S(x) &=& \frac{10x\times x}{x} + \frac{4\times x}{x} + \frac{20}{x}\\
S(x) &=& \frac{10x^2 + 4x + 20}{x}
\end{eqnarray*}
\item On retrouve la formule $\frac{u}{v}$ à dériver
\[
u(x) = 6x^2 + 18x + 54 \Rightarrow u'(x) = 12x + 18
u(x) = 10x^2 + 4x + 20 \Rightarrow u'(x) = 20x + 4
\]
\[
v(x) = x \Rightarrow v'(x) = 1
\]
Donc au numérateur on obtient
\begin{eqnarray*}
u'(x)\times v(x) - u(x)\times v'(x) &=& (12x + 18)\times x - (6x^2 + 18x + 54)\times 1\\
&=& 6x^2 - 54
u'(x)\times v(x) - u(x)\times v'(x) &=& (20x + 4)\times x - (10x^2 + 4x + 20)\times 1\\
&=& 10x^2 - 20
\end{eqnarray*}
Donc
\[
S'(x) = \frac{6x^2 - 54}{x^2}
S'(x) = \frac{10x^2 - 20}{x^2}
\]
\item Tableau de variations de $S$
\begin{itemize}
\item Valeur interdite: $x^2 = 0 \equiv x = 0$
\item Signe de $6x^2 - 54$: c'est un polynôme du 2e degré
\item Signe de $10x^2 - 20$: c'est un polynôme du 2e degré
\[
\Delta = 1296 > 0
\Delta = 800 > 0
\]
Il y a donc 2 racines
\[
x_1 = - 3 \qquad
x_2 = 3
x_1 = - 1.4142135623730951 \qquad
x_2 = 1.4142135623730951
\]
Et on sait que $6x^2 - 54$ est du signe de $a$ donc positif en dehors des racines
Et on sait que $10x^2 - 20$ est du signe de $a$ donc positif en dehors des racines
\item Le dénominateur $x^2$ est toujours positif.
\item Tableau de variations
\begin{tikzpicture}[baseline=(a.north)]
\tkzTabInit[lgt=3,espcl=3]{$x$/1,$6x^2 - 54$/1, $x^2$/1, $S'$/1, $S$/2}{$0$, $- 3$, $10$}
\tkzTabInit[lgt=3,espcl=3]{$x$/1,$10x^2 - 20$/1, $x^2$/1, $S'$/1, $S$/2}{$0$, $- 1.4142135623730951$, $10$}
\tkzTabLine{d,-, z, +, }
\tkzTabLine{d,+, , +, }
\tkzTabLine{d,-, z, +, }
@ -120,7 +120,7 @@ Les valeurs des exercices sont générés automatiquement. Si une valeur a un no
\end{tikzpicture}
\end{itemize}
\item On a donc une surface minimal pour $x=3$ et $h = 27$.
\item On a donc une surface minimal pour $x=1.4142135623730951$ et $h = 2.8284271247461902$.
\end{enumerate}
\end{solution}
@ -133,14 +133,14 @@ Les valeurs des exercices sont générés automatiquement. Si une valeur a un no
Le tour d'un bassin au niveau du sol présente deux axes de symétrie : laxe des abscisses et la droite déquation $x=4$. Il est obtenu par symétrie de la courbe $\mathcal{C}_f$ sur $\intFF{0}{4}$$f$ est la fonction définie par
\[
f(x) = \left(- x^{2} + 3.0 x - 6.6\right) e^{- x} + 6.6
f(x) = \left(- x^{2} + 4.3 x - 6.7\right) e^{- x} + 6.7
\]
On admet que sur $\intFF{0}{4}$ la fonction $f$ est positive.
\begin{enumerate}
\item Sur un repère, tracer l'allure de la courbe $\mathcal{C}_f$, les axes de symétries puis compléter pour dessiner la forme du bassin.
\item Montrer que la fonction $f$ admet comme primitive sur $\R$ la fonction $F$ définie par
\[
F(x) = 6.6 x + \left( x^{2} - x + 5.6\right) e^{- x}
F(x) = 6.7 x + \left( x^{2} - 2.3 x + 4.4\right) e^{- x}
\]
\item Calculer la quantité $\ds \int_0^4 f(x) \; dx$, vous donnerez le résultat sous forme exacte. Interpréter le résultat et reportez cette quantité sur le graphique.
\item On considère que l'échelle de votre graphique est de 1unité pour 15m. Calculer l'aire du bassin. Vous donnerez un résultat arrondi au $m^2$ près.
@ -156,15 +156,14 @@ Les valeurs des exercices sont générés automatiquement. Si une valeur a un no
\tkzGrid
\tkzAxeXY
\tkzFct[domain=0:10,color=red,very thick]%
{ (-x**2 + 3.0*x - 6.6)*exp(-x) + 6.6 };
{ (-x**2 + 4.3*x - 6.7)*exp(-x) + 6.7 };
\end{tikzpicture}
\item Il faut dériver $F(x)$ et vérifier que $F'(x) = f(x)$.
\item $\ds \int_0^4 f(x) \; dx = F(4) - F(0) = \frac{17.6}{e^{4}} + 20.8$
\item $\ds \int_0^4 f(x) \; dx = F(4) - F(0) = \frac{11.2}{e^{4}} + 22.4$
\item La quantité calculée à la question précédente se retrouve 4fois pour former le bassin. Il faut ensuite prendre en compte l'échelle, comme 1unité de longueur correspond à 15m, une unité d'air correspond à $15\times15 = 225m^2$. Ainsi l'aire du bassin est égale à
\[
(\frac{17.6}{e^{4}} + 20.8)\times 4 \times 15^2 = 19010.00000
(\frac{11.2}{e^{4}} + 22.4)\times 4 \times 15^2 = 20345.00000
\]
\end{enumerate}
\end{solution}
@ -185,9 +184,9 @@ Les valeurs des exercices sont générés automatiquement. Si une valeur a un no
Deux ateliers A et B fabriquent des stylos pour une entreprise.
L'atelier A fabrique 43.0\,\% des stylos, et parmi ceux-là, 31.0\,\% possèdent un défaut de fabrication.
L'atelier A fabrique 24.0\,\% des stylos, et parmi ceux-là, 52.0\,\% possèdent un défaut de fabrication.
De plus, 19.0\,\% des stylos possèdent un défaut de fabrication et sortent de l'atelier B.
De plus, 71.0\,\% des stylos possèdent un défaut de fabrication et sortent de l'atelier B.
Un stylo est prélevé au hasard dans le stock de l'entreprise.
@ -241,7 +240,7 @@ Les valeurs des exercices sont générés automatiquement. Si une valeur a un no
\item
\begin{enumerate}
\item Calculer la probabilité qu'un stylo provienne de l'atelier A et possède un défaut de fabrication.
\item En déduire que la probabilité qu'un stylo possède un défaut de fabrication est de $0.32$.
\item En déduire que la probabilité qu'un stylo possède un défaut de fabrication est de $0.83$.
\end{enumerate}
\item On prélève un stylo au hasard avec un défaut. Quelle est la probabilité qu'il vienne de l'atelier A?
\end{enumerate}
@ -251,7 +250,7 @@ Les valeurs des exercices sont générés automatiquement. Si une valeur a un no
\textbf{Partie B}
\medskip
Dans cette partie, on suppose que 32.0\,\% des stylos possèdent un défaut de fabrication.
Dans cette partie, on suppose que 83.0\,\% des stylos possèdent un défaut de fabrication.
L'entreprise confectionne des paquets contenant chacun $4$~stylos.
@ -266,7 +265,7 @@ Les valeurs des exercices sont générés automatiquement. Si une valeur a un no
\begin{enumerate}
\setcounter{enumi}{4}
\item Avec quelle loi peut-on modéliser $X$. Préciser les paramètres.
\item Calculer et interpréter la probabilité $P(X = 7)$.
\item Calculer et interpréter la probabilité $P(X = 12)$.
\item Le directeur de l'entreprise affirme qu'il y a plus d'une chance sur deux qu'un paquet ne comporte aucun stylo défectueux. A-t-il raison ?
\item Combien de stylos peut-on espérer avoir en moyenne?
\end{enumerate}
@ -282,27 +281,27 @@ Les valeurs des exercices sont générés automatiquement. Si une valeur a un no
child {node {$A$}
child {node {$D$}
edge from parent
node[above] {0.31}
node[above] {0.52}
}
child {node {$\overline{D}$}
edge from parent
node[above] {0.69}
node[above] {0.48}
}
edge from parent
node[above] {0.43}
node[above] {0.24}
}
child[missing] {}
child { node {$B$}
child {node {$D$}
edge from parent
node[above] {0.34}
node[above] {0.94}
}
child {node {$\overline{D}$}
edge from parent
node[above] {0.66}
node[above] {0.06}
}
edge from parent
node[above] {0.57}
node[above] {0.76}
} ;
\end{tikzpicture}
\end{center}
@ -310,51 +309,51 @@ Les valeurs des exercices sont générés automatiquement. Si une valeur a un no
\begin{itemize}
\item Probabilité que le stylo vienne de l'atelier A
\[
P(A) = 0.43
P(A) = 0.24
\]
\item Probabilité que le stylo vienne de l'atelier B
\[
P(B) = 0.57
P(B) = 0.76
\]
\item Probabilité que le stylo ait un défaut sachant qu'il vient de l'atelier A.
\[
P_A(D) = 0.31
P_A(D) = 0.52
\]
\item Probabilité que le stylo vienne de l'atelier B et qu'il ait un défaut.
\[
P(D \cap D) = 0.19
P(D \cap D) = 0.71
\]
\end{itemize}
\item
\begin{enumerate}
\item Probabilité qu'un stylo vienne de l'atelier A et qu'il ait un defaut
\[
P(A\cap D) = P(A) \times P_A(D) = 0.43 \times 0.31 = 0.13
P(A\cap D) = P(A) \times P_A(D) = 0.24 \times 0.52 = 0.12
\]
\item Probabilité que le stylo ai un défaut de fabrication.
\[
P(D) = P(A\cap D) + P(B\cap D) = 0.13 + 0.19 = 0.32
P(D) = P(A\cap D) + P(B\cap D) = 0.12 + 0.71 = 0.83
\]
\end{enumerate}
\item Probabilité qu'il vienne de l'atelier A sachant qu'il a un defaut
\[
P_D(A) = \frac{P(A\cap D)}{P(D)} = \frac{0.13}{0.32} = 0.41
P_D(A) = \frac{P(A\cap D)}{P(D)} = \frac{0.12}{0.83} = 0.14
\]
\item $X$ peut être modélisée par une loi binomiale de paramètres $n=10$ et $p=0.32$.
\item $X$ peut être modélisée par une loi binomiale de paramètres $n=18$ et $p=0.83$.
\item (\textit{par de correction automatique disponible pour le résultat final}
\[
P(X = 7) = \coefBino{10}{7}\times 0.32^{7} \times 0.68^{3}
P(X = 12) = \coefBino{18}{12}\times 0.83^{12} \times 0.17^{6}
\]
\item (\textit{par de correction automatique disponible pour le résultat final}
Il faut calculer la probabilité qu'il y ait 0 stylo avec un defaut.
\[
P(X = 0) = \coefBino{10}{0}\times 0.32^{0} \times 0.68^{10}
P(X = 0) = \coefBino{18}{0}\times 0.83^{0} \times 0.17^{18}
\]
Puis comparer ce nombre à 0,5.
\item Il faut calculer l'espérance
\[
E[X] = n\times p = 10 \times 0.32 = 3.2
E[X] = n\times p = 18 \times 0.83 = 14.94
\]
\end{enumerate}
\end{solution}

View File

@ -3,7 +3,7 @@
% Title Page
\title{DM1 \hfill VANDROUX Guillemette}
\tribe{Maths complémentaire}
\tribe{Maths complémentaires}
\date{\hfillÀ render pour le jeudi 27 mai}
\xsimsetup{
@ -17,7 +17,7 @@ Les valeurs des exercices sont générés automatiquement. Si une valeur a un no
\begin{exercise}[subtitle={Optimisation de matière}]
\begin{minipage}{0.6\textwidth}
On se propose de fabriquer avec le moins de tôle possible une citerne fermée en forme de parallélépipède rectangle dont le volume intérieur doit être de $30m^3$. La longueur est aussi fixée à $3m$ par le cahier des charges.
On se propose de fabriquer avec le moins de tôle possible une citerne fermée en forme de parallélépipède rectangle dont le volume intérieur doit être de $27m^3$. La longueur est aussi fixée à $3m$ par le cahier des charges.
On peut donc faire varier uniquement la largeur (notée $x$) et la hauteur (notée $h$) de la cuve.
\end{minipage}
@ -35,18 +35,18 @@ Les valeurs des exercices sont générés automatiquement. Si une valeur a un no
\begin{enumerate}
\item Expliquer pourquoi quand la largeur $x$ change, la hauteur $h$ doit elle aussi changer pour respecter les contraintes.
\item Démontrer que l'on doit avoir $h = \dfrac{10}{x}$.
\item Démontrer que l'on doit avoir $h = \dfrac{9}{x}$.
\item On note $S(x)$ l'aire totale de la citerne (c'est à dire la somme des aires des six faces). Montrer que l'on peut écrire
\[
S(x) = 6x + 20 + \frac{60}{x}
S(x) = 6x + 18 + \frac{54}{x}
\]
\item Démontrer que
\[
S(x) = \frac{6x^2 + 20x + 60}{x}
S(x) = \frac{6x^2 + 18x + 54}{x}
\]
\item Démontrer que
\[
S'(x) = \frac{6x^2 - 60}{x^2}
S'(x) = \frac{6x^2 - 54}{x^2}
\]
\item En déduire le tableau de variation de $S(x)$ sur $\intOF{0}{10}$.
\item Déterminer les valeurs de $x$ et $h$ correspondant à une utilisation minimal de tôle.
@ -57,62 +57,62 @@ Les valeurs des exercices sont générés automatiquement. Si une valeur a un no
\begin{enumerate}
\item Le volume étant fixe si l'on fait varier $x$, $h$ doit aussi varier.
\begin{itemize}
\item Si $x = 2$ alors conserver un volume de $V=30$, $h$ doit être égale à $10 / 2$
\item Si $x = 3$ alors conserver un volume de $V=30$, $h$ doit être égale à $10 / 3$
\item Si $x = 2$ alors conserver un volume de $V=27$, $h$ doit être égale à $9 / 2$
\item Si $x = 3$ alors conserver un volume de $V=27$, $h$ doit être égale à $9 / 3$
\end{itemize}
\item Pour calculer le volume, on a
\begin{eqnarray*}
V &=& h\times x \times 3 \\
30 &=& h\times x \times 3 \\
x &=& \frac{30}{h\times 3} = \frac{10}{h}
27 &=& h\times x \times 3 \\
x &=& \frac{27}{h\times 3} = \frac{9}{h}
\end{eqnarray*}
\item Pour calculer la surface totale, on ajoute la surface de chaque face. On a donc le calcul suivant
\begin{eqnarray*}
S(x) &=& x\times h \times 2 + x\times3\times2 + h\times 3\times 2\\
S(x) &=& x\times \frac{10}{x} \times 2 + x\times3\times2 + \frac{10}{x}\times 3\times 2\\
S(x) &=& 6x + 20 + \frac{60}{x}
S(x) &=& x\times \frac{9}{x} \times 2 + x\times3\times2 + \frac{9}{x}\times 3\times 2\\
S(x) &=& 6x + 18 + \frac{54}{x}
\end{eqnarray*}
\item Pour trouver cette nouvelle forme, on met chaque élément sur le même dénominateur
\begin{eqnarray*}
S(x) &=& 6x + 20 + \frac{60}{x}\\
S(x) &=& \frac{6x\times x}{x} + \frac{20\times x}{x} + \frac{60}{x}\\
S(x) &=& \frac{6x^2 + 20x + 60}{x}
S(x) &=& 6x + 18 + \frac{54}{x}\\
S(x) &=& \frac{6x\times x}{x} + \frac{18\times x}{x} + \frac{54}{x}\\
S(x) &=& \frac{6x^2 + 18x + 54}{x}
\end{eqnarray*}
\item On retrouve la formule $\frac{u}{v}$ à dériver
\[
u(x) = 6x^2 + 20x + 60 \Rightarrow u'(x) = 12x + 20
u(x) = 6x^2 + 18x + 54 \Rightarrow u'(x) = 12x + 18
\]
\[
v(x) = x \Rightarrow v'(x) = 1
\]
Donc au numérateur on obtient
\begin{eqnarray*}
u'(x)\times v(x) - u(x)\times v'(x) &=& (12x + 20)\times x - (6x^2 + 20x + 60)\times 1\\
&=& 6x^2 - 60
u'(x)\times v(x) - u(x)\times v'(x) &=& (12x + 18)\times x - (6x^2 + 18x + 54)\times 1\\
&=& 6x^2 - 54
\end{eqnarray*}
Donc
\[
S'(x) = \frac{6x^2 - 60}{x^2}
S'(x) = \frac{6x^2 - 54}{x^2}
\]
\item Tableau de variations de $S$
\begin{itemize}
\item Valeur interdite: $x^2 = 0 \equiv x = 0$
\item Signe de $6x^2 - 60$: c'est un polynôme du 2e degré
\item Signe de $6x^2 - 54$: c'est un polynôme du 2e degré
\[
\Delta = 1440 > 0
\Delta = 1296 > 0
\]
Il y a donc 2 racines
\[
x_1 = - 3.1622776601683795 \qquad
x_2 = 3.1622776601683795
x_1 = - 3 \qquad
x_2 = 3
\]
Et on sait que $6x^2 - 60$ est du signe de $a$ donc positif en dehors des racines
Et on sait que $6x^2 - 54$ est du signe de $a$ donc positif en dehors des racines
\item Le dénominateur $x^2$ est toujours positif.
\item Tableau de variations
\begin{tikzpicture}[baseline=(a.north)]
\tkzTabInit[lgt=3,espcl=3]{$x$/1,$6x^2 - 60$/1, $x^2$/1, $S'$/1, $S$/2}{$0$, $- 3.1622776601683795$, $10$}
\tkzTabInit[lgt=3,espcl=3]{$x$/1,$6x^2 - 54$/1, $x^2$/1, $S'$/1, $S$/2}{$0$, $- 3$, $10$}
\tkzTabLine{d,-, z, +, }
\tkzTabLine{d,+, , +, }
\tkzTabLine{d,-, z, +, }
@ -120,7 +120,7 @@ Les valeurs des exercices sont générés automatiquement. Si une valeur a un no
\end{tikzpicture}
\end{itemize}
\item On a donc une surface minimal pour $x=3.1622776601683795$ et $h = 31.6227766016837950$.
\item On a donc une surface minimal pour $x=3$ et $h = 27$.
\end{enumerate}
\end{solution}
@ -133,14 +133,14 @@ Les valeurs des exercices sont générés automatiquement. Si une valeur a un no
Le tour d'un bassin au niveau du sol présente deux axes de symétrie : laxe des abscisses et la droite déquation $x=4$. Il est obtenu par symétrie de la courbe $\mathcal{C}_f$ sur $\intFF{0}{4}$$f$ est la fonction définie par
\[
f(x) = \left(- x^{2} + 5.0 x - 1.8\right) e^{- x} + 1.8
f(x) = \left(- x^{2} + 2.3 x - 7.0\right) e^{- x} + 7.0
\]
On admet que sur $\intFF{0}{4}$ la fonction $f$ est positive.
\begin{enumerate}
\item Sur un repère, tracer l'allure de la courbe $\mathcal{C}_f$, les axes de symétries puis compléter pour dessiner la forme du bassin.
\item Montrer que la fonction $f$ admet comme primitive sur $\R$ la fonction $F$ définie par
\[
F(x) = 1.8 x + \left( x^{2} - 3.0 x - 1.2\right) e^{- x}
F(x) = 7.0 x + \left( x^{2} - 0.3 x + 6.7\right) e^{- x}
\]
\item Calculer la quantité $\ds \int_0^4 f(x) \; dx$, vous donnerez le résultat sous forme exacte. Interpréter le résultat et reportez cette quantité sur le graphique.
\item On considère que l'échelle de votre graphique est de 1unité pour 15m. Calculer l'aire du bassin. Vous donnerez un résultat arrondi au $m^2$ près.
@ -156,15 +156,14 @@ Les valeurs des exercices sont générés automatiquement. Si une valeur a un no
\tkzGrid
\tkzAxeXY
\tkzFct[domain=0:10,color=red,very thick]%
{ (-x**2 + 5.0*x - 1.8)*exp(-x) + 1.8 };
{ (-x**2 + 2.3*x - 7.0)*exp(-x) + 7.0 };
\end{tikzpicture}
\item Il faut dériver $F(x)$ et vérifier que $F'(x) = f(x)$.
\item $\ds \int_0^4 f(x) \; dx = F(4) - F(0) = \frac{2.8}{e^{4}} + 8.4$
\item $\ds \int_0^4 f(x) \; dx = F(4) - F(0) = \frac{21.5}{e^{4}} + 21.3$
\item La quantité calculée à la question précédente se retrouve 4fois pour former le bassin. Il faut ensuite prendre en compte l'échelle, comme 1unité de longueur correspond à 15m, une unité d'air correspond à $15\times15 = 225m^2$. Ainsi l'aire du bassin est égale à
\[
(\frac{2.8}{e^{4}} + 8.4)\times 4 \times 15^2 = 7606.000000
(\frac{21.5}{e^{4}} + 21.3)\times 4 \times 15^2 = 19524.00000
\]
\end{enumerate}
\end{solution}
@ -185,9 +184,9 @@ Les valeurs des exercices sont générés automatiquement. Si une valeur a un no
Deux ateliers A et B fabriquent des stylos pour une entreprise.
L'atelier A fabrique 55.00000000000001\,\% des stylos, et parmi ceux-là, 46.0\,\% possèdent un défaut de fabrication.
L'atelier A fabrique 43.0\,\% des stylos, et parmi ceux-là, 9.0\,\% possèdent un défaut de fabrication.
De plus, 20.0\,\% des stylos possèdent un défaut de fabrication et sortent de l'atelier B.
De plus, 27.0\,\% des stylos possèdent un défaut de fabrication et sortent de l'atelier B.
Un stylo est prélevé au hasard dans le stock de l'entreprise.
@ -241,7 +240,7 @@ Les valeurs des exercices sont générés automatiquement. Si une valeur a un no
\item
\begin{enumerate}
\item Calculer la probabilité qu'un stylo provienne de l'atelier A et possède un défaut de fabrication.
\item En déduire que la probabilité qu'un stylo possède un défaut de fabrication est de $0.45$.
\item En déduire que la probabilité qu'un stylo possède un défaut de fabrication est de $0.31$.
\end{enumerate}
\item On prélève un stylo au hasard avec un défaut. Quelle est la probabilité qu'il vienne de l'atelier A?
\end{enumerate}
@ -251,7 +250,7 @@ Les valeurs des exercices sont générés automatiquement. Si une valeur a un no
\textbf{Partie B}
\medskip
Dans cette partie, on suppose que 45.0\,\% des stylos possèdent un défaut de fabrication.
Dans cette partie, on suppose que 31.0\,\% des stylos possèdent un défaut de fabrication.
L'entreprise confectionne des paquets contenant chacun $4$~stylos.
@ -266,7 +265,7 @@ Les valeurs des exercices sont générés automatiquement. Si une valeur a un no
\begin{enumerate}
\setcounter{enumi}{4}
\item Avec quelle loi peut-on modéliser $X$. Préciser les paramètres.
\item Calculer et interpréter la probabilité $P(X = 13)$.
\item Calculer et interpréter la probabilité $P(X = 10)$.
\item Le directeur de l'entreprise affirme qu'il y a plus d'une chance sur deux qu'un paquet ne comporte aucun stylo défectueux. A-t-il raison ?
\item Combien de stylos peut-on espérer avoir en moyenne?
\end{enumerate}
@ -282,27 +281,27 @@ Les valeurs des exercices sont générés automatiquement. Si une valeur a un no
child {node {$A$}
child {node {$D$}
edge from parent
node[above] {0.46}
node[above] {0.09}
}
child {node {$\overline{D}$}
edge from parent
node[above] {0.54}
node[above] {0.91}
}
edge from parent
node[above] {0.55}
node[above] {0.43}
}
child[missing] {}
child { node {$B$}
child {node {$D$}
edge from parent
node[above] {0.44}
node[above] {0.48}
}
child {node {$\overline{D}$}
edge from parent
node[above] {0.56}
node[above] {0.52}
}
edge from parent
node[above] {0.45}
node[above] {0.57}
} ;
\end{tikzpicture}
\end{center}
@ -310,51 +309,51 @@ Les valeurs des exercices sont générés automatiquement. Si une valeur a un no
\begin{itemize}
\item Probabilité que le stylo vienne de l'atelier A
\[
P(A) = 0.55
P(A) = 0.43
\]
\item Probabilité que le stylo vienne de l'atelier B
\[
P(B) = 0.45
P(B) = 0.57
\]
\item Probabilité que le stylo ait un défaut sachant qu'il vient de l'atelier A.
\[
P_A(D) = 0.46
P_A(D) = 0.09
\]
\item Probabilité que le stylo vienne de l'atelier B et qu'il ait un défaut.
\[
P(D \cap D) = 0.2
P(D \cap D) = 0.27
\]
\end{itemize}
\item
\begin{enumerate}
\item Probabilité qu'un stylo vienne de l'atelier A et qu'il ait un defaut
\[
P(A\cap D) = P(A) \times P_A(D) = 0.55 \times 0.46 = 0.25
P(A\cap D) = P(A) \times P_A(D) = 0.43 \times 0.09 = 0.04
\]
\item Probabilité que le stylo ai un défaut de fabrication.
\[
P(D) = P(A\cap D) + P(B\cap D) = 0.25 + 0.2 = 0.45
P(D) = P(A\cap D) + P(B\cap D) = 0.04 + 0.27 = 0.31
\]
\end{enumerate}
\item Probabilité qu'il vienne de l'atelier A sachant qu'il a un defaut
\[
P_D(A) = \frac{P(A\cap D)}{P(D)} = \frac{0.25}{0.45} = 0.56
P_D(A) = \frac{P(A\cap D)}{P(D)} = \frac{0.04}{0.31} = 0.13
\]
\item $X$ peut être modélisée par une loi binomiale de paramètres $n=15$ et $p=0.45$.
\item $X$ peut être modélisée par une loi binomiale de paramètres $n=11$ et $p=0.31$.
\item (\textit{par de correction automatique disponible pour le résultat final}
\[
P(X = 13) = \coefBino{15}{13}\times 0.45^{13} \times 0.55^{2}
P(X = 10) = \coefBino{11}{10}\times 0.31^{10} \times 0.69^{1}
\]
\item (\textit{par de correction automatique disponible pour le résultat final}
Il faut calculer la probabilité qu'il y ait 0 stylo avec un defaut.
\[
P(X = 0) = \coefBino{15}{0}\times 0.45^{0} \times 0.55^{15}
P(X = 0) = \coefBino{11}{0}\times 0.31^{0} \times 0.69^{11}
\]
Puis comparer ce nombre à 0,5.
\item Il faut calculer l'espérance
\[
E[X] = n\times p = 15 \times 0.45 = 6.75
E[X] = n\times p = 11 \times 0.31 = 3.41
\]
\end{enumerate}
\end{solution}

View File

@ -3,7 +3,7 @@
% Title Page
\title{DM1 \hfill BALLARD Antoine}
\tribe{Maths complémentaire}
\tribe{Maths complémentaires}
\date{\hfillÀ render pour le jeudi 27 mai}
\xsimsetup{
@ -17,7 +17,7 @@ Les valeurs des exercices sont générés automatiquement. Si une valeur a un no
\begin{exercise}[subtitle={Optimisation de matière}]
\begin{minipage}{0.6\textwidth}
On se propose de fabriquer avec le moins de tôle possible une citerne fermée en forme de parallélépipède rectangle dont le volume intérieur doit être de $8m^3$. La longueur est aussi fixée à $4m$ par le cahier des charges.
On se propose de fabriquer avec le moins de tôle possible une citerne fermée en forme de parallélépipède rectangle dont le volume intérieur doit être de $20m^3$. La longueur est aussi fixée à $2m$ par le cahier des charges.
On peut donc faire varier uniquement la largeur (notée $x$) et la hauteur (notée $h$) de la cuve.
\end{minipage}
@ -28,25 +28,25 @@ Les valeurs des exercices sont générés automatiquement. Si une valeur a un no
\pgfmathsetmacro{\cubey}{1}
\pgfmathsetmacro{\cubez}{2}
\draw[black,fill=gray] (0,0,0) -- ++(-\cubex,0,0) -- ++(0,-\cubey,0) node [midway, left] {$h$} -- ++(\cubex,0,0) node [midway, below] {$x$} -- cycle;
\draw[black,fill=gray] (0,0,0) -- ++(0,0,-\cubez) -- ++(0,-\cubey,0) -- ++(0,0,\cubez) node [midway, right] {$4m$} -- cycle;
\draw[black,fill=gray] (0,0,0) -- ++(0,0,-\cubez) -- ++(0,-\cubey,0) -- ++(0,0,\cubez) node [midway, right] {$2m$} -- cycle;
\draw[black,fill=gray] (0,0,0) -- ++(-\cubex,0,0) -- ++(0,0,-\cubez) -- ++(\cubex,0,0) -- cycle;
\end{tikzpicture}
\end{minipage}
\begin{enumerate}
\item Expliquer pourquoi quand la largeur $x$ change, la hauteur $h$ doit elle aussi changer pour respecter les contraintes.
\item Démontrer que l'on doit avoir $h = \dfrac{2}{x}$.
\item Démontrer que l'on doit avoir $h = \dfrac{10}{x}$.
\item On note $S(x)$ l'aire totale de la citerne (c'est à dire la somme des aires des six faces). Montrer que l'on peut écrire
\[
S(x) = 8x + 4 + \frac{16}{x}
S(x) = 4x + 20 + \frac{40}{x}
\]
\item Démontrer que
\[
S(x) = \frac{8x^2 + 4x + 16}{x}
S(x) = \frac{4x^2 + 20x + 40}{x}
\]
\item Démontrer que
\[
S'(x) = \frac{8x^2 - 16}{x^2}
S'(x) = \frac{4x^2 - 40}{x^2}
\]
\item En déduire le tableau de variation de $S(x)$ sur $\intOF{0}{10}$.
\item Déterminer les valeurs de $x$ et $h$ correspondant à une utilisation minimal de tôle.
@ -57,62 +57,62 @@ Les valeurs des exercices sont générés automatiquement. Si une valeur a un no
\begin{enumerate}
\item Le volume étant fixe si l'on fait varier $x$, $h$ doit aussi varier.
\begin{itemize}
\item Si $x = 2$ alors conserver un volume de $V=8$, $h$ doit être égale à $2 / 2$
\item Si $x = 3$ alors conserver un volume de $V=8$, $h$ doit être égale à $2 / 3$
\item Si $x = 2$ alors conserver un volume de $V=20$, $h$ doit être égale à $10 / 2$
\item Si $x = 3$ alors conserver un volume de $V=20$, $h$ doit être égale à $10 / 3$
\end{itemize}
\item Pour calculer le volume, on a
\begin{eqnarray*}
V &=& h\times x \times 4 \\
8 &=& h\times x \times 4 \\
x &=& \frac{8}{h\times 4} = \frac{2}{h}
V &=& h\times x \times 2 \\
20 &=& h\times x \times 2 \\
x &=& \frac{20}{h\times 2} = \frac{10}{h}
\end{eqnarray*}
\item Pour calculer la surface totale, on ajoute la surface de chaque face. On a donc le calcul suivant
\begin{eqnarray*}
S(x) &=& x\times h \times 2 + x\times4\times2 + h\times 4\times 2\\
S(x) &=& x\times \frac{2}{x} \times 2 + x\times4\times2 + \frac{2}{x}\times 4\times 2\\
S(x) &=& 8x + 4 + \frac{16}{x}
S(x) &=& x\times h \times 2 + x\times2\times2 + h\times 2\times 2\\
S(x) &=& x\times \frac{10}{x} \times 2 + x\times2\times2 + \frac{10}{x}\times 2\times 2\\
S(x) &=& 4x + 20 + \frac{40}{x}
\end{eqnarray*}
\item Pour trouver cette nouvelle forme, on met chaque élément sur le même dénominateur
\begin{eqnarray*}
S(x) &=& 8x + 4 + \frac{16}{x}\\
S(x) &=& \frac{8x\times x}{x} + \frac{4\times x}{x} + \frac{16}{x}\\
S(x) &=& \frac{8x^2 + 4x + 16}{x}
S(x) &=& 4x + 20 + \frac{40}{x}\\
S(x) &=& \frac{4x\times x}{x} + \frac{20\times x}{x} + \frac{40}{x}\\
S(x) &=& \frac{4x^2 + 20x + 40}{x}
\end{eqnarray*}
\item On retrouve la formule $\frac{u}{v}$ à dériver
\[
u(x) = 8x^2 + 4x + 16 \Rightarrow u'(x) = 16x + 4
u(x) = 4x^2 + 20x + 40 \Rightarrow u'(x) = 8x + 20
\]
\[
v(x) = x \Rightarrow v'(x) = 1
\]
Donc au numérateur on obtient
\begin{eqnarray*}
u'(x)\times v(x) - u(x)\times v'(x) &=& (16x + 4)\times x - (8x^2 + 4x + 16)\times 1\\
&=& 8x^2 - 16
u'(x)\times v(x) - u(x)\times v'(x) &=& (8x + 20)\times x - (4x^2 + 20x + 40)\times 1\\
&=& 4x^2 - 40
\end{eqnarray*}
Donc
\[
S'(x) = \frac{8x^2 - 16}{x^2}
S'(x) = \frac{4x^2 - 40}{x^2}
\]
\item Tableau de variations de $S$
\begin{itemize}
\item Valeur interdite: $x^2 = 0 \equiv x = 0$
\item Signe de $8x^2 - 16$: c'est un polynôme du 2e degré
\item Signe de $4x^2 - 40$: c'est un polynôme du 2e degré
\[
\Delta = 512 > 0
\Delta = 640 > 0
\]
Il y a donc 2 racines
\[
x_1 = - 1.4142135623730951 \qquad
x_2 = 1.4142135623730951
x_1 = - 3.1622776601683795 \qquad
x_2 = 3.1622776601683795
\]
Et on sait que $8x^2 - 16$ est du signe de $a$ donc positif en dehors des racines
Et on sait que $4x^2 - 40$ est du signe de $a$ donc positif en dehors des racines
\item Le dénominateur $x^2$ est toujours positif.
\item Tableau de variations
\begin{tikzpicture}[baseline=(a.north)]
\tkzTabInit[lgt=3,espcl=3]{$x$/1,$8x^2 - 16$/1, $x^2$/1, $S'$/1, $S$/2}{$0$, $- 1.4142135623730951$, $10$}
\tkzTabInit[lgt=3,espcl=3]{$x$/1,$4x^2 - 40$/1, $x^2$/1, $S'$/1, $S$/2}{$0$, $- 3.1622776601683795$, $10$}
\tkzTabLine{d,-, z, +, }
\tkzTabLine{d,+, , +, }
\tkzTabLine{d,-, z, +, }
@ -120,7 +120,7 @@ Les valeurs des exercices sont générés automatiquement. Si une valeur a un no
\end{tikzpicture}
\end{itemize}
\item On a donc une surface minimal pour $x=1.4142135623730951$ et $h = 2.8284271247461902$.
\item On a donc une surface minimal pour $x=3.1622776601683795$ et $h = 31.6227766016837950$.
\end{enumerate}
\end{solution}
@ -133,14 +133,14 @@ Les valeurs des exercices sont générés automatiquement. Si une valeur a un no
Le tour d'un bassin au niveau du sol présente deux axes de symétrie : laxe des abscisses et la droite déquation $x=4$. Il est obtenu par symétrie de la courbe $\mathcal{C}_f$ sur $\intFF{0}{4}$$f$ est la fonction définie par
\[
f(x) = \left(- x^{2} + 0.3 x - 7.3\right) e^{- x} + 7.3
f(x) = \left(- x^{2} + 4.7 x - 7.9\right) e^{- x} + 7.9
\]
On admet que sur $\intFF{0}{4}$ la fonction $f$ est positive.
\begin{enumerate}
\item Sur un repère, tracer l'allure de la courbe $\mathcal{C}_f$, les axes de symétries puis compléter pour dessiner la forme du bassin.
\item Montrer que la fonction $f$ admet comme primitive sur $\R$ la fonction $F$ définie par
\[
F(x) = 7.3 x + \left( x^{2} + 1.7 x + 9.0\right) e^{- x}
F(x) = 7.9 x + \left( x^{2} - 2.7 x + 5.2\right) e^{- x}
\]
\item Calculer la quantité $\ds \int_0^4 f(x) \; dx$, vous donnerez le résultat sous forme exacte. Interpréter le résultat et reportez cette quantité sur le graphique.
\item On considère que l'échelle de votre graphique est de 1unité pour 15m. Calculer l'aire du bassin. Vous donnerez un résultat arrondi au $m^2$ près.
@ -156,15 +156,14 @@ Les valeurs des exercices sont générés automatiquement. Si une valeur a un no
\tkzGrid
\tkzAxeXY
\tkzFct[domain=0:10,color=red,very thick]%
{ (-x**2 + 0.3*x - 7.3)*exp(-x) + 7.3 };
{ (-x**2 + 4.7*x - 7.9)*exp(-x) + 7.9 };
\end{tikzpicture}
\item Il faut dériver $F(x)$ et vérifier que $F'(x) = f(x)$.
\item $\ds \int_0^4 f(x) \; dx = F(4) - F(0) = \frac{31.8}{e^{4}} + 20.2$
\item $\ds \int_0^4 f(x) \; dx = F(4) - F(0) = \frac{10.4}{e^{4}} + 26.4$
\item La quantité calculée à la question précédente se retrouve 4fois pour former le bassin. Il faut ensuite prendre en compte l'échelle, comme 1unité de longueur correspond à 15m, une unité d'air correspond à $15\times15 = 225m^2$. Ainsi l'aire du bassin est égale à
\[
(\frac{31.8}{e^{4}} + 20.2)\times 4 \times 15^2 = 18704.00000
(\frac{10.4}{e^{4}} + 26.4)\times 4 \times 15^2 = 23931.00000
\]
\end{enumerate}
\end{solution}
@ -185,9 +184,9 @@ Les valeurs des exercices sont générés automatiquement. Si une valeur a un no
Deux ateliers A et B fabriquent des stylos pour une entreprise.
L'atelier A fabrique 86.0\,\% des stylos, et parmi ceux-là, 34.0\,\% possèdent un défaut de fabrication.
L'atelier A fabrique 40.0\,\% des stylos, et parmi ceux-là, 24.0\,\% possèdent un défaut de fabrication.
De plus, 8.0\,\% des stylos possèdent un défaut de fabrication et sortent de l'atelier B.
De plus, 50.0\,\% des stylos possèdent un défaut de fabrication et sortent de l'atelier B.
Un stylo est prélevé au hasard dans le stock de l'entreprise.
@ -241,7 +240,7 @@ Les valeurs des exercices sont générés automatiquement. Si une valeur a un no
\item
\begin{enumerate}
\item Calculer la probabilité qu'un stylo provienne de l'atelier A et possède un défaut de fabrication.
\item En déduire que la probabilité qu'un stylo possède un défaut de fabrication est de $0.37$.
\item En déduire que la probabilité qu'un stylo possède un défaut de fabrication est de $0.6$.
\end{enumerate}
\item On prélève un stylo au hasard avec un défaut. Quelle est la probabilité qu'il vienne de l'atelier A?
\end{enumerate}
@ -251,7 +250,7 @@ Les valeurs des exercices sont générés automatiquement. Si une valeur a un no
\textbf{Partie B}
\medskip
Dans cette partie, on suppose que 37.0\,\% des stylos possèdent un défaut de fabrication.
Dans cette partie, on suppose que 60.0\,\% des stylos possèdent un défaut de fabrication.
L'entreprise confectionne des paquets contenant chacun $4$~stylos.
@ -266,7 +265,7 @@ Les valeurs des exercices sont générés automatiquement. Si une valeur a un no
\begin{enumerate}
\setcounter{enumi}{4}
\item Avec quelle loi peut-on modéliser $X$. Préciser les paramètres.
\item Calculer et interpréter la probabilité $P(X = 10)$.
\item Calculer et interpréter la probabilité $P(X = 12)$.
\item Le directeur de l'entreprise affirme qu'il y a plus d'une chance sur deux qu'un paquet ne comporte aucun stylo défectueux. A-t-il raison ?
\item Combien de stylos peut-on espérer avoir en moyenne?
\end{enumerate}
@ -282,27 +281,27 @@ Les valeurs des exercices sont générés automatiquement. Si une valeur a un no
child {node {$A$}
child {node {$D$}
edge from parent
node[above] {0.34}
node[above] {0.24}
}
child {node {$\overline{D}$}
edge from parent
node[above] {0.66}
node[above] {0.76}
}
edge from parent
node[above] {0.86}
node[above] {0.4}
}
child[missing] {}
child { node {$B$}
child {node {$D$}
edge from parent
node[above] {0.58}
node[above] {0.83}
}
child {node {$\overline{D}$}
edge from parent
node[above] {0.42}
node[above] {0.17}
}
edge from parent
node[above] {0.14}
node[above] {0.6}
} ;
\end{tikzpicture}
\end{center}
@ -310,51 +309,51 @@ Les valeurs des exercices sont générés automatiquement. Si une valeur a un no
\begin{itemize}
\item Probabilité que le stylo vienne de l'atelier A
\[
P(A) = 0.86
P(A) = 0.4
\]
\item Probabilité que le stylo vienne de l'atelier B
\[
P(B) = 0.14
P(B) = 0.6
\]
\item Probabilité que le stylo ait un défaut sachant qu'il vient de l'atelier A.
\[
P_A(D) = 0.34
P_A(D) = 0.24
\]
\item Probabilité que le stylo vienne de l'atelier B et qu'il ait un défaut.
\[
P(D \cap D) = 0.08
P(D \cap D) = 0.5
\]
\end{itemize}
\item
\begin{enumerate}
\item Probabilité qu'un stylo vienne de l'atelier A et qu'il ait un defaut
\[
P(A\cap D) = P(A) \times P_A(D) = 0.86 \times 0.34 = 0.29
P(A\cap D) = P(A) \times P_A(D) = 0.4 \times 0.24 = 0.1
\]
\item Probabilité que le stylo ai un défaut de fabrication.
\[
P(D) = P(A\cap D) + P(B\cap D) = 0.29 + 0.08 = 0.37
P(D) = P(A\cap D) + P(B\cap D) = 0.1 + 0.5 = 0.6
\]
\end{enumerate}
\item Probabilité qu'il vienne de l'atelier A sachant qu'il a un defaut
\[
P_D(A) = \frac{P(A\cap D)}{P(D)} = \frac{0.29}{0.37} = 0.78
P_D(A) = \frac{P(A\cap D)}{P(D)} = \frac{0.1}{0.6} = 0.17
\]
\item $X$ peut être modélisée par une loi binomiale de paramètres $n=20$ et $p=0.37$.
\item $X$ peut être modélisée par une loi binomiale de paramètres $n=15$ et $p=0.6$.
\item (\textit{par de correction automatique disponible pour le résultat final}
\[
P(X = 10) = \coefBino{20}{10}\times 0.37^{10} \times 0.63^{10}
P(X = 12) = \coefBino{15}{12}\times 0.6^{12} \times 0.4^{3}
\]
\item (\textit{par de correction automatique disponible pour le résultat final}
Il faut calculer la probabilité qu'il y ait 0 stylo avec un defaut.
\[
P(X = 0) = \coefBino{20}{0}\times 0.37^{0} \times 0.63^{20}
P(X = 0) = \coefBino{15}{0}\times 0.6^{0} \times 0.4^{15}
\]
Puis comparer ce nombre à 0,5.
\item Il faut calculer l'espérance
\[
E[X] = n\times p = 20 \times 0.37 = 7.4
E[X] = n\times p = 15 \times 0.6 = 9.0
\]
\end{enumerate}
\end{solution}

View File

@ -3,7 +3,7 @@
% Title Page
\title{DM1 \hfill BALUKHATYY Alexandre}
\tribe{Maths complémentaire}
\tribe{Maths complémentaires}
\date{\hfillÀ render pour le jeudi 27 mai}
\xsimsetup{
@ -17,7 +17,7 @@ Les valeurs des exercices sont générés automatiquement. Si une valeur a un no
\begin{exercise}[subtitle={Optimisation de matière}]
\begin{minipage}{0.6\textwidth}
On se propose de fabriquer avec le moins de tôle possible une citerne fermée en forme de parallélépipède rectangle dont le volume intérieur doit être de $15m^3$. La longueur est aussi fixée à $3m$ par le cahier des charges.
On se propose de fabriquer avec le moins de tôle possible une citerne fermée en forme de parallélépipède rectangle dont le volume intérieur doit être de $14m^3$. La longueur est aussi fixée à $2m$ par le cahier des charges.
On peut donc faire varier uniquement la largeur (notée $x$) et la hauteur (notée $h$) de la cuve.
\end{minipage}
@ -28,25 +28,25 @@ Les valeurs des exercices sont générés automatiquement. Si une valeur a un no
\pgfmathsetmacro{\cubey}{1}
\pgfmathsetmacro{\cubez}{2}
\draw[black,fill=gray] (0,0,0) -- ++(-\cubex,0,0) -- ++(0,-\cubey,0) node [midway, left] {$h$} -- ++(\cubex,0,0) node [midway, below] {$x$} -- cycle;
\draw[black,fill=gray] (0,0,0) -- ++(0,0,-\cubez) -- ++(0,-\cubey,0) -- ++(0,0,\cubez) node [midway, right] {$3m$} -- cycle;
\draw[black,fill=gray] (0,0,0) -- ++(0,0,-\cubez) -- ++(0,-\cubey,0) -- ++(0,0,\cubez) node [midway, right] {$2m$} -- cycle;
\draw[black,fill=gray] (0,0,0) -- ++(-\cubex,0,0) -- ++(0,0,-\cubez) -- ++(\cubex,0,0) -- cycle;
\end{tikzpicture}
\end{minipage}
\begin{enumerate}
\item Expliquer pourquoi quand la largeur $x$ change, la hauteur $h$ doit elle aussi changer pour respecter les contraintes.
\item Démontrer que l'on doit avoir $h = \dfrac{5}{x}$.
\item Démontrer que l'on doit avoir $h = \dfrac{7}{x}$.
\item On note $S(x)$ l'aire totale de la citerne (c'est à dire la somme des aires des six faces). Montrer que l'on peut écrire
\[
S(x) = 6x + 10 + \frac{30}{x}
S(x) = 4x + 14 + \frac{28}{x}
\]
\item Démontrer que
\[
S(x) = \frac{6x^2 + 10x + 30}{x}
S(x) = \frac{4x^2 + 14x + 28}{x}
\]
\item Démontrer que
\[
S'(x) = \frac{6x^2 - 30}{x^2}
S'(x) = \frac{4x^2 - 28}{x^2}
\]
\item En déduire le tableau de variation de $S(x)$ sur $\intOF{0}{10}$.
\item Déterminer les valeurs de $x$ et $h$ correspondant à une utilisation minimal de tôle.
@ -57,62 +57,62 @@ Les valeurs des exercices sont générés automatiquement. Si une valeur a un no
\begin{enumerate}
\item Le volume étant fixe si l'on fait varier $x$, $h$ doit aussi varier.
\begin{itemize}
\item Si $x = 2$ alors conserver un volume de $V=15$, $h$ doit être égale à $5 / 2$
\item Si $x = 3$ alors conserver un volume de $V=15$, $h$ doit être égale à $5 / 3$
\item Si $x = 2$ alors conserver un volume de $V=14$, $h$ doit être égale à $7 / 2$
\item Si $x = 3$ alors conserver un volume de $V=14$, $h$ doit être égale à $7 / 3$
\end{itemize}
\item Pour calculer le volume, on a
\begin{eqnarray*}
V &=& h\times x \times 3 \\
15 &=& h\times x \times 3 \\
x &=& \frac{15}{h\times 3} = \frac{5}{h}
V &=& h\times x \times 2 \\
14 &=& h\times x \times 2 \\
x &=& \frac{14}{h\times 2} = \frac{7}{h}
\end{eqnarray*}
\item Pour calculer la surface totale, on ajoute la surface de chaque face. On a donc le calcul suivant
\begin{eqnarray*}
S(x) &=& x\times h \times 2 + x\times3\times2 + h\times 3\times 2\\
S(x) &=& x\times \frac{5}{x} \times 2 + x\times3\times2 + \frac{5}{x}\times 3\times 2\\
S(x) &=& 6x + 10 + \frac{30}{x}
S(x) &=& x\times h \times 2 + x\times2\times2 + h\times 2\times 2\\
S(x) &=& x\times \frac{7}{x} \times 2 + x\times2\times2 + \frac{7}{x}\times 2\times 2\\
S(x) &=& 4x + 14 + \frac{28}{x}
\end{eqnarray*}
\item Pour trouver cette nouvelle forme, on met chaque élément sur le même dénominateur
\begin{eqnarray*}
S(x) &=& 6x + 10 + \frac{30}{x}\\
S(x) &=& \frac{6x\times x}{x} + \frac{10\times x}{x} + \frac{30}{x}\\
S(x) &=& \frac{6x^2 + 10x + 30}{x}
S(x) &=& 4x + 14 + \frac{28}{x}\\
S(x) &=& \frac{4x\times x}{x} + \frac{14\times x}{x} + \frac{28}{x}\\
S(x) &=& \frac{4x^2 + 14x + 28}{x}
\end{eqnarray*}
\item On retrouve la formule $\frac{u}{v}$ à dériver
\[
u(x) = 6x^2 + 10x + 30 \Rightarrow u'(x) = 12x + 10
u(x) = 4x^2 + 14x + 28 \Rightarrow u'(x) = 8x + 14
\]
\[
v(x) = x \Rightarrow v'(x) = 1
\]
Donc au numérateur on obtient
\begin{eqnarray*}
u'(x)\times v(x) - u(x)\times v'(x) &=& (12x + 10)\times x - (6x^2 + 10x + 30)\times 1\\
&=& 6x^2 - 30
u'(x)\times v(x) - u(x)\times v'(x) &=& (8x + 14)\times x - (4x^2 + 14x + 28)\times 1\\
&=& 4x^2 - 28
\end{eqnarray*}
Donc
\[
S'(x) = \frac{6x^2 - 30}{x^2}
S'(x) = \frac{4x^2 - 28}{x^2}
\]
\item Tableau de variations de $S$
\begin{itemize}
\item Valeur interdite: $x^2 = 0 \equiv x = 0$
\item Signe de $6x^2 - 30$: c'est un polynôme du 2e degré
\item Signe de $4x^2 - 28$: c'est un polynôme du 2e degré
\[
\Delta = 720 > 0
\Delta = 448 > 0
\]
Il y a donc 2 racines
\[
x_1 = - 2.23606797749979 \qquad
x_2 = 2.23606797749979
x_1 = - 2.6457513110645907 \qquad
x_2 = 2.6457513110645907
\]
Et on sait que $6x^2 - 30$ est du signe de $a$ donc positif en dehors des racines
Et on sait que $4x^2 - 28$ est du signe de $a$ donc positif en dehors des racines
\item Le dénominateur $x^2$ est toujours positif.
\item Tableau de variations
\begin{tikzpicture}[baseline=(a.north)]
\tkzTabInit[lgt=3,espcl=3]{$x$/1,$6x^2 - 30$/1, $x^2$/1, $S'$/1, $S$/2}{$0$, $- 2.23606797749979$, $10$}
\tkzTabInit[lgt=3,espcl=3]{$x$/1,$4x^2 - 28$/1, $x^2$/1, $S'$/1, $S$/2}{$0$, $- 2.6457513110645907$, $10$}
\tkzTabLine{d,-, z, +, }
\tkzTabLine{d,+, , +, }
\tkzTabLine{d,-, z, +, }
@ -120,7 +120,7 @@ Les valeurs des exercices sont générés automatiquement. Si une valeur a un no
\end{tikzpicture}
\end{itemize}
\item On a donc une surface minimal pour $x=2.23606797749979$ et $h = 11.18033988749895$.
\item On a donc une surface minimal pour $x=2.6457513110645907$ et $h = 18.5202591774521349$.
\end{enumerate}
\end{solution}
@ -133,14 +133,14 @@ Les valeurs des exercices sont générés automatiquement. Si une valeur a un no
Le tour d'un bassin au niveau du sol présente deux axes de symétrie : laxe des abscisses et la droite déquation $x=4$. Il est obtenu par symétrie de la courbe $\mathcal{C}_f$ sur $\intFF{0}{4}$$f$ est la fonction définie par
\[
f(x) = \left(- x^{2} + 8.4 x - 0.6\right) e^{- x} + 0.6
f(x) = \left(- x^{2} + 7.7 x - 8.4\right) e^{- x} + 8.4
\]
On admet que sur $\intFF{0}{4}$ la fonction $f$ est positive.
\begin{enumerate}
\item Sur un repère, tracer l'allure de la courbe $\mathcal{C}_f$, les axes de symétries puis compléter pour dessiner la forme du bassin.
\item Montrer que la fonction $f$ admet comme primitive sur $\R$ la fonction $F$ définie par
\[
F(x) = 0.6 x + \left( x^{2} - 6.4 x - 5.8\right) e^{- x}
F(x) = 8.4 x + \left( x^{2} - 5.7 x + 2.7\right) e^{- x}
\]
\item Calculer la quantité $\ds \int_0^4 f(x) \; dx$, vous donnerez le résultat sous forme exacte. Interpréter le résultat et reportez cette quantité sur le graphique.
\item On considère que l'échelle de votre graphique est de 1unité pour 15m. Calculer l'aire du bassin. Vous donnerez un résultat arrondi au $m^2$ près.
@ -156,15 +156,14 @@ Les valeurs des exercices sont générés automatiquement. Si une valeur a un no
\tkzGrid
\tkzAxeXY
\tkzFct[domain=0:10,color=red,very thick]%
{ (-x**2 + 8.4*x - 0.6)*exp(-x) + 0.6 };
{ (-x**2 + 7.7*x - 8.4)*exp(-x) + 8.4 };
\end{tikzpicture}
\item Il faut dériver $F(x)$ et vérifier que $F'(x) = f(x)$.
\item $\ds \int_0^4 f(x) \; dx = F(4) - F(0) = 8.2 - \frac{15.4}{e^{4}}$
\item $\ds \int_0^4 f(x) \; dx = F(4) - F(0) = 30.9 - \frac{4.1}{e^{4}}$
\item La quantité calculée à la question précédente se retrouve 4fois pour former le bassin. Il faut ensuite prendre en compte l'échelle, comme 1unité de longueur correspond à 15m, une unité d'air correspond à $15\times15 = 225m^2$. Ainsi l'aire du bassin est égale à
\[
(8.2 - \frac{15.4}{e^{4}})\times 4 \times 15^2 = 7126.000000
(30.9 - \frac{4.1}{e^{4}})\times 4 \times 15^2 = 27742.00000
\]
\end{enumerate}
\end{solution}
@ -185,9 +184,9 @@ Les valeurs des exercices sont générés automatiquement. Si une valeur a un no
Deux ateliers A et B fabriquent des stylos pour une entreprise.
L'atelier A fabrique 25.0\,\% des stylos, et parmi ceux-là, 84.0\,\% possèdent un défaut de fabrication.
L'atelier A fabrique 14.000000000000002\,\% des stylos, et parmi ceux-là, 51.0\,\% possèdent un défaut de fabrication.
De plus, 57.99999999999999\,\% des stylos possèdent un défaut de fabrication et sortent de l'atelier B.
De plus, 61.0\,\% des stylos possèdent un défaut de fabrication et sortent de l'atelier B.
Un stylo est prélevé au hasard dans le stock de l'entreprise.
@ -241,7 +240,7 @@ Les valeurs des exercices sont générés automatiquement. Si une valeur a un no
\item
\begin{enumerate}
\item Calculer la probabilité qu'un stylo provienne de l'atelier A et possède un défaut de fabrication.
\item En déduire que la probabilité qu'un stylo possède un défaut de fabrication est de $0.79$.
\item En déduire que la probabilité qu'un stylo possède un défaut de fabrication est de $0.68$.
\end{enumerate}
\item On prélève un stylo au hasard avec un défaut. Quelle est la probabilité qu'il vienne de l'atelier A?
\end{enumerate}
@ -251,7 +250,7 @@ Les valeurs des exercices sont générés automatiquement. Si une valeur a un no
\textbf{Partie B}
\medskip
Dans cette partie, on suppose que 79.0\,\% des stylos possèdent un défaut de fabrication.
Dans cette partie, on suppose que 68.0\,\% des stylos possèdent un défaut de fabrication.
L'entreprise confectionne des paquets contenant chacun $4$~stylos.
@ -266,7 +265,7 @@ Les valeurs des exercices sont générés automatiquement. Si une valeur a un no
\begin{enumerate}
\setcounter{enumi}{4}
\item Avec quelle loi peut-on modéliser $X$. Préciser les paramètres.
\item Calculer et interpréter la probabilité $P(X = 11)$.
\item Calculer et interpréter la probabilité $P(X = 10)$.
\item Le directeur de l'entreprise affirme qu'il y a plus d'une chance sur deux qu'un paquet ne comporte aucun stylo défectueux. A-t-il raison ?
\item Combien de stylos peut-on espérer avoir en moyenne?
\end{enumerate}
@ -282,27 +281,27 @@ Les valeurs des exercices sont générés automatiquement. Si une valeur a un no
child {node {$A$}
child {node {$D$}
edge from parent
node[above] {0.84}
node[above] {0.51}
}
child {node {$\overline{D}$}
edge from parent
node[above] {0.16}
node[above] {0.49}
}
edge from parent
node[above] {0.25}
node[above] {0.14}
}
child[missing] {}
child { node {$B$}
child {node {$D$}
edge from parent
node[above] {0.78}
node[above] {0.71}
}
child {node {$\overline{D}$}
edge from parent
node[above] {0.22}
node[above] {0.29}
}
edge from parent
node[above] {0.75}
node[above] {0.86}
} ;
\end{tikzpicture}
\end{center}
@ -310,51 +309,51 @@ Les valeurs des exercices sont générés automatiquement. Si une valeur a un no
\begin{itemize}
\item Probabilité que le stylo vienne de l'atelier A
\[
P(A) = 0.25
P(A) = 0.14
\]
\item Probabilité que le stylo vienne de l'atelier B
\[
P(B) = 0.75
P(B) = 0.86
\]
\item Probabilité que le stylo ait un défaut sachant qu'il vient de l'atelier A.
\[
P_A(D) = 0.84
P_A(D) = 0.51
\]
\item Probabilité que le stylo vienne de l'atelier B et qu'il ait un défaut.
\[
P(D \cap D) = 0.58
P(D \cap D) = 0.61
\]
\end{itemize}
\item
\begin{enumerate}
\item Probabilité qu'un stylo vienne de l'atelier A et qu'il ait un defaut
\[
P(A\cap D) = P(A) \times P_A(D) = 0.25 \times 0.84 = 0.21
P(A\cap D) = P(A) \times P_A(D) = 0.14 \times 0.51 = 0.07
\]
\item Probabilité que le stylo ai un défaut de fabrication.
\[
P(D) = P(A\cap D) + P(B\cap D) = 0.21 + 0.58 = 0.79
P(D) = P(A\cap D) + P(B\cap D) = 0.07 + 0.61 = 0.68
\]
\end{enumerate}
\item Probabilité qu'il vienne de l'atelier A sachant qu'il a un defaut
\[
P_D(A) = \frac{P(A\cap D)}{P(D)} = \frac{0.21}{0.79} = 0.27
P_D(A) = \frac{P(A\cap D)}{P(D)} = \frac{0.07}{0.68} = 0.1
\]
\item $X$ peut être modélisée par une loi binomiale de paramètres $n=12$ et $p=0.79$.
\item $X$ peut être modélisée par une loi binomiale de paramètres $n=16$ et $p=0.68$.
\item (\textit{par de correction automatique disponible pour le résultat final}
\[
P(X = 11) = \coefBino{12}{11}\times 0.79^{11} \times 0.21^{1}
P(X = 10) = \coefBino{16}{10}\times 0.68^{10} \times 0.32^{6}
\]
\item (\textit{par de correction automatique disponible pour le résultat final}
Il faut calculer la probabilité qu'il y ait 0 stylo avec un defaut.
\[
P(X = 0) = \coefBino{12}{0}\times 0.79^{0} \times 0.21^{12}
P(X = 0) = \coefBino{16}{0}\times 0.68^{0} \times 0.32^{16}
\]
Puis comparer ce nombre à 0,5.
\item Il faut calculer l'espérance
\[
E[X] = n\times p = 12 \times 0.79 = 9.48
E[X] = n\times p = 16 \times 0.68 = 10.88
\]
\end{enumerate}
\end{solution}

View File

@ -3,7 +3,7 @@
% Title Page
\title{DM1 \hfill CALES Mathis}
\tribe{Maths complémentaire}
\tribe{Maths complémentaires}
\date{\hfillÀ render pour le jeudi 27 mai}
\xsimsetup{
@ -17,7 +17,7 @@ Les valeurs des exercices sont générés automatiquement. Si une valeur a un no
\begin{exercise}[subtitle={Optimisation de matière}]
\begin{minipage}{0.6\textwidth}
On se propose de fabriquer avec le moins de tôle possible une citerne fermée en forme de parallélépipède rectangle dont le volume intérieur doit être de $6m^3$. La longueur est aussi fixée à $2m$ par le cahier des charges.
On se propose de fabriquer avec le moins de tôle possible une citerne fermée en forme de parallélépipède rectangle dont le volume intérieur doit être de $8m^3$. La longueur est aussi fixée à $2m$ par le cahier des charges.
On peut donc faire varier uniquement la largeur (notée $x$) et la hauteur (notée $h$) de la cuve.
\end{minipage}
@ -35,18 +35,18 @@ Les valeurs des exercices sont générés automatiquement. Si une valeur a un no
\begin{enumerate}
\item Expliquer pourquoi quand la largeur $x$ change, la hauteur $h$ doit elle aussi changer pour respecter les contraintes.
\item Démontrer que l'on doit avoir $h = \dfrac{3}{x}$.
\item Démontrer que l'on doit avoir $h = \dfrac{4}{x}$.
\item On note $S(x)$ l'aire totale de la citerne (c'est à dire la somme des aires des six faces). Montrer que l'on peut écrire
\[
S(x) = 4x + 6 + \frac{12}{x}
S(x) = 4x + 8 + \frac{16}{x}
\]
\item Démontrer que
\[
S(x) = \frac{4x^2 + 6x + 12}{x}
S(x) = \frac{4x^2 + 8x + 16}{x}
\]
\item Démontrer que
\[
S'(x) = \frac{4x^2 - 12}{x^2}
S'(x) = \frac{4x^2 - 16}{x^2}
\]
\item En déduire le tableau de variation de $S(x)$ sur $\intOF{0}{10}$.
\item Déterminer les valeurs de $x$ et $h$ correspondant à une utilisation minimal de tôle.
@ -57,62 +57,62 @@ Les valeurs des exercices sont générés automatiquement. Si une valeur a un no
\begin{enumerate}
\item Le volume étant fixe si l'on fait varier $x$, $h$ doit aussi varier.
\begin{itemize}
\item Si $x = 2$ alors conserver un volume de $V=6$, $h$ doit être égale à $3 / 2$
\item Si $x = 3$ alors conserver un volume de $V=6$, $h$ doit être égale à $3 / 3$
\item Si $x = 2$ alors conserver un volume de $V=8$, $h$ doit être égale à $4 / 2$
\item Si $x = 3$ alors conserver un volume de $V=8$, $h$ doit être égale à $4 / 3$
\end{itemize}
\item Pour calculer le volume, on a
\begin{eqnarray*}
V &=& h\times x \times 2 \\
6 &=& h\times x \times 2 \\
x &=& \frac{6}{h\times 2} = \frac{3}{h}
8 &=& h\times x \times 2 \\
x &=& \frac{8}{h\times 2} = \frac{4}{h}
\end{eqnarray*}
\item Pour calculer la surface totale, on ajoute la surface de chaque face. On a donc le calcul suivant
\begin{eqnarray*}
S(x) &=& x\times h \times 2 + x\times2\times2 + h\times 2\times 2\\
S(x) &=& x\times \frac{3}{x} \times 2 + x\times2\times2 + \frac{3}{x}\times 2\times 2\\
S(x) &=& 4x + 6 + \frac{12}{x}
S(x) &=& x\times \frac{4}{x} \times 2 + x\times2\times2 + \frac{4}{x}\times 2\times 2\\
S(x) &=& 4x + 8 + \frac{16}{x}
\end{eqnarray*}
\item Pour trouver cette nouvelle forme, on met chaque élément sur le même dénominateur
\begin{eqnarray*}
S(x) &=& 4x + 6 + \frac{12}{x}\\
S(x) &=& \frac{4x\times x}{x} + \frac{6\times x}{x} + \frac{12}{x}\\
S(x) &=& \frac{4x^2 + 6x + 12}{x}
S(x) &=& 4x + 8 + \frac{16}{x}\\
S(x) &=& \frac{4x\times x}{x} + \frac{8\times x}{x} + \frac{16}{x}\\
S(x) &=& \frac{4x^2 + 8x + 16}{x}
\end{eqnarray*}
\item On retrouve la formule $\frac{u}{v}$ à dériver
\[
u(x) = 4x^2 + 6x + 12 \Rightarrow u'(x) = 8x + 6
u(x) = 4x^2 + 8x + 16 \Rightarrow u'(x) = 8x + 8
\]
\[
v(x) = x \Rightarrow v'(x) = 1
\]
Donc au numérateur on obtient
\begin{eqnarray*}
u'(x)\times v(x) - u(x)\times v'(x) &=& (8x + 6)\times x - (4x^2 + 6x + 12)\times 1\\
&=& 4x^2 - 12
u'(x)\times v(x) - u(x)\times v'(x) &=& (8x + 8)\times x - (4x^2 + 8x + 16)\times 1\\
&=& 4x^2 - 16
\end{eqnarray*}
Donc
\[
S'(x) = \frac{4x^2 - 12}{x^2}
S'(x) = \frac{4x^2 - 16}{x^2}
\]
\item Tableau de variations de $S$
\begin{itemize}
\item Valeur interdite: $x^2 = 0 \equiv x = 0$
\item Signe de $4x^2 - 12$: c'est un polynôme du 2e degré
\item Signe de $4x^2 - 16$: c'est un polynôme du 2e degré
\[
\Delta = 192 > 0
\Delta = 256 > 0
\]
Il y a donc 2 racines
\[
x_1 = - 1.7320508075688772 \qquad
x_2 = 1.7320508075688772
x_1 = - 2 \qquad
x_2 = 2
\]
Et on sait que $4x^2 - 12$ est du signe de $a$ donc positif en dehors des racines
Et on sait que $4x^2 - 16$ est du signe de $a$ donc positif en dehors des racines
\item Le dénominateur $x^2$ est toujours positif.
\item Tableau de variations
\begin{tikzpicture}[baseline=(a.north)]
\tkzTabInit[lgt=3,espcl=3]{$x$/1,$4x^2 - 12$/1, $x^2$/1, $S'$/1, $S$/2}{$0$, $- 1.7320508075688772$, $10$}
\tkzTabInit[lgt=3,espcl=3]{$x$/1,$4x^2 - 16$/1, $x^2$/1, $S'$/1, $S$/2}{$0$, $- 2$, $10$}
\tkzTabLine{d,-, z, +, }
\tkzTabLine{d,+, , +, }
\tkzTabLine{d,-, z, +, }
@ -120,7 +120,7 @@ Les valeurs des exercices sont générés automatiquement. Si une valeur a un no
\end{tikzpicture}
\end{itemize}
\item On a donc une surface minimal pour $x=1.7320508075688772$ et $h = 5.1961524227066316$.
\item On a donc une surface minimal pour $x=2$ et $h = 8$.
\end{enumerate}
\end{solution}
@ -133,14 +133,14 @@ Les valeurs des exercices sont générés automatiquement. Si une valeur a un no
Le tour d'un bassin au niveau du sol présente deux axes de symétrie : laxe des abscisses et la droite déquation $x=4$. Il est obtenu par symétrie de la courbe $\mathcal{C}_f$ sur $\intFF{0}{4}$$f$ est la fonction définie par
\[
f(x) = \left(- x^{2} + 2.8 x - 8.7\right) e^{- x} + 8.7
f(x) = \left(- x^{2} + 9.4 x - 4.9\right) e^{- x} + 4.9
\]
On admet que sur $\intFF{0}{4}$ la fonction $f$ est positive.
\begin{enumerate}
\item Sur un repère, tracer l'allure de la courbe $\mathcal{C}_f$, les axes de symétries puis compléter pour dessiner la forme du bassin.
\item Montrer que la fonction $f$ admet comme primitive sur $\R$ la fonction $F$ définie par
\[
F(x) = 8.7 x + \left( x^{2} - 0.8 x + 7.9\right) e^{- x}
F(x) = 4.9 x + \left( x^{2} - 7.4 x - 2.5\right) e^{- x}
\]
\item Calculer la quantité $\ds \int_0^4 f(x) \; dx$, vous donnerez le résultat sous forme exacte. Interpréter le résultat et reportez cette quantité sur le graphique.
\item On considère que l'échelle de votre graphique est de 1unité pour 15m. Calculer l'aire du bassin. Vous donnerez un résultat arrondi au $m^2$ près.
@ -156,15 +156,14 @@ Les valeurs des exercices sont générés automatiquement. Si une valeur a un no
\tkzGrid
\tkzAxeXY
\tkzFct[domain=0:10,color=red,very thick]%
{ (-x**2 + 2.8*x - 8.7)*exp(-x) + 8.7 };
{ (-x**2 + 9.4*x - 4.9)*exp(-x) + 4.9 };
\end{tikzpicture}
\item Il faut dériver $F(x)$ et vérifier que $F'(x) = f(x)$.
\item $\ds \int_0^4 f(x) \; dx = F(4) - F(0) = \frac{20.7}{e^{4}} + 26.9$
\item $\ds \int_0^4 f(x) \; dx = F(4) - F(0) = 22.1 - \frac{16.1}{e^{4}}$
\item La quantité calculée à la question précédente se retrouve 4fois pour former le bassin. Il faut ensuite prendre en compte l'échelle, comme 1unité de longueur correspond à 15m, une unité d'air correspond à $15\times15 = 225m^2$. Ainsi l'aire du bassin est égale à
\[
(\frac{20.7}{e^{4}} + 26.9)\times 4 \times 15^2 = 24551.00000
(22.1 - \frac{16.1}{e^{4}})\times 4 \times 15^2 = 19625.00000
\]
\end{enumerate}
\end{solution}
@ -185,9 +184,9 @@ Les valeurs des exercices sont générés automatiquement. Si une valeur a un no
Deux ateliers A et B fabriquent des stylos pour une entreprise.
L'atelier A fabrique 48.0\,\% des stylos, et parmi ceux-là, 99.0\,\% possèdent un défaut de fabrication.
L'atelier A fabrique 37.0\,\% des stylos, et parmi ceux-là, 24.0\,\% possèdent un défaut de fabrication.
De plus, 10.0\,\% des stylos possèdent un défaut de fabrication et sortent de l'atelier B.
De plus, 22.0\,\% des stylos possèdent un défaut de fabrication et sortent de l'atelier B.
Un stylo est prélevé au hasard dans le stock de l'entreprise.
@ -241,7 +240,7 @@ Les valeurs des exercices sont générés automatiquement. Si une valeur a un no
\item
\begin{enumerate}
\item Calculer la probabilité qu'un stylo provienne de l'atelier A et possède un défaut de fabrication.
\item En déduire que la probabilité qu'un stylo possède un défaut de fabrication est de $0.58$.
\item En déduire que la probabilité qu'un stylo possède un défaut de fabrication est de $0.31$.
\end{enumerate}
\item On prélève un stylo au hasard avec un défaut. Quelle est la probabilité qu'il vienne de l'atelier A?
\end{enumerate}
@ -251,7 +250,7 @@ Les valeurs des exercices sont générés automatiquement. Si une valeur a un no
\textbf{Partie B}
\medskip
Dans cette partie, on suppose que 57.99999999999999\,\% des stylos possèdent un défaut de fabrication.
Dans cette partie, on suppose que 31.0\,\% des stylos possèdent un défaut de fabrication.
L'entreprise confectionne des paquets contenant chacun $4$~stylos.
@ -266,7 +265,7 @@ Les valeurs des exercices sont générés automatiquement. Si une valeur a un no
\begin{enumerate}
\setcounter{enumi}{4}
\item Avec quelle loi peut-on modéliser $X$. Préciser les paramètres.
\item Calculer et interpréter la probabilité $P(X = 10)$.
\item Calculer et interpréter la probabilité $P(X = 12)$.
\item Le directeur de l'entreprise affirme qu'il y a plus d'une chance sur deux qu'un paquet ne comporte aucun stylo défectueux. A-t-il raison ?
\item Combien de stylos peut-on espérer avoir en moyenne?
\end{enumerate}
@ -282,27 +281,27 @@ Les valeurs des exercices sont générés automatiquement. Si une valeur a un no
child {node {$A$}
child {node {$D$}
edge from parent
node[above] {0.99}
node[above] {0.24}
}
child {node {$\overline{D}$}
edge from parent
node[above] {0.01}
node[above] {0.76}
}
edge from parent
node[above] {0.48}
node[above] {0.37}
}
child[missing] {}
child { node {$B$}
child {node {$D$}
edge from parent
node[above] {0.2}
node[above] {0.35}
}
child {node {$\overline{D}$}
edge from parent
node[above] {0.8}
node[above] {0.65}
}
edge from parent
node[above] {0.52}
node[above] {0.63}
} ;
\end{tikzpicture}
\end{center}
@ -310,51 +309,51 @@ Les valeurs des exercices sont générés automatiquement. Si une valeur a un no
\begin{itemize}
\item Probabilité que le stylo vienne de l'atelier A
\[
P(A) = 0.48
P(A) = 0.37
\]
\item Probabilité que le stylo vienne de l'atelier B
\[
P(B) = 0.52
P(B) = 0.63
\]
\item Probabilité que le stylo ait un défaut sachant qu'il vient de l'atelier A.
\[
P_A(D) = 0.99
P_A(D) = 0.24
\]
\item Probabilité que le stylo vienne de l'atelier B et qu'il ait un défaut.
\[
P(D \cap D) = 0.1
P(D \cap D) = 0.22
\]
\end{itemize}
\item
\begin{enumerate}
\item Probabilité qu'un stylo vienne de l'atelier A et qu'il ait un defaut
\[
P(A\cap D) = P(A) \times P_A(D) = 0.48 \times 0.99 = 0.48
P(A\cap D) = P(A) \times P_A(D) = 0.37 \times 0.24 = 0.09
\]
\item Probabilité que le stylo ai un défaut de fabrication.
\[
P(D) = P(A\cap D) + P(B\cap D) = 0.48 + 0.1 = 0.58
P(D) = P(A\cap D) + P(B\cap D) = 0.09 + 0.22 = 0.31
\]
\end{enumerate}
\item Probabilité qu'il vienne de l'atelier A sachant qu'il a un defaut
\[
P_D(A) = \frac{P(A\cap D)}{P(D)} = \frac{0.48}{0.58} = 0.83
P_D(A) = \frac{P(A\cap D)}{P(D)} = \frac{0.09}{0.31} = 0.29
\]
\item $X$ peut être modélisée par une loi binomiale de paramètres $n=17$ et $p=0.58$.
\item $X$ peut être modélisée par une loi binomiale de paramètres $n=18$ et $p=0.31$.
\item (\textit{par de correction automatique disponible pour le résultat final}
\[
P(X = 10) = \coefBino{17}{10}\times 0.58^{10} \times 0.42^{7}
P(X = 12) = \coefBino{18}{12}\times 0.31^{12} \times 0.69^{6}
\]
\item (\textit{par de correction automatique disponible pour le résultat final}
Il faut calculer la probabilité qu'il y ait 0 stylo avec un defaut.
\[
P(X = 0) = \coefBino{17}{0}\times 0.58^{0} \times 0.42^{17}
P(X = 0) = \coefBino{18}{0}\times 0.31^{0} \times 0.69^{18}
\]
Puis comparer ce nombre à 0,5.
\item Il faut calculer l'espérance
\[
E[X] = n\times p = 17 \times 0.58 = 9.86
E[X] = n\times p = 18 \times 0.31 = 5.58
\]
\end{enumerate}
\end{solution}

View File

@ -3,7 +3,7 @@
% Title Page
\title{DM1 \hfill CHAKIR Iman}
\tribe{Maths complémentaire}
\tribe{Maths complémentaires}
\date{\hfillÀ render pour le jeudi 27 mai}
\xsimsetup{
@ -17,7 +17,7 @@ Les valeurs des exercices sont générés automatiquement. Si une valeur a un no
\begin{exercise}[subtitle={Optimisation de matière}]
\begin{minipage}{0.6\textwidth}
On se propose de fabriquer avec le moins de tôle possible une citerne fermée en forme de parallélépipède rectangle dont le volume intérieur doit être de $20m^3$. La longueur est aussi fixée à $4m$ par le cahier des charges.
On se propose de fabriquer avec le moins de tôle possible une citerne fermée en forme de parallélépipède rectangle dont le volume intérieur doit être de $16m^3$. La longueur est aussi fixée à $2m$ par le cahier des charges.
On peut donc faire varier uniquement la largeur (notée $x$) et la hauteur (notée $h$) de la cuve.
\end{minipage}
@ -28,25 +28,25 @@ Les valeurs des exercices sont générés automatiquement. Si une valeur a un no
\pgfmathsetmacro{\cubey}{1}
\pgfmathsetmacro{\cubez}{2}
\draw[black,fill=gray] (0,0,0) -- ++(-\cubex,0,0) -- ++(0,-\cubey,0) node [midway, left] {$h$} -- ++(\cubex,0,0) node [midway, below] {$x$} -- cycle;
\draw[black,fill=gray] (0,0,0) -- ++(0,0,-\cubez) -- ++(0,-\cubey,0) -- ++(0,0,\cubez) node [midway, right] {$4m$} -- cycle;
\draw[black,fill=gray] (0,0,0) -- ++(0,0,-\cubez) -- ++(0,-\cubey,0) -- ++(0,0,\cubez) node [midway, right] {$2m$} -- cycle;
\draw[black,fill=gray] (0,0,0) -- ++(-\cubex,0,0) -- ++(0,0,-\cubez) -- ++(\cubex,0,0) -- cycle;
\end{tikzpicture}
\end{minipage}
\begin{enumerate}
\item Expliquer pourquoi quand la largeur $x$ change, la hauteur $h$ doit elle aussi changer pour respecter les contraintes.
\item Démontrer que l'on doit avoir $h = \dfrac{5}{x}$.
\item Démontrer que l'on doit avoir $h = \dfrac{8}{x}$.
\item On note $S(x)$ l'aire totale de la citerne (c'est à dire la somme des aires des six faces). Montrer que l'on peut écrire
\[
S(x) = 8x + 10 + \frac{40}{x}
S(x) = 4x + 16 + \frac{32}{x}
\]
\item Démontrer que
\[
S(x) = \frac{8x^2 + 10x + 40}{x}
S(x) = \frac{4x^2 + 16x + 32}{x}
\]
\item Démontrer que
\[
S'(x) = \frac{8x^2 - 40}{x^2}
S'(x) = \frac{4x^2 - 32}{x^2}
\]
\item En déduire le tableau de variation de $S(x)$ sur $\intOF{0}{10}$.
\item Déterminer les valeurs de $x$ et $h$ correspondant à une utilisation minimal de tôle.
@ -57,62 +57,62 @@ Les valeurs des exercices sont générés automatiquement. Si une valeur a un no
\begin{enumerate}
\item Le volume étant fixe si l'on fait varier $x$, $h$ doit aussi varier.
\begin{itemize}
\item Si $x = 2$ alors conserver un volume de $V=20$, $h$ doit être égale à $5 / 2$
\item Si $x = 3$ alors conserver un volume de $V=20$, $h$ doit être égale à $5 / 3$
\item Si $x = 2$ alors conserver un volume de $V=16$, $h$ doit être égale à $8 / 2$
\item Si $x = 3$ alors conserver un volume de $V=16$, $h$ doit être égale à $8 / 3$
\end{itemize}
\item Pour calculer le volume, on a
\begin{eqnarray*}
V &=& h\times x \times 4 \\
20 &=& h\times x \times 4 \\
x &=& \frac{20}{h\times 4} = \frac{5}{h}
V &=& h\times x \times 2 \\
16 &=& h\times x \times 2 \\
x &=& \frac{16}{h\times 2} = \frac{8}{h}
\end{eqnarray*}
\item Pour calculer la surface totale, on ajoute la surface de chaque face. On a donc le calcul suivant
\begin{eqnarray*}
S(x) &=& x\times h \times 2 + x\times4\times2 + h\times 4\times 2\\
S(x) &=& x\times \frac{5}{x} \times 2 + x\times4\times2 + \frac{5}{x}\times 4\times 2\\
S(x) &=& 8x + 10 + \frac{40}{x}
S(x) &=& x\times h \times 2 + x\times2\times2 + h\times 2\times 2\\
S(x) &=& x\times \frac{8}{x} \times 2 + x\times2\times2 + \frac{8}{x}\times 2\times 2\\
S(x) &=& 4x + 16 + \frac{32}{x}
\end{eqnarray*}
\item Pour trouver cette nouvelle forme, on met chaque élément sur le même dénominateur
\begin{eqnarray*}
S(x) &=& 8x + 10 + \frac{40}{x}\\
S(x) &=& \frac{8x\times x}{x} + \frac{10\times x}{x} + \frac{40}{x}\\
S(x) &=& \frac{8x^2 + 10x + 40}{x}
S(x) &=& 4x + 16 + \frac{32}{x}\\
S(x) &=& \frac{4x\times x}{x} + \frac{16\times x}{x} + \frac{32}{x}\\
S(x) &=& \frac{4x^2 + 16x + 32}{x}
\end{eqnarray*}
\item On retrouve la formule $\frac{u}{v}$ à dériver
\[
u(x) = 8x^2 + 10x + 40 \Rightarrow u'(x) = 16x + 10
u(x) = 4x^2 + 16x + 32 \Rightarrow u'(x) = 8x + 16
\]
\[
v(x) = x \Rightarrow v'(x) = 1
\]
Donc au numérateur on obtient
\begin{eqnarray*}
u'(x)\times v(x) - u(x)\times v'(x) &=& (16x + 10)\times x - (8x^2 + 10x + 40)\times 1\\
&=& 8x^2 - 40
u'(x)\times v(x) - u(x)\times v'(x) &=& (8x + 16)\times x - (4x^2 + 16x + 32)\times 1\\
&=& 4x^2 - 32
\end{eqnarray*}
Donc
\[
S'(x) = \frac{8x^2 - 40}{x^2}
S'(x) = \frac{4x^2 - 32}{x^2}
\]
\item Tableau de variations de $S$
\begin{itemize}
\item Valeur interdite: $x^2 = 0 \equiv x = 0$
\item Signe de $8x^2 - 40$: c'est un polynôme du 2e degré
\item Signe de $4x^2 - 32$: c'est un polynôme du 2e degré
\[
\Delta = 1280 > 0
\Delta = 512 > 0
\]
Il y a donc 2 racines
\[
x_1 = - 2.23606797749979 \qquad
x_2 = 2.23606797749979
x_1 = - 2.8284271247461903 \qquad
x_2 = 2.8284271247461903
\]
Et on sait que $8x^2 - 40$ est du signe de $a$ donc positif en dehors des racines
Et on sait que $4x^2 - 32$ est du signe de $a$ donc positif en dehors des racines
\item Le dénominateur $x^2$ est toujours positif.
\item Tableau de variations
\begin{tikzpicture}[baseline=(a.north)]
\tkzTabInit[lgt=3,espcl=3]{$x$/1,$8x^2 - 40$/1, $x^2$/1, $S'$/1, $S$/2}{$0$, $- 2.23606797749979$, $10$}
\tkzTabInit[lgt=3,espcl=3]{$x$/1,$4x^2 - 32$/1, $x^2$/1, $S'$/1, $S$/2}{$0$, $- 2.8284271247461903$, $10$}
\tkzTabLine{d,-, z, +, }
\tkzTabLine{d,+, , +, }
\tkzTabLine{d,-, z, +, }
@ -120,7 +120,7 @@ Les valeurs des exercices sont générés automatiquement. Si une valeur a un no
\end{tikzpicture}
\end{itemize}
\item On a donc une surface minimal pour $x=2.23606797749979$ et $h = 11.18033988749895$.
\item On a donc une surface minimal pour $x=2.8284271247461903$ et $h = 22.6274169979695224$.
\end{enumerate}
\end{solution}
@ -133,14 +133,14 @@ Les valeurs des exercices sont générés automatiquement. Si une valeur a un no
Le tour d'un bassin au niveau du sol présente deux axes de symétrie : laxe des abscisses et la droite déquation $x=4$. Il est obtenu par symétrie de la courbe $\mathcal{C}_f$ sur $\intFF{0}{4}$$f$ est la fonction définie par
\[
f(x) = \left(- x^{2} + 9.0 x - 9.0\right) e^{- x} + 9.0
f(x) = \left(- x^{2} + 6.1 x - 2.6\right) e^{- x} + 2.6
\]
On admet que sur $\intFF{0}{4}$ la fonction $f$ est positive.
\begin{enumerate}
\item Sur un repère, tracer l'allure de la courbe $\mathcal{C}_f$, les axes de symétries puis compléter pour dessiner la forme du bassin.
\item Montrer que la fonction $f$ admet comme primitive sur $\R$ la fonction $F$ définie par
\[
F(x) = 9.0 x + \left( x^{2} - 7.0 x + 2.0\right) e^{- x}
F(x) = 2.6 x + \left( x^{2} - 4.1 x - 1.5\right) e^{- x}
\]
\item Calculer la quantité $\ds \int_0^4 f(x) \; dx$, vous donnerez le résultat sous forme exacte. Interpréter le résultat et reportez cette quantité sur le graphique.
\item On considère que l'échelle de votre graphique est de 1unité pour 15m. Calculer l'aire du bassin. Vous donnerez un résultat arrondi au $m^2$ près.
@ -156,15 +156,14 @@ Les valeurs des exercices sont générés automatiquement. Si une valeur a un no
\tkzGrid
\tkzAxeXY
\tkzFct[domain=0:10,color=red,very thick]%
{ (-x**2 + 9.0*x - 9.0)*exp(-x) + 9.0 };
{ (-x**2 + 6.1*x - 2.6)*exp(-x) + 2.6 };
\end{tikzpicture}
\item Il faut dériver $F(x)$ et vérifier que $F'(x) = f(x)$.
\item $\ds \int_0^4 f(x) \; dx = F(4) - F(0) = 34.0 - \frac{10.0}{e^{4}}$
\item $\ds \int_0^4 f(x) \; dx = F(4) - F(0) = 11.9 - \frac{1.9}{e^{4}}$
\item La quantité calculée à la question précédente se retrouve 4fois pour former le bassin. Il faut ensuite prendre en compte l'échelle, comme 1unité de longueur correspond à 15m, une unité d'air correspond à $15\times15 = 225m^2$. Ainsi l'aire du bassin est égale à
\[
(34.0 - \frac{10.0}{e^{4}})\times 4 \times 15^2 = 30435.00000
(11.9 - \frac{1.9}{e^{4}})\times 4 \times 15^2 = 10679.00000
\]
\end{enumerate}
\end{solution}
@ -185,9 +184,9 @@ Les valeurs des exercices sont générés automatiquement. Si une valeur a un no
Deux ateliers A et B fabriquent des stylos pour une entreprise.
L'atelier A fabrique 27.0\,\% des stylos, et parmi ceux-là, 39.0\,\% possèdent un défaut de fabrication.
L'atelier A fabrique 57.99999999999999\,\% des stylos, et parmi ceux-là, 5.0\,\% possèdent un défaut de fabrication.
De plus, 4.0\,\% des stylos possèdent un défaut de fabrication et sortent de l'atelier B.
De plus, 14.000000000000002\,\% des stylos possèdent un défaut de fabrication et sortent de l'atelier B.
Un stylo est prélevé au hasard dans le stock de l'entreprise.
@ -241,7 +240,7 @@ Les valeurs des exercices sont générés automatiquement. Si une valeur a un no
\item
\begin{enumerate}
\item Calculer la probabilité qu'un stylo provienne de l'atelier A et possède un défaut de fabrication.
\item En déduire que la probabilité qu'un stylo possède un défaut de fabrication est de $0.15$.
\item En déduire que la probabilité qu'un stylo possède un défaut de fabrication est de $0.17$.
\end{enumerate}
\item On prélève un stylo au hasard avec un défaut. Quelle est la probabilité qu'il vienne de l'atelier A?
\end{enumerate}
@ -251,7 +250,7 @@ Les valeurs des exercices sont générés automatiquement. Si une valeur a un no
\textbf{Partie B}
\medskip
Dans cette partie, on suppose que 15.0\,\% des stylos possèdent un défaut de fabrication.
Dans cette partie, on suppose que 17.0\,\% des stylos possèdent un défaut de fabrication.
L'entreprise confectionne des paquets contenant chacun $4$~stylos.
@ -266,7 +265,7 @@ Les valeurs des exercices sont générés automatiquement. Si une valeur a un no
\begin{enumerate}
\setcounter{enumi}{4}
\item Avec quelle loi peut-on modéliser $X$. Préciser les paramètres.
\item Calculer et interpréter la probabilité $P(X = 9)$.
\item Calculer et interpréter la probabilité $P(X = 17)$.
\item Le directeur de l'entreprise affirme qu'il y a plus d'une chance sur deux qu'un paquet ne comporte aucun stylo défectueux. A-t-il raison ?
\item Combien de stylos peut-on espérer avoir en moyenne?
\end{enumerate}
@ -281,19 +280,6 @@ Les valeurs des exercices sont générés automatiquement. Si une valeur a un no
\node {.}
child {node {$A$}
child {node {$D$}
edge from parent
node[above] {0.39}
}
child {node {$\overline{D}$}
edge from parent
node[above] {0.61}
}
edge from parent
node[above] {0.27}
}
child[missing] {}
child { node {$B$}
child {node {$D$}
edge from parent
node[above] {0.05}
}
@ -302,7 +288,20 @@ Les valeurs des exercices sont générés automatiquement. Si une valeur a un no
node[above] {0.95}
}
edge from parent
node[above] {0.73}
node[above] {0.58}
}
child[missing] {}
child { node {$B$}
child {node {$D$}
edge from parent
node[above] {0.33}
}
child {node {$\overline{D}$}
edge from parent
node[above] {0.67}
}
edge from parent
node[above] {0.42}
} ;
\end{tikzpicture}
\end{center}
@ -310,51 +309,51 @@ Les valeurs des exercices sont générés automatiquement. Si une valeur a un no
\begin{itemize}
\item Probabilité que le stylo vienne de l'atelier A
\[
P(A) = 0.27
P(A) = 0.58
\]
\item Probabilité que le stylo vienne de l'atelier B
\[
P(B) = 0.73
P(B) = 0.42
\]
\item Probabilité que le stylo ait un défaut sachant qu'il vient de l'atelier A.
\[
P_A(D) = 0.39
P_A(D) = 0.05
\]
\item Probabilité que le stylo vienne de l'atelier B et qu'il ait un défaut.
\[
P(D \cap D) = 0.04
P(D \cap D) = 0.14
\]
\end{itemize}
\item
\begin{enumerate}
\item Probabilité qu'un stylo vienne de l'atelier A et qu'il ait un defaut
\[
P(A\cap D) = P(A) \times P_A(D) = 0.27 \times 0.39 = 0.11
P(A\cap D) = P(A) \times P_A(D) = 0.58 \times 0.05 = 0.03
\]
\item Probabilité que le stylo ai un défaut de fabrication.
\[
P(D) = P(A\cap D) + P(B\cap D) = 0.11 + 0.04 = 0.15
P(D) = P(A\cap D) + P(B\cap D) = 0.03 + 0.14 = 0.17
\]
\end{enumerate}
\item Probabilité qu'il vienne de l'atelier A sachant qu'il a un defaut
\[
P_D(A) = \frac{P(A\cap D)}{P(D)} = \frac{0.11}{0.15} = 0.73
P_D(A) = \frac{P(A\cap D)}{P(D)} = \frac{0.03}{0.17} = 0.18
\]
\item $X$ peut être modélisée par une loi binomiale de paramètres $n=17$ et $p=0.15$.
\item $X$ peut être modélisée par une loi binomiale de paramètres $n=19$ et $p=0.17$.
\item (\textit{par de correction automatique disponible pour le résultat final}
\[
P(X = 9) = \coefBino{17}{9}\times 0.15^{9} \times 0.85^{8}
P(X = 17) = \coefBino{19}{17}\times 0.17^{17} \times 0.83^{2}
\]
\item (\textit{par de correction automatique disponible pour le résultat final}
Il faut calculer la probabilité qu'il y ait 0 stylo avec un defaut.
\[
P(X = 0) = \coefBino{17}{0}\times 0.15^{0} \times 0.85^{17}
P(X = 0) = \coefBino{19}{0}\times 0.17^{0} \times 0.83^{19}
\]
Puis comparer ce nombre à 0,5.
\item Il faut calculer l'espérance
\[
E[X] = n\times p = 17 \times 0.15 = 2.55
E[X] = n\times p = 19 \times 0.17 = 3.23
\]
\end{enumerate}
\end{solution}

View File

@ -3,7 +3,7 @@
% Title Page
\title{DM1 \hfill GERMAIN Margot}
\tribe{Maths complémentaire}
\tribe{Maths complémentaires}
\date{\hfillÀ render pour le jeudi 27 mai}
\xsimsetup{
@ -17,7 +17,7 @@ Les valeurs des exercices sont générés automatiquement. Si une valeur a un no
\begin{exercise}[subtitle={Optimisation de matière}]
\begin{minipage}{0.6\textwidth}
On se propose de fabriquer avec le moins de tôle possible une citerne fermée en forme de parallélépipède rectangle dont le volume intérieur doit être de $35m^3$. La longueur est aussi fixée à $5m$ par le cahier des charges.
On se propose de fabriquer avec le moins de tôle possible une citerne fermée en forme de parallélépipède rectangle dont le volume intérieur doit être de $45m^3$. La longueur est aussi fixée à $5m$ par le cahier des charges.
On peut donc faire varier uniquement la largeur (notée $x$) et la hauteur (notée $h$) de la cuve.
\end{minipage}
@ -35,18 +35,18 @@ Les valeurs des exercices sont générés automatiquement. Si une valeur a un no
\begin{enumerate}
\item Expliquer pourquoi quand la largeur $x$ change, la hauteur $h$ doit elle aussi changer pour respecter les contraintes.
\item Démontrer que l'on doit avoir $h = \dfrac{7}{x}$.
\item Démontrer que l'on doit avoir $h = \dfrac{9}{x}$.
\item On note $S(x)$ l'aire totale de la citerne (c'est à dire la somme des aires des six faces). Montrer que l'on peut écrire
\[
S(x) = 10x + 14 + \frac{70}{x}
S(x) = 10x + 18 + \frac{90}{x}
\]
\item Démontrer que
\[
S(x) = \frac{10x^2 + 14x + 70}{x}
S(x) = \frac{10x^2 + 18x + 90}{x}
\]
\item Démontrer que
\[
S'(x) = \frac{10x^2 - 70}{x^2}
S'(x) = \frac{10x^2 - 90}{x^2}
\]
\item En déduire le tableau de variation de $S(x)$ sur $\intOF{0}{10}$.
\item Déterminer les valeurs de $x$ et $h$ correspondant à une utilisation minimal de tôle.
@ -57,62 +57,62 @@ Les valeurs des exercices sont générés automatiquement. Si une valeur a un no
\begin{enumerate}
\item Le volume étant fixe si l'on fait varier $x$, $h$ doit aussi varier.
\begin{itemize}
\item Si $x = 2$ alors conserver un volume de $V=35$, $h$ doit être égale à $7 / 2$
\item Si $x = 3$ alors conserver un volume de $V=35$, $h$ doit être égale à $7 / 3$
\item Si $x = 2$ alors conserver un volume de $V=45$, $h$ doit être égale à $9 / 2$
\item Si $x = 3$ alors conserver un volume de $V=45$, $h$ doit être égale à $9 / 3$
\end{itemize}
\item Pour calculer le volume, on a
\begin{eqnarray*}
V &=& h\times x \times 5 \\
35 &=& h\times x \times 5 \\
x &=& \frac{35}{h\times 5} = \frac{7}{h}
45 &=& h\times x \times 5 \\
x &=& \frac{45}{h\times 5} = \frac{9}{h}
\end{eqnarray*}
\item Pour calculer la surface totale, on ajoute la surface de chaque face. On a donc le calcul suivant
\begin{eqnarray*}
S(x) &=& x\times h \times 2 + x\times5\times2 + h\times 5\times 2\\
S(x) &=& x\times \frac{7}{x} \times 2 + x\times5\times2 + \frac{7}{x}\times 5\times 2\\
S(x) &=& 10x + 14 + \frac{70}{x}
S(x) &=& x\times \frac{9}{x} \times 2 + x\times5\times2 + \frac{9}{x}\times 5\times 2\\
S(x) &=& 10x + 18 + \frac{90}{x}
\end{eqnarray*}
\item Pour trouver cette nouvelle forme, on met chaque élément sur le même dénominateur
\begin{eqnarray*}
S(x) &=& 10x + 14 + \frac{70}{x}\\
S(x) &=& \frac{10x\times x}{x} + \frac{14\times x}{x} + \frac{70}{x}\\
S(x) &=& \frac{10x^2 + 14x + 70}{x}
S(x) &=& 10x + 18 + \frac{90}{x}\\
S(x) &=& \frac{10x\times x}{x} + \frac{18\times x}{x} + \frac{90}{x}\\
S(x) &=& \frac{10x^2 + 18x + 90}{x}
\end{eqnarray*}
\item On retrouve la formule $\frac{u}{v}$ à dériver
\[
u(x) = 10x^2 + 14x + 70 \Rightarrow u'(x) = 20x + 14
u(x) = 10x^2 + 18x + 90 \Rightarrow u'(x) = 20x + 18
\]
\[
v(x) = x \Rightarrow v'(x) = 1
\]
Donc au numérateur on obtient
\begin{eqnarray*}
u'(x)\times v(x) - u(x)\times v'(x) &=& (20x + 14)\times x - (10x^2 + 14x + 70)\times 1\\
&=& 10x^2 - 70
u'(x)\times v(x) - u(x)\times v'(x) &=& (20x + 18)\times x - (10x^2 + 18x + 90)\times 1\\
&=& 10x^2 - 90
\end{eqnarray*}
Donc
\[
S'(x) = \frac{10x^2 - 70}{x^2}
S'(x) = \frac{10x^2 - 90}{x^2}
\]
\item Tableau de variations de $S$
\begin{itemize}
\item Valeur interdite: $x^2 = 0 \equiv x = 0$
\item Signe de $10x^2 - 70$: c'est un polynôme du 2e degré
\item Signe de $10x^2 - 90$: c'est un polynôme du 2e degré
\[
\Delta = 2800 > 0
\Delta = 3600 > 0
\]
Il y a donc 2 racines
\[
x_1 = - 2.6457513110645907 \qquad
x_2 = 2.6457513110645907
x_1 = - 3 \qquad
x_2 = 3
\]
Et on sait que $10x^2 - 70$ est du signe de $a$ donc positif en dehors des racines
Et on sait que $10x^2 - 90$ est du signe de $a$ donc positif en dehors des racines
\item Le dénominateur $x^2$ est toujours positif.
\item Tableau de variations
\begin{tikzpicture}[baseline=(a.north)]
\tkzTabInit[lgt=3,espcl=3]{$x$/1,$10x^2 - 70$/1, $x^2$/1, $S'$/1, $S$/2}{$0$, $- 2.6457513110645907$, $10$}
\tkzTabInit[lgt=3,espcl=3]{$x$/1,$10x^2 - 90$/1, $x^2$/1, $S'$/1, $S$/2}{$0$, $- 3$, $10$}
\tkzTabLine{d,-, z, +, }
\tkzTabLine{d,+, , +, }
\tkzTabLine{d,-, z, +, }
@ -120,7 +120,7 @@ Les valeurs des exercices sont générés automatiquement. Si une valeur a un no
\end{tikzpicture}
\end{itemize}
\item On a donc une surface minimal pour $x=2.6457513110645907$ et $h = 18.5202591774521349$.
\item On a donc une surface minimal pour $x=3$ et $h = 27$.
\end{enumerate}
\end{solution}
@ -133,14 +133,14 @@ Les valeurs des exercices sont générés automatiquement. Si une valeur a un no
Le tour d'un bassin au niveau du sol présente deux axes de symétrie : laxe des abscisses et la droite déquation $x=4$. Il est obtenu par symétrie de la courbe $\mathcal{C}_f$ sur $\intFF{0}{4}$$f$ est la fonction définie par
\[
f(x) = \left(- x^{2} + 8.0 x - 1.1\right) e^{- x} + 1.1
f(x) = \left(- x^{2} + 1.2 x - 9.5\right) e^{- x} + 9.5
\]
On admet que sur $\intFF{0}{4}$ la fonction $f$ est positive.
\begin{enumerate}
\item Sur un repère, tracer l'allure de la courbe $\mathcal{C}_f$, les axes de symétries puis compléter pour dessiner la forme du bassin.
\item Montrer que la fonction $f$ admet comme primitive sur $\R$ la fonction $F$ définie par
\[
F(x) = 1.1 x + \left( x^{2} - 6.0 x - 4.9\right) e^{- x}
F(x) = 9.5 x + \left( x^{2} + 0.8 x + 10.3\right) e^{- x}
\]
\item Calculer la quantité $\ds \int_0^4 f(x) \; dx$, vous donnerez le résultat sous forme exacte. Interpréter le résultat et reportez cette quantité sur le graphique.
\item On considère que l'échelle de votre graphique est de 1unité pour 15m. Calculer l'aire du bassin. Vous donnerez un résultat arrondi au $m^2$ près.
@ -156,15 +156,14 @@ Les valeurs des exercices sont générés automatiquement. Si une valeur a un no
\tkzGrid
\tkzAxeXY
\tkzFct[domain=0:10,color=red,very thick]%
{ (-x**2 + 8.0*x - 1.1)*exp(-x) + 1.1 };
{ (-x**2 + 1.2*x - 9.5)*exp(-x) + 9.5 };
\end{tikzpicture}
\item Il faut dériver $F(x)$ et vérifier que $F'(x) = f(x)$.
\item $\ds \int_0^4 f(x) \; dx = F(4) - F(0) = 9.3 - \frac{12.9}{e^{4}}$
\item $\ds \int_0^4 f(x) \; dx = F(4) - F(0) = \frac{29.5}{e^{4}} + 27.7$
\item La quantité calculée à la question précédente se retrouve 4fois pour former le bassin. Il faut ensuite prendre en compte l'échelle, comme 1unité de longueur correspond à 15m, une unité d'air correspond à $15\times15 = 225m^2$. Ainsi l'aire du bassin est égale à
\[
(9.3 - \frac{12.9}{e^{4}})\times 4 \times 15^2 = 8157.000000
(\frac{29.5}{e^{4}} + 27.7)\times 4 \times 15^2 = 25416.00000
\]
\end{enumerate}
\end{solution}
@ -185,9 +184,9 @@ Les valeurs des exercices sont générés automatiquement. Si une valeur a un no
Deux ateliers A et B fabriquent des stylos pour une entreprise.
L'atelier A fabrique 81.0\,\% des stylos, et parmi ceux-là, 57.99999999999999\,\% possèdent un défaut de fabrication.
L'atelier A fabrique 31.0\,\% des stylos, et parmi ceux-là, 4.0\,\% possèdent un défaut de fabrication.
De plus, 2.0\,\% des stylos possèdent un défaut de fabrication et sortent de l'atelier B.
De plus, 15.0\,\% des stylos possèdent un défaut de fabrication et sortent de l'atelier B.
Un stylo est prélevé au hasard dans le stock de l'entreprise.
@ -241,7 +240,7 @@ Les valeurs des exercices sont générés automatiquement. Si une valeur a un no
\item
\begin{enumerate}
\item Calculer la probabilité qu'un stylo provienne de l'atelier A et possède un défaut de fabrication.
\item En déduire que la probabilité qu'un stylo possède un défaut de fabrication est de $0.49$.
\item En déduire que la probabilité qu'un stylo possède un défaut de fabrication est de $0.16$.
\end{enumerate}
\item On prélève un stylo au hasard avec un défaut. Quelle est la probabilité qu'il vienne de l'atelier A?
\end{enumerate}
@ -251,7 +250,7 @@ Les valeurs des exercices sont générés automatiquement. Si une valeur a un no
\textbf{Partie B}
\medskip
Dans cette partie, on suppose que 49.0\,\% des stylos possèdent un défaut de fabrication.
Dans cette partie, on suppose que 16.0\,\% des stylos possèdent un défaut de fabrication.
L'entreprise confectionne des paquets contenant chacun $4$~stylos.
@ -266,7 +265,7 @@ Les valeurs des exercices sont générés automatiquement. Si une valeur a un no
\begin{enumerate}
\setcounter{enumi}{4}
\item Avec quelle loi peut-on modéliser $X$. Préciser les paramètres.
\item Calculer et interpréter la probabilité $P(X = 14)$.
\item Calculer et interpréter la probabilité $P(X = 13)$.
\item Le directeur de l'entreprise affirme qu'il y a plus d'une chance sur deux qu'un paquet ne comporte aucun stylo défectueux. A-t-il raison ?
\item Combien de stylos peut-on espérer avoir en moyenne?
\end{enumerate}
@ -282,27 +281,27 @@ Les valeurs des exercices sont générés automatiquement. Si une valeur a un no
child {node {$A$}
child {node {$D$}
edge from parent
node[above] {0.58}
node[above] {0.04}
}
child {node {$\overline{D}$}
edge from parent
node[above] {0.42}
node[above] {0.96}
}
edge from parent
node[above] {0.81}
node[above] {0.31}
}
child[missing] {}
child { node {$B$}
child {node {$D$}
edge from parent
node[above] {0.13}
node[above] {0.22}
}
child {node {$\overline{D}$}
edge from parent
node[above] {0.87}
node[above] {0.78}
}
edge from parent
node[above] {0.19}
node[above] {0.69}
} ;
\end{tikzpicture}
\end{center}
@ -310,51 +309,51 @@ Les valeurs des exercices sont générés automatiquement. Si une valeur a un no
\begin{itemize}
\item Probabilité que le stylo vienne de l'atelier A
\[
P(A) = 0.81
P(A) = 0.31
\]
\item Probabilité que le stylo vienne de l'atelier B
\[
P(B) = 0.19
P(B) = 0.69
\]
\item Probabilité que le stylo ait un défaut sachant qu'il vient de l'atelier A.
\[
P_A(D) = 0.58
P_A(D) = 0.04
\]
\item Probabilité que le stylo vienne de l'atelier B et qu'il ait un défaut.
\[
P(D \cap D) = 0.02
P(D \cap D) = 0.15
\]
\end{itemize}
\item
\begin{enumerate}
\item Probabilité qu'un stylo vienne de l'atelier A et qu'il ait un defaut
\[
P(A\cap D) = P(A) \times P_A(D) = 0.81 \times 0.58 = 0.47
P(A\cap D) = P(A) \times P_A(D) = 0.31 \times 0.04 = 0.01
\]
\item Probabilité que le stylo ai un défaut de fabrication.
\[
P(D) = P(A\cap D) + P(B\cap D) = 0.47 + 0.02 = 0.49
P(D) = P(A\cap D) + P(B\cap D) = 0.01 + 0.15 = 0.16
\]
\end{enumerate}
\item Probabilité qu'il vienne de l'atelier A sachant qu'il a un defaut
\[
P_D(A) = \frac{P(A\cap D)}{P(D)} = \frac{0.47}{0.49} = 0.96
P_D(A) = \frac{P(A\cap D)}{P(D)} = \frac{0.01}{0.16} = 0.06
\]
\item $X$ peut être modélisée par une loi binomiale de paramètres $n=20$ et $p=0.49$.
\item $X$ peut être modélisée par une loi binomiale de paramètres $n=14$ et $p=0.16$.
\item (\textit{par de correction automatique disponible pour le résultat final}
\[
P(X = 14) = \coefBino{20}{14}\times 0.49^{14} \times 0.51^{6}
P(X = 13) = \coefBino{14}{13}\times 0.16^{13} \times 0.84^{1}
\]
\item (\textit{par de correction automatique disponible pour le résultat final}
Il faut calculer la probabilité qu'il y ait 0 stylo avec un defaut.
\[
P(X = 0) = \coefBino{20}{0}\times 0.49^{0} \times 0.51^{20}
P(X = 0) = \coefBino{14}{0}\times 0.16^{0} \times 0.84^{14}
\]
Puis comparer ce nombre à 0,5.
\item Il faut calculer l'espérance
\[
E[X] = n\times p = 20 \times 0.49 = 9.8
E[X] = n\times p = 14 \times 0.16 = 2.24
\]
\end{enumerate}
\end{solution}

View File

@ -3,7 +3,7 @@
% Title Page
\title{DM1 \hfill HOKELEKLI Damla}
\tribe{Maths complémentaire}
\tribe{Maths complémentaires}
\date{\hfillÀ render pour le jeudi 27 mai}
\xsimsetup{
@ -17,7 +17,7 @@ Les valeurs des exercices sont générés automatiquement. Si une valeur a un no
\begin{exercise}[subtitle={Optimisation de matière}]
\begin{minipage}{0.6\textwidth}
On se propose de fabriquer avec le moins de tôle possible une citerne fermée en forme de parallélépipède rectangle dont le volume intérieur doit être de $10m^3$. La longueur est aussi fixée à $2m$ par le cahier des charges.
On se propose de fabriquer avec le moins de tôle possible une citerne fermée en forme de parallélépipède rectangle dont le volume intérieur doit être de $21m^3$. La longueur est aussi fixée à $3m$ par le cahier des charges.
On peut donc faire varier uniquement la largeur (notée $x$) et la hauteur (notée $h$) de la cuve.
\end{minipage}
@ -28,25 +28,25 @@ Les valeurs des exercices sont générés automatiquement. Si une valeur a un no
\pgfmathsetmacro{\cubey}{1}
\pgfmathsetmacro{\cubez}{2}
\draw[black,fill=gray] (0,0,0) -- ++(-\cubex,0,0) -- ++(0,-\cubey,0) node [midway, left] {$h$} -- ++(\cubex,0,0) node [midway, below] {$x$} -- cycle;
\draw[black,fill=gray] (0,0,0) -- ++(0,0,-\cubez) -- ++(0,-\cubey,0) -- ++(0,0,\cubez) node [midway, right] {$2m$} -- cycle;
\draw[black,fill=gray] (0,0,0) -- ++(0,0,-\cubez) -- ++(0,-\cubey,0) -- ++(0,0,\cubez) node [midway, right] {$3m$} -- cycle;
\draw[black,fill=gray] (0,0,0) -- ++(-\cubex,0,0) -- ++(0,0,-\cubez) -- ++(\cubex,0,0) -- cycle;
\end{tikzpicture}
\end{minipage}
\begin{enumerate}
\item Expliquer pourquoi quand la largeur $x$ change, la hauteur $h$ doit elle aussi changer pour respecter les contraintes.
\item Démontrer que l'on doit avoir $h = \dfrac{5}{x}$.
\item Démontrer que l'on doit avoir $h = \dfrac{7}{x}$.
\item On note $S(x)$ l'aire totale de la citerne (c'est à dire la somme des aires des six faces). Montrer que l'on peut écrire
\[
S(x) = 4x + 10 + \frac{20}{x}
S(x) = 6x + 14 + \frac{42}{x}
\]
\item Démontrer que
\[
S(x) = \frac{4x^2 + 10x + 20}{x}
S(x) = \frac{6x^2 + 14x + 42}{x}
\]
\item Démontrer que
\[
S'(x) = \frac{4x^2 - 20}{x^2}
S'(x) = \frac{6x^2 - 42}{x^2}
\]
\item En déduire le tableau de variation de $S(x)$ sur $\intOF{0}{10}$.
\item Déterminer les valeurs de $x$ et $h$ correspondant à une utilisation minimal de tôle.
@ -57,62 +57,62 @@ Les valeurs des exercices sont générés automatiquement. Si une valeur a un no
\begin{enumerate}
\item Le volume étant fixe si l'on fait varier $x$, $h$ doit aussi varier.
\begin{itemize}
\item Si $x = 2$ alors conserver un volume de $V=10$, $h$ doit être égale à $5 / 2$
\item Si $x = 3$ alors conserver un volume de $V=10$, $h$ doit être égale à $5 / 3$
\item Si $x = 2$ alors conserver un volume de $V=21$, $h$ doit être égale à $7 / 2$
\item Si $x = 3$ alors conserver un volume de $V=21$, $h$ doit être égale à $7 / 3$
\end{itemize}
\item Pour calculer le volume, on a
\begin{eqnarray*}
V &=& h\times x \times 2 \\
10 &=& h\times x \times 2 \\
x &=& \frac{10}{h\times 2} = \frac{5}{h}
V &=& h\times x \times 3 \\
21 &=& h\times x \times 3 \\
x &=& \frac{21}{h\times 3} = \frac{7}{h}
\end{eqnarray*}
\item Pour calculer la surface totale, on ajoute la surface de chaque face. On a donc le calcul suivant
\begin{eqnarray*}
S(x) &=& x\times h \times 2 + x\times2\times2 + h\times 2\times 2\\
S(x) &=& x\times \frac{5}{x} \times 2 + x\times2\times2 + \frac{5}{x}\times 2\times 2\\
S(x) &=& 4x + 10 + \frac{20}{x}
S(x) &=& x\times h \times 2 + x\times3\times2 + h\times 3\times 2\\
S(x) &=& x\times \frac{7}{x} \times 2 + x\times3\times2 + \frac{7}{x}\times 3\times 2\\
S(x) &=& 6x + 14 + \frac{42}{x}
\end{eqnarray*}
\item Pour trouver cette nouvelle forme, on met chaque élément sur le même dénominateur
\begin{eqnarray*}
S(x) &=& 4x + 10 + \frac{20}{x}\\
S(x) &=& \frac{4x\times x}{x} + \frac{10\times x}{x} + \frac{20}{x}\\
S(x) &=& \frac{4x^2 + 10x + 20}{x}
S(x) &=& 6x + 14 + \frac{42}{x}\\
S(x) &=& \frac{6x\times x}{x} + \frac{14\times x}{x} + \frac{42}{x}\\
S(x) &=& \frac{6x^2 + 14x + 42}{x}
\end{eqnarray*}
\item On retrouve la formule $\frac{u}{v}$ à dériver
\[
u(x) = 4x^2 + 10x + 20 \Rightarrow u'(x) = 8x + 10
u(x) = 6x^2 + 14x + 42 \Rightarrow u'(x) = 12x + 14
\]
\[
v(x) = x \Rightarrow v'(x) = 1
\]
Donc au numérateur on obtient
\begin{eqnarray*}
u'(x)\times v(x) - u(x)\times v'(x) &=& (8x + 10)\times x - (4x^2 + 10x + 20)\times 1\\
&=& 4x^2 - 20
u'(x)\times v(x) - u(x)\times v'(x) &=& (12x + 14)\times x - (6x^2 + 14x + 42)\times 1\\
&=& 6x^2 - 42
\end{eqnarray*}
Donc
\[
S'(x) = \frac{4x^2 - 20}{x^2}
S'(x) = \frac{6x^2 - 42}{x^2}
\]
\item Tableau de variations de $S$
\begin{itemize}
\item Valeur interdite: $x^2 = 0 \equiv x = 0$
\item Signe de $4x^2 - 20$: c'est un polynôme du 2e degré
\item Signe de $6x^2 - 42$: c'est un polynôme du 2e degré
\[
\Delta = 320 > 0
\Delta = 1008 > 0
\]
Il y a donc 2 racines
\[
x_1 = - 2.23606797749979 \qquad
x_2 = 2.23606797749979
x_1 = - 2.6457513110645907 \qquad
x_2 = 2.6457513110645907
\]
Et on sait que $4x^2 - 20$ est du signe de $a$ donc positif en dehors des racines
Et on sait que $6x^2 - 42$ est du signe de $a$ donc positif en dehors des racines
\item Le dénominateur $x^2$ est toujours positif.
\item Tableau de variations
\begin{tikzpicture}[baseline=(a.north)]
\tkzTabInit[lgt=3,espcl=3]{$x$/1,$4x^2 - 20$/1, $x^2$/1, $S'$/1, $S$/2}{$0$, $- 2.23606797749979$, $10$}
\tkzTabInit[lgt=3,espcl=3]{$x$/1,$6x^2 - 42$/1, $x^2$/1, $S'$/1, $S$/2}{$0$, $- 2.6457513110645907$, $10$}
\tkzTabLine{d,-, z, +, }
\tkzTabLine{d,+, , +, }
\tkzTabLine{d,-, z, +, }
@ -120,7 +120,7 @@ Les valeurs des exercices sont générés automatiquement. Si une valeur a un no
\end{tikzpicture}
\end{itemize}
\item On a donc une surface minimal pour $x=2.23606797749979$ et $h = 11.18033988749895$.
\item On a donc une surface minimal pour $x=2.6457513110645907$ et $h = 18.5202591774521349$.
\end{enumerate}
\end{solution}
@ -133,14 +133,14 @@ Les valeurs des exercices sont générés automatiquement. Si une valeur a un no
Le tour d'un bassin au niveau du sol présente deux axes de symétrie : laxe des abscisses et la droite déquation $x=4$. Il est obtenu par symétrie de la courbe $\mathcal{C}_f$ sur $\intFF{0}{4}$$f$ est la fonction définie par
\[
f(x) = \left(- x^{2} + 5.6 x - 1.6\right) e^{- x} + 1.6
f(x) = \left(- x^{2} + 2.6 x - 4.1\right) e^{- x} + 4.1
\]
On admet que sur $\intFF{0}{4}$ la fonction $f$ est positive.
\begin{enumerate}
\item Sur un repère, tracer l'allure de la courbe $\mathcal{C}_f$, les axes de symétries puis compléter pour dessiner la forme du bassin.
\item Montrer que la fonction $f$ admet comme primitive sur $\R$ la fonction $F$ définie par
\[
F(x) = 1.6 x + \left( x^{2} - 3.6 x - 2.0\right) e^{- x}
F(x) = 4.1 x + \left( x^{2} - 0.6 x + 3.5\right) e^{- x}
\]
\item Calculer la quantité $\ds \int_0^4 f(x) \; dx$, vous donnerez le résultat sous forme exacte. Interpréter le résultat et reportez cette quantité sur le graphique.
\item On considère que l'échelle de votre graphique est de 1unité pour 15m. Calculer l'aire du bassin. Vous donnerez un résultat arrondi au $m^2$ près.
@ -156,15 +156,14 @@ Les valeurs des exercices sont générés automatiquement. Si une valeur a un no
\tkzGrid
\tkzAxeXY
\tkzFct[domain=0:10,color=red,very thick]%
{ (-x**2 + 5.6*x - 1.6)*exp(-x) + 1.6 };
{ (-x**2 + 2.6*x - 4.1)*exp(-x) + 4.1 };
\end{tikzpicture}
\item Il faut dériver $F(x)$ et vérifier que $F'(x) = f(x)$.
\item $\ds \int_0^4 f(x) \; dx = F(4) - F(0) = 8.4 - \frac{0.399999999999999}{e^{4}}$
\item $\ds \int_0^4 f(x) \; dx = F(4) - F(0) = \frac{17.1}{e^{4}} + 12.9$
\item La quantité calculée à la question précédente se retrouve 4fois pour former le bassin. Il faut ensuite prendre en compte l'échelle, comme 1unité de longueur correspond à 15m, une unité d'air correspond à $15\times15 = 225m^2$. Ainsi l'aire du bassin est égale à
\[
(8.4 - \frac{0.399999999999999}{e^{4}})\times 4 \times 15^2 = 7553.000000
(\frac{17.1}{e^{4}} + 12.9)\times 4 \times 15^2 = 11892.00000
\]
\end{enumerate}
\end{solution}
@ -185,9 +184,9 @@ Les valeurs des exercices sont générés automatiquement. Si une valeur a un no
Deux ateliers A et B fabriquent des stylos pour une entreprise.
L'atelier A fabrique 47.0\,\% des stylos, et parmi ceux-là, 25.0\,\% possèdent un défaut de fabrication.
L'atelier A fabrique 26.0\,\% des stylos, et parmi ceux-là, 60.0\,\% possèdent un défaut de fabrication.
De plus, 23.0\,\% des stylos possèdent un défaut de fabrication et sortent de l'atelier B.
De plus, 74.0\,\% des stylos possèdent un défaut de fabrication et sortent de l'atelier B.
Un stylo est prélevé au hasard dans le stock de l'entreprise.
@ -241,7 +240,7 @@ Les valeurs des exercices sont générés automatiquement. Si une valeur a un no
\item
\begin{enumerate}
\item Calculer la probabilité qu'un stylo provienne de l'atelier A et possède un défaut de fabrication.
\item En déduire que la probabilité qu'un stylo possède un défaut de fabrication est de $0.35$.
\item En déduire que la probabilité qu'un stylo possède un défaut de fabrication est de $0.9$.
\end{enumerate}
\item On prélève un stylo au hasard avec un défaut. Quelle est la probabilité qu'il vienne de l'atelier A?
\end{enumerate}
@ -251,7 +250,7 @@ Les valeurs des exercices sont générés automatiquement. Si une valeur a un no
\textbf{Partie B}
\medskip
Dans cette partie, on suppose que 35.0\,\% des stylos possèdent un défaut de fabrication.
Dans cette partie, on suppose que 90.0\,\% des stylos possèdent un défaut de fabrication.
L'entreprise confectionne des paquets contenant chacun $4$~stylos.
@ -266,7 +265,7 @@ Les valeurs des exercices sont générés automatiquement. Si une valeur a un no
\begin{enumerate}
\setcounter{enumi}{4}
\item Avec quelle loi peut-on modéliser $X$. Préciser les paramètres.
\item Calculer et interpréter la probabilité $P(X = 17)$.
\item Calculer et interpréter la probabilité $P(X = 13)$.
\item Le directeur de l'entreprise affirme qu'il y a plus d'une chance sur deux qu'un paquet ne comporte aucun stylo défectueux. A-t-il raison ?
\item Combien de stylos peut-on espérer avoir en moyenne?
\end{enumerate}
@ -282,27 +281,27 @@ Les valeurs des exercices sont générés automatiquement. Si une valeur a un no
child {node {$A$}
child {node {$D$}
edge from parent
node[above] {0.25}
node[above] {0.6}
}
child {node {$\overline{D}$}
edge from parent
node[above] {0.75}
node[above] {0.4}
}
edge from parent
node[above] {0.47}
node[above] {0.26}
}
child[missing] {}
child { node {$B$}
child {node {$D$}
edge from parent
node[above] {0.44}
node[above] {1.0}
}
child {node {$\overline{D}$}
edge from parent
node[above] {0.56}
node[above] {0.0}
}
edge from parent
node[above] {0.53}
node[above] {0.74}
} ;
\end{tikzpicture}
\end{center}
@ -310,51 +309,51 @@ Les valeurs des exercices sont générés automatiquement. Si une valeur a un no
\begin{itemize}
\item Probabilité que le stylo vienne de l'atelier A
\[
P(A) = 0.47
P(A) = 0.26
\]
\item Probabilité que le stylo vienne de l'atelier B
\[
P(B) = 0.53
P(B) = 0.74
\]
\item Probabilité que le stylo ait un défaut sachant qu'il vient de l'atelier A.
\[
P_A(D) = 0.25
P_A(D) = 0.6
\]
\item Probabilité que le stylo vienne de l'atelier B et qu'il ait un défaut.
\[
P(D \cap D) = 0.23
P(D \cap D) = 0.74
\]
\end{itemize}
\item
\begin{enumerate}
\item Probabilité qu'un stylo vienne de l'atelier A et qu'il ait un defaut
\[
P(A\cap D) = P(A) \times P_A(D) = 0.47 \times 0.25 = 0.12
P(A\cap D) = P(A) \times P_A(D) = 0.26 \times 0.6 = 0.16
\]
\item Probabilité que le stylo ai un défaut de fabrication.
\[
P(D) = P(A\cap D) + P(B\cap D) = 0.12 + 0.23 = 0.35
P(D) = P(A\cap D) + P(B\cap D) = 0.16 + 0.74 = 0.9
\]
\end{enumerate}
\item Probabilité qu'il vienne de l'atelier A sachant qu'il a un defaut
\[
P_D(A) = \frac{P(A\cap D)}{P(D)} = \frac{0.12}{0.35} = 0.34
P_D(A) = \frac{P(A\cap D)}{P(D)} = \frac{0.16}{0.9} = 0.18
\]
\item $X$ peut être modélisée par une loi binomiale de paramètres $n=19$ et $p=0.35$.
\item $X$ peut être modélisée par une loi binomiale de paramètres $n=15$ et $p=0.9$.
\item (\textit{par de correction automatique disponible pour le résultat final}
\[
P(X = 17) = \coefBino{19}{17}\times 0.35^{17} \times 0.65^{2}
P(X = 13) = \coefBino{15}{13}\times 0.9^{13} \times 0.1^{2}
\]
\item (\textit{par de correction automatique disponible pour le résultat final}
Il faut calculer la probabilité qu'il y ait 0 stylo avec un defaut.
\[
P(X = 0) = \coefBino{19}{0}\times 0.35^{0} \times 0.65^{19}
P(X = 0) = \coefBino{15}{0}\times 0.9^{0} \times 0.1^{15}
\]
Puis comparer ce nombre à 0,5.
\item Il faut calculer l'espérance
\[
E[X] = n\times p = 19 \times 0.35 = 6.65
E[X] = n\times p = 15 \times 0.9 = 13.5
\]
\end{enumerate}
\end{solution}

View File

@ -3,7 +3,7 @@
% Title Page
\title{DM1 \hfill KICHENASSAMY Kévin}
\tribe{Maths complémentaire}
\tribe{Maths complémentaires}
\date{\hfillÀ render pour le jeudi 27 mai}
\xsimsetup{
@ -133,14 +133,14 @@ Les valeurs des exercices sont générés automatiquement. Si une valeur a un no
Le tour d'un bassin au niveau du sol présente deux axes de symétrie : laxe des abscisses et la droite déquation $x=4$. Il est obtenu par symétrie de la courbe $\mathcal{C}_f$ sur $\intFF{0}{4}$$f$ est la fonction définie par
\[
f(x) = \left(- x^{2} + 0.4 x - 6.0\right) e^{- x} + 6.0
f(x) = \left(- x^{2} + 8.1 x - 0.8\right) e^{- x} + 0.8
\]
On admet que sur $\intFF{0}{4}$ la fonction $f$ est positive.
\begin{enumerate}
\item Sur un repère, tracer l'allure de la courbe $\mathcal{C}_f$, les axes de symétries puis compléter pour dessiner la forme du bassin.
\item Montrer que la fonction $f$ admet comme primitive sur $\R$ la fonction $F$ définie par
\[
F(x) = 6.0 x + \left( x^{2} + 1.6 x + 7.6\right) e^{- x}
F(x) = 0.8 x + \left( x^{2} - 6.1 x - 5.3\right) e^{- x}
\]
\item Calculer la quantité $\ds \int_0^4 f(x) \; dx$, vous donnerez le résultat sous forme exacte. Interpréter le résultat et reportez cette quantité sur le graphique.
\item On considère que l'échelle de votre graphique est de 1unité pour 15m. Calculer l'aire du bassin. Vous donnerez un résultat arrondi au $m^2$ près.
@ -156,15 +156,14 @@ Les valeurs des exercices sont générés automatiquement. Si une valeur a un no
\tkzGrid
\tkzAxeXY
\tkzFct[domain=0:10,color=red,very thick]%
{ (-x**2 + 0.4*x - 6.0)*exp(-x) + 6.0 };
{ (-x**2 + 8.1*x - 0.8)*exp(-x) + 0.8 };
\end{tikzpicture}
\item Il faut dériver $F(x)$ et vérifier que $F'(x) = f(x)$.
\item $\ds \int_0^4 f(x) \; dx = F(4) - F(0) = \frac{30.0}{e^{4}} + 16.4$
\item $\ds \int_0^4 f(x) \; dx = F(4) - F(0) = 8.5 - \frac{13.7}{e^{4}}$
\item La quantité calculée à la question précédente se retrouve 4fois pour former le bassin. Il faut ensuite prendre en compte l'échelle, comme 1unité de longueur correspond à 15m, une unité d'air correspond à $15\times15 = 225m^2$. Ainsi l'aire du bassin est égale à
\[
(\frac{30.0}{e^{4}} + 16.4)\times 4 \times 15^2 = 15255.00000
(8.5 - \frac{13.7}{e^{4}})\times 4 \times 15^2 = 7424.000000
\]
\end{enumerate}
\end{solution}
@ -185,9 +184,9 @@ Les valeurs des exercices sont générés automatiquement. Si une valeur a un no
Deux ateliers A et B fabriquent des stylos pour une entreprise.
L'atelier A fabrique 84.0\,\% des stylos, et parmi ceux-là, 85.0\,\% possèdent un défaut de fabrication.
L'atelier A fabrique 68.0\,\% des stylos, et parmi ceux-là, 79.0\,\% possèdent un défaut de fabrication.
De plus, 4.0\,\% des stylos possèdent un défaut de fabrication et sortent de l'atelier B.
De plus, 26.0\,\% des stylos possèdent un défaut de fabrication et sortent de l'atelier B.
Un stylo est prélevé au hasard dans le stock de l'entreprise.
@ -241,7 +240,7 @@ Les valeurs des exercices sont générés automatiquement. Si une valeur a un no
\item
\begin{enumerate}
\item Calculer la probabilité qu'un stylo provienne de l'atelier A et possède un défaut de fabrication.
\item En déduire que la probabilité qu'un stylo possède un défaut de fabrication est de $0.75$.
\item En déduire que la probabilité qu'un stylo possède un défaut de fabrication est de $0.8$.
\end{enumerate}
\item On prélève un stylo au hasard avec un défaut. Quelle est la probabilité qu'il vienne de l'atelier A?
\end{enumerate}
@ -251,7 +250,7 @@ Les valeurs des exercices sont générés automatiquement. Si une valeur a un no
\textbf{Partie B}
\medskip
Dans cette partie, on suppose que 75.0\,\% des stylos possèdent un défaut de fabrication.
Dans cette partie, on suppose que 80.0\,\% des stylos possèdent un défaut de fabrication.
L'entreprise confectionne des paquets contenant chacun $4$~stylos.
@ -266,7 +265,7 @@ Les valeurs des exercices sont générés automatiquement. Si une valeur a un no
\begin{enumerate}
\setcounter{enumi}{4}
\item Avec quelle loi peut-on modéliser $X$. Préciser les paramètres.
\item Calculer et interpréter la probabilité $P(X = 14)$.
\item Calculer et interpréter la probabilité $P(X = 12)$.
\item Le directeur de l'entreprise affirme qu'il y a plus d'une chance sur deux qu'un paquet ne comporte aucun stylo défectueux. A-t-il raison ?
\item Combien de stylos peut-on espérer avoir en moyenne?
\end{enumerate}
@ -282,27 +281,27 @@ Les valeurs des exercices sont générés automatiquement. Si une valeur a un no
child {node {$A$}
child {node {$D$}
edge from parent
node[above] {0.85}
node[above] {0.79}
}
child {node {$\overline{D}$}
edge from parent
node[above] {0.15}
node[above] {0.21}
}
edge from parent
node[above] {0.84}
node[above] {0.68}
}
child[missing] {}
child { node {$B$}
child {node {$D$}
edge from parent
node[above] {0.26}
node[above] {0.81}
}
child {node {$\overline{D}$}
edge from parent
node[above] {0.74}
node[above] {0.19}
}
edge from parent
node[above] {0.16}
node[above] {0.32}
} ;
\end{tikzpicture}
\end{center}
@ -310,51 +309,51 @@ Les valeurs des exercices sont générés automatiquement. Si une valeur a un no
\begin{itemize}
\item Probabilité que le stylo vienne de l'atelier A
\[
P(A) = 0.84
P(A) = 0.68
\]
\item Probabilité que le stylo vienne de l'atelier B
\[
P(B) = 0.16
P(B) = 0.32
\]
\item Probabilité que le stylo ait un défaut sachant qu'il vient de l'atelier A.
\[
P_A(D) = 0.85
P_A(D) = 0.79
\]
\item Probabilité que le stylo vienne de l'atelier B et qu'il ait un défaut.
\[
P(D \cap D) = 0.04
P(D \cap D) = 0.26
\]
\end{itemize}
\item
\begin{enumerate}
\item Probabilité qu'un stylo vienne de l'atelier A et qu'il ait un defaut
\[
P(A\cap D) = P(A) \times P_A(D) = 0.84 \times 0.85 = 0.71
P(A\cap D) = P(A) \times P_A(D) = 0.68 \times 0.79 = 0.54
\]
\item Probabilité que le stylo ai un défaut de fabrication.
\[
P(D) = P(A\cap D) + P(B\cap D) = 0.71 + 0.04 = 0.75
P(D) = P(A\cap D) + P(B\cap D) = 0.54 + 0.26 = 0.8
\]
\end{enumerate}
\item Probabilité qu'il vienne de l'atelier A sachant qu'il a un defaut
\[
P_D(A) = \frac{P(A\cap D)}{P(D)} = \frac{0.71}{0.75} = 0.95
P_D(A) = \frac{P(A\cap D)}{P(D)} = \frac{0.54}{0.8} = 0.68
\]
\item $X$ peut être modélisée par une loi binomiale de paramètres $n=17$ et $p=0.75$.
\item $X$ peut être modélisée par une loi binomiale de paramètres $n=16$ et $p=0.8$.
\item (\textit{par de correction automatique disponible pour le résultat final}
\[
P(X = 14) = \coefBino{17}{14}\times 0.75^{14} \times 0.25^{3}
P(X = 12) = \coefBino{16}{12}\times 0.8^{12} \times 0.2^{4}
\]
\item (\textit{par de correction automatique disponible pour le résultat final}
Il faut calculer la probabilité qu'il y ait 0 stylo avec un defaut.
\[
P(X = 0) = \coefBino{17}{0}\times 0.75^{0} \times 0.25^{17}
P(X = 0) = \coefBino{16}{0}\times 0.8^{0} \times 0.2^{16}
\]
Puis comparer ce nombre à 0,5.
\item Il faut calculer l'espérance
\[
E[X] = n\times p = 17 \times 0.75 = 12.75
E[X] = n\times p = 16 \times 0.8 = 12.8
\]
\end{enumerate}
\end{solution}

View File

@ -3,7 +3,7 @@
% Title Page
\title{DM1 \hfill MATHIEU Allan}
\tribe{Maths complémentaire}
\tribe{Maths complémentaires}
\date{\hfillÀ render pour le jeudi 27 mai}
\xsimsetup{
@ -17,7 +17,7 @@ Les valeurs des exercices sont générés automatiquement. Si une valeur a un no
\begin{exercise}[subtitle={Optimisation de matière}]
\begin{minipage}{0.6\textwidth}
On se propose de fabriquer avec le moins de tôle possible une citerne fermée en forme de parallélépipède rectangle dont le volume intérieur doit être de $40m^3$. La longueur est aussi fixée à $4m$ par le cahier des charges.
On se propose de fabriquer avec le moins de tôle possible une citerne fermée en forme de parallélépipède rectangle dont le volume intérieur doit être de $24m^3$. La longueur est aussi fixée à $3m$ par le cahier des charges.
On peut donc faire varier uniquement la largeur (notée $x$) et la hauteur (notée $h$) de la cuve.
\end{minipage}
@ -28,25 +28,25 @@ Les valeurs des exercices sont générés automatiquement. Si une valeur a un no
\pgfmathsetmacro{\cubey}{1}
\pgfmathsetmacro{\cubez}{2}
\draw[black,fill=gray] (0,0,0) -- ++(-\cubex,0,0) -- ++(0,-\cubey,0) node [midway, left] {$h$} -- ++(\cubex,0,0) node [midway, below] {$x$} -- cycle;
\draw[black,fill=gray] (0,0,0) -- ++(0,0,-\cubez) -- ++(0,-\cubey,0) -- ++(0,0,\cubez) node [midway, right] {$4m$} -- cycle;
\draw[black,fill=gray] (0,0,0) -- ++(0,0,-\cubez) -- ++(0,-\cubey,0) -- ++(0,0,\cubez) node [midway, right] {$3m$} -- cycle;
\draw[black,fill=gray] (0,0,0) -- ++(-\cubex,0,0) -- ++(0,0,-\cubez) -- ++(\cubex,0,0) -- cycle;
\end{tikzpicture}
\end{minipage}
\begin{enumerate}
\item Expliquer pourquoi quand la largeur $x$ change, la hauteur $h$ doit elle aussi changer pour respecter les contraintes.
\item Démontrer que l'on doit avoir $h = \dfrac{10}{x}$.
\item Démontrer que l'on doit avoir $h = \dfrac{8}{x}$.
\item On note $S(x)$ l'aire totale de la citerne (c'est à dire la somme des aires des six faces). Montrer que l'on peut écrire
\[
S(x) = 8x + 20 + \frac{80}{x}
S(x) = 6x + 16 + \frac{48}{x}
\]
\item Démontrer que
\[
S(x) = \frac{8x^2 + 20x + 80}{x}
S(x) = \frac{6x^2 + 16x + 48}{x}
\]
\item Démontrer que
\[
S'(x) = \frac{8x^2 - 80}{x^2}
S'(x) = \frac{6x^2 - 48}{x^2}
\]
\item En déduire le tableau de variation de $S(x)$ sur $\intOF{0}{10}$.
\item Déterminer les valeurs de $x$ et $h$ correspondant à une utilisation minimal de tôle.
@ -57,62 +57,62 @@ Les valeurs des exercices sont générés automatiquement. Si une valeur a un no
\begin{enumerate}
\item Le volume étant fixe si l'on fait varier $x$, $h$ doit aussi varier.
\begin{itemize}
\item Si $x = 2$ alors conserver un volume de $V=40$, $h$ doit être égale à $10 / 2$
\item Si $x = 3$ alors conserver un volume de $V=40$, $h$ doit être égale à $10 / 3$
\item Si $x = 2$ alors conserver un volume de $V=24$, $h$ doit être égale à $8 / 2$
\item Si $x = 3$ alors conserver un volume de $V=24$, $h$ doit être égale à $8 / 3$
\end{itemize}
\item Pour calculer le volume, on a
\begin{eqnarray*}
V &=& h\times x \times 4 \\
40 &=& h\times x \times 4 \\
x &=& \frac{40}{h\times 4} = \frac{10}{h}
V &=& h\times x \times 3 \\
24 &=& h\times x \times 3 \\
x &=& \frac{24}{h\times 3} = \frac{8}{h}
\end{eqnarray*}
\item Pour calculer la surface totale, on ajoute la surface de chaque face. On a donc le calcul suivant
\begin{eqnarray*}
S(x) &=& x\times h \times 2 + x\times4\times2 + h\times 4\times 2\\
S(x) &=& x\times \frac{10}{x} \times 2 + x\times4\times2 + \frac{10}{x}\times 4\times 2\\
S(x) &=& 8x + 20 + \frac{80}{x}
S(x) &=& x\times h \times 2 + x\times3\times2 + h\times 3\times 2\\
S(x) &=& x\times \frac{8}{x} \times 2 + x\times3\times2 + \frac{8}{x}\times 3\times 2\\
S(x) &=& 6x + 16 + \frac{48}{x}
\end{eqnarray*}
\item Pour trouver cette nouvelle forme, on met chaque élément sur le même dénominateur
\begin{eqnarray*}
S(x) &=& 8x + 20 + \frac{80}{x}\\
S(x) &=& \frac{8x\times x}{x} + \frac{20\times x}{x} + \frac{80}{x}\\
S(x) &=& \frac{8x^2 + 20x + 80}{x}
S(x) &=& 6x + 16 + \frac{48}{x}\\
S(x) &=& \frac{6x\times x}{x} + \frac{16\times x}{x} + \frac{48}{x}\\
S(x) &=& \frac{6x^2 + 16x + 48}{x}
\end{eqnarray*}
\item On retrouve la formule $\frac{u}{v}$ à dériver
\[
u(x) = 8x^2 + 20x + 80 \Rightarrow u'(x) = 16x + 20
u(x) = 6x^2 + 16x + 48 \Rightarrow u'(x) = 12x + 16
\]
\[
v(x) = x \Rightarrow v'(x) = 1
\]
Donc au numérateur on obtient
\begin{eqnarray*}
u'(x)\times v(x) - u(x)\times v'(x) &=& (16x + 20)\times x - (8x^2 + 20x + 80)\times 1\\
&=& 8x^2 - 80
u'(x)\times v(x) - u(x)\times v'(x) &=& (12x + 16)\times x - (6x^2 + 16x + 48)\times 1\\
&=& 6x^2 - 48
\end{eqnarray*}
Donc
\[
S'(x) = \frac{8x^2 - 80}{x^2}
S'(x) = \frac{6x^2 - 48}{x^2}
\]
\item Tableau de variations de $S$
\begin{itemize}
\item Valeur interdite: $x^2 = 0 \equiv x = 0$
\item Signe de $8x^2 - 80$: c'est un polynôme du 2e degré
\item Signe de $6x^2 - 48$: c'est un polynôme du 2e degré
\[
\Delta = 2560 > 0
\Delta = 1152 > 0
\]
Il y a donc 2 racines
\[
x_1 = - 3.1622776601683795 \qquad
x_2 = 3.1622776601683795
x_1 = - 2.82842712474619 \qquad
x_2 = 2.82842712474619
\]
Et on sait que $8x^2 - 80$ est du signe de $a$ donc positif en dehors des racines
Et on sait que $6x^2 - 48$ est du signe de $a$ donc positif en dehors des racines
\item Le dénominateur $x^2$ est toujours positif.
\item Tableau de variations
\begin{tikzpicture}[baseline=(a.north)]
\tkzTabInit[lgt=3,espcl=3]{$x$/1,$8x^2 - 80$/1, $x^2$/1, $S'$/1, $S$/2}{$0$, $- 3.1622776601683795$, $10$}
\tkzTabInit[lgt=3,espcl=3]{$x$/1,$6x^2 - 48$/1, $x^2$/1, $S'$/1, $S$/2}{$0$, $- 2.82842712474619$, $10$}
\tkzTabLine{d,-, z, +, }
\tkzTabLine{d,+, , +, }
\tkzTabLine{d,-, z, +, }
@ -120,7 +120,7 @@ Les valeurs des exercices sont générés automatiquement. Si une valeur a un no
\end{tikzpicture}
\end{itemize}
\item On a donc une surface minimal pour $x=3.1622776601683795$ et $h = 31.6227766016837950$.
\item On a donc une surface minimal pour $x=2.82842712474619$ et $h = 22.62741699796952$.
\end{enumerate}
\end{solution}
@ -133,14 +133,14 @@ Les valeurs des exercices sont générés automatiquement. Si une valeur a un no
Le tour d'un bassin au niveau du sol présente deux axes de symétrie : laxe des abscisses et la droite déquation $x=4$. Il est obtenu par symétrie de la courbe $\mathcal{C}_f$ sur $\intFF{0}{4}$$f$ est la fonction définie par
\[
f(x) = \left(- x^{2} + 6.1 x - 9.3\right) e^{- x} + 9.3
f(x) = \left(- x^{2} + 5.6 x - 1.3\right) e^{- x} + 1.3
\]
On admet que sur $\intFF{0}{4}$ la fonction $f$ est positive.
\begin{enumerate}
\item Sur un repère, tracer l'allure de la courbe $\mathcal{C}_f$, les axes de symétries puis compléter pour dessiner la forme du bassin.
\item Montrer que la fonction $f$ admet comme primitive sur $\R$ la fonction $F$ définie par
\[
F(x) = 9.3 x + \left( x^{2} - 4.1 x + 5.2\right) e^{- x}
F(x) = 1.3 x + \left( x^{2} - 3.6 x - 2.3\right) e^{- x}
\]
\item Calculer la quantité $\ds \int_0^4 f(x) \; dx$, vous donnerez le résultat sous forme exacte. Interpréter le résultat et reportez cette quantité sur le graphique.
\item On considère que l'échelle de votre graphique est de 1unité pour 15m. Calculer l'aire du bassin. Vous donnerez un résultat arrondi au $m^2$ près.
@ -156,15 +156,14 @@ Les valeurs des exercices sont générés automatiquement. Si une valeur a un no
\tkzGrid
\tkzAxeXY
\tkzFct[domain=0:10,color=red,very thick]%
{ (-x**2 + 6.1*x - 9.3)*exp(-x) + 9.3 };
{ (-x**2 + 5.6*x - 1.3)*exp(-x) + 1.3 };
\end{tikzpicture}
\item Il faut dériver $F(x)$ et vérifier que $F'(x) = f(x)$.
\item $\ds \int_0^4 f(x) \; dx = F(4) - F(0) = \frac{4.8}{e^{4}} + 32.0$
\item $\ds \int_0^4 f(x) \; dx = F(4) - F(0) = 7.5 - \frac{0.699999999999999}{e^{4}}$
\item La quantité calculée à la question précédente se retrouve 4fois pour former le bassin. Il faut ensuite prendre en compte l'échelle, comme 1unité de longueur correspond à 15m, une unité d'air correspond à $15\times15 = 225m^2$. Ainsi l'aire du bassin est égale à
\[
(\frac{4.8}{e^{4}} + 32.0)\times 4 \times 15^2 = 28879.00000
(7.5 - \frac{0.699999999999999}{e^{4}})\times 4 \times 15^2 = 6738.000000
\]
\end{enumerate}
\end{solution}
@ -185,9 +184,9 @@ Les valeurs des exercices sont générés automatiquement. Si une valeur a un no
Deux ateliers A et B fabriquent des stylos pour une entreprise.
L'atelier A fabrique 92.0\,\% des stylos, et parmi ceux-là, 47.0\,\% possèdent un défaut de fabrication.
L'atelier A fabrique 53.0\,\% des stylos, et parmi ceux-là, 59.0\,\% possèdent un défaut de fabrication.
De plus, 1.0\,\% des stylos possèdent un défaut de fabrication et sortent de l'atelier B.
De plus, 7.000000000000001\,\% des stylos possèdent un défaut de fabrication et sortent de l'atelier B.
Un stylo est prélevé au hasard dans le stock de l'entreprise.
@ -241,7 +240,7 @@ Les valeurs des exercices sont générés automatiquement. Si une valeur a un no
\item
\begin{enumerate}
\item Calculer la probabilité qu'un stylo provienne de l'atelier A et possède un défaut de fabrication.
\item En déduire que la probabilité qu'un stylo possède un défaut de fabrication est de $0.44$.
\item En déduire que la probabilité qu'un stylo possède un défaut de fabrication est de $0.38$.
\end{enumerate}
\item On prélève un stylo au hasard avec un défaut. Quelle est la probabilité qu'il vienne de l'atelier A?
\end{enumerate}
@ -251,7 +250,7 @@ Les valeurs des exercices sont générés automatiquement. Si une valeur a un no
\textbf{Partie B}
\medskip
Dans cette partie, on suppose que 44.0\,\% des stylos possèdent un défaut de fabrication.
Dans cette partie, on suppose que 38.0\,\% des stylos possèdent un défaut de fabrication.
L'entreprise confectionne des paquets contenant chacun $4$~stylos.
@ -266,7 +265,7 @@ Les valeurs des exercices sont générés automatiquement. Si une valeur a un no
\begin{enumerate}
\setcounter{enumi}{4}
\item Avec quelle loi peut-on modéliser $X$. Préciser les paramètres.
\item Calculer et interpréter la probabilité $P(X = 15)$.
\item Calculer et interpréter la probabilité $P(X = 7)$.
\item Le directeur de l'entreprise affirme qu'il y a plus d'une chance sur deux qu'un paquet ne comporte aucun stylo défectueux. A-t-il raison ?
\item Combien de stylos peut-on espérer avoir en moyenne?
\end{enumerate}
@ -282,27 +281,27 @@ Les valeurs des exercices sont générés automatiquement. Si une valeur a un no
child {node {$A$}
child {node {$D$}
edge from parent
node[above] {0.47}
node[above] {0.59}
}
child {node {$\overline{D}$}
edge from parent
node[above] {0.53}
node[above] {0.41}
}
edge from parent
node[above] {0.92}
node[above] {0.53}
}
child[missing] {}
child { node {$B$}
child {node {$D$}
edge from parent
node[above] {0.17}
node[above] {0.14}
}
child {node {$\overline{D}$}
edge from parent
node[above] {0.83}
node[above] {0.86}
}
edge from parent
node[above] {0.08}
node[above] {0.47}
} ;
\end{tikzpicture}
\end{center}
@ -310,51 +309,51 @@ Les valeurs des exercices sont générés automatiquement. Si une valeur a un no
\begin{itemize}
\item Probabilité que le stylo vienne de l'atelier A
\[
P(A) = 0.92
P(A) = 0.53
\]
\item Probabilité que le stylo vienne de l'atelier B
\[
P(B) = 0.08
P(B) = 0.47
\]
\item Probabilité que le stylo ait un défaut sachant qu'il vient de l'atelier A.
\[
P_A(D) = 0.47
P_A(D) = 0.59
\]
\item Probabilité que le stylo vienne de l'atelier B et qu'il ait un défaut.
\[
P(D \cap D) = 0.01
P(D \cap D) = 0.07
\]
\end{itemize}
\item
\begin{enumerate}
\item Probabilité qu'un stylo vienne de l'atelier A et qu'il ait un defaut
\[
P(A\cap D) = P(A) \times P_A(D) = 0.92 \times 0.47 = 0.43
P(A\cap D) = P(A) \times P_A(D) = 0.53 \times 0.59 = 0.31
\]
\item Probabilité que le stylo ai un défaut de fabrication.
\[
P(D) = P(A\cap D) + P(B\cap D) = 0.43 + 0.01 = 0.44
P(D) = P(A\cap D) + P(B\cap D) = 0.31 + 0.07 = 0.38
\]
\end{enumerate}
\item Probabilité qu'il vienne de l'atelier A sachant qu'il a un defaut
\[
P_D(A) = \frac{P(A\cap D)}{P(D)} = \frac{0.43}{0.44} = 0.98
P_D(A) = \frac{P(A\cap D)}{P(D)} = \frac{0.31}{0.38} = 0.82
\]
\item $X$ peut être modélisée par une loi binomiale de paramètres $n=17$ et $p=0.44$.
\item $X$ peut être modélisée par une loi binomiale de paramètres $n=12$ et $p=0.38$.
\item (\textit{par de correction automatique disponible pour le résultat final}
\[
P(X = 15) = \coefBino{17}{15}\times 0.44^{15} \times 0.56^{2}
P(X = 7) = \coefBino{12}{7}\times 0.38^{7} \times 0.62^{5}
\]
\item (\textit{par de correction automatique disponible pour le résultat final}
Il faut calculer la probabilité qu'il y ait 0 stylo avec un defaut.
\[
P(X = 0) = \coefBino{17}{0}\times 0.44^{0} \times 0.56^{17}
P(X = 0) = \coefBino{12}{0}\times 0.38^{0} \times 0.62^{12}
\]
Puis comparer ce nombre à 0,5.
\item Il faut calculer l'espérance
\[
E[X] = n\times p = 17 \times 0.44 = 7.48
E[X] = n\times p = 12 \times 0.38 = 4.56
\]
\end{enumerate}
\end{solution}

View File

@ -3,7 +3,7 @@
% Title Page
\title{DM1 \hfill MOLINIER Annelise}
\tribe{Maths complémentaire}
\tribe{Maths complémentaires}
\date{\hfillÀ render pour le jeudi 27 mai}
\xsimsetup{
@ -17,7 +17,7 @@ Les valeurs des exercices sont générés automatiquement. Si une valeur a un no
\begin{exercise}[subtitle={Optimisation de matière}]
\begin{minipage}{0.6\textwidth}
On se propose de fabriquer avec le moins de tôle possible une citerne fermée en forme de parallélépipède rectangle dont le volume intérieur doit être de $25m^3$. La longueur est aussi fixée à $5m$ par le cahier des charges.
On se propose de fabriquer avec le moins de tôle possible une citerne fermée en forme de parallélépipède rectangle dont le volume intérieur doit être de $20m^3$. La longueur est aussi fixée à $2m$ par le cahier des charges.
On peut donc faire varier uniquement la largeur (notée $x$) et la hauteur (notée $h$) de la cuve.
\end{minipage}
@ -28,25 +28,25 @@ Les valeurs des exercices sont générés automatiquement. Si une valeur a un no
\pgfmathsetmacro{\cubey}{1}
\pgfmathsetmacro{\cubez}{2}
\draw[black,fill=gray] (0,0,0) -- ++(-\cubex,0,0) -- ++(0,-\cubey,0) node [midway, left] {$h$} -- ++(\cubex,0,0) node [midway, below] {$x$} -- cycle;
\draw[black,fill=gray] (0,0,0) -- ++(0,0,-\cubez) -- ++(0,-\cubey,0) -- ++(0,0,\cubez) node [midway, right] {$5m$} -- cycle;
\draw[black,fill=gray] (0,0,0) -- ++(0,0,-\cubez) -- ++(0,-\cubey,0) -- ++(0,0,\cubez) node [midway, right] {$2m$} -- cycle;
\draw[black,fill=gray] (0,0,0) -- ++(-\cubex,0,0) -- ++(0,0,-\cubez) -- ++(\cubex,0,0) -- cycle;
\end{tikzpicture}
\end{minipage}
\begin{enumerate}
\item Expliquer pourquoi quand la largeur $x$ change, la hauteur $h$ doit elle aussi changer pour respecter les contraintes.
\item Démontrer que l'on doit avoir $h = \dfrac{5}{x}$.
\item Démontrer que l'on doit avoir $h = \dfrac{10}{x}$.
\item On note $S(x)$ l'aire totale de la citerne (c'est à dire la somme des aires des six faces). Montrer que l'on peut écrire
\[
S(x) = 10x + 10 + \frac{50}{x}
S(x) = 4x + 20 + \frac{40}{x}
\]
\item Démontrer que
\[
S(x) = \frac{10x^2 + 10x + 50}{x}
S(x) = \frac{4x^2 + 20x + 40}{x}
\]
\item Démontrer que
\[
S'(x) = \frac{10x^2 - 50}{x^2}
S'(x) = \frac{4x^2 - 40}{x^2}
\]
\item En déduire le tableau de variation de $S(x)$ sur $\intOF{0}{10}$.
\item Déterminer les valeurs de $x$ et $h$ correspondant à une utilisation minimal de tôle.
@ -57,62 +57,62 @@ Les valeurs des exercices sont générés automatiquement. Si une valeur a un no
\begin{enumerate}
\item Le volume étant fixe si l'on fait varier $x$, $h$ doit aussi varier.
\begin{itemize}
\item Si $x = 2$ alors conserver un volume de $V=25$, $h$ doit être égale à $5 / 2$
\item Si $x = 3$ alors conserver un volume de $V=25$, $h$ doit être égale à $5 / 3$
\item Si $x = 2$ alors conserver un volume de $V=20$, $h$ doit être égale à $10 / 2$
\item Si $x = 3$ alors conserver un volume de $V=20$, $h$ doit être égale à $10 / 3$
\end{itemize}
\item Pour calculer le volume, on a
\begin{eqnarray*}
V &=& h\times x \times 5 \\
25 &=& h\times x \times 5 \\
x &=& \frac{25}{h\times 5} = \frac{5}{h}
V &=& h\times x \times 2 \\
20 &=& h\times x \times 2 \\
x &=& \frac{20}{h\times 2} = \frac{10}{h}
\end{eqnarray*}
\item Pour calculer la surface totale, on ajoute la surface de chaque face. On a donc le calcul suivant
\begin{eqnarray*}
S(x) &=& x\times h \times 2 + x\times5\times2 + h\times 5\times 2\\
S(x) &=& x\times \frac{5}{x} \times 2 + x\times5\times2 + \frac{5}{x}\times 5\times 2\\
S(x) &=& 10x + 10 + \frac{50}{x}
S(x) &=& x\times h \times 2 + x\times2\times2 + h\times 2\times 2\\
S(x) &=& x\times \frac{10}{x} \times 2 + x\times2\times2 + \frac{10}{x}\times 2\times 2\\
S(x) &=& 4x + 20 + \frac{40}{x}
\end{eqnarray*}
\item Pour trouver cette nouvelle forme, on met chaque élément sur le même dénominateur
\begin{eqnarray*}
S(x) &=& 10x + 10 + \frac{50}{x}\\
S(x) &=& \frac{10x\times x}{x} + \frac{10\times x}{x} + \frac{50}{x}\\
S(x) &=& \frac{10x^2 + 10x + 50}{x}
S(x) &=& 4x + 20 + \frac{40}{x}\\
S(x) &=& \frac{4x\times x}{x} + \frac{20\times x}{x} + \frac{40}{x}\\
S(x) &=& \frac{4x^2 + 20x + 40}{x}
\end{eqnarray*}
\item On retrouve la formule $\frac{u}{v}$ à dériver
\[
u(x) = 10x^2 + 10x + 50 \Rightarrow u'(x) = 20x + 10
u(x) = 4x^2 + 20x + 40 \Rightarrow u'(x) = 8x + 20
\]
\[
v(x) = x \Rightarrow v'(x) = 1
\]
Donc au numérateur on obtient
\begin{eqnarray*}
u'(x)\times v(x) - u(x)\times v'(x) &=& (20x + 10)\times x - (10x^2 + 10x + 50)\times 1\\
&=& 10x^2 - 50
u'(x)\times v(x) - u(x)\times v'(x) &=& (8x + 20)\times x - (4x^2 + 20x + 40)\times 1\\
&=& 4x^2 - 40
\end{eqnarray*}
Donc
\[
S'(x) = \frac{10x^2 - 50}{x^2}
S'(x) = \frac{4x^2 - 40}{x^2}
\]
\item Tableau de variations de $S$
\begin{itemize}
\item Valeur interdite: $x^2 = 0 \equiv x = 0$
\item Signe de $10x^2 - 50$: c'est un polynôme du 2e degré
\item Signe de $4x^2 - 40$: c'est un polynôme du 2e degré
\[
\Delta = 2000 > 0
\Delta = 640 > 0
\]
Il y a donc 2 racines
\[
x_1 = - 2.23606797749979 \qquad
x_2 = 2.23606797749979
x_1 = - 3.1622776601683795 \qquad
x_2 = 3.1622776601683795
\]
Et on sait que $10x^2 - 50$ est du signe de $a$ donc positif en dehors des racines
Et on sait que $4x^2 - 40$ est du signe de $a$ donc positif en dehors des racines
\item Le dénominateur $x^2$ est toujours positif.
\item Tableau de variations
\begin{tikzpicture}[baseline=(a.north)]
\tkzTabInit[lgt=3,espcl=3]{$x$/1,$10x^2 - 50$/1, $x^2$/1, $S'$/1, $S$/2}{$0$, $- 2.23606797749979$, $10$}
\tkzTabInit[lgt=3,espcl=3]{$x$/1,$4x^2 - 40$/1, $x^2$/1, $S'$/1, $S$/2}{$0$, $- 3.1622776601683795$, $10$}
\tkzTabLine{d,-, z, +, }
\tkzTabLine{d,+, , +, }
\tkzTabLine{d,-, z, +, }
@ -120,7 +120,7 @@ Les valeurs des exercices sont générés automatiquement. Si une valeur a un no
\end{tikzpicture}
\end{itemize}
\item On a donc une surface minimal pour $x=2.23606797749979$ et $h = 11.18033988749895$.
\item On a donc une surface minimal pour $x=3.1622776601683795$ et $h = 31.6227766016837950$.
\end{enumerate}
\end{solution}
@ -133,14 +133,14 @@ Les valeurs des exercices sont générés automatiquement. Si une valeur a un no
Le tour d'un bassin au niveau du sol présente deux axes de symétrie : laxe des abscisses et la droite déquation $x=4$. Il est obtenu par symétrie de la courbe $\mathcal{C}_f$ sur $\intFF{0}{4}$$f$ est la fonction définie par
\[
f(x) = \left(- x^{2} + 6.9 x - 3.3\right) e^{- x} + 3.3
f(x) = \left(- x^{2} + 5.1 x - 3.5\right) e^{- x} + 3.5
\]
On admet que sur $\intFF{0}{4}$ la fonction $f$ est positive.
\begin{enumerate}
\item Sur un repère, tracer l'allure de la courbe $\mathcal{C}_f$, les axes de symétries puis compléter pour dessiner la forme du bassin.
\item Montrer que la fonction $f$ admet comme primitive sur $\R$ la fonction $F$ définie par
\[
F(x) = 3.3 x + \left( x^{2} - 4.9 x - 1.6\right) e^{- x}
F(x) = 3.5 x + \left( x^{2} - 3.1 x + 0.4\right) e^{- x}
\]
\item Calculer la quantité $\ds \int_0^4 f(x) \; dx$, vous donnerez le résultat sous forme exacte. Interpréter le résultat et reportez cette quantité sur le graphique.
\item On considère que l'échelle de votre graphique est de 1unité pour 15m. Calculer l'aire du bassin. Vous donnerez un résultat arrondi au $m^2$ près.
@ -156,15 +156,14 @@ Les valeurs des exercices sont générés automatiquement. Si une valeur a un no
\tkzGrid
\tkzAxeXY
\tkzFct[domain=0:10,color=red,very thick]%
{ (-x**2 + 6.9*x - 3.3)*exp(-x) + 3.3 };
{ (-x**2 + 5.1*x - 3.5)*exp(-x) + 3.5 };
\end{tikzpicture}
\item Il faut dériver $F(x)$ et vérifier que $F'(x) = f(x)$.
\item $\ds \int_0^4 f(x) \; dx = F(4) - F(0) = 14.8 - \frac{5.2}{e^{4}}$
\item $\ds \int_0^4 f(x) \; dx = F(4) - F(0) = \frac{4.0}{e^{4}} + 13.6$
\item La quantité calculée à la question précédente se retrouve 4fois pour former le bassin. Il faut ensuite prendre en compte l'échelle, comme 1unité de longueur correspond à 15m, une unité d'air correspond à $15\times15 = 225m^2$. Ainsi l'aire du bassin est égale à
\[
(14.8 - \frac{5.2}{e^{4}})\times 4 \times 15^2 = 13234.00000
(\frac{4.0}{e^{4}} + 13.6)\times 4 \times 15^2 = 12306.00000
\]
\end{enumerate}
\end{solution}
@ -185,9 +184,9 @@ Les valeurs des exercices sont générés automatiquement. Si une valeur a un no
Deux ateliers A et B fabriquent des stylos pour une entreprise.
L'atelier A fabrique 56.00000000000001\,\% des stylos, et parmi ceux-là, 22.0\,\% possèdent un défaut de fabrication.
L'atelier A fabrique 52.0\,\% des stylos, et parmi ceux-là, 81.0\,\% possèdent un défaut de fabrication.
De plus, 10.0\,\% des stylos possèdent un défaut de fabrication et sortent de l'atelier B.
De plus, 31.0\,\% des stylos possèdent un défaut de fabrication et sortent de l'atelier B.
Un stylo est prélevé au hasard dans le stock de l'entreprise.
@ -241,7 +240,7 @@ Les valeurs des exercices sont générés automatiquement. Si une valeur a un no
\item
\begin{enumerate}
\item Calculer la probabilité qu'un stylo provienne de l'atelier A et possède un défaut de fabrication.
\item En déduire que la probabilité qu'un stylo possède un défaut de fabrication est de $0.22$.
\item En déduire que la probabilité qu'un stylo possède un défaut de fabrication est de $0.73$.
\end{enumerate}
\item On prélève un stylo au hasard avec un défaut. Quelle est la probabilité qu'il vienne de l'atelier A?
\end{enumerate}
@ -251,7 +250,7 @@ Les valeurs des exercices sont générés automatiquement. Si une valeur a un no
\textbf{Partie B}
\medskip
Dans cette partie, on suppose que 22.0\,\% des stylos possèdent un défaut de fabrication.
Dans cette partie, on suppose que 73.0\,\% des stylos possèdent un défaut de fabrication.
L'entreprise confectionne des paquets contenant chacun $4$~stylos.
@ -266,7 +265,7 @@ Les valeurs des exercices sont générés automatiquement. Si une valeur a un no
\begin{enumerate}
\setcounter{enumi}{4}
\item Avec quelle loi peut-on modéliser $X$. Préciser les paramètres.
\item Calculer et interpréter la probabilité $P(X = 19)$.
\item Calculer et interpréter la probabilité $P(X = 6)$.
\item Le directeur de l'entreprise affirme qu'il y a plus d'une chance sur deux qu'un paquet ne comporte aucun stylo défectueux. A-t-il raison ?
\item Combien de stylos peut-on espérer avoir en moyenne?
\end{enumerate}
@ -282,27 +281,27 @@ Les valeurs des exercices sont générés automatiquement. Si une valeur a un no
child {node {$A$}
child {node {$D$}
edge from parent
node[above] {0.22}
node[above] {0.81}
}
child {node {$\overline{D}$}
edge from parent
node[above] {0.78}
node[above] {0.19}
}
edge from parent
node[above] {0.56}
node[above] {0.52}
}
child[missing] {}
child { node {$B$}
child {node {$D$}
edge from parent
node[above] {0.23}
node[above] {0.65}
}
child {node {$\overline{D}$}
edge from parent
node[above] {0.77}
node[above] {0.35}
}
edge from parent
node[above] {0.44}
node[above] {0.48}
} ;
\end{tikzpicture}
\end{center}
@ -310,51 +309,51 @@ Les valeurs des exercices sont générés automatiquement. Si une valeur a un no
\begin{itemize}
\item Probabilité que le stylo vienne de l'atelier A
\[
P(A) = 0.56
P(A) = 0.52
\]
\item Probabilité que le stylo vienne de l'atelier B
\[
P(B) = 0.44
P(B) = 0.48
\]
\item Probabilité que le stylo ait un défaut sachant qu'il vient de l'atelier A.
\[
P_A(D) = 0.22
P_A(D) = 0.81
\]
\item Probabilité que le stylo vienne de l'atelier B et qu'il ait un défaut.
\[
P(D \cap D) = 0.1
P(D \cap D) = 0.31
\]
\end{itemize}
\item
\begin{enumerate}
\item Probabilité qu'un stylo vienne de l'atelier A et qu'il ait un defaut
\[
P(A\cap D) = P(A) \times P_A(D) = 0.56 \times 0.22 = 0.12
P(A\cap D) = P(A) \times P_A(D) = 0.52 \times 0.81 = 0.42
\]
\item Probabilité que le stylo ai un défaut de fabrication.
\[
P(D) = P(A\cap D) + P(B\cap D) = 0.12 + 0.1 = 0.22
P(D) = P(A\cap D) + P(B\cap D) = 0.42 + 0.31 = 0.73
\]
\end{enumerate}
\item Probabilité qu'il vienne de l'atelier A sachant qu'il a un defaut
\[
P_D(A) = \frac{P(A\cap D)}{P(D)} = \frac{0.12}{0.22} = 0.55
P_D(A) = \frac{P(A\cap D)}{P(D)} = \frac{0.42}{0.73} = 0.58
\]
\item $X$ peut être modélisée par une loi binomiale de paramètres $n=19$ et $p=0.22$.
\item $X$ peut être modélisée par une loi binomiale de paramètres $n=10$ et $p=0.73$.
\item (\textit{par de correction automatique disponible pour le résultat final}
\[
P(X = 19) = \coefBino{19}{19}\times 0.22^{19} \times 0.78^{0}
P(X = 6) = \coefBino{10}{6}\times 0.73^{6} \times 0.27^{4}
\]
\item (\textit{par de correction automatique disponible pour le résultat final}
Il faut calculer la probabilité qu'il y ait 0 stylo avec un defaut.
\[
P(X = 0) = \coefBino{19}{0}\times 0.22^{0} \times 0.78^{19}
P(X = 0) = \coefBino{10}{0}\times 0.73^{0} \times 0.27^{10}
\]
Puis comparer ce nombre à 0,5.
\item Il faut calculer l'espérance
\[
E[X] = n\times p = 19 \times 0.22 = 4.18
E[X] = n\times p = 10 \times 0.73 = 7.3
\]
\end{enumerate}
\end{solution}

View File

@ -3,7 +3,7 @@
% Title Page
\title{DM1 \hfill MOUHOUBI Maïssa}
\tribe{Maths complémentaire}
\tribe{Maths complémentaires}
\date{\hfillÀ render pour le jeudi 27 mai}
\xsimsetup{
@ -17,7 +17,7 @@ Les valeurs des exercices sont générés automatiquement. Si une valeur a un no
\begin{exercise}[subtitle={Optimisation de matière}]
\begin{minipage}{0.6\textwidth}
On se propose de fabriquer avec le moins de tôle possible une citerne fermée en forme de parallélépipède rectangle dont le volume intérieur doit être de $35m^3$. La longueur est aussi fixée à $5m$ par le cahier des charges.
On se propose de fabriquer avec le moins de tôle possible une citerne fermée en forme de parallélépipède rectangle dont le volume intérieur doit être de $9m^3$. La longueur est aussi fixée à $3m$ par le cahier des charges.
On peut donc faire varier uniquement la largeur (notée $x$) et la hauteur (notée $h$) de la cuve.
\end{minipage}
@ -28,25 +28,25 @@ Les valeurs des exercices sont générés automatiquement. Si une valeur a un no
\pgfmathsetmacro{\cubey}{1}
\pgfmathsetmacro{\cubez}{2}
\draw[black,fill=gray] (0,0,0) -- ++(-\cubex,0,0) -- ++(0,-\cubey,0) node [midway, left] {$h$} -- ++(\cubex,0,0) node [midway, below] {$x$} -- cycle;
\draw[black,fill=gray] (0,0,0) -- ++(0,0,-\cubez) -- ++(0,-\cubey,0) -- ++(0,0,\cubez) node [midway, right] {$5m$} -- cycle;
\draw[black,fill=gray] (0,0,0) -- ++(0,0,-\cubez) -- ++(0,-\cubey,0) -- ++(0,0,\cubez) node [midway, right] {$3m$} -- cycle;
\draw[black,fill=gray] (0,0,0) -- ++(-\cubex,0,0) -- ++(0,0,-\cubez) -- ++(\cubex,0,0) -- cycle;
\end{tikzpicture}
\end{minipage}
\begin{enumerate}
\item Expliquer pourquoi quand la largeur $x$ change, la hauteur $h$ doit elle aussi changer pour respecter les contraintes.
\item Démontrer que l'on doit avoir $h = \dfrac{7}{x}$.
\item Démontrer que l'on doit avoir $h = \dfrac{3}{x}$.
\item On note $S(x)$ l'aire totale de la citerne (c'est à dire la somme des aires des six faces). Montrer que l'on peut écrire
\[
S(x) = 10x + 14 + \frac{70}{x}
S(x) = 6x + 6 + \frac{18}{x}
\]
\item Démontrer que
\[
S(x) = \frac{10x^2 + 14x + 70}{x}
S(x) = \frac{6x^2 + 6x + 18}{x}
\]
\item Démontrer que
\[
S'(x) = \frac{10x^2 - 70}{x^2}
S'(x) = \frac{6x^2 - 18}{x^2}
\]
\item En déduire le tableau de variation de $S(x)$ sur $\intOF{0}{10}$.
\item Déterminer les valeurs de $x$ et $h$ correspondant à une utilisation minimal de tôle.
@ -57,62 +57,62 @@ Les valeurs des exercices sont générés automatiquement. Si une valeur a un no
\begin{enumerate}
\item Le volume étant fixe si l'on fait varier $x$, $h$ doit aussi varier.
\begin{itemize}
\item Si $x = 2$ alors conserver un volume de $V=35$, $h$ doit être égale à $7 / 2$
\item Si $x = 3$ alors conserver un volume de $V=35$, $h$ doit être égale à $7 / 3$
\item Si $x = 2$ alors conserver un volume de $V=9$, $h$ doit être égale à $3 / 2$
\item Si $x = 3$ alors conserver un volume de $V=9$, $h$ doit être égale à $3 / 3$
\end{itemize}
\item Pour calculer le volume, on a
\begin{eqnarray*}
V &=& h\times x \times 5 \\
35 &=& h\times x \times 5 \\
x &=& \frac{35}{h\times 5} = \frac{7}{h}
V &=& h\times x \times 3 \\
9 &=& h\times x \times 3 \\
x &=& \frac{9}{h\times 3} = \frac{3}{h}
\end{eqnarray*}
\item Pour calculer la surface totale, on ajoute la surface de chaque face. On a donc le calcul suivant
\begin{eqnarray*}
S(x) &=& x\times h \times 2 + x\times5\times2 + h\times 5\times 2\\
S(x) &=& x\times \frac{7}{x} \times 2 + x\times5\times2 + \frac{7}{x}\times 5\times 2\\
S(x) &=& 10x + 14 + \frac{70}{x}
S(x) &=& x\times h \times 2 + x\times3\times2 + h\times 3\times 2\\
S(x) &=& x\times \frac{3}{x} \times 2 + x\times3\times2 + \frac{3}{x}\times 3\times 2\\
S(x) &=& 6x + 6 + \frac{18}{x}
\end{eqnarray*}
\item Pour trouver cette nouvelle forme, on met chaque élément sur le même dénominateur
\begin{eqnarray*}
S(x) &=& 10x + 14 + \frac{70}{x}\\
S(x) &=& \frac{10x\times x}{x} + \frac{14\times x}{x} + \frac{70}{x}\\
S(x) &=& \frac{10x^2 + 14x + 70}{x}
S(x) &=& 6x + 6 + \frac{18}{x}\\
S(x) &=& \frac{6x\times x}{x} + \frac{6\times x}{x} + \frac{18}{x}\\
S(x) &=& \frac{6x^2 + 6x + 18}{x}
\end{eqnarray*}
\item On retrouve la formule $\frac{u}{v}$ à dériver
\[
u(x) = 10x^2 + 14x + 70 \Rightarrow u'(x) = 20x + 14
u(x) = 6x^2 + 6x + 18 \Rightarrow u'(x) = 12x + 6
\]
\[
v(x) = x \Rightarrow v'(x) = 1
\]
Donc au numérateur on obtient
\begin{eqnarray*}
u'(x)\times v(x) - u(x)\times v'(x) &=& (20x + 14)\times x - (10x^2 + 14x + 70)\times 1\\
&=& 10x^2 - 70
u'(x)\times v(x) - u(x)\times v'(x) &=& (12x + 6)\times x - (6x^2 + 6x + 18)\times 1\\
&=& 6x^2 - 18
\end{eqnarray*}
Donc
\[
S'(x) = \frac{10x^2 - 70}{x^2}
S'(x) = \frac{6x^2 - 18}{x^2}
\]
\item Tableau de variations de $S$
\begin{itemize}
\item Valeur interdite: $x^2 = 0 \equiv x = 0$
\item Signe de $10x^2 - 70$: c'est un polynôme du 2e degré
\item Signe de $6x^2 - 18$: c'est un polynôme du 2e degré
\[
\Delta = 2800 > 0
\Delta = 432 > 0
\]
Il y a donc 2 racines
\[
x_1 = - 2.6457513110645907 \qquad
x_2 = 2.6457513110645907
x_1 = - 1.7320508075688774 \qquad
x_2 = 1.7320508075688774
\]
Et on sait que $10x^2 - 70$ est du signe de $a$ donc positif en dehors des racines
Et on sait que $6x^2 - 18$ est du signe de $a$ donc positif en dehors des racines
\item Le dénominateur $x^2$ est toujours positif.
\item Tableau de variations
\begin{tikzpicture}[baseline=(a.north)]
\tkzTabInit[lgt=3,espcl=3]{$x$/1,$10x^2 - 70$/1, $x^2$/1, $S'$/1, $S$/2}{$0$, $- 2.6457513110645907$, $10$}
\tkzTabInit[lgt=3,espcl=3]{$x$/1,$6x^2 - 18$/1, $x^2$/1, $S'$/1, $S$/2}{$0$, $- 1.7320508075688774$, $10$}
\tkzTabLine{d,-, z, +, }
\tkzTabLine{d,+, , +, }
\tkzTabLine{d,-, z, +, }
@ -120,7 +120,7 @@ Les valeurs des exercices sont générés automatiquement. Si une valeur a un no
\end{tikzpicture}
\end{itemize}
\item On a donc une surface minimal pour $x=2.6457513110645907$ et $h = 18.5202591774521349$.
\item On a donc une surface minimal pour $x=1.7320508075688774$ et $h = 5.1961524227066322$.
\end{enumerate}
\end{solution}
@ -133,14 +133,14 @@ Les valeurs des exercices sont générés automatiquement. Si une valeur a un no
Le tour d'un bassin au niveau du sol présente deux axes de symétrie : laxe des abscisses et la droite déquation $x=4$. Il est obtenu par symétrie de la courbe $\mathcal{C}_f$ sur $\intFF{0}{4}$$f$ est la fonction définie par
\[
f(x) = \left(- x^{2} + 1.1 x - 6.1\right) e^{- x} + 6.1
f(x) = \left(- x^{2} + 7.0 x - 7.2\right) e^{- x} + 7.2
\]
On admet que sur $\intFF{0}{4}$ la fonction $f$ est positive.
\begin{enumerate}
\item Sur un repère, tracer l'allure de la courbe $\mathcal{C}_f$, les axes de symétries puis compléter pour dessiner la forme du bassin.
\item Montrer que la fonction $f$ admet comme primitive sur $\R$ la fonction $F$ définie par
\[
F(x) = 6.1 x + \left( x^{2} + 0.9 x + 7.0\right) e^{- x}
F(x) = 7.2 x + \left( x^{2} - 5.0 x + 2.2\right) e^{- x}
\]
\item Calculer la quantité $\ds \int_0^4 f(x) \; dx$, vous donnerez le résultat sous forme exacte. Interpréter le résultat et reportez cette quantité sur le graphique.
\item On considère que l'échelle de votre graphique est de 1unité pour 15m. Calculer l'aire du bassin. Vous donnerez un résultat arrondi au $m^2$ près.
@ -156,15 +156,14 @@ Les valeurs des exercices sont générés automatiquement. Si une valeur a un no
\tkzGrid
\tkzAxeXY
\tkzFct[domain=0:10,color=red,very thick]%
{ (-x**2 + 1.1*x - 6.1)*exp(-x) + 6.1 };
{ (-x**2 + 7.0*x - 7.2)*exp(-x) + 7.2 };
\end{tikzpicture}
\item Il faut dériver $F(x)$ et vérifier que $F'(x) = f(x)$.
\item $\ds \int_0^4 f(x) \; dx = F(4) - F(0) = \frac{26.6}{e^{4}} + 17.4$
\item $\ds \int_0^4 f(x) \; dx = F(4) - F(0) = 26.6 - \frac{1.8}{e^{4}}$
\item La quantité calculée à la question précédente se retrouve 4fois pour former le bassin. Il faut ensuite prendre en compte l'échelle, comme 1unité de longueur correspond à 15m, une unité d'air correspond à $15\times15 = 225m^2$. Ainsi l'aire du bassin est égale à
\[
(\frac{26.6}{e^{4}} + 17.4)\times 4 \times 15^2 = 16098.00000
(26.6 - \frac{1.8}{e^{4}})\times 4 \times 15^2 = 23910.00000
\]
\end{enumerate}
\end{solution}
@ -185,9 +184,9 @@ Les valeurs des exercices sont générés automatiquement. Si une valeur a un no
Deux ateliers A et B fabriquent des stylos pour une entreprise.
L'atelier A fabrique 23.0\,\% des stylos, et parmi ceux-là, 66.0\,\% possèdent un défaut de fabrication.
L'atelier A fabrique 57.99999999999999\,\% des stylos, et parmi ceux-là, 48.0\,\% possèdent un défaut de fabrication.
De plus, 68.0\,\% des stylos possèdent un défaut de fabrication et sortent de l'atelier B.
De plus, 11.0\,\% des stylos possèdent un défaut de fabrication et sortent de l'atelier B.
Un stylo est prélevé au hasard dans le stock de l'entreprise.
@ -241,7 +240,7 @@ Les valeurs des exercices sont générés automatiquement. Si une valeur a un no
\item
\begin{enumerate}
\item Calculer la probabilité qu'un stylo provienne de l'atelier A et possède un défaut de fabrication.
\item En déduire que la probabilité qu'un stylo possède un défaut de fabrication est de $0.83$.
\item En déduire que la probabilité qu'un stylo possède un défaut de fabrication est de $0.39$.
\end{enumerate}
\item On prélève un stylo au hasard avec un défaut. Quelle est la probabilité qu'il vienne de l'atelier A?
\end{enumerate}
@ -251,7 +250,7 @@ Les valeurs des exercices sont générés automatiquement. Si une valeur a un no
\textbf{Partie B}
\medskip
Dans cette partie, on suppose que 83.0\,\% des stylos possèdent un défaut de fabrication.
Dans cette partie, on suppose que 39.0\,\% des stylos possèdent un défaut de fabrication.
L'entreprise confectionne des paquets contenant chacun $4$~stylos.
@ -266,7 +265,7 @@ Les valeurs des exercices sont générés automatiquement. Si une valeur a un no
\begin{enumerate}
\setcounter{enumi}{4}
\item Avec quelle loi peut-on modéliser $X$. Préciser les paramètres.
\item Calculer et interpréter la probabilité $P(X = 10)$.
\item Calculer et interpréter la probabilité $P(X = 11)$.
\item Le directeur de l'entreprise affirme qu'il y a plus d'une chance sur deux qu'un paquet ne comporte aucun stylo défectueux. A-t-il raison ?
\item Combien de stylos peut-on espérer avoir en moyenne?
\end{enumerate}
@ -282,27 +281,27 @@ Les valeurs des exercices sont générés automatiquement. Si une valeur a un no
child {node {$A$}
child {node {$D$}
edge from parent
node[above] {0.66}
node[above] {0.48}
}
child {node {$\overline{D}$}
edge from parent
node[above] {0.34}
node[above] {0.52}
}
edge from parent
node[above] {0.23}
node[above] {0.58}
}
child[missing] {}
child { node {$B$}
child {node {$D$}
edge from parent
node[above] {0.88}
node[above] {0.26}
}
child {node {$\overline{D}$}
edge from parent
node[above] {0.12}
node[above] {0.74}
}
edge from parent
node[above] {0.77}
node[above] {0.42}
} ;
\end{tikzpicture}
\end{center}
@ -310,51 +309,51 @@ Les valeurs des exercices sont générés automatiquement. Si une valeur a un no
\begin{itemize}
\item Probabilité que le stylo vienne de l'atelier A
\[
P(A) = 0.23
P(A) = 0.58
\]
\item Probabilité que le stylo vienne de l'atelier B
\[
P(B) = 0.77
P(B) = 0.42
\]
\item Probabilité que le stylo ait un défaut sachant qu'il vient de l'atelier A.
\[
P_A(D) = 0.66
P_A(D) = 0.48
\]
\item Probabilité que le stylo vienne de l'atelier B et qu'il ait un défaut.
\[
P(D \cap D) = 0.68
P(D \cap D) = 0.11
\]
\end{itemize}
\item
\begin{enumerate}
\item Probabilité qu'un stylo vienne de l'atelier A et qu'il ait un defaut
\[
P(A\cap D) = P(A) \times P_A(D) = 0.23 \times 0.66 = 0.15
P(A\cap D) = P(A) \times P_A(D) = 0.58 \times 0.48 = 0.28
\]
\item Probabilité que le stylo ai un défaut de fabrication.
\[
P(D) = P(A\cap D) + P(B\cap D) = 0.15 + 0.68 = 0.83
P(D) = P(A\cap D) + P(B\cap D) = 0.28 + 0.11 = 0.39
\]
\end{enumerate}
\item Probabilité qu'il vienne de l'atelier A sachant qu'il a un defaut
\[
P_D(A) = \frac{P(A\cap D)}{P(D)} = \frac{0.15}{0.83} = 0.18
P_D(A) = \frac{P(A\cap D)}{P(D)} = \frac{0.28}{0.39} = 0.72
\]
\item $X$ peut être modélisée par une loi binomiale de paramètres $n=17$ et $p=0.83$.
\item $X$ peut être modélisée par une loi binomiale de paramètres $n=20$ et $p=0.39$.
\item (\textit{par de correction automatique disponible pour le résultat final}
\[
P(X = 10) = \coefBino{17}{10}\times 0.83^{10} \times 0.17^{7}
P(X = 11) = \coefBino{20}{11}\times 0.39^{11} \times 0.61^{9}
\]
\item (\textit{par de correction automatique disponible pour le résultat final}
Il faut calculer la probabilité qu'il y ait 0 stylo avec un defaut.
\[
P(X = 0) = \coefBino{17}{0}\times 0.83^{0} \times 0.17^{17}
P(X = 0) = \coefBino{20}{0}\times 0.39^{0} \times 0.61^{20}
\]
Puis comparer ce nombre à 0,5.
\item Il faut calculer l'espérance
\[
E[X] = n\times p = 17 \times 0.83 = 14.11
E[X] = n\times p = 20 \times 0.39 = 7.8
\]
\end{enumerate}
\end{solution}

View File

@ -3,7 +3,7 @@
% Title Page
\title{DM1 \hfill PERDRIX Camille}
\tribe{Maths complémentaire}
\tribe{Maths complémentaires}
\date{\hfillÀ render pour le jeudi 27 mai}
\xsimsetup{
@ -17,7 +17,7 @@ Les valeurs des exercices sont générés automatiquement. Si une valeur a un no
\begin{exercise}[subtitle={Optimisation de matière}]
\begin{minipage}{0.6\textwidth}
On se propose de fabriquer avec le moins de tôle possible une citerne fermée en forme de parallélépipède rectangle dont le volume intérieur doit être de $8m^3$. La longueur est aussi fixée à $4m$ par le cahier des charges.
On se propose de fabriquer avec le moins de tôle possible une citerne fermée en forme de parallélépipède rectangle dont le volume intérieur doit être de $30m^3$. La longueur est aussi fixée à $5m$ par le cahier des charges.
On peut donc faire varier uniquement la largeur (notée $x$) et la hauteur (notée $h$) de la cuve.
\end{minipage}
@ -28,25 +28,25 @@ Les valeurs des exercices sont générés automatiquement. Si une valeur a un no
\pgfmathsetmacro{\cubey}{1}
\pgfmathsetmacro{\cubez}{2}
\draw[black,fill=gray] (0,0,0) -- ++(-\cubex,0,0) -- ++(0,-\cubey,0) node [midway, left] {$h$} -- ++(\cubex,0,0) node [midway, below] {$x$} -- cycle;
\draw[black,fill=gray] (0,0,0) -- ++(0,0,-\cubez) -- ++(0,-\cubey,0) -- ++(0,0,\cubez) node [midway, right] {$4m$} -- cycle;
\draw[black,fill=gray] (0,0,0) -- ++(0,0,-\cubez) -- ++(0,-\cubey,0) -- ++(0,0,\cubez) node [midway, right] {$5m$} -- cycle;
\draw[black,fill=gray] (0,0,0) -- ++(-\cubex,0,0) -- ++(0,0,-\cubez) -- ++(\cubex,0,0) -- cycle;
\end{tikzpicture}
\end{minipage}
\begin{enumerate}
\item Expliquer pourquoi quand la largeur $x$ change, la hauteur $h$ doit elle aussi changer pour respecter les contraintes.
\item Démontrer que l'on doit avoir $h = \dfrac{2}{x}$.
\item Démontrer que l'on doit avoir $h = \dfrac{6}{x}$.
\item On note $S(x)$ l'aire totale de la citerne (c'est à dire la somme des aires des six faces). Montrer que l'on peut écrire
\[
S(x) = 8x + 4 + \frac{16}{x}
S(x) = 10x + 12 + \frac{60}{x}
\]
\item Démontrer que
\[
S(x) = \frac{8x^2 + 4x + 16}{x}
S(x) = \frac{10x^2 + 12x + 60}{x}
\]
\item Démontrer que
\[
S'(x) = \frac{8x^2 - 16}{x^2}
S'(x) = \frac{10x^2 - 60}{x^2}
\]
\item En déduire le tableau de variation de $S(x)$ sur $\intOF{0}{10}$.
\item Déterminer les valeurs de $x$ et $h$ correspondant à une utilisation minimal de tôle.
@ -57,62 +57,62 @@ Les valeurs des exercices sont générés automatiquement. Si une valeur a un no
\begin{enumerate}
\item Le volume étant fixe si l'on fait varier $x$, $h$ doit aussi varier.
\begin{itemize}
\item Si $x = 2$ alors conserver un volume de $V=8$, $h$ doit être égale à $2 / 2$
\item Si $x = 3$ alors conserver un volume de $V=8$, $h$ doit être égale à $2 / 3$
\item Si $x = 2$ alors conserver un volume de $V=30$, $h$ doit être égale à $6 / 2$
\item Si $x = 3$ alors conserver un volume de $V=30$, $h$ doit être égale à $6 / 3$
\end{itemize}
\item Pour calculer le volume, on a
\begin{eqnarray*}
V &=& h\times x \times 4 \\
8 &=& h\times x \times 4 \\
x &=& \frac{8}{h\times 4} = \frac{2}{h}
V &=& h\times x \times 5 \\
30 &=& h\times x \times 5 \\
x &=& \frac{30}{h\times 5} = \frac{6}{h}
\end{eqnarray*}
\item Pour calculer la surface totale, on ajoute la surface de chaque face. On a donc le calcul suivant
\begin{eqnarray*}
S(x) &=& x\times h \times 2 + x\times4\times2 + h\times 4\times 2\\
S(x) &=& x\times \frac{2}{x} \times 2 + x\times4\times2 + \frac{2}{x}\times 4\times 2\\
S(x) &=& 8x + 4 + \frac{16}{x}
S(x) &=& x\times h \times 2 + x\times5\times2 + h\times 5\times 2\\
S(x) &=& x\times \frac{6}{x} \times 2 + x\times5\times2 + \frac{6}{x}\times 5\times 2\\
S(x) &=& 10x + 12 + \frac{60}{x}
\end{eqnarray*}
\item Pour trouver cette nouvelle forme, on met chaque élément sur le même dénominateur
\begin{eqnarray*}
S(x) &=& 8x + 4 + \frac{16}{x}\\
S(x) &=& \frac{8x\times x}{x} + \frac{4\times x}{x} + \frac{16}{x}\\
S(x) &=& \frac{8x^2 + 4x + 16}{x}
S(x) &=& 10x + 12 + \frac{60}{x}\\
S(x) &=& \frac{10x\times x}{x} + \frac{12\times x}{x} + \frac{60}{x}\\
S(x) &=& \frac{10x^2 + 12x + 60}{x}
\end{eqnarray*}
\item On retrouve la formule $\frac{u}{v}$ à dériver
\[
u(x) = 8x^2 + 4x + 16 \Rightarrow u'(x) = 16x + 4
u(x) = 10x^2 + 12x + 60 \Rightarrow u'(x) = 20x + 12
\]
\[
v(x) = x \Rightarrow v'(x) = 1
\]
Donc au numérateur on obtient
\begin{eqnarray*}
u'(x)\times v(x) - u(x)\times v'(x) &=& (16x + 4)\times x - (8x^2 + 4x + 16)\times 1\\
&=& 8x^2 - 16
u'(x)\times v(x) - u(x)\times v'(x) &=& (20x + 12)\times x - (10x^2 + 12x + 60)\times 1\\
&=& 10x^2 - 60
\end{eqnarray*}
Donc
\[
S'(x) = \frac{8x^2 - 16}{x^2}
S'(x) = \frac{10x^2 - 60}{x^2}
\]
\item Tableau de variations de $S$
\begin{itemize}
\item Valeur interdite: $x^2 = 0 \equiv x = 0$
\item Signe de $8x^2 - 16$: c'est un polynôme du 2e degré
\item Signe de $10x^2 - 60$: c'est un polynôme du 2e degré
\[
\Delta = 512 > 0
\Delta = 2400 > 0
\]
Il y a donc 2 racines
\[
x_1 = - 1.4142135623730951 \qquad
x_2 = 1.4142135623730951
x_1 = - 2.449489742783178 \qquad
x_2 = 2.449489742783178
\]
Et on sait que $8x^2 - 16$ est du signe de $a$ donc positif en dehors des racines
Et on sait que $10x^2 - 60$ est du signe de $a$ donc positif en dehors des racines
\item Le dénominateur $x^2$ est toujours positif.
\item Tableau de variations
\begin{tikzpicture}[baseline=(a.north)]
\tkzTabInit[lgt=3,espcl=3]{$x$/1,$8x^2 - 16$/1, $x^2$/1, $S'$/1, $S$/2}{$0$, $- 1.4142135623730951$, $10$}
\tkzTabInit[lgt=3,espcl=3]{$x$/1,$10x^2 - 60$/1, $x^2$/1, $S'$/1, $S$/2}{$0$, $- 2.449489742783178$, $10$}
\tkzTabLine{d,-, z, +, }
\tkzTabLine{d,+, , +, }
\tkzTabLine{d,-, z, +, }
@ -120,7 +120,7 @@ Les valeurs des exercices sont générés automatiquement. Si une valeur a un no
\end{tikzpicture}
\end{itemize}
\item On a donc une surface minimal pour $x=1.4142135623730951$ et $h = 2.8284271247461902$.
\item On a donc une surface minimal pour $x=2.449489742783178$ et $h = 14.696938456699068$.
\end{enumerate}
\end{solution}
@ -133,14 +133,14 @@ Les valeurs des exercices sont générés automatiquement. Si une valeur a un no
Le tour d'un bassin au niveau du sol présente deux axes de symétrie : laxe des abscisses et la droite déquation $x=4$. Il est obtenu par symétrie de la courbe $\mathcal{C}_f$ sur $\intFF{0}{4}$$f$ est la fonction définie par
\[
f(x) = \left(- x^{2} + 8.3 x - 5.0\right) e^{- x} + 5.0
f(x) = \left(- x^{2} + 6.1 x - 9.6\right) e^{- x} + 9.6
\]
On admet que sur $\intFF{0}{4}$ la fonction $f$ est positive.
\begin{enumerate}
\item Sur un repère, tracer l'allure de la courbe $\mathcal{C}_f$, les axes de symétries puis compléter pour dessiner la forme du bassin.
\item Montrer que la fonction $f$ admet comme primitive sur $\R$ la fonction $F$ définie par
\[
F(x) = 5.0 x + \left( x^{2} - 6.3 x - 1.3\right) e^{- x}
F(x) = 9.6 x + \left( x^{2} - 4.1 x + 5.5\right) e^{- x}
\]
\item Calculer la quantité $\ds \int_0^4 f(x) \; dx$, vous donnerez le résultat sous forme exacte. Interpréter le résultat et reportez cette quantité sur le graphique.
\item On considère que l'échelle de votre graphique est de 1unité pour 15m. Calculer l'aire du bassin. Vous donnerez un résultat arrondi au $m^2$ près.
@ -156,15 +156,14 @@ Les valeurs des exercices sont générés automatiquement. Si une valeur a un no
\tkzGrid
\tkzAxeXY
\tkzFct[domain=0:10,color=red,very thick]%
{ (-x**2 + 8.3*x - 5.0)*exp(-x) + 5.0 };
{ (-x**2 + 6.1*x - 9.6)*exp(-x) + 9.6 };
\end{tikzpicture}
\item Il faut dériver $F(x)$ et vérifier que $F'(x) = f(x)$.
\item $\ds \int_0^4 f(x) \; dx = F(4) - F(0) = 21.3 - \frac{10.5}{e^{4}}$
\item $\ds \int_0^4 f(x) \; dx = F(4) - F(0) = \frac{5.1}{e^{4}} + 32.9$
\item La quantité calculée à la question précédente se retrouve 4fois pour former le bassin. Il faut ensuite prendre en compte l'échelle, comme 1unité de longueur correspond à 15m, une unité d'air correspond à $15\times15 = 225m^2$. Ainsi l'aire du bassin est égale à
\[
(21.3 - \frac{10.5}{e^{4}})\times 4 \times 15^2 = 18997.00000
(\frac{5.1}{e^{4}} + 32.9)\times 4 \times 15^2 = 29694.00000
\]
\end{enumerate}
\end{solution}
@ -185,9 +184,9 @@ Les valeurs des exercices sont générés automatiquement. Si une valeur a un no
Deux ateliers A et B fabriquent des stylos pour une entreprise.
L'atelier A fabrique 66.0\,\% des stylos, et parmi ceux-là, 35.0\,\% possèdent un défaut de fabrication.
L'atelier A fabrique 12.0\,\% des stylos, et parmi ceux-là, 55.00000000000001\,\% possèdent un défaut de fabrication.
De plus, 28.000000000000004\,\% des stylos possèdent un défaut de fabrication et sortent de l'atelier B.
De plus, 81.0\,\% des stylos possèdent un défaut de fabrication et sortent de l'atelier B.
Un stylo est prélevé au hasard dans le stock de l'entreprise.
@ -241,7 +240,7 @@ Les valeurs des exercices sont générés automatiquement. Si une valeur a un no
\item
\begin{enumerate}
\item Calculer la probabilité qu'un stylo provienne de l'atelier A et possède un défaut de fabrication.
\item En déduire que la probabilité qu'un stylo possède un défaut de fabrication est de $0.51$.
\item En déduire que la probabilité qu'un stylo possède un défaut de fabrication est de $0.88$.
\end{enumerate}
\item On prélève un stylo au hasard avec un défaut. Quelle est la probabilité qu'il vienne de l'atelier A?
\end{enumerate}
@ -251,7 +250,7 @@ Les valeurs des exercices sont générés automatiquement. Si une valeur a un no
\textbf{Partie B}
\medskip
Dans cette partie, on suppose que 51.0\,\% des stylos possèdent un défaut de fabrication.
Dans cette partie, on suppose que 88.0\,\% des stylos possèdent un défaut de fabrication.
L'entreprise confectionne des paquets contenant chacun $4$~stylos.
@ -266,7 +265,7 @@ Les valeurs des exercices sont générés automatiquement. Si une valeur a un no
\begin{enumerate}
\setcounter{enumi}{4}
\item Avec quelle loi peut-on modéliser $X$. Préciser les paramètres.
\item Calculer et interpréter la probabilité $P(X = 11)$.
\item Calculer et interpréter la probabilité $P(X = 8)$.
\item Le directeur de l'entreprise affirme qu'il y a plus d'une chance sur deux qu'un paquet ne comporte aucun stylo défectueux. A-t-il raison ?
\item Combien de stylos peut-on espérer avoir en moyenne?
\end{enumerate}
@ -282,27 +281,27 @@ Les valeurs des exercices sont générés automatiquement. Si une valeur a un no
child {node {$A$}
child {node {$D$}
edge from parent
node[above] {0.35}
node[above] {0.55}
}
child {node {$\overline{D}$}
edge from parent
node[above] {0.65}
node[above] {0.45}
}
edge from parent
node[above] {0.66}
node[above] {0.12}
}
child[missing] {}
child { node {$B$}
child {node {$D$}
edge from parent
node[above] {0.81}
node[above] {0.92}
}
child {node {$\overline{D}$}
edge from parent
node[above] {0.19}
node[above] {0.08}
}
edge from parent
node[above] {0.34}
node[above] {0.88}
} ;
\end{tikzpicture}
\end{center}
@ -310,51 +309,51 @@ Les valeurs des exercices sont générés automatiquement. Si une valeur a un no
\begin{itemize}
\item Probabilité que le stylo vienne de l'atelier A
\[
P(A) = 0.66
P(A) = 0.12
\]
\item Probabilité que le stylo vienne de l'atelier B
\[
P(B) = 0.34
P(B) = 0.88
\]
\item Probabilité que le stylo ait un défaut sachant qu'il vient de l'atelier A.
\[
P_A(D) = 0.35
P_A(D) = 0.55
\]
\item Probabilité que le stylo vienne de l'atelier B et qu'il ait un défaut.
\[
P(D \cap D) = 0.28
P(D \cap D) = 0.81
\]
\end{itemize}
\item
\begin{enumerate}
\item Probabilité qu'un stylo vienne de l'atelier A et qu'il ait un defaut
\[
P(A\cap D) = P(A) \times P_A(D) = 0.66 \times 0.35 = 0.23
P(A\cap D) = P(A) \times P_A(D) = 0.12 \times 0.55 = 0.07
\]
\item Probabilité que le stylo ai un défaut de fabrication.
\[
P(D) = P(A\cap D) + P(B\cap D) = 0.23 + 0.28 = 0.51
P(D) = P(A\cap D) + P(B\cap D) = 0.07 + 0.81 = 0.88
\]
\end{enumerate}
\item Probabilité qu'il vienne de l'atelier A sachant qu'il a un defaut
\[
P_D(A) = \frac{P(A\cap D)}{P(D)} = \frac{0.23}{0.51} = 0.45
P_D(A) = \frac{P(A\cap D)}{P(D)} = \frac{0.07}{0.88} = 0.08
\]
\item $X$ peut être modélisée par une loi binomiale de paramètres $n=19$ et $p=0.51$.
\item $X$ peut être modélisée par une loi binomiale de paramètres $n=15$ et $p=0.88$.
\item (\textit{par de correction automatique disponible pour le résultat final}
\[
P(X = 11) = \coefBino{19}{11}\times 0.51^{11} \times 0.49^{8}
P(X = 8) = \coefBino{15}{8}\times 0.88^{8} \times 0.12^{7}
\]
\item (\textit{par de correction automatique disponible pour le résultat final}
Il faut calculer la probabilité qu'il y ait 0 stylo avec un defaut.
\[
P(X = 0) = \coefBino{19}{0}\times 0.51^{0} \times 0.49^{19}
P(X = 0) = \coefBino{15}{0}\times 0.88^{0} \times 0.12^{15}
\]
Puis comparer ce nombre à 0,5.
\item Il faut calculer l'espérance
\[
E[X] = n\times p = 19 \times 0.51 = 9.69
E[X] = n\times p = 15 \times 0.88 = 13.2
\]
\end{enumerate}
\end{solution}

View File

@ -3,7 +3,7 @@
% Title Page
\title{DM1 \hfill POISON Lorette}
\tribe{Maths complémentaire}
\tribe{Maths complémentaires}
\date{\hfillÀ render pour le jeudi 27 mai}
\xsimsetup{
@ -17,7 +17,7 @@ Les valeurs des exercices sont générés automatiquement. Si une valeur a un no
\begin{exercise}[subtitle={Optimisation de matière}]
\begin{minipage}{0.6\textwidth}
On se propose de fabriquer avec le moins de tôle possible une citerne fermée en forme de parallélépipède rectangle dont le volume intérieur doit être de $35m^3$. La longueur est aussi fixée à $5m$ par le cahier des charges.
On se propose de fabriquer avec le moins de tôle possible une citerne fermée en forme de parallélépipède rectangle dont le volume intérieur doit être de $16m^3$. La longueur est aussi fixée à $4m$ par le cahier des charges.
On peut donc faire varier uniquement la largeur (notée $x$) et la hauteur (notée $h$) de la cuve.
\end{minipage}
@ -28,25 +28,25 @@ Les valeurs des exercices sont générés automatiquement. Si une valeur a un no
\pgfmathsetmacro{\cubey}{1}
\pgfmathsetmacro{\cubez}{2}
\draw[black,fill=gray] (0,0,0) -- ++(-\cubex,0,0) -- ++(0,-\cubey,0) node [midway, left] {$h$} -- ++(\cubex,0,0) node [midway, below] {$x$} -- cycle;
\draw[black,fill=gray] (0,0,0) -- ++(0,0,-\cubez) -- ++(0,-\cubey,0) -- ++(0,0,\cubez) node [midway, right] {$5m$} -- cycle;
\draw[black,fill=gray] (0,0,0) -- ++(0,0,-\cubez) -- ++(0,-\cubey,0) -- ++(0,0,\cubez) node [midway, right] {$4m$} -- cycle;
\draw[black,fill=gray] (0,0,0) -- ++(-\cubex,0,0) -- ++(0,0,-\cubez) -- ++(\cubex,0,0) -- cycle;
\end{tikzpicture}
\end{minipage}
\begin{enumerate}
\item Expliquer pourquoi quand la largeur $x$ change, la hauteur $h$ doit elle aussi changer pour respecter les contraintes.
\item Démontrer que l'on doit avoir $h = \dfrac{7}{x}$.
\item Démontrer que l'on doit avoir $h = \dfrac{4}{x}$.
\item On note $S(x)$ l'aire totale de la citerne (c'est à dire la somme des aires des six faces). Montrer que l'on peut écrire
\[
S(x) = 10x + 14 + \frac{70}{x}
S(x) = 8x + 8 + \frac{32}{x}
\]
\item Démontrer que
\[
S(x) = \frac{10x^2 + 14x + 70}{x}
S(x) = \frac{8x^2 + 8x + 32}{x}
\]
\item Démontrer que
\[
S'(x) = \frac{10x^2 - 70}{x^2}
S'(x) = \frac{8x^2 - 32}{x^2}
\]
\item En déduire le tableau de variation de $S(x)$ sur $\intOF{0}{10}$.
\item Déterminer les valeurs de $x$ et $h$ correspondant à une utilisation minimal de tôle.
@ -57,62 +57,62 @@ Les valeurs des exercices sont générés automatiquement. Si une valeur a un no
\begin{enumerate}
\item Le volume étant fixe si l'on fait varier $x$, $h$ doit aussi varier.
\begin{itemize}
\item Si $x = 2$ alors conserver un volume de $V=35$, $h$ doit être égale à $7 / 2$
\item Si $x = 3$ alors conserver un volume de $V=35$, $h$ doit être égale à $7 / 3$
\item Si $x = 2$ alors conserver un volume de $V=16$, $h$ doit être égale à $4 / 2$
\item Si $x = 3$ alors conserver un volume de $V=16$, $h$ doit être égale à $4 / 3$
\end{itemize}
\item Pour calculer le volume, on a
\begin{eqnarray*}
V &=& h\times x \times 5 \\
35 &=& h\times x \times 5 \\
x &=& \frac{35}{h\times 5} = \frac{7}{h}
V &=& h\times x \times 4 \\
16 &=& h\times x \times 4 \\
x &=& \frac{16}{h\times 4} = \frac{4}{h}
\end{eqnarray*}
\item Pour calculer la surface totale, on ajoute la surface de chaque face. On a donc le calcul suivant
\begin{eqnarray*}
S(x) &=& x\times h \times 2 + x\times5\times2 + h\times 5\times 2\\
S(x) &=& x\times \frac{7}{x} \times 2 + x\times5\times2 + \frac{7}{x}\times 5\times 2\\
S(x) &=& 10x + 14 + \frac{70}{x}
S(x) &=& x\times h \times 2 + x\times4\times2 + h\times 4\times 2\\
S(x) &=& x\times \frac{4}{x} \times 2 + x\times4\times2 + \frac{4}{x}\times 4\times 2\\
S(x) &=& 8x + 8 + \frac{32}{x}
\end{eqnarray*}
\item Pour trouver cette nouvelle forme, on met chaque élément sur le même dénominateur
\begin{eqnarray*}
S(x) &=& 10x + 14 + \frac{70}{x}\\
S(x) &=& \frac{10x\times x}{x} + \frac{14\times x}{x} + \frac{70}{x}\\
S(x) &=& \frac{10x^2 + 14x + 70}{x}
S(x) &=& 8x + 8 + \frac{32}{x}\\
S(x) &=& \frac{8x\times x}{x} + \frac{8\times x}{x} + \frac{32}{x}\\
S(x) &=& \frac{8x^2 + 8x + 32}{x}
\end{eqnarray*}
\item On retrouve la formule $\frac{u}{v}$ à dériver
\[
u(x) = 10x^2 + 14x + 70 \Rightarrow u'(x) = 20x + 14
u(x) = 8x^2 + 8x + 32 \Rightarrow u'(x) = 16x + 8
\]
\[
v(x) = x \Rightarrow v'(x) = 1
\]
Donc au numérateur on obtient
\begin{eqnarray*}
u'(x)\times v(x) - u(x)\times v'(x) &=& (20x + 14)\times x - (10x^2 + 14x + 70)\times 1\\
&=& 10x^2 - 70
u'(x)\times v(x) - u(x)\times v'(x) &=& (16x + 8)\times x - (8x^2 + 8x + 32)\times 1\\
&=& 8x^2 - 32
\end{eqnarray*}
Donc
\[
S'(x) = \frac{10x^2 - 70}{x^2}
S'(x) = \frac{8x^2 - 32}{x^2}
\]
\item Tableau de variations de $S$
\begin{itemize}
\item Valeur interdite: $x^2 = 0 \equiv x = 0$
\item Signe de $10x^2 - 70$: c'est un polynôme du 2e degré
\item Signe de $8x^2 - 32$: c'est un polynôme du 2e degré
\[
\Delta = 2800 > 0
\Delta = 1024 > 0
\]
Il y a donc 2 racines
\[
x_1 = - 2.6457513110645907 \qquad
x_2 = 2.6457513110645907
x_1 = - 2 \qquad
x_2 = 2
\]
Et on sait que $10x^2 - 70$ est du signe de $a$ donc positif en dehors des racines
Et on sait que $8x^2 - 32$ est du signe de $a$ donc positif en dehors des racines
\item Le dénominateur $x^2$ est toujours positif.
\item Tableau de variations
\begin{tikzpicture}[baseline=(a.north)]
\tkzTabInit[lgt=3,espcl=3]{$x$/1,$10x^2 - 70$/1, $x^2$/1, $S'$/1, $S$/2}{$0$, $- 2.6457513110645907$, $10$}
\tkzTabInit[lgt=3,espcl=3]{$x$/1,$8x^2 - 32$/1, $x^2$/1, $S'$/1, $S$/2}{$0$, $- 2$, $10$}
\tkzTabLine{d,-, z, +, }
\tkzTabLine{d,+, , +, }
\tkzTabLine{d,-, z, +, }
@ -120,7 +120,7 @@ Les valeurs des exercices sont générés automatiquement. Si une valeur a un no
\end{tikzpicture}
\end{itemize}
\item On a donc une surface minimal pour $x=2.6457513110645907$ et $h = 18.5202591774521349$.
\item On a donc une surface minimal pour $x=2$ et $h = 8$.
\end{enumerate}
\end{solution}
@ -133,14 +133,14 @@ Les valeurs des exercices sont générés automatiquement. Si une valeur a un no
Le tour d'un bassin au niveau du sol présente deux axes de symétrie : laxe des abscisses et la droite déquation $x=4$. Il est obtenu par symétrie de la courbe $\mathcal{C}_f$ sur $\intFF{0}{4}$$f$ est la fonction définie par
\[
f(x) = \left(- x^{2} + 6.1 x - 3.4\right) e^{- x} + 3.4
f(x) = \left(- x^{2} + 6.8 x - 6.4\right) e^{- x} + 6.4
\]
On admet que sur $\intFF{0}{4}$ la fonction $f$ est positive.
\begin{enumerate}
\item Sur un repère, tracer l'allure de la courbe $\mathcal{C}_f$, les axes de symétries puis compléter pour dessiner la forme du bassin.
\item Montrer que la fonction $f$ admet comme primitive sur $\R$ la fonction $F$ définie par
\[
F(x) = 3.4 x + \left( x^{2} - 4.1 x - 0.7\right) e^{- x}
F(x) = 6.4 x + \left( x^{2} - 4.8 x + 1.6\right) e^{- x}
\]
\item Calculer la quantité $\ds \int_0^4 f(x) \; dx$, vous donnerez le résultat sous forme exacte. Interpréter le résultat et reportez cette quantité sur le graphique.
\item On considère que l'échelle de votre graphique est de 1unité pour 15m. Calculer l'aire du bassin. Vous donnerez un résultat arrondi au $m^2$ près.
@ -156,15 +156,14 @@ Les valeurs des exercices sont générés automatiquement. Si une valeur a un no
\tkzGrid
\tkzAxeXY
\tkzFct[domain=0:10,color=red,very thick]%
{ (-x**2 + 6.1*x - 3.4)*exp(-x) + 3.4 };
{ (-x**2 + 6.8*x - 6.4)*exp(-x) + 6.4 };
\end{tikzpicture}
\item Il faut dériver $F(x)$ et vérifier que $F'(x) = f(x)$.
\item $\ds \int_0^4 f(x) \; dx = F(4) - F(0) = 14.3 - \frac{1.1}{e^{4}}$
\item $\ds \int_0^4 f(x) \; dx = F(4) - F(0) = 24.0 - \frac{1.6}{e^{4}}$
\item La quantité calculée à la question précédente se retrouve 4fois pour former le bassin. Il faut ensuite prendre en compte l'échelle, comme 1unité de longueur correspond à 15m, une unité d'air correspond à $15\times15 = 225m^2$. Ainsi l'aire du bassin est égale à
\[
(14.3 - \frac{1.1}{e^{4}})\times 4 \times 15^2 = 12852.00000
(24.0 - \frac{1.6}{e^{4}})\times 4 \times 15^2 = 21574.00000
\]
\end{enumerate}
\end{solution}
@ -185,9 +184,9 @@ Les valeurs des exercices sont générés automatiquement. Si une valeur a un no
Deux ateliers A et B fabriquent des stylos pour une entreprise.
L'atelier A fabrique 47.0\,\% des stylos, et parmi ceux-là, 95.0\,\% possèdent un défaut de fabrication.
L'atelier A fabrique 61.0\,\% des stylos, et parmi ceux-là, 21.0\,\% possèdent un défaut de fabrication.
De plus, 50.0\,\% des stylos possèdent un défaut de fabrication et sortent de l'atelier B.
De plus, 37.0\,\% des stylos possèdent un défaut de fabrication et sortent de l'atelier B.
Un stylo est prélevé au hasard dans le stock de l'entreprise.
@ -241,7 +240,7 @@ Les valeurs des exercices sont générés automatiquement. Si une valeur a un no
\item
\begin{enumerate}
\item Calculer la probabilité qu'un stylo provienne de l'atelier A et possède un défaut de fabrication.
\item En déduire que la probabilité qu'un stylo possède un défaut de fabrication est de $0.95$.
\item En déduire que la probabilité qu'un stylo possède un défaut de fabrication est de $0.5$.
\end{enumerate}
\item On prélève un stylo au hasard avec un défaut. Quelle est la probabilité qu'il vienne de l'atelier A?
\end{enumerate}
@ -251,7 +250,7 @@ Les valeurs des exercices sont générés automatiquement. Si une valeur a un no
\textbf{Partie B}
\medskip
Dans cette partie, on suppose que 95.0\,\% des stylos possèdent un défaut de fabrication.
Dans cette partie, on suppose que 50.0\,\% des stylos possèdent un défaut de fabrication.
L'entreprise confectionne des paquets contenant chacun $4$~stylos.
@ -266,7 +265,7 @@ Les valeurs des exercices sont générés automatiquement. Si une valeur a un no
\begin{enumerate}
\setcounter{enumi}{4}
\item Avec quelle loi peut-on modéliser $X$. Préciser les paramètres.
\item Calculer et interpréter la probabilité $P(X = 12)$.
\item Calculer et interpréter la probabilité $P(X = 10)$.
\item Le directeur de l'entreprise affirme qu'il y a plus d'une chance sur deux qu'un paquet ne comporte aucun stylo défectueux. A-t-il raison ?
\item Combien de stylos peut-on espérer avoir en moyenne?
\end{enumerate}
@ -282,27 +281,27 @@ Les valeurs des exercices sont générés automatiquement. Si une valeur a un no
child {node {$A$}
child {node {$D$}
edge from parent
node[above] {0.95}
node[above] {0.21}
}
child {node {$\overline{D}$}
edge from parent
node[above] {0.05}
node[above] {0.79}
}
edge from parent
node[above] {0.47}
node[above] {0.61}
}
child[missing] {}
child { node {$B$}
child {node {$D$}
edge from parent
node[above] {0.95}
node[above] {0.96}
}
child {node {$\overline{D}$}
edge from parent
node[above] {0.05}
node[above] {0.04}
}
edge from parent
node[above] {0.53}
node[above] {0.39}
} ;
\end{tikzpicture}
\end{center}
@ -310,51 +309,51 @@ Les valeurs des exercices sont générés automatiquement. Si une valeur a un no
\begin{itemize}
\item Probabilité que le stylo vienne de l'atelier A
\[
P(A) = 0.47
P(A) = 0.61
\]
\item Probabilité que le stylo vienne de l'atelier B
\[
P(B) = 0.53
P(B) = 0.39
\]
\item Probabilité que le stylo ait un défaut sachant qu'il vient de l'atelier A.
\[
P_A(D) = 0.95
P_A(D) = 0.21
\]
\item Probabilité que le stylo vienne de l'atelier B et qu'il ait un défaut.
\[
P(D \cap D) = 0.5
P(D \cap D) = 0.37
\]
\end{itemize}
\item
\begin{enumerate}
\item Probabilité qu'un stylo vienne de l'atelier A et qu'il ait un defaut
\[
P(A\cap D) = P(A) \times P_A(D) = 0.47 \times 0.95 = 0.45
P(A\cap D) = P(A) \times P_A(D) = 0.61 \times 0.21 = 0.13
\]
\item Probabilité que le stylo ai un défaut de fabrication.
\[
P(D) = P(A\cap D) + P(B\cap D) = 0.45 + 0.5 = 0.95
P(D) = P(A\cap D) + P(B\cap D) = 0.13 + 0.37 = 0.5
\]
\end{enumerate}
\item Probabilité qu'il vienne de l'atelier A sachant qu'il a un defaut
\[
P_D(A) = \frac{P(A\cap D)}{P(D)} = \frac{0.45}{0.95} = 0.47
P_D(A) = \frac{P(A\cap D)}{P(D)} = \frac{0.13}{0.5} = 0.26
\]
\item $X$ peut être modélisée par une loi binomiale de paramètres $n=15$ et $p=0.95$.
\item $X$ peut être modélisée par une loi binomiale de paramètres $n=10$ et $p=0.5$.
\item (\textit{par de correction automatique disponible pour le résultat final}
\[
P(X = 12) = \coefBino{15}{12}\times 0.95^{12} \times 0.05^{3}
P(X = 10) = \coefBino{10}{10}\times 0.5^{10} \times 0.5^{0}
\]
\item (\textit{par de correction automatique disponible pour le résultat final}
Il faut calculer la probabilité qu'il y ait 0 stylo avec un defaut.
\[
P(X = 0) = \coefBino{15}{0}\times 0.95^{0} \times 0.05^{15}
P(X = 0) = \coefBino{10}{0}\times 0.5^{0} \times 0.5^{10}
\]
Puis comparer ce nombre à 0,5.
\item Il faut calculer l'espérance
\[
E[X] = n\times p = 15 \times 0.95 = 14.25
E[X] = n\times p = 10 \times 0.5 = 5.0
\]
\end{enumerate}
\end{solution}

View File

@ -3,7 +3,7 @@
% Title Page
\title{DM1 \hfill RODRIGUEZ Teddy}
\tribe{Maths complémentaire}
\tribe{Maths complémentaires}
\date{\hfillÀ render pour le jeudi 27 mai}
\xsimsetup{
@ -17,7 +17,7 @@ Les valeurs des exercices sont générés automatiquement. Si une valeur a un no
\begin{exercise}[subtitle={Optimisation de matière}]
\begin{minipage}{0.6\textwidth}
On se propose de fabriquer avec le moins de tôle possible une citerne fermée en forme de parallélépipède rectangle dont le volume intérieur doit être de $50m^3$. La longueur est aussi fixée à $5m$ par le cahier des charges.
On se propose de fabriquer avec le moins de tôle possible une citerne fermée en forme de parallélépipède rectangle dont le volume intérieur doit être de $24m^3$. La longueur est aussi fixée à $3m$ par le cahier des charges.
On peut donc faire varier uniquement la largeur (notée $x$) et la hauteur (notée $h$) de la cuve.
\end{minipage}
@ -28,25 +28,25 @@ Les valeurs des exercices sont générés automatiquement. Si une valeur a un no
\pgfmathsetmacro{\cubey}{1}
\pgfmathsetmacro{\cubez}{2}
\draw[black,fill=gray] (0,0,0) -- ++(-\cubex,0,0) -- ++(0,-\cubey,0) node [midway, left] {$h$} -- ++(\cubex,0,0) node [midway, below] {$x$} -- cycle;
\draw[black,fill=gray] (0,0,0) -- ++(0,0,-\cubez) -- ++(0,-\cubey,0) -- ++(0,0,\cubez) node [midway, right] {$5m$} -- cycle;
\draw[black,fill=gray] (0,0,0) -- ++(0,0,-\cubez) -- ++(0,-\cubey,0) -- ++(0,0,\cubez) node [midway, right] {$3m$} -- cycle;
\draw[black,fill=gray] (0,0,0) -- ++(-\cubex,0,0) -- ++(0,0,-\cubez) -- ++(\cubex,0,0) -- cycle;
\end{tikzpicture}
\end{minipage}
\begin{enumerate}
\item Expliquer pourquoi quand la largeur $x$ change, la hauteur $h$ doit elle aussi changer pour respecter les contraintes.
\item Démontrer que l'on doit avoir $h = \dfrac{10}{x}$.
\item Démontrer que l'on doit avoir $h = \dfrac{8}{x}$.
\item On note $S(x)$ l'aire totale de la citerne (c'est à dire la somme des aires des six faces). Montrer que l'on peut écrire
\[
S(x) = 10x + 20 + \frac{100}{x}
S(x) = 6x + 16 + \frac{48}{x}
\]
\item Démontrer que
\[
S(x) = \frac{10x^2 + 20x + 100}{x}
S(x) = \frac{6x^2 + 16x + 48}{x}
\]
\item Démontrer que
\[
S'(x) = \frac{10x^2 - 100}{x^2}
S'(x) = \frac{6x^2 - 48}{x^2}
\]
\item En déduire le tableau de variation de $S(x)$ sur $\intOF{0}{10}$.
\item Déterminer les valeurs de $x$ et $h$ correspondant à une utilisation minimal de tôle.
@ -57,62 +57,62 @@ Les valeurs des exercices sont générés automatiquement. Si une valeur a un no
\begin{enumerate}
\item Le volume étant fixe si l'on fait varier $x$, $h$ doit aussi varier.
\begin{itemize}
\item Si $x = 2$ alors conserver un volume de $V=50$, $h$ doit être égale à $10 / 2$
\item Si $x = 3$ alors conserver un volume de $V=50$, $h$ doit être égale à $10 / 3$
\item Si $x = 2$ alors conserver un volume de $V=24$, $h$ doit être égale à $8 / 2$
\item Si $x = 3$ alors conserver un volume de $V=24$, $h$ doit être égale à $8 / 3$
\end{itemize}
\item Pour calculer le volume, on a
\begin{eqnarray*}
V &=& h\times x \times 5 \\
50 &=& h\times x \times 5 \\
x &=& \frac{50}{h\times 5} = \frac{10}{h}
V &=& h\times x \times 3 \\
24 &=& h\times x \times 3 \\
x &=& \frac{24}{h\times 3} = \frac{8}{h}
\end{eqnarray*}
\item Pour calculer la surface totale, on ajoute la surface de chaque face. On a donc le calcul suivant
\begin{eqnarray*}
S(x) &=& x\times h \times 2 + x\times5\times2 + h\times 5\times 2\\
S(x) &=& x\times \frac{10}{x} \times 2 + x\times5\times2 + \frac{10}{x}\times 5\times 2\\
S(x) &=& 10x + 20 + \frac{100}{x}
S(x) &=& x\times h \times 2 + x\times3\times2 + h\times 3\times 2\\
S(x) &=& x\times \frac{8}{x} \times 2 + x\times3\times2 + \frac{8}{x}\times 3\times 2\\
S(x) &=& 6x + 16 + \frac{48}{x}
\end{eqnarray*}
\item Pour trouver cette nouvelle forme, on met chaque élément sur le même dénominateur
\begin{eqnarray*}
S(x) &=& 10x + 20 + \frac{100}{x}\\
S(x) &=& \frac{10x\times x}{x} + \frac{20\times x}{x} + \frac{100}{x}\\
S(x) &=& \frac{10x^2 + 20x + 100}{x}
S(x) &=& 6x + 16 + \frac{48}{x}\\
S(x) &=& \frac{6x\times x}{x} + \frac{16\times x}{x} + \frac{48}{x}\\
S(x) &=& \frac{6x^2 + 16x + 48}{x}
\end{eqnarray*}
\item On retrouve la formule $\frac{u}{v}$ à dériver
\[
u(x) = 10x^2 + 20x + 100 \Rightarrow u'(x) = 20x + 20
u(x) = 6x^2 + 16x + 48 \Rightarrow u'(x) = 12x + 16
\]
\[
v(x) = x \Rightarrow v'(x) = 1
\]
Donc au numérateur on obtient
\begin{eqnarray*}
u'(x)\times v(x) - u(x)\times v'(x) &=& (20x + 20)\times x - (10x^2 + 20x + 100)\times 1\\
&=& 10x^2 - 100
u'(x)\times v(x) - u(x)\times v'(x) &=& (12x + 16)\times x - (6x^2 + 16x + 48)\times 1\\
&=& 6x^2 - 48
\end{eqnarray*}
Donc
\[
S'(x) = \frac{10x^2 - 100}{x^2}
S'(x) = \frac{6x^2 - 48}{x^2}
\]
\item Tableau de variations de $S$
\begin{itemize}
\item Valeur interdite: $x^2 = 0 \equiv x = 0$
\item Signe de $10x^2 - 100$: c'est un polynôme du 2e degré
\item Signe de $6x^2 - 48$: c'est un polynôme du 2e degré
\[
\Delta = 4000 > 0
\Delta = 1152 > 0
\]
Il y a donc 2 racines
\[
x_1 = - 3.162277660168379 \qquad
x_2 = 3.162277660168379
x_1 = - 2.82842712474619 \qquad
x_2 = 2.82842712474619
\]
Et on sait que $10x^2 - 100$ est du signe de $a$ donc positif en dehors des racines
Et on sait que $6x^2 - 48$ est du signe de $a$ donc positif en dehors des racines
\item Le dénominateur $x^2$ est toujours positif.
\item Tableau de variations
\begin{tikzpicture}[baseline=(a.north)]
\tkzTabInit[lgt=3,espcl=3]{$x$/1,$10x^2 - 100$/1, $x^2$/1, $S'$/1, $S$/2}{$0$, $- 3.162277660168379$, $10$}
\tkzTabInit[lgt=3,espcl=3]{$x$/1,$6x^2 - 48$/1, $x^2$/1, $S'$/1, $S$/2}{$0$, $- 2.82842712474619$, $10$}
\tkzTabLine{d,-, z, +, }
\tkzTabLine{d,+, , +, }
\tkzTabLine{d,-, z, +, }
@ -120,7 +120,7 @@ Les valeurs des exercices sont générés automatiquement. Si une valeur a un no
\end{tikzpicture}
\end{itemize}
\item On a donc une surface minimal pour $x=3.162277660168379$ et $h = 31.622776601683790$.
\item On a donc une surface minimal pour $x=2.82842712474619$ et $h = 22.62741699796952$.
\end{enumerate}
\end{solution}
@ -133,14 +133,14 @@ Les valeurs des exercices sont générés automatiquement. Si une valeur a un no
Le tour d'un bassin au niveau du sol présente deux axes de symétrie : laxe des abscisses et la droite déquation $x=4$. Il est obtenu par symétrie de la courbe $\mathcal{C}_f$ sur $\intFF{0}{4}$$f$ est la fonction définie par
\[
f(x) = \left(- x^{2} + 1.0 x - 3.6\right) e^{- x} + 3.6
f(x) = \left(- x^{2} + 3.8 x - 4.0\right) e^{- x} + 4.0
\]
On admet que sur $\intFF{0}{4}$ la fonction $f$ est positive.
\begin{enumerate}
\item Sur un repère, tracer l'allure de la courbe $\mathcal{C}_f$, les axes de symétries puis compléter pour dessiner la forme du bassin.
\item Montrer que la fonction $f$ admet comme primitive sur $\R$ la fonction $F$ définie par
\[
F(x) = 3.6 x + \left( x^{2} + x + 4.6\right) e^{- x}
F(x) = 4.0 x + \left( x^{2} - 1.8 x + 2.2\right) e^{- x}
\]
\item Calculer la quantité $\ds \int_0^4 f(x) \; dx$, vous donnerez le résultat sous forme exacte. Interpréter le résultat et reportez cette quantité sur le graphique.
\item On considère que l'échelle de votre graphique est de 1unité pour 15m. Calculer l'aire du bassin. Vous donnerez un résultat arrondi au $m^2$ près.
@ -156,15 +156,14 @@ Les valeurs des exercices sont générés automatiquement. Si une valeur a un no
\tkzGrid
\tkzAxeXY
\tkzFct[domain=0:10,color=red,very thick]%
{ (-x**2 + 1.0*x - 3.6)*exp(-x) + 3.6 };
{ (-x**2 + 3.8*x - 4.0)*exp(-x) + 4.0 };
\end{tikzpicture}
\item Il faut dériver $F(x)$ et vérifier que $F'(x) = f(x)$.
\item $\ds \int_0^4 f(x) \; dx = F(4) - F(0) = \frac{24.6}{e^{4}} + 9.8$
\item $\ds \int_0^4 f(x) \; dx = F(4) - F(0) = \frac{11.0}{e^{4}} + 13.8$
\item La quantité calculée à la question précédente se retrouve 4fois pour former le bassin. Il faut ensuite prendre en compte l'échelle, comme 1unité de longueur correspond à 15m, une unité d'air correspond à $15\times15 = 225m^2$. Ainsi l'aire du bassin est égale à
\[
(\frac{24.6}{e^{4}} + 9.8)\times 4 \times 15^2 = 9226.000000
(\frac{11.0}{e^{4}} + 13.8)\times 4 \times 15^2 = 12601.00000
\]
\end{enumerate}
\end{solution}
@ -185,9 +184,9 @@ Les valeurs des exercices sont générés automatiquement. Si une valeur a un no
Deux ateliers A et B fabriquent des stylos pour une entreprise.
L'atelier A fabrique 64.0\,\% des stylos, et parmi ceux-là, 4.0\,\% possèdent un défaut de fabrication.
L'atelier A fabrique 56.99999999999999\,\% des stylos, et parmi ceux-là, 6.0\,\% possèdent un défaut de fabrication.
De plus, 6.0\,\% des stylos possèdent un défaut de fabrication et sortent de l'atelier B.
De plus, 42.0\,\% des stylos possèdent un défaut de fabrication et sortent de l'atelier B.
Un stylo est prélevé au hasard dans le stock de l'entreprise.
@ -241,7 +240,7 @@ Les valeurs des exercices sont générés automatiquement. Si une valeur a un no
\item
\begin{enumerate}
\item Calculer la probabilité qu'un stylo provienne de l'atelier A et possède un défaut de fabrication.
\item En déduire que la probabilité qu'un stylo possède un défaut de fabrication est de $0.09$.
\item En déduire que la probabilité qu'un stylo possède un défaut de fabrication est de $0.45$.
\end{enumerate}
\item On prélève un stylo au hasard avec un défaut. Quelle est la probabilité qu'il vienne de l'atelier A?
\end{enumerate}
@ -251,7 +250,7 @@ Les valeurs des exercices sont générés automatiquement. Si une valeur a un no
\textbf{Partie B}
\medskip
Dans cette partie, on suppose que 9.0\,\% des stylos possèdent un défaut de fabrication.
Dans cette partie, on suppose que 45.0\,\% des stylos possèdent un défaut de fabrication.
L'entreprise confectionne des paquets contenant chacun $4$~stylos.
@ -266,7 +265,7 @@ Les valeurs des exercices sont générés automatiquement. Si une valeur a un no
\begin{enumerate}
\setcounter{enumi}{4}
\item Avec quelle loi peut-on modéliser $X$. Préciser les paramètres.
\item Calculer et interpréter la probabilité $P(X = 9)$.
\item Calculer et interpréter la probabilité $P(X = 8)$.
\item Le directeur de l'entreprise affirme qu'il y a plus d'une chance sur deux qu'un paquet ne comporte aucun stylo défectueux. A-t-il raison ?
\item Combien de stylos peut-on espérer avoir en moyenne?
\end{enumerate}
@ -282,27 +281,27 @@ Les valeurs des exercices sont générés automatiquement. Si une valeur a un no
child {node {$A$}
child {node {$D$}
edge from parent
node[above] {0.04}
node[above] {0.06}
}
child {node {$\overline{D}$}
edge from parent
node[above] {0.96}
node[above] {0.94}
}
edge from parent
node[above] {0.64}
node[above] {0.57}
}
child[missing] {}
child { node {$B$}
child {node {$D$}
edge from parent
node[above] {0.18}
node[above] {0.97}
}
child {node {$\overline{D}$}
edge from parent
node[above] {0.82}
node[above] {0.03}
}
edge from parent
node[above] {0.36}
node[above] {0.43}
} ;
\end{tikzpicture}
\end{center}
@ -310,51 +309,51 @@ Les valeurs des exercices sont générés automatiquement. Si une valeur a un no
\begin{itemize}
\item Probabilité que le stylo vienne de l'atelier A
\[
P(A) = 0.64
P(A) = 0.57
\]
\item Probabilité que le stylo vienne de l'atelier B
\[
P(B) = 0.36
P(B) = 0.43
\]
\item Probabilité que le stylo ait un défaut sachant qu'il vient de l'atelier A.
\[
P_A(D) = 0.04
P_A(D) = 0.06
\]
\item Probabilité que le stylo vienne de l'atelier B et qu'il ait un défaut.
\[
P(D \cap D) = 0.06
P(D \cap D) = 0.42
\]
\end{itemize}
\item
\begin{enumerate}
\item Probabilité qu'un stylo vienne de l'atelier A et qu'il ait un defaut
\[
P(A\cap D) = P(A) \times P_A(D) = 0.64 \times 0.04 = 0.03
P(A\cap D) = P(A) \times P_A(D) = 0.57 \times 0.06 = 0.03
\]
\item Probabilité que le stylo ai un défaut de fabrication.
\[
P(D) = P(A\cap D) + P(B\cap D) = 0.03 + 0.06 = 0.09
P(D) = P(A\cap D) + P(B\cap D) = 0.03 + 0.42 = 0.45
\]
\end{enumerate}
\item Probabilité qu'il vienne de l'atelier A sachant qu'il a un defaut
\[
P_D(A) = \frac{P(A\cap D)}{P(D)} = \frac{0.03}{0.09} = 0.33
P_D(A) = \frac{P(A\cap D)}{P(D)} = \frac{0.03}{0.45} = 0.07
\]
\item $X$ peut être modélisée par une loi binomiale de paramètres $n=16$ et $p=0.09$.
\item $X$ peut être modélisée par une loi binomiale de paramètres $n=11$ et $p=0.45$.
\item (\textit{par de correction automatique disponible pour le résultat final}
\[
P(X = 9) = \coefBino{16}{9}\times 0.09^{9} \times 0.91^{7}
P(X = 8) = \coefBino{11}{8}\times 0.45^{8} \times 0.55^{3}
\]
\item (\textit{par de correction automatique disponible pour le résultat final}
Il faut calculer la probabilité qu'il y ait 0 stylo avec un defaut.
\[
P(X = 0) = \coefBino{16}{0}\times 0.09^{0} \times 0.91^{16}
P(X = 0) = \coefBino{11}{0}\times 0.45^{0} \times 0.55^{11}
\]
Puis comparer ce nombre à 0,5.
\item Il faut calculer l'espérance
\[
E[X] = n\times p = 16 \times 0.09 = 1.44
E[X] = n\times p = 11 \times 0.45 = 4.95
\]
\end{enumerate}
\end{solution}

View File

@ -3,7 +3,7 @@
% Title Page
\title{DM1 \hfill SAINT CYR Louis}
\tribe{Maths complémentaire}
\tribe{Maths complémentaires}
\date{\hfillÀ render pour le jeudi 27 mai}
\xsimsetup{
@ -17,7 +17,7 @@ Les valeurs des exercices sont générés automatiquement. Si une valeur a un no
\begin{exercise}[subtitle={Optimisation de matière}]
\begin{minipage}{0.6\textwidth}
On se propose de fabriquer avec le moins de tôle possible une citerne fermée en forme de parallélépipède rectangle dont le volume intérieur doit être de $45m^3$. La longueur est aussi fixée à $5m$ par le cahier des charges.
On se propose de fabriquer avec le moins de tôle possible une citerne fermée en forme de parallélépipède rectangle dont le volume intérieur doit être de $8m^3$. La longueur est aussi fixée à $2m$ par le cahier des charges.
On peut donc faire varier uniquement la largeur (notée $x$) et la hauteur (notée $h$) de la cuve.
\end{minipage}
@ -28,25 +28,25 @@ Les valeurs des exercices sont générés automatiquement. Si une valeur a un no
\pgfmathsetmacro{\cubey}{1}
\pgfmathsetmacro{\cubez}{2}
\draw[black,fill=gray] (0,0,0) -- ++(-\cubex,0,0) -- ++(0,-\cubey,0) node [midway, left] {$h$} -- ++(\cubex,0,0) node [midway, below] {$x$} -- cycle;
\draw[black,fill=gray] (0,0,0) -- ++(0,0,-\cubez) -- ++(0,-\cubey,0) -- ++(0,0,\cubez) node [midway, right] {$5m$} -- cycle;
\draw[black,fill=gray] (0,0,0) -- ++(0,0,-\cubez) -- ++(0,-\cubey,0) -- ++(0,0,\cubez) node [midway, right] {$2m$} -- cycle;
\draw[black,fill=gray] (0,0,0) -- ++(-\cubex,0,0) -- ++(0,0,-\cubez) -- ++(\cubex,0,0) -- cycle;
\end{tikzpicture}
\end{minipage}
\begin{enumerate}
\item Expliquer pourquoi quand la largeur $x$ change, la hauteur $h$ doit elle aussi changer pour respecter les contraintes.
\item Démontrer que l'on doit avoir $h = \dfrac{9}{x}$.
\item Démontrer que l'on doit avoir $h = \dfrac{4}{x}$.
\item On note $S(x)$ l'aire totale de la citerne (c'est à dire la somme des aires des six faces). Montrer que l'on peut écrire
\[
S(x) = 10x + 18 + \frac{90}{x}
S(x) = 4x + 8 + \frac{16}{x}
\]
\item Démontrer que
\[
S(x) = \frac{10x^2 + 18x + 90}{x}
S(x) = \frac{4x^2 + 8x + 16}{x}
\]
\item Démontrer que
\[
S'(x) = \frac{10x^2 - 90}{x^2}
S'(x) = \frac{4x^2 - 16}{x^2}
\]
\item En déduire le tableau de variation de $S(x)$ sur $\intOF{0}{10}$.
\item Déterminer les valeurs de $x$ et $h$ correspondant à une utilisation minimal de tôle.
@ -57,62 +57,62 @@ Les valeurs des exercices sont générés automatiquement. Si une valeur a un no
\begin{enumerate}
\item Le volume étant fixe si l'on fait varier $x$, $h$ doit aussi varier.
\begin{itemize}
\item Si $x = 2$ alors conserver un volume de $V=45$, $h$ doit être égale à $9 / 2$
\item Si $x = 3$ alors conserver un volume de $V=45$, $h$ doit être égale à $9 / 3$
\item Si $x = 2$ alors conserver un volume de $V=8$, $h$ doit être égale à $4 / 2$
\item Si $x = 3$ alors conserver un volume de $V=8$, $h$ doit être égale à $4 / 3$
\end{itemize}
\item Pour calculer le volume, on a
\begin{eqnarray*}
V &=& h\times x \times 5 \\
45 &=& h\times x \times 5 \\
x &=& \frac{45}{h\times 5} = \frac{9}{h}
V &=& h\times x \times 2 \\
8 &=& h\times x \times 2 \\
x &=& \frac{8}{h\times 2} = \frac{4}{h}
\end{eqnarray*}
\item Pour calculer la surface totale, on ajoute la surface de chaque face. On a donc le calcul suivant
\begin{eqnarray*}
S(x) &=& x\times h \times 2 + x\times5\times2 + h\times 5\times 2\\
S(x) &=& x\times \frac{9}{x} \times 2 + x\times5\times2 + \frac{9}{x}\times 5\times 2\\
S(x) &=& 10x + 18 + \frac{90}{x}
S(x) &=& x\times h \times 2 + x\times2\times2 + h\times 2\times 2\\
S(x) &=& x\times \frac{4}{x} \times 2 + x\times2\times2 + \frac{4}{x}\times 2\times 2\\
S(x) &=& 4x + 8 + \frac{16}{x}
\end{eqnarray*}
\item Pour trouver cette nouvelle forme, on met chaque élément sur le même dénominateur
\begin{eqnarray*}
S(x) &=& 10x + 18 + \frac{90}{x}\\
S(x) &=& \frac{10x\times x}{x} + \frac{18\times x}{x} + \frac{90}{x}\\
S(x) &=& \frac{10x^2 + 18x + 90}{x}
S(x) &=& 4x + 8 + \frac{16}{x}\\
S(x) &=& \frac{4x\times x}{x} + \frac{8\times x}{x} + \frac{16}{x}\\
S(x) &=& \frac{4x^2 + 8x + 16}{x}
\end{eqnarray*}
\item On retrouve la formule $\frac{u}{v}$ à dériver
\[
u(x) = 10x^2 + 18x + 90 \Rightarrow u'(x) = 20x + 18
u(x) = 4x^2 + 8x + 16 \Rightarrow u'(x) = 8x + 8
\]
\[
v(x) = x \Rightarrow v'(x) = 1
\]
Donc au numérateur on obtient
\begin{eqnarray*}
u'(x)\times v(x) - u(x)\times v'(x) &=& (20x + 18)\times x - (10x^2 + 18x + 90)\times 1\\
&=& 10x^2 - 90
u'(x)\times v(x) - u(x)\times v'(x) &=& (8x + 8)\times x - (4x^2 + 8x + 16)\times 1\\
&=& 4x^2 - 16
\end{eqnarray*}
Donc
\[
S'(x) = \frac{10x^2 - 90}{x^2}
S'(x) = \frac{4x^2 - 16}{x^2}
\]
\item Tableau de variations de $S$
\begin{itemize}
\item Valeur interdite: $x^2 = 0 \equiv x = 0$
\item Signe de $10x^2 - 90$: c'est un polynôme du 2e degré
\item Signe de $4x^2 - 16$: c'est un polynôme du 2e degré
\[
\Delta = 3600 > 0
\Delta = 256 > 0
\]
Il y a donc 2 racines
\[
x_1 = - 3 \qquad
x_2 = 3
x_1 = - 2 \qquad
x_2 = 2
\]
Et on sait que $10x^2 - 90$ est du signe de $a$ donc positif en dehors des racines
Et on sait que $4x^2 - 16$ est du signe de $a$ donc positif en dehors des racines
\item Le dénominateur $x^2$ est toujours positif.
\item Tableau de variations
\begin{tikzpicture}[baseline=(a.north)]
\tkzTabInit[lgt=3,espcl=3]{$x$/1,$10x^2 - 90$/1, $x^2$/1, $S'$/1, $S$/2}{$0$, $- 3$, $10$}
\tkzTabInit[lgt=3,espcl=3]{$x$/1,$4x^2 - 16$/1, $x^2$/1, $S'$/1, $S$/2}{$0$, $- 2$, $10$}
\tkzTabLine{d,-, z, +, }
\tkzTabLine{d,+, , +, }
\tkzTabLine{d,-, z, +, }
@ -120,7 +120,7 @@ Les valeurs des exercices sont générés automatiquement. Si une valeur a un no
\end{tikzpicture}
\end{itemize}
\item On a donc une surface minimal pour $x=3$ et $h = 27$.
\item On a donc une surface minimal pour $x=2$ et $h = 8$.
\end{enumerate}
\end{solution}
@ -133,14 +133,14 @@ Les valeurs des exercices sont générés automatiquement. Si une valeur a un no
Le tour d'un bassin au niveau du sol présente deux axes de symétrie : laxe des abscisses et la droite déquation $x=4$. Il est obtenu par symétrie de la courbe $\mathcal{C}_f$ sur $\intFF{0}{4}$$f$ est la fonction définie par
\[
f(x) = \left(- x^{2} + 8.0 x - 0.9\right) e^{- x} + 0.9
f(x) = \left(- x^{2} + 2.5 x - 6.7\right) e^{- x} + 6.7
\]
On admet que sur $\intFF{0}{4}$ la fonction $f$ est positive.
\begin{enumerate}
\item Sur un repère, tracer l'allure de la courbe $\mathcal{C}_f$, les axes de symétries puis compléter pour dessiner la forme du bassin.
\item Montrer que la fonction $f$ admet comme primitive sur $\R$ la fonction $F$ définie par
\[
F(x) = 0.9 x + \left( x^{2} - 6.0 x - 5.1\right) e^{- x}
F(x) = 6.7 x + \left( x^{2} - 0.5 x + 6.2\right) e^{- x}
\]
\item Calculer la quantité $\ds \int_0^4 f(x) \; dx$, vous donnerez le résultat sous forme exacte. Interpréter le résultat et reportez cette quantité sur le graphique.
\item On considère que l'échelle de votre graphique est de 1unité pour 15m. Calculer l'aire du bassin. Vous donnerez un résultat arrondi au $m^2$ près.
@ -156,15 +156,14 @@ Les valeurs des exercices sont générés automatiquement. Si une valeur a un no
\tkzGrid
\tkzAxeXY
\tkzFct[domain=0:10,color=red,very thick]%
{ (-x**2 + 8.0*x - 0.9)*exp(-x) + 0.9 };
{ (-x**2 + 2.5*x - 6.7)*exp(-x) + 6.7 };
\end{tikzpicture}
\item Il faut dériver $F(x)$ et vérifier que $F'(x) = f(x)$.
\item $\ds \int_0^4 f(x) \; dx = F(4) - F(0) = 8.7 - \frac{13.1}{e^{4}}$
\item $\ds \int_0^4 f(x) \; dx = F(4) - F(0) = \frac{20.2}{e^{4}} + 20.6$
\item La quantité calculée à la question précédente se retrouve 4fois pour former le bassin. Il faut ensuite prendre en compte l'échelle, comme 1unité de longueur correspond à 15m, une unité d'air correspond à $15\times15 = 225m^2$. Ainsi l'aire du bassin est égale à
\[
(8.7 - \frac{13.1}{e^{4}})\times 4 \times 15^2 = 7614.000000
(\frac{20.2}{e^{4}} + 20.6)\times 4 \times 15^2 = 18873.00000
\]
\end{enumerate}
\end{solution}
@ -185,9 +184,9 @@ Les valeurs des exercices sont générés automatiquement. Si une valeur a un no
Deux ateliers A et B fabriquent des stylos pour une entreprise.
L'atelier A fabrique 88.0\,\% des stylos, et parmi ceux-là, 75.0\,\% possèdent un défaut de fabrication.
L'atelier A fabrique 99.0\,\% des stylos, et parmi ceux-là, 28.000000000000004\,\% possèdent un défaut de fabrication.
De plus, 4.0\,\% des stylos possèdent un défaut de fabrication et sortent de l'atelier B.
De plus, 1.0\,\% des stylos possèdent un défaut de fabrication et sortent de l'atelier B.
Un stylo est prélevé au hasard dans le stock de l'entreprise.
@ -241,7 +240,7 @@ Les valeurs des exercices sont générés automatiquement. Si une valeur a un no
\item
\begin{enumerate}
\item Calculer la probabilité qu'un stylo provienne de l'atelier A et possède un défaut de fabrication.
\item En déduire que la probabilité qu'un stylo possède un défaut de fabrication est de $0.7$.
\item En déduire que la probabilité qu'un stylo possède un défaut de fabrication est de $0.29$.
\end{enumerate}
\item On prélève un stylo au hasard avec un défaut. Quelle est la probabilité qu'il vienne de l'atelier A?
\end{enumerate}
@ -251,7 +250,7 @@ Les valeurs des exercices sont générés automatiquement. Si une valeur a un no
\textbf{Partie B}
\medskip
Dans cette partie, on suppose que 70.0\,\% des stylos possèdent un défaut de fabrication.
Dans cette partie, on suppose que 28.999999999999996\,\% des stylos possèdent un défaut de fabrication.
L'entreprise confectionne des paquets contenant chacun $4$~stylos.
@ -266,7 +265,7 @@ Les valeurs des exercices sont générés automatiquement. Si une valeur a un no
\begin{enumerate}
\setcounter{enumi}{4}
\item Avec quelle loi peut-on modéliser $X$. Préciser les paramètres.
\item Calculer et interpréter la probabilité $P(X = 14)$.
\item Calculer et interpréter la probabilité $P(X = 9)$.
\item Le directeur de l'entreprise affirme qu'il y a plus d'une chance sur deux qu'un paquet ne comporte aucun stylo défectueux. A-t-il raison ?
\item Combien de stylos peut-on espérer avoir en moyenne?
\end{enumerate}
@ -282,27 +281,27 @@ Les valeurs des exercices sont générés automatiquement. Si une valeur a un no
child {node {$A$}
child {node {$D$}
edge from parent
node[above] {0.75}
node[above] {0.28}
}
child {node {$\overline{D}$}
edge from parent
node[above] {0.25}
node[above] {0.72}
}
edge from parent
node[above] {0.88}
node[above] {0.99}
}
child[missing] {}
child { node {$B$}
child {node {$D$}
edge from parent
node[above] {0.34}
node[above] {0.82}
}
child {node {$\overline{D}$}
edge from parent
node[above] {0.66}
node[above] {0.18}
}
edge from parent
node[above] {0.12}
node[above] {0.01}
} ;
\end{tikzpicture}
\end{center}
@ -310,51 +309,51 @@ Les valeurs des exercices sont générés automatiquement. Si une valeur a un no
\begin{itemize}
\item Probabilité que le stylo vienne de l'atelier A
\[
P(A) = 0.88
P(A) = 0.99
\]
\item Probabilité que le stylo vienne de l'atelier B
\[
P(B) = 0.12
P(B) = 0.01
\]
\item Probabilité que le stylo ait un défaut sachant qu'il vient de l'atelier A.
\[
P_A(D) = 0.75
P_A(D) = 0.28
\]
\item Probabilité que le stylo vienne de l'atelier B et qu'il ait un défaut.
\[
P(D \cap D) = 0.04
P(D \cap D) = 0.01
\]
\end{itemize}
\item
\begin{enumerate}
\item Probabilité qu'un stylo vienne de l'atelier A et qu'il ait un defaut
\[
P(A\cap D) = P(A) \times P_A(D) = 0.88 \times 0.75 = 0.66
P(A\cap D) = P(A) \times P_A(D) = 0.99 \times 0.28 = 0.28
\]
\item Probabilité que le stylo ai un défaut de fabrication.
\[
P(D) = P(A\cap D) + P(B\cap D) = 0.66 + 0.04 = 0.7
P(D) = P(A\cap D) + P(B\cap D) = 0.28 + 0.01 = 0.29
\]
\end{enumerate}
\item Probabilité qu'il vienne de l'atelier A sachant qu'il a un defaut
\[
P_D(A) = \frac{P(A\cap D)}{P(D)} = \frac{0.66}{0.7} = 0.94
P_D(A) = \frac{P(A\cap D)}{P(D)} = \frac{0.28}{0.29} = 0.97
\]
\item $X$ peut être modélisée par une loi binomiale de paramètres $n=15$ et $p=0.7$.
\item $X$ peut être modélisée par une loi binomiale de paramètres $n=10$ et $p=0.29$.
\item (\textit{par de correction automatique disponible pour le résultat final}
\[
P(X = 14) = \coefBino{15}{14}\times 0.7^{14} \times 0.3^{1}
P(X = 9) = \coefBino{10}{9}\times 0.29^{9} \times 0.71^{1}
\]
\item (\textit{par de correction automatique disponible pour le résultat final}
Il faut calculer la probabilité qu'il y ait 0 stylo avec un defaut.
\[
P(X = 0) = \coefBino{15}{0}\times 0.7^{0} \times 0.3^{15}
P(X = 0) = \coefBino{10}{0}\times 0.29^{0} \times 0.71^{10}
\]
Puis comparer ce nombre à 0,5.
\item Il faut calculer l'espérance
\[
E[X] = n\times p = 15 \times 0.7 = 10.5
E[X] = n\times p = 10 \times 0.29 = 2.9
\]
\end{enumerate}
\end{solution}

View File

@ -3,7 +3,7 @@
% Title Page
\title{DM1 \hfill SAVIN Lou-Ann}
\tribe{Maths complémentaire}
\tribe{Maths complémentaires}
\date{\hfillÀ render pour le jeudi 27 mai}
\xsimsetup{
@ -17,7 +17,7 @@ Les valeurs des exercices sont générés automatiquement. Si une valeur a un no
\begin{exercise}[subtitle={Optimisation de matière}]
\begin{minipage}{0.6\textwidth}
On se propose de fabriquer avec le moins de tôle possible une citerne fermée en forme de parallélépipède rectangle dont le volume intérieur doit être de $18m^3$. La longueur est aussi fixée à $3m$ par le cahier des charges.
On se propose de fabriquer avec le moins de tôle possible une citerne fermée en forme de parallélépipède rectangle dont le volume intérieur doit être de $4m^3$. La longueur est aussi fixée à $2m$ par le cahier des charges.
On peut donc faire varier uniquement la largeur (notée $x$) et la hauteur (notée $h$) de la cuve.
\end{minipage}
@ -28,25 +28,25 @@ Les valeurs des exercices sont générés automatiquement. Si une valeur a un no
\pgfmathsetmacro{\cubey}{1}
\pgfmathsetmacro{\cubez}{2}
\draw[black,fill=gray] (0,0,0) -- ++(-\cubex,0,0) -- ++(0,-\cubey,0) node [midway, left] {$h$} -- ++(\cubex,0,0) node [midway, below] {$x$} -- cycle;
\draw[black,fill=gray] (0,0,0) -- ++(0,0,-\cubez) -- ++(0,-\cubey,0) -- ++(0,0,\cubez) node [midway, right] {$3m$} -- cycle;
\draw[black,fill=gray] (0,0,0) -- ++(0,0,-\cubez) -- ++(0,-\cubey,0) -- ++(0,0,\cubez) node [midway, right] {$2m$} -- cycle;
\draw[black,fill=gray] (0,0,0) -- ++(-\cubex,0,0) -- ++(0,0,-\cubez) -- ++(\cubex,0,0) -- cycle;
\end{tikzpicture}
\end{minipage}
\begin{enumerate}
\item Expliquer pourquoi quand la largeur $x$ change, la hauteur $h$ doit elle aussi changer pour respecter les contraintes.
\item Démontrer que l'on doit avoir $h = \dfrac{6}{x}$.
\item Démontrer que l'on doit avoir $h = \dfrac{2}{x}$.
\item On note $S(x)$ l'aire totale de la citerne (c'est à dire la somme des aires des six faces). Montrer que l'on peut écrire
\[
S(x) = 6x + 12 + \frac{36}{x}
S(x) = 4x + 4 + \frac{8}{x}
\]
\item Démontrer que
\[
S(x) = \frac{6x^2 + 12x + 36}{x}
S(x) = \frac{4x^2 + 4x + 8}{x}
\]
\item Démontrer que
\[
S'(x) = \frac{6x^2 - 36}{x^2}
S'(x) = \frac{4x^2 - 8}{x^2}
\]
\item En déduire le tableau de variation de $S(x)$ sur $\intOF{0}{10}$.
\item Déterminer les valeurs de $x$ et $h$ correspondant à une utilisation minimal de tôle.
@ -57,62 +57,62 @@ Les valeurs des exercices sont générés automatiquement. Si une valeur a un no
\begin{enumerate}
\item Le volume étant fixe si l'on fait varier $x$, $h$ doit aussi varier.
\begin{itemize}
\item Si $x = 2$ alors conserver un volume de $V=18$, $h$ doit être égale à $6 / 2$
\item Si $x = 3$ alors conserver un volume de $V=18$, $h$ doit être égale à $6 / 3$
\item Si $x = 2$ alors conserver un volume de $V=4$, $h$ doit être égale à $2 / 2$
\item Si $x = 3$ alors conserver un volume de $V=4$, $h$ doit être égale à $2 / 3$
\end{itemize}
\item Pour calculer le volume, on a
\begin{eqnarray*}
V &=& h\times x \times 3 \\
18 &=& h\times x \times 3 \\
x &=& \frac{18}{h\times 3} = \frac{6}{h}
V &=& h\times x \times 2 \\
4 &=& h\times x \times 2 \\
x &=& \frac{4}{h\times 2} = \frac{2}{h}
\end{eqnarray*}
\item Pour calculer la surface totale, on ajoute la surface de chaque face. On a donc le calcul suivant
\begin{eqnarray*}
S(x) &=& x\times h \times 2 + x\times3\times2 + h\times 3\times 2\\
S(x) &=& x\times \frac{6}{x} \times 2 + x\times3\times2 + \frac{6}{x}\times 3\times 2\\
S(x) &=& 6x + 12 + \frac{36}{x}
S(x) &=& x\times h \times 2 + x\times2\times2 + h\times 2\times 2\\
S(x) &=& x\times \frac{2}{x} \times 2 + x\times2\times2 + \frac{2}{x}\times 2\times 2\\
S(x) &=& 4x + 4 + \frac{8}{x}
\end{eqnarray*}
\item Pour trouver cette nouvelle forme, on met chaque élément sur le même dénominateur
\begin{eqnarray*}
S(x) &=& 6x + 12 + \frac{36}{x}\\
S(x) &=& \frac{6x\times x}{x} + \frac{12\times x}{x} + \frac{36}{x}\\
S(x) &=& \frac{6x^2 + 12x + 36}{x}
S(x) &=& 4x + 4 + \frac{8}{x}\\
S(x) &=& \frac{4x\times x}{x} + \frac{4\times x}{x} + \frac{8}{x}\\
S(x) &=& \frac{4x^2 + 4x + 8}{x}
\end{eqnarray*}
\item On retrouve la formule $\frac{u}{v}$ à dériver
\[
u(x) = 6x^2 + 12x + 36 \Rightarrow u'(x) = 12x + 12
u(x) = 4x^2 + 4x + 8 \Rightarrow u'(x) = 8x + 4
\]
\[
v(x) = x \Rightarrow v'(x) = 1
\]
Donc au numérateur on obtient
\begin{eqnarray*}
u'(x)\times v(x) - u(x)\times v'(x) &=& (12x + 12)\times x - (6x^2 + 12x + 36)\times 1\\
&=& 6x^2 - 36
u'(x)\times v(x) - u(x)\times v'(x) &=& (8x + 4)\times x - (4x^2 + 4x + 8)\times 1\\
&=& 4x^2 - 8
\end{eqnarray*}
Donc
\[
S'(x) = \frac{6x^2 - 36}{x^2}
S'(x) = \frac{4x^2 - 8}{x^2}
\]
\item Tableau de variations de $S$
\begin{itemize}
\item Valeur interdite: $x^2 = 0 \equiv x = 0$
\item Signe de $6x^2 - 36$: c'est un polynôme du 2e degré
\item Signe de $4x^2 - 8$: c'est un polynôme du 2e degré
\[
\Delta = 864 > 0
\Delta = 128 > 0
\]
Il y a donc 2 racines
\[
x_1 = - 2.4494897427831783 \qquad
x_2 = 2.4494897427831783
x_1 = - 1.4142135623730951 \qquad
x_2 = 1.4142135623730951
\]
Et on sait que $6x^2 - 36$ est du signe de $a$ donc positif en dehors des racines
Et on sait que $4x^2 - 8$ est du signe de $a$ donc positif en dehors des racines
\item Le dénominateur $x^2$ est toujours positif.
\item Tableau de variations
\begin{tikzpicture}[baseline=(a.north)]
\tkzTabInit[lgt=3,espcl=3]{$x$/1,$6x^2 - 36$/1, $x^2$/1, $S'$/1, $S$/2}{$0$, $- 2.4494897427831783$, $10$}
\tkzTabInit[lgt=3,espcl=3]{$x$/1,$4x^2 - 8$/1, $x^2$/1, $S'$/1, $S$/2}{$0$, $- 1.4142135623730951$, $10$}
\tkzTabLine{d,-, z, +, }
\tkzTabLine{d,+, , +, }
\tkzTabLine{d,-, z, +, }
@ -120,7 +120,7 @@ Les valeurs des exercices sont générés automatiquement. Si une valeur a un no
\end{tikzpicture}
\end{itemize}
\item On a donc une surface minimal pour $x=2.4494897427831783$ et $h = 14.6969384566990698$.
\item On a donc une surface minimal pour $x=1.4142135623730951$ et $h = 2.8284271247461902$.
\end{enumerate}
\end{solution}
@ -133,14 +133,14 @@ Les valeurs des exercices sont générés automatiquement. Si une valeur a un no
Le tour d'un bassin au niveau du sol présente deux axes de symétrie : laxe des abscisses et la droite déquation $x=4$. Il est obtenu par symétrie de la courbe $\mathcal{C}_f$ sur $\intFF{0}{4}$$f$ est la fonction définie par
\[
f(x) = \left(- x^{2} + 2.7 x - 6.1\right) e^{- x} + 6.1
f(x) = \left(- x^{2} + 2.2 x - 5.8\right) e^{- x} + 5.8
\]
On admet que sur $\intFF{0}{4}$ la fonction $f$ est positive.
\begin{enumerate}
\item Sur un repère, tracer l'allure de la courbe $\mathcal{C}_f$, les axes de symétries puis compléter pour dessiner la forme du bassin.
\item Montrer que la fonction $f$ admet comme primitive sur $\R$ la fonction $F$ définie par
\[
F(x) = 6.1 x + \left( x^{2} - 0.7 x + 5.4\right) e^{- x}
F(x) = 5.8 x + \left( x^{2} - 0.2 x + 5.6\right) e^{- x}
\]
\item Calculer la quantité $\ds \int_0^4 f(x) \; dx$, vous donnerez le résultat sous forme exacte. Interpréter le résultat et reportez cette quantité sur le graphique.
\item On considère que l'échelle de votre graphique est de 1unité pour 15m. Calculer l'aire du bassin. Vous donnerez un résultat arrondi au $m^2$ près.
@ -156,15 +156,14 @@ Les valeurs des exercices sont générés automatiquement. Si une valeur a un no
\tkzGrid
\tkzAxeXY
\tkzFct[domain=0:10,color=red,very thick]%
{ (-x**2 + 2.7*x - 6.1)*exp(-x) + 6.1 };
{ (-x**2 + 2.2*x - 5.8)*exp(-x) + 5.8 };
\end{tikzpicture}
\item Il faut dériver $F(x)$ et vérifier que $F'(x) = f(x)$.
\item $\ds \int_0^4 f(x) \; dx = F(4) - F(0) = \frac{18.6}{e^{4}} + 19.0$
\item $\ds \int_0^4 f(x) \; dx = F(4) - F(0) = \frac{20.8}{e^{4}} + 17.6$
\item La quantité calculée à la question précédente se retrouve 4fois pour former le bassin. Il faut ensuite prendre en compte l'échelle, comme 1unité de longueur correspond à 15m, une unité d'air correspond à $15\times15 = 225m^2$. Ainsi l'aire du bassin est égale à
\[
(\frac{18.6}{e^{4}} + 19.0)\times 4 \times 15^2 = 17407.00000
(\frac{20.8}{e^{4}} + 17.6)\times 4 \times 15^2 = 16183.00000
\]
\end{enumerate}
\end{solution}
@ -185,9 +184,9 @@ Les valeurs des exercices sont générés automatiquement. Si une valeur a un no
Deux ateliers A et B fabriquent des stylos pour une entreprise.
L'atelier A fabrique 47.0\,\% des stylos, et parmi ceux-là, 45.0\,\% possèdent un défaut de fabrication.
L'atelier A fabrique 93.0\,\% des stylos, et parmi ceux-là, 62.0\,\% possèdent un défaut de fabrication.
De plus, 3.0\,\% des stylos possèdent un défaut de fabrication et sortent de l'atelier B.
De plus, 1.0\,\% des stylos possèdent un défaut de fabrication et sortent de l'atelier B.
Un stylo est prélevé au hasard dans le stock de l'entreprise.
@ -241,7 +240,7 @@ Les valeurs des exercices sont générés automatiquement. Si une valeur a un no
\item
\begin{enumerate}
\item Calculer la probabilité qu'un stylo provienne de l'atelier A et possède un défaut de fabrication.
\item En déduire que la probabilité qu'un stylo possède un défaut de fabrication est de $0.24$.
\item En déduire que la probabilité qu'un stylo possède un défaut de fabrication est de $0.59$.
\end{enumerate}
\item On prélève un stylo au hasard avec un défaut. Quelle est la probabilité qu'il vienne de l'atelier A?
\end{enumerate}
@ -251,7 +250,7 @@ Les valeurs des exercices sont générés automatiquement. Si une valeur a un no
\textbf{Partie B}
\medskip
Dans cette partie, on suppose que 24.0\,\% des stylos possèdent un défaut de fabrication.
Dans cette partie, on suppose que 59.0\,\% des stylos possèdent un défaut de fabrication.
L'entreprise confectionne des paquets contenant chacun $4$~stylos.
@ -266,7 +265,7 @@ Les valeurs des exercices sont générés automatiquement. Si une valeur a un no
\begin{enumerate}
\setcounter{enumi}{4}
\item Avec quelle loi peut-on modéliser $X$. Préciser les paramètres.
\item Calculer et interpréter la probabilité $P(X = 9)$.
\item Calculer et interpréter la probabilité $P(X = 11)$.
\item Le directeur de l'entreprise affirme qu'il y a plus d'une chance sur deux qu'un paquet ne comporte aucun stylo défectueux. A-t-il raison ?
\item Combien de stylos peut-on espérer avoir en moyenne?
\end{enumerate}
@ -282,27 +281,27 @@ Les valeurs des exercices sont générés automatiquement. Si une valeur a un no
child {node {$A$}
child {node {$D$}
edge from parent
node[above] {0.45}
node[above] {0.62}
}
child {node {$\overline{D}$}
edge from parent
node[above] {0.55}
node[above] {0.38}
}
edge from parent
node[above] {0.47}
node[above] {0.93}
}
child[missing] {}
child { node {$B$}
child {node {$D$}
edge from parent
node[above] {0.05}
node[above] {0.16}
}
child {node {$\overline{D}$}
edge from parent
node[above] {0.95}
node[above] {0.84}
}
edge from parent
node[above] {0.53}
node[above] {0.07}
} ;
\end{tikzpicture}
\end{center}
@ -310,51 +309,51 @@ Les valeurs des exercices sont générés automatiquement. Si une valeur a un no
\begin{itemize}
\item Probabilité que le stylo vienne de l'atelier A
\[
P(A) = 0.47
P(A) = 0.93
\]
\item Probabilité que le stylo vienne de l'atelier B
\[
P(B) = 0.53
P(B) = 0.07
\]
\item Probabilité que le stylo ait un défaut sachant qu'il vient de l'atelier A.
\[
P_A(D) = 0.45
P_A(D) = 0.62
\]
\item Probabilité que le stylo vienne de l'atelier B et qu'il ait un défaut.
\[
P(D \cap D) = 0.03
P(D \cap D) = 0.01
\]
\end{itemize}
\item
\begin{enumerate}
\item Probabilité qu'un stylo vienne de l'atelier A et qu'il ait un defaut
\[
P(A\cap D) = P(A) \times P_A(D) = 0.47 \times 0.45 = 0.21
P(A\cap D) = P(A) \times P_A(D) = 0.93 \times 0.62 = 0.58
\]
\item Probabilité que le stylo ai un défaut de fabrication.
\[
P(D) = P(A\cap D) + P(B\cap D) = 0.21 + 0.03 = 0.24
P(D) = P(A\cap D) + P(B\cap D) = 0.58 + 0.01 = 0.59
\]
\end{enumerate}
\item Probabilité qu'il vienne de l'atelier A sachant qu'il a un defaut
\[
P_D(A) = \frac{P(A\cap D)}{P(D)} = \frac{0.21}{0.24} = 0.88
P_D(A) = \frac{P(A\cap D)}{P(D)} = \frac{0.58}{0.59} = 0.98
\]
\item $X$ peut être modélisée par une loi binomiale de paramètres $n=19$ et $p=0.24$.
\item $X$ peut être modélisée par une loi binomiale de paramètres $n=13$ et $p=0.59$.
\item (\textit{par de correction automatique disponible pour le résultat final}
\[
P(X = 9) = \coefBino{19}{9}\times 0.24^{9} \times 0.76^{10}
P(X = 11) = \coefBino{13}{11}\times 0.59^{11} \times 0.41^{2}
\]
\item (\textit{par de correction automatique disponible pour le résultat final}
Il faut calculer la probabilité qu'il y ait 0 stylo avec un defaut.
\[
P(X = 0) = \coefBino{19}{0}\times 0.24^{0} \times 0.76^{19}
P(X = 0) = \coefBino{13}{0}\times 0.59^{0} \times 0.41^{13}
\]
Puis comparer ce nombre à 0,5.
\item Il faut calculer l'espérance
\[
E[X] = n\times p = 19 \times 0.24 = 4.56
E[X] = n\times p = 13 \times 0.59 = 7.67
\]
\end{enumerate}
\end{solution}

View File

@ -3,7 +3,7 @@
% Title Page
\title{DM1 \hfill SILVA LOPES Katleen}
\tribe{Maths complémentaire}
\tribe{Maths complémentaires}
\date{\hfillÀ render pour le jeudi 27 mai}
\xsimsetup{
@ -17,7 +17,7 @@ Les valeurs des exercices sont générés automatiquement. Si une valeur a un no
\begin{exercise}[subtitle={Optimisation de matière}]
\begin{minipage}{0.6\textwidth}
On se propose de fabriquer avec le moins de tôle possible une citerne fermée en forme de parallélépipède rectangle dont le volume intérieur doit être de $27m^3$. La longueur est aussi fixée à $3m$ par le cahier des charges.
On se propose de fabriquer avec le moins de tôle possible une citerne fermée en forme de parallélépipède rectangle dont le volume intérieur doit être de $10m^3$. La longueur est aussi fixée à $5m$ par le cahier des charges.
On peut donc faire varier uniquement la largeur (notée $x$) et la hauteur (notée $h$) de la cuve.
\end{minipage}
@ -28,25 +28,25 @@ Les valeurs des exercices sont générés automatiquement. Si une valeur a un no
\pgfmathsetmacro{\cubey}{1}
\pgfmathsetmacro{\cubez}{2}
\draw[black,fill=gray] (0,0,0) -- ++(-\cubex,0,0) -- ++(0,-\cubey,0) node [midway, left] {$h$} -- ++(\cubex,0,0) node [midway, below] {$x$} -- cycle;
\draw[black,fill=gray] (0,0,0) -- ++(0,0,-\cubez) -- ++(0,-\cubey,0) -- ++(0,0,\cubez) node [midway, right] {$3m$} -- cycle;
\draw[black,fill=gray] (0,0,0) -- ++(0,0,-\cubez) -- ++(0,-\cubey,0) -- ++(0,0,\cubez) node [midway, right] {$5m$} -- cycle;
\draw[black,fill=gray] (0,0,0) -- ++(-\cubex,0,0) -- ++(0,0,-\cubez) -- ++(\cubex,0,0) -- cycle;
\end{tikzpicture}
\end{minipage}
\begin{enumerate}
\item Expliquer pourquoi quand la largeur $x$ change, la hauteur $h$ doit elle aussi changer pour respecter les contraintes.
\item Démontrer que l'on doit avoir $h = \dfrac{9}{x}$.
\item Démontrer que l'on doit avoir $h = \dfrac{2}{x}$.
\item On note $S(x)$ l'aire totale de la citerne (c'est à dire la somme des aires des six faces). Montrer que l'on peut écrire
\[
S(x) = 6x + 18 + \frac{54}{x}
S(x) = 10x + 4 + \frac{20}{x}
\]
\item Démontrer que
\[
S(x) = \frac{6x^2 + 18x + 54}{x}
S(x) = \frac{10x^2 + 4x + 20}{x}
\]
\item Démontrer que
\[
S'(x) = \frac{6x^2 - 54}{x^2}
S'(x) = \frac{10x^2 - 20}{x^2}
\]
\item En déduire le tableau de variation de $S(x)$ sur $\intOF{0}{10}$.
\item Déterminer les valeurs de $x$ et $h$ correspondant à une utilisation minimal de tôle.
@ -57,62 +57,62 @@ Les valeurs des exercices sont générés automatiquement. Si une valeur a un no
\begin{enumerate}
\item Le volume étant fixe si l'on fait varier $x$, $h$ doit aussi varier.
\begin{itemize}
\item Si $x = 2$ alors conserver un volume de $V=27$, $h$ doit être égale à $9 / 2$
\item Si $x = 3$ alors conserver un volume de $V=27$, $h$ doit être égale à $9 / 3$
\item Si $x = 2$ alors conserver un volume de $V=10$, $h$ doit être égale à $2 / 2$
\item Si $x = 3$ alors conserver un volume de $V=10$, $h$ doit être égale à $2 / 3$
\end{itemize}
\item Pour calculer le volume, on a
\begin{eqnarray*}
V &=& h\times x \times 3 \\
27 &=& h\times x \times 3 \\
x &=& \frac{27}{h\times 3} = \frac{9}{h}
V &=& h\times x \times 5 \\
10 &=& h\times x \times 5 \\
x &=& \frac{10}{h\times 5} = \frac{2}{h}
\end{eqnarray*}
\item Pour calculer la surface totale, on ajoute la surface de chaque face. On a donc le calcul suivant
\begin{eqnarray*}
S(x) &=& x\times h \times 2 + x\times3\times2 + h\times 3\times 2\\
S(x) &=& x\times \frac{9}{x} \times 2 + x\times3\times2 + \frac{9}{x}\times 3\times 2\\
S(x) &=& 6x + 18 + \frac{54}{x}
S(x) &=& x\times h \times 2 + x\times5\times2 + h\times 5\times 2\\
S(x) &=& x\times \frac{2}{x} \times 2 + x\times5\times2 + \frac{2}{x}\times 5\times 2\\
S(x) &=& 10x + 4 + \frac{20}{x}
\end{eqnarray*}
\item Pour trouver cette nouvelle forme, on met chaque élément sur le même dénominateur
\begin{eqnarray*}
S(x) &=& 6x + 18 + \frac{54}{x}\\
S(x) &=& \frac{6x\times x}{x} + \frac{18\times x}{x} + \frac{54}{x}\\
S(x) &=& \frac{6x^2 + 18x + 54}{x}
S(x) &=& 10x + 4 + \frac{20}{x}\\
S(x) &=& \frac{10x\times x}{x} + \frac{4\times x}{x} + \frac{20}{x}\\
S(x) &=& \frac{10x^2 + 4x + 20}{x}
\end{eqnarray*}
\item On retrouve la formule $\frac{u}{v}$ à dériver
\[
u(x) = 6x^2 + 18x + 54 \Rightarrow u'(x) = 12x + 18
u(x) = 10x^2 + 4x + 20 \Rightarrow u'(x) = 20x + 4
\]
\[
v(x) = x \Rightarrow v'(x) = 1
\]
Donc au numérateur on obtient
\begin{eqnarray*}
u'(x)\times v(x) - u(x)\times v'(x) &=& (12x + 18)\times x - (6x^2 + 18x + 54)\times 1\\
&=& 6x^2 - 54
u'(x)\times v(x) - u(x)\times v'(x) &=& (20x + 4)\times x - (10x^2 + 4x + 20)\times 1\\
&=& 10x^2 - 20
\end{eqnarray*}
Donc
\[
S'(x) = \frac{6x^2 - 54}{x^2}
S'(x) = \frac{10x^2 - 20}{x^2}
\]
\item Tableau de variations de $S$
\begin{itemize}
\item Valeur interdite: $x^2 = 0 \equiv x = 0$
\item Signe de $6x^2 - 54$: c'est un polynôme du 2e degré
\item Signe de $10x^2 - 20$: c'est un polynôme du 2e degré
\[
\Delta = 1296 > 0
\Delta = 800 > 0
\]
Il y a donc 2 racines
\[
x_1 = - 3 \qquad
x_2 = 3
x_1 = - 1.4142135623730951 \qquad
x_2 = 1.4142135623730951
\]
Et on sait que $6x^2 - 54$ est du signe de $a$ donc positif en dehors des racines
Et on sait que $10x^2 - 20$ est du signe de $a$ donc positif en dehors des racines
\item Le dénominateur $x^2$ est toujours positif.
\item Tableau de variations
\begin{tikzpicture}[baseline=(a.north)]
\tkzTabInit[lgt=3,espcl=3]{$x$/1,$6x^2 - 54$/1, $x^2$/1, $S'$/1, $S$/2}{$0$, $- 3$, $10$}
\tkzTabInit[lgt=3,espcl=3]{$x$/1,$10x^2 - 20$/1, $x^2$/1, $S'$/1, $S$/2}{$0$, $- 1.4142135623730951$, $10$}
\tkzTabLine{d,-, z, +, }
\tkzTabLine{d,+, , +, }
\tkzTabLine{d,-, z, +, }
@ -120,7 +120,7 @@ Les valeurs des exercices sont générés automatiquement. Si une valeur a un no
\end{tikzpicture}
\end{itemize}
\item On a donc une surface minimal pour $x=3$ et $h = 27$.
\item On a donc une surface minimal pour $x=1.4142135623730951$ et $h = 2.8284271247461902$.
\end{enumerate}
\end{solution}
@ -133,14 +133,14 @@ Les valeurs des exercices sont générés automatiquement. Si une valeur a un no
Le tour d'un bassin au niveau du sol présente deux axes de symétrie : laxe des abscisses et la droite déquation $x=4$. Il est obtenu par symétrie de la courbe $\mathcal{C}_f$ sur $\intFF{0}{4}$$f$ est la fonction définie par
\[
f(x) = \left(- x^{2} + 3.0 x - 6.6\right) e^{- x} + 6.6
f(x) = \left(- x^{2} + 4.3 x - 6.7\right) e^{- x} + 6.7
\]
On admet que sur $\intFF{0}{4}$ la fonction $f$ est positive.
\begin{enumerate}
\item Sur un repère, tracer l'allure de la courbe $\mathcal{C}_f$, les axes de symétries puis compléter pour dessiner la forme du bassin.
\item Montrer que la fonction $f$ admet comme primitive sur $\R$ la fonction $F$ définie par
\[
F(x) = 6.6 x + \left( x^{2} - x + 5.6\right) e^{- x}
F(x) = 6.7 x + \left( x^{2} - 2.3 x + 4.4\right) e^{- x}
\]
\item Calculer la quantité $\ds \int_0^4 f(x) \; dx$, vous donnerez le résultat sous forme exacte. Interpréter le résultat et reportez cette quantité sur le graphique.
\item On considère que l'échelle de votre graphique est de 1unité pour 15m. Calculer l'aire du bassin. Vous donnerez un résultat arrondi au $m^2$ près.
@ -156,15 +156,14 @@ Les valeurs des exercices sont générés automatiquement. Si une valeur a un no
\tkzGrid
\tkzAxeXY
\tkzFct[domain=0:10,color=red,very thick]%
{ (-x**2 + 3.0*x - 6.6)*exp(-x) + 6.6 };
{ (-x**2 + 4.3*x - 6.7)*exp(-x) + 6.7 };
\end{tikzpicture}
\item Il faut dériver $F(x)$ et vérifier que $F'(x) = f(x)$.
\item $\ds \int_0^4 f(x) \; dx = F(4) - F(0) = \frac{17.6}{e^{4}} + 20.8$
\item $\ds \int_0^4 f(x) \; dx = F(4) - F(0) = \frac{11.2}{e^{4}} + 22.4$
\item La quantité calculée à la question précédente se retrouve 4fois pour former le bassin. Il faut ensuite prendre en compte l'échelle, comme 1unité de longueur correspond à 15m, une unité d'air correspond à $15\times15 = 225m^2$. Ainsi l'aire du bassin est égale à
\[
(\frac{17.6}{e^{4}} + 20.8)\times 4 \times 15^2 = 19010.00000
(\frac{11.2}{e^{4}} + 22.4)\times 4 \times 15^2 = 20345.00000
\]
\end{enumerate}
\end{solution}
@ -185,9 +184,9 @@ Les valeurs des exercices sont générés automatiquement. Si une valeur a un no
Deux ateliers A et B fabriquent des stylos pour une entreprise.
L'atelier A fabrique 43.0\,\% des stylos, et parmi ceux-là, 31.0\,\% possèdent un défaut de fabrication.
L'atelier A fabrique 24.0\,\% des stylos, et parmi ceux-là, 52.0\,\% possèdent un défaut de fabrication.
De plus, 19.0\,\% des stylos possèdent un défaut de fabrication et sortent de l'atelier B.
De plus, 71.0\,\% des stylos possèdent un défaut de fabrication et sortent de l'atelier B.
Un stylo est prélevé au hasard dans le stock de l'entreprise.
@ -241,7 +240,7 @@ Les valeurs des exercices sont générés automatiquement. Si une valeur a un no
\item
\begin{enumerate}
\item Calculer la probabilité qu'un stylo provienne de l'atelier A et possède un défaut de fabrication.
\item En déduire que la probabilité qu'un stylo possède un défaut de fabrication est de $0.32$.
\item En déduire que la probabilité qu'un stylo possède un défaut de fabrication est de $0.83$.
\end{enumerate}
\item On prélève un stylo au hasard avec un défaut. Quelle est la probabilité qu'il vienne de l'atelier A?
\end{enumerate}
@ -251,7 +250,7 @@ Les valeurs des exercices sont générés automatiquement. Si une valeur a un no
\textbf{Partie B}
\medskip
Dans cette partie, on suppose que 32.0\,\% des stylos possèdent un défaut de fabrication.
Dans cette partie, on suppose que 83.0\,\% des stylos possèdent un défaut de fabrication.
L'entreprise confectionne des paquets contenant chacun $4$~stylos.
@ -266,7 +265,7 @@ Les valeurs des exercices sont générés automatiquement. Si une valeur a un no
\begin{enumerate}
\setcounter{enumi}{4}
\item Avec quelle loi peut-on modéliser $X$. Préciser les paramètres.
\item Calculer et interpréter la probabilité $P(X = 7)$.
\item Calculer et interpréter la probabilité $P(X = 12)$.
\item Le directeur de l'entreprise affirme qu'il y a plus d'une chance sur deux qu'un paquet ne comporte aucun stylo défectueux. A-t-il raison ?
\item Combien de stylos peut-on espérer avoir en moyenne?
\end{enumerate}
@ -282,27 +281,27 @@ Les valeurs des exercices sont générés automatiquement. Si une valeur a un no
child {node {$A$}
child {node {$D$}
edge from parent
node[above] {0.31}
node[above] {0.52}
}
child {node {$\overline{D}$}
edge from parent
node[above] {0.69}
node[above] {0.48}
}
edge from parent
node[above] {0.43}
node[above] {0.24}
}
child[missing] {}
child { node {$B$}
child {node {$D$}
edge from parent
node[above] {0.34}
node[above] {0.94}
}
child {node {$\overline{D}$}
edge from parent
node[above] {0.66}
node[above] {0.06}
}
edge from parent
node[above] {0.57}
node[above] {0.76}
} ;
\end{tikzpicture}
\end{center}
@ -310,51 +309,51 @@ Les valeurs des exercices sont générés automatiquement. Si une valeur a un no
\begin{itemize}
\item Probabilité que le stylo vienne de l'atelier A
\[
P(A) = 0.43
P(A) = 0.24
\]
\item Probabilité que le stylo vienne de l'atelier B
\[
P(B) = 0.57
P(B) = 0.76
\]
\item Probabilité que le stylo ait un défaut sachant qu'il vient de l'atelier A.
\[
P_A(D) = 0.31
P_A(D) = 0.52
\]
\item Probabilité que le stylo vienne de l'atelier B et qu'il ait un défaut.
\[
P(D \cap D) = 0.19
P(D \cap D) = 0.71
\]
\end{itemize}
\item
\begin{enumerate}
\item Probabilité qu'un stylo vienne de l'atelier A et qu'il ait un defaut
\[
P(A\cap D) = P(A) \times P_A(D) = 0.43 \times 0.31 = 0.13
P(A\cap D) = P(A) \times P_A(D) = 0.24 \times 0.52 = 0.12
\]
\item Probabilité que le stylo ai un défaut de fabrication.
\[
P(D) = P(A\cap D) + P(B\cap D) = 0.13 + 0.19 = 0.32
P(D) = P(A\cap D) + P(B\cap D) = 0.12 + 0.71 = 0.83
\]
\end{enumerate}
\item Probabilité qu'il vienne de l'atelier A sachant qu'il a un defaut
\[
P_D(A) = \frac{P(A\cap D)}{P(D)} = \frac{0.13}{0.32} = 0.41
P_D(A) = \frac{P(A\cap D)}{P(D)} = \frac{0.12}{0.83} = 0.14
\]
\item $X$ peut être modélisée par une loi binomiale de paramètres $n=10$ et $p=0.32$.
\item $X$ peut être modélisée par une loi binomiale de paramètres $n=18$ et $p=0.83$.
\item (\textit{par de correction automatique disponible pour le résultat final}
\[
P(X = 7) = \coefBino{10}{7}\times 0.32^{7} \times 0.68^{3}
P(X = 12) = \coefBino{18}{12}\times 0.83^{12} \times 0.17^{6}
\]
\item (\textit{par de correction automatique disponible pour le résultat final}
Il faut calculer la probabilité qu'il y ait 0 stylo avec un defaut.
\[
P(X = 0) = \coefBino{10}{0}\times 0.32^{0} \times 0.68^{10}
P(X = 0) = \coefBino{18}{0}\times 0.83^{0} \times 0.17^{18}
\]
Puis comparer ce nombre à 0,5.
\item Il faut calculer l'espérance
\[
E[X] = n\times p = 10 \times 0.32 = 3.2
E[X] = n\times p = 18 \times 0.83 = 14.94
\]
\end{enumerate}
\end{solution}

View File

@ -3,7 +3,7 @@
% Title Page
\title{DM1 \hfill VANDROUX Guillemette}
\tribe{Maths complémentaire}
\tribe{Maths complémentaires}
\date{\hfillÀ render pour le jeudi 27 mai}
\xsimsetup{
@ -17,7 +17,7 @@ Les valeurs des exercices sont générés automatiquement. Si une valeur a un no
\begin{exercise}[subtitle={Optimisation de matière}]
\begin{minipage}{0.6\textwidth}
On se propose de fabriquer avec le moins de tôle possible une citerne fermée en forme de parallélépipède rectangle dont le volume intérieur doit être de $30m^3$. La longueur est aussi fixée à $3m$ par le cahier des charges.
On se propose de fabriquer avec le moins de tôle possible une citerne fermée en forme de parallélépipède rectangle dont le volume intérieur doit être de $27m^3$. La longueur est aussi fixée à $3m$ par le cahier des charges.
On peut donc faire varier uniquement la largeur (notée $x$) et la hauteur (notée $h$) de la cuve.
\end{minipage}
@ -35,18 +35,18 @@ Les valeurs des exercices sont générés automatiquement. Si une valeur a un no
\begin{enumerate}
\item Expliquer pourquoi quand la largeur $x$ change, la hauteur $h$ doit elle aussi changer pour respecter les contraintes.
\item Démontrer que l'on doit avoir $h = \dfrac{10}{x}$.
\item Démontrer que l'on doit avoir $h = \dfrac{9}{x}$.
\item On note $S(x)$ l'aire totale de la citerne (c'est à dire la somme des aires des six faces). Montrer que l'on peut écrire
\[
S(x) = 6x + 20 + \frac{60}{x}
S(x) = 6x + 18 + \frac{54}{x}
\]
\item Démontrer que
\[
S(x) = \frac{6x^2 + 20x + 60}{x}
S(x) = \frac{6x^2 + 18x + 54}{x}
\]
\item Démontrer que
\[
S'(x) = \frac{6x^2 - 60}{x^2}
S'(x) = \frac{6x^2 - 54}{x^2}
\]
\item En déduire le tableau de variation de $S(x)$ sur $\intOF{0}{10}$.
\item Déterminer les valeurs de $x$ et $h$ correspondant à une utilisation minimal de tôle.
@ -57,62 +57,62 @@ Les valeurs des exercices sont générés automatiquement. Si une valeur a un no
\begin{enumerate}
\item Le volume étant fixe si l'on fait varier $x$, $h$ doit aussi varier.
\begin{itemize}
\item Si $x = 2$ alors conserver un volume de $V=30$, $h$ doit être égale à $10 / 2$
\item Si $x = 3$ alors conserver un volume de $V=30$, $h$ doit être égale à $10 / 3$
\item Si $x = 2$ alors conserver un volume de $V=27$, $h$ doit être égale à $9 / 2$
\item Si $x = 3$ alors conserver un volume de $V=27$, $h$ doit être égale à $9 / 3$
\end{itemize}
\item Pour calculer le volume, on a
\begin{eqnarray*}
V &=& h\times x \times 3 \\
30 &=& h\times x \times 3 \\
x &=& \frac{30}{h\times 3} = \frac{10}{h}
27 &=& h\times x \times 3 \\
x &=& \frac{27}{h\times 3} = \frac{9}{h}
\end{eqnarray*}
\item Pour calculer la surface totale, on ajoute la surface de chaque face. On a donc le calcul suivant
\begin{eqnarray*}
S(x) &=& x\times h \times 2 + x\times3\times2 + h\times 3\times 2\\
S(x) &=& x\times \frac{10}{x} \times 2 + x\times3\times2 + \frac{10}{x}\times 3\times 2\\
S(x) &=& 6x + 20 + \frac{60}{x}
S(x) &=& x\times \frac{9}{x} \times 2 + x\times3\times2 + \frac{9}{x}\times 3\times 2\\
S(x) &=& 6x + 18 + \frac{54}{x}
\end{eqnarray*}
\item Pour trouver cette nouvelle forme, on met chaque élément sur le même dénominateur
\begin{eqnarray*}
S(x) &=& 6x + 20 + \frac{60}{x}\\
S(x) &=& \frac{6x\times x}{x} + \frac{20\times x}{x} + \frac{60}{x}\\
S(x) &=& \frac{6x^2 + 20x + 60}{x}
S(x) &=& 6x + 18 + \frac{54}{x}\\
S(x) &=& \frac{6x\times x}{x} + \frac{18\times x}{x} + \frac{54}{x}\\
S(x) &=& \frac{6x^2 + 18x + 54}{x}
\end{eqnarray*}
\item On retrouve la formule $\frac{u}{v}$ à dériver
\[
u(x) = 6x^2 + 20x + 60 \Rightarrow u'(x) = 12x + 20
u(x) = 6x^2 + 18x + 54 \Rightarrow u'(x) = 12x + 18
\]
\[
v(x) = x \Rightarrow v'(x) = 1
\]
Donc au numérateur on obtient
\begin{eqnarray*}
u'(x)\times v(x) - u(x)\times v'(x) &=& (12x + 20)\times x - (6x^2 + 20x + 60)\times 1\\
&=& 6x^2 - 60
u'(x)\times v(x) - u(x)\times v'(x) &=& (12x + 18)\times x - (6x^2 + 18x + 54)\times 1\\
&=& 6x^2 - 54
\end{eqnarray*}
Donc
\[
S'(x) = \frac{6x^2 - 60}{x^2}
S'(x) = \frac{6x^2 - 54}{x^2}
\]
\item Tableau de variations de $S$
\begin{itemize}
\item Valeur interdite: $x^2 = 0 \equiv x = 0$
\item Signe de $6x^2 - 60$: c'est un polynôme du 2e degré
\item Signe de $6x^2 - 54$: c'est un polynôme du 2e degré
\[
\Delta = 1440 > 0
\Delta = 1296 > 0
\]
Il y a donc 2 racines
\[
x_1 = - 3.1622776601683795 \qquad
x_2 = 3.1622776601683795
x_1 = - 3 \qquad
x_2 = 3
\]
Et on sait que $6x^2 - 60$ est du signe de $a$ donc positif en dehors des racines
Et on sait que $6x^2 - 54$ est du signe de $a$ donc positif en dehors des racines
\item Le dénominateur $x^2$ est toujours positif.
\item Tableau de variations
\begin{tikzpicture}[baseline=(a.north)]
\tkzTabInit[lgt=3,espcl=3]{$x$/1,$6x^2 - 60$/1, $x^2$/1, $S'$/1, $S$/2}{$0$, $- 3.1622776601683795$, $10$}
\tkzTabInit[lgt=3,espcl=3]{$x$/1,$6x^2 - 54$/1, $x^2$/1, $S'$/1, $S$/2}{$0$, $- 3$, $10$}
\tkzTabLine{d,-, z, +, }
\tkzTabLine{d,+, , +, }
\tkzTabLine{d,-, z, +, }
@ -120,7 +120,7 @@ Les valeurs des exercices sont générés automatiquement. Si une valeur a un no
\end{tikzpicture}
\end{itemize}
\item On a donc une surface minimal pour $x=3.1622776601683795$ et $h = 31.6227766016837950$.
\item On a donc une surface minimal pour $x=3$ et $h = 27$.
\end{enumerate}
\end{solution}
@ -133,14 +133,14 @@ Les valeurs des exercices sont générés automatiquement. Si une valeur a un no
Le tour d'un bassin au niveau du sol présente deux axes de symétrie : laxe des abscisses et la droite déquation $x=4$. Il est obtenu par symétrie de la courbe $\mathcal{C}_f$ sur $\intFF{0}{4}$$f$ est la fonction définie par
\[
f(x) = \left(- x^{2} + 5.0 x - 1.8\right) e^{- x} + 1.8
f(x) = \left(- x^{2} + 2.3 x - 7.0\right) e^{- x} + 7.0
\]
On admet que sur $\intFF{0}{4}$ la fonction $f$ est positive.
\begin{enumerate}
\item Sur un repère, tracer l'allure de la courbe $\mathcal{C}_f$, les axes de symétries puis compléter pour dessiner la forme du bassin.
\item Montrer que la fonction $f$ admet comme primitive sur $\R$ la fonction $F$ définie par
\[
F(x) = 1.8 x + \left( x^{2} - 3.0 x - 1.2\right) e^{- x}
F(x) = 7.0 x + \left( x^{2} - 0.3 x + 6.7\right) e^{- x}
\]
\item Calculer la quantité $\ds \int_0^4 f(x) \; dx$, vous donnerez le résultat sous forme exacte. Interpréter le résultat et reportez cette quantité sur le graphique.
\item On considère que l'échelle de votre graphique est de 1unité pour 15m. Calculer l'aire du bassin. Vous donnerez un résultat arrondi au $m^2$ près.
@ -156,15 +156,14 @@ Les valeurs des exercices sont générés automatiquement. Si une valeur a un no
\tkzGrid
\tkzAxeXY
\tkzFct[domain=0:10,color=red,very thick]%
{ (-x**2 + 5.0*x - 1.8)*exp(-x) + 1.8 };
{ (-x**2 + 2.3*x - 7.0)*exp(-x) + 7.0 };
\end{tikzpicture}
\item Il faut dériver $F(x)$ et vérifier que $F'(x) = f(x)$.
\item $\ds \int_0^4 f(x) \; dx = F(4) - F(0) = \frac{2.8}{e^{4}} + 8.4$
\item $\ds \int_0^4 f(x) \; dx = F(4) - F(0) = \frac{21.5}{e^{4}} + 21.3$
\item La quantité calculée à la question précédente se retrouve 4fois pour former le bassin. Il faut ensuite prendre en compte l'échelle, comme 1unité de longueur correspond à 15m, une unité d'air correspond à $15\times15 = 225m^2$. Ainsi l'aire du bassin est égale à
\[
(\frac{2.8}{e^{4}} + 8.4)\times 4 \times 15^2 = 7606.000000
(\frac{21.5}{e^{4}} + 21.3)\times 4 \times 15^2 = 19524.00000
\]
\end{enumerate}
\end{solution}
@ -185,9 +184,9 @@ Les valeurs des exercices sont générés automatiquement. Si une valeur a un no
Deux ateliers A et B fabriquent des stylos pour une entreprise.
L'atelier A fabrique 55.00000000000001\,\% des stylos, et parmi ceux-là, 46.0\,\% possèdent un défaut de fabrication.
L'atelier A fabrique 43.0\,\% des stylos, et parmi ceux-là, 9.0\,\% possèdent un défaut de fabrication.
De plus, 20.0\,\% des stylos possèdent un défaut de fabrication et sortent de l'atelier B.
De plus, 27.0\,\% des stylos possèdent un défaut de fabrication et sortent de l'atelier B.
Un stylo est prélevé au hasard dans le stock de l'entreprise.
@ -241,7 +240,7 @@ Les valeurs des exercices sont générés automatiquement. Si une valeur a un no
\item
\begin{enumerate}
\item Calculer la probabilité qu'un stylo provienne de l'atelier A et possède un défaut de fabrication.
\item En déduire que la probabilité qu'un stylo possède un défaut de fabrication est de $0.45$.
\item En déduire que la probabilité qu'un stylo possède un défaut de fabrication est de $0.31$.
\end{enumerate}
\item On prélève un stylo au hasard avec un défaut. Quelle est la probabilité qu'il vienne de l'atelier A?
\end{enumerate}
@ -251,7 +250,7 @@ Les valeurs des exercices sont générés automatiquement. Si une valeur a un no
\textbf{Partie B}
\medskip
Dans cette partie, on suppose que 45.0\,\% des stylos possèdent un défaut de fabrication.
Dans cette partie, on suppose que 31.0\,\% des stylos possèdent un défaut de fabrication.
L'entreprise confectionne des paquets contenant chacun $4$~stylos.
@ -266,7 +265,7 @@ Les valeurs des exercices sont générés automatiquement. Si une valeur a un no
\begin{enumerate}
\setcounter{enumi}{4}
\item Avec quelle loi peut-on modéliser $X$. Préciser les paramètres.
\item Calculer et interpréter la probabilité $P(X = 13)$.
\item Calculer et interpréter la probabilité $P(X = 10)$.
\item Le directeur de l'entreprise affirme qu'il y a plus d'une chance sur deux qu'un paquet ne comporte aucun stylo défectueux. A-t-il raison ?
\item Combien de stylos peut-on espérer avoir en moyenne?
\end{enumerate}
@ -282,27 +281,27 @@ Les valeurs des exercices sont générés automatiquement. Si une valeur a un no
child {node {$A$}
child {node {$D$}
edge from parent
node[above] {0.46}
node[above] {0.09}
}
child {node {$\overline{D}$}
edge from parent
node[above] {0.54}
node[above] {0.91}
}
edge from parent
node[above] {0.55}
node[above] {0.43}
}
child[missing] {}
child { node {$B$}
child {node {$D$}
edge from parent
node[above] {0.44}
node[above] {0.48}
}
child {node {$\overline{D}$}
edge from parent
node[above] {0.56}
node[above] {0.52}
}
edge from parent
node[above] {0.45}
node[above] {0.57}
} ;
\end{tikzpicture}
\end{center}
@ -310,51 +309,51 @@ Les valeurs des exercices sont générés automatiquement. Si une valeur a un no
\begin{itemize}
\item Probabilité que le stylo vienne de l'atelier A
\[
P(A) = 0.55
P(A) = 0.43
\]
\item Probabilité que le stylo vienne de l'atelier B
\[
P(B) = 0.45
P(B) = 0.57
\]
\item Probabilité que le stylo ait un défaut sachant qu'il vient de l'atelier A.
\[
P_A(D) = 0.46
P_A(D) = 0.09
\]
\item Probabilité que le stylo vienne de l'atelier B et qu'il ait un défaut.
\[
P(D \cap D) = 0.2
P(D \cap D) = 0.27
\]
\end{itemize}
\item
\begin{enumerate}
\item Probabilité qu'un stylo vienne de l'atelier A et qu'il ait un defaut
\[
P(A\cap D) = P(A) \times P_A(D) = 0.55 \times 0.46 = 0.25
P(A\cap D) = P(A) \times P_A(D) = 0.43 \times 0.09 = 0.04
\]
\item Probabilité que le stylo ai un défaut de fabrication.
\[
P(D) = P(A\cap D) + P(B\cap D) = 0.25 + 0.2 = 0.45
P(D) = P(A\cap D) + P(B\cap D) = 0.04 + 0.27 = 0.31
\]
\end{enumerate}
\item Probabilité qu'il vienne de l'atelier A sachant qu'il a un defaut
\[
P_D(A) = \frac{P(A\cap D)}{P(D)} = \frac{0.25}{0.45} = 0.56
P_D(A) = \frac{P(A\cap D)}{P(D)} = \frac{0.04}{0.31} = 0.13
\]
\item $X$ peut être modélisée par une loi binomiale de paramètres $n=15$ et $p=0.45$.
\item $X$ peut être modélisée par une loi binomiale de paramètres $n=11$ et $p=0.31$.
\item (\textit{par de correction automatique disponible pour le résultat final}
\[
P(X = 13) = \coefBino{15}{13}\times 0.45^{13} \times 0.55^{2}
P(X = 10) = \coefBino{11}{10}\times 0.31^{10} \times 0.69^{1}
\]
\item (\textit{par de correction automatique disponible pour le résultat final}
Il faut calculer la probabilité qu'il y ait 0 stylo avec un defaut.
\[
P(X = 0) = \coefBino{15}{0}\times 0.45^{0} \times 0.55^{15}
P(X = 0) = \coefBino{11}{0}\times 0.31^{0} \times 0.69^{11}
\]
Puis comparer ce nombre à 0,5.
\item Il faut calculer l'espérance
\[
E[X] = n\times p = 15 \times 0.45 = 6.75
E[X] = n\times p = 11 \times 0.31 = 3.41
\]
\end{enumerate}
\end{solution}